You are on page 1of 467

ARCH-MLC

Fall 2007 SOA MLC



Exam Date: November 6, 2007 8:30 am 11:30 am

by Yufeng Guo, ASA

www.archactuarial.com www.guo.coursehost.com 2
Feature of Arch MLC Manual
Cover everything in the SOA MLC syllabus.
Thorough explanation of the core concepts with worked out problems
Reference to Actuarial Mathematics and Models for Quantifying Risk. You can use
either textbook as a companion to go with Arch MLC.
Low Cost: $99. For $99, you bring a quality study manual home.
No shipping charge. Good saving for international exam candidates.
Convenience. After you buy Arch MLC PDF, you can print a hard copy. You can also
install the PDF in your computer.
PDF has detailed bookmarks for quick reference.
PDF has a clickable table of contents for quick reference.
Arch MLC, Fall 2007 c _Yufeng Guo 2
Contents
0 INTRODUCTION 9
The origin of Arch Manual . . . . . . . . . . . . . . . . . . . . . . . . . . . . 9
Praises of the Arch Manual originally written by Nathan and Robin . . . . . 10
About Yufeng Guo . . . . . . . . . . . . . . . . . . . . . . . . . . . . . . . . . 10
How to use this manual . . . . . . . . . . . . . . . . . . . . . . . . . . . . . . 11
1 ACTUARIAL MATHEMATICS: CHAPTER 3 SURVIVAL DISTRIBU-
TIONS AND LIFE TABLES 13
3.2.1 The Survival Function . . . . . . . . . . . . . . . . . . . . . . . . . . . . 15
3.2.2 Time-until-Death for a Person Age x . . . . . . . . . . . . . . . . . . . . 16
3.2.3 Curtate-Future-Lifetime . . . . . . . . . . . . . . . . . . . . . . . . . . . 19
3.2.4 Force of Mortality . . . . . . . . . . . . . . . . . . . . . . . . . . . . . . 20
3.3-3.5 Life Tables . . . . . . . . . . . . . . . . . . . . . . . . . . . . . . . . . 21
3.5.2 Recursion Formulas . . . . . . . . . . . . . . . . . . . . . . . . . . . . . 27
3.6 Assumptions for Fractional Ages . . . . . . . . . . . . . . . . . . . . . . . 28
3.7 Some Analytical Laws of Mortality . . . . . . . . . . . . . . . . . . . . . . 33
Modied DeMoivres Law . . . . . . . . . . . . . . . . . . . . . . . . . . . . . 35
3.8 Select and Ultimate Tables . . . . . . . . . . . . . . . . . . . . . . . . . . 35
Conclusion . . . . . . . . . . . . . . . . . . . . . . . . . . . . . . . . . . . . . 38
CHAPTER 3 Formula Summary . . . . . . . . . . . . . . . . . . . . . . . . . 39
Past SOA/CAS Exam Questions: . . . . . . . . . . . . . . . . . . . . . . . . . 43
Problems from Pre-2000 SOA-CAS exams . . . . . . . . . . . . . . . . . . . . 55
Solutions to Chapter 3 . . . . . . . . . . . . . . . . . . . . . . . . . . . . . . . 57
2 ACTUARIAL MATHEMATICS CHAPTER 4 LIFE INSURANCE 59
4.2 Insurances Payable at the Moment of Death . . . . . . . . . . . . . . . . . 60
TYPES OF INSURANCE . . . . . . . . . . . . . . . . . . . . . . . . . . . . . 61
4.2.1 Level Benet Insurance . . . . . . . . . . . . . . . . . . . . . . . 61
4.2.2 Endowment Insurance . . . . . . . . . . . . . . . . . . . . . . . . 64
4.2.3 Deferred Insurance . . . . . . . . . . . . . . . . . . . . . . . . . . 66
4.2.4 Varying Benet Insurance . . . . . . . . . . . . . . . . . . . . . . 68
4.3 Insurances Payable at the End of the Year of Death . . . . . . . . . . . . 71
4.4 Relationships between Insurances Payable at the Moment of death and the
End of the Year of Death . . . . . . . . . . . . . . . . . . . . . . . . . . . . . 77
CHAPTER 4 Formula Summary . . . . . . . . . . . . . . . . . . . . . . . . . 79
Past SOA/CAS Exam Questions: . . . . . . . . . . . . . . . . . . . . . . . . . 81
3
www.archactuarial.com www.guo.coursehost.com 4
Problems from Pre-2000 SOA-CAS exams . . . . . . . . . . . . . . . . . . . . 91
Solutions . . . . . . . . . . . . . . . . . . . . . . . . . . . . . . . . . . . . . . 93
3 ACTUARIAL MATHEMATICS: CHAPTER 5 LIFE ANNUITIES 95
5.2 Continuous Life Annuities . . . . . . . . . . . . . . . . . . . . . . . . . . . 95
The most important equation so far(!) . . . . . . . . . . . . . . . . . . . . . . 98
5.3 Discrete Life Annuities . . . . . . . . . . . . . . . . . . . . . . . . . . . . . 104
5.4 Life Annuities with m-thly Payments . . . . . . . . . . . . . . . . . . . . . 110
CHAPTER 5 Formula Summary . . . . . . . . . . . . . . . . . . . . . . . . . 114
Continuous Annuities: . . . . . . . . . . . . . . . . . . . . . . . . . . . 114
Discrete annuities: . . . . . . . . . . . . . . . . . . . . . . . . . . . . . 115
Past SOA/CAS Exam Questions: . . . . . . . . . . . . . . . . . . . . . . . . . 116
Problems from Pre-2000 SOA-CAS exams . . . . . . . . . . . . . . . . . . . . 129
Solutions to Pre-2000 Problems: Chapter 5 . . . . . . . . . . . . . . . . . . . 131
4 ACTUARIAL MATHEMATICS: CHAPTER 6 BENEFIT PREMIUMS 133
6.2 Fully Continuous Premiums . . . . . . . . . . . . . . . . . . . . . . . . . . 134
6.3 Fully Discrete Premiums . . . . . . . . . . . . . . . . . . . . . . . . . . . . 140
6.4 True m-thly Payment Premiums . . . . . . . . . . . . . . . . . . . . . . . 145
CHAPTER 6 Formula Summary . . . . . . . . . . . . . . . . . . . . . . . . . 149
Past SOA/CAS Exam Questions: . . . . . . . . . . . . . . . . . . . . . . . . . 150
Problems from Pre-2000 SOA-CAS exams . . . . . . . . . . . . . . . . . . . . 166
Solutions to Pre-2000 Exam Questions: Chapter 6 . . . . . . . . . . . . . . . 168
5 ACTUARIAL MATHEMATICS: CHAPTER 7 BENEFIT RESERVES 171
7.2 Fully Continuous Benet Reserves . . . . . . . . . . . . . . . . . . . . . . 172
7.3 Other Methods for Calculating the Benet Reserve . . . . . . . . . . . . . 175
1) Prospective Formula . . . . . . . . . . . . . . . . . . . . . . . . . . 175
2) Retrospective Formula . . . . . . . . . . . . . . . . . . . . . . . . . 175
3) Premium Dierence Formula . . . . . . . . . . . . . . . . . . . . . . 178
4) Paid-Up Insurance Formula . . . . . . . . . . . . . . . . . . . . . . 178
5) Other Reserve Formulas . . . . . . . . . . . . . . . . . . . . . . . . 179
7.4 Fully Discrete Reserves . . . . . . . . . . . . . . . . . . . . . . . . . . . . 181
7.5 Benet Reserves on a Semi-Continuous Basis . . . . . . . . . . . . . . . . 184
7.6 Benet Reserves Based on True m-thly Benet Premiums . . . . . . . . . 185
CHAPTER 7 Formula Summary . . . . . . . . . . . . . . . . . . . . . . . . . 186
Continuous Reserve Formulas: . . . . . . . . . . . . . . . . . . . . . . . . . . . 186
Discrete Reserves: . . . . . . . . . . . . . . . . . . . . . . . . . . . . . 186
Past SOA/CAS Exam Questions: . . . . . . . . . . . . . . . . . . . . . . . . . 188
Problems from Pre-2000 SOA-CAS exams . . . . . . . . . . . . . . . . . . . . 192
Solutions to Pre-2000 Questions: Chapter 7 . . . . . . . . . . . . . . . . . . . 194
6 ACTUARIAL MATHEMATICS: CHAPTER 8 ANALYSIS OF BENEFIT
RESERVES 195
8.2 Benet Reserves for General Insurances . . . . . . . . . . . . . . . . . . . 195
8.3 Recursion Relations for Fully Discrete Benet Reserves . . . . . . . . . . 200
8.4 Benet Reserves at Fractional Durations . . . . . . . . . . . . . . . . . . . 203
Arch MLC, Fall 2007 c _Yufeng Guo 4
www.archactuarial.com www.guo.coursehost.com 5
8.5 The Hattendorf Theorem . . . . . . . . . . . . . . . . . . . . . . . . . . . 206
CHAPTER 8 Formula Summary . . . . . . . . . . . . . . . . . . . . . . . . . 210
Chapter 8 More Formulas . . . . . . . . . . . . . . . . . . . . . . . . . . . . . 211
Past SOA/CAS Exam Questions: . . . . . . . . . . . . . . . . . . . . . . . . . 212
Problems from Pre-2000 SOA-CAS exams . . . . . . . . . . . . . . . . . . . . 232
Solutions to Pre-2000 Problems: Chapter 8 . . . . . . . . . . . . . . . . . . . 235
7 ACTUARIAL MATHEMATICS: CHAPTER 9 MULTIPLE LIFE FUNC-
TIONS 239
9.2 Joint Distributions of Future Lifetimes . . . . . . . . . . . . . . . . . . . . 239
9.3 Joint Life Status . . . . . . . . . . . . . . . . . . . . . . . . . . . . . . . . 241
The following is important! . . . . . . . . . . . . . . . . . . . . . . . . 241
9.4 Last Survivor Status . . . . . . . . . . . . . . . . . . . . . . . . . . . . . . 246
9.5 More Probabilities and Expectations . . . . . . . . . . . . . . . . . . . . . 248
9.6 Dependent Lifetime Models . . . . . . . . . . . . . . . . . . . . . . . . . . 251
9.6.1 Common Shock (Non-Theoretical Version) . . . . . . . . . . . . 251
9.7 Insurance and Annuity Benets . . . . . . . . . . . . . . . . . . . . . . . . 254
9.7.1 Survival Statuses . . . . . . . . . . . . . . . . . . . . . . . . . . . 254
9.7.2 Special Two-Life Annuities . . . . . . . . . . . . . . . . . . . . . 260
9.7.3 Reversionary Annuities . . . . . . . . . . . . . . . . . . . . . . . 261
9.9 Simple Contingent Functions . . . . . . . . . . . . . . . . . . . . . . . . . 263
CHAPTER 9 Formula Summary . . . . . . . . . . . . . . . . . . . . . . . . . 266
Past SOA/CAS Exam Questions: . . . . . . . . . . . . . . . . . . . . . . . . . 268
Problems from Pre-2000 SOA-CAS exams . . . . . . . . . . . . . . . . . . . . 281
Solutions to Pre-2000 Exam Questions: Chapter 9 . . . . . . . . . . . . . . . 283
8 ACTUARIAL MATHEMATICS: CHAPTER 10 MULTIPLE DECREMENT
MODELS 285
10.2 Two Random Variables . . . . . . . . . . . . . . . . . . . . . . . . . . . . 285
Probability density functions: . . . . . . . . . . . . . . . . . . . . . . . 287
10.3 Random Survivorship Group . . . . . . . . . . . . . . . . . . . . . . . . . 290
10.4 Deterministic Survivorship Group . . . . . . . . . . . . . . . . . . . . . . 290
10.5 Associated Single Decrement Tables . . . . . . . . . . . . . . . . . . . . . 294
10.5.1 Basic Relationships . . . . . . . . . . . . . . . . . . . . . . . . . 295
10.5.4 Uniform Distribution Assumption for Multiple Decrements . . . 296
10.6 Construction of a Multiple Decrement Table . . . . . . . . . . . . . . . . 298
CASE I : Two decrements that are uniformly distributed in the asso-
ciated single decrement table . . . . . . . . . . . . . . . . . . . . . . . 298
CASE II : Three decrements that are uniformly distributed in the as-
sociated single decrement table . . . . . . . . . . . . . . . . . . . . . . 298
CASE III : Multiple decrements some are uniformly distributed in
the associated single decrement table and some are not. . . . . . . . . 298
CHAPTER 10 Formula Summary . . . . . . . . . . . . . . . . . . . . . . . . . 303
Past SOA/CAS Exam Questions: . . . . . . . . . . . . . . . . . . . . . . . . . 305
Problems from Pre-2000 SOA-CAS Exams . . . . . . . . . . . . . . . . . . . . 315
Solutions to Pre-2000 Problems: Chapter 10 . . . . . . . . . . . . . . . . . . . 318
Arch MLC, Fall 2007 c _Yufeng Guo 5
www.archactuarial.com www.guo.coursehost.com 6
9 ACTUARIAL MATHEMATICS: CHAPTER 11 APPLICATIONS OF MUL-
TIPLE DECREMENT THEORY 321
11.2 Actuarial Present Values and Their Numerical Estimation . . . . . . . . 321
11.3 Benet Premiums and Reserves . . . . . . . . . . . . . . . . . . . . . . . 322
CHAPTER 11 Formula Summary . . . . . . . . . . . . . . . . . . . . . . . . . 325
ARCH Sample Exam Problem . . . . . . . . . . . . . . . . . . . . . . . . . . 325
Solution: . . . . . . . . . . . . . . . . . . . . . . . . . . . . . . . . . . . . . . . 325
Past SOA/CAS Exam Questions: . . . . . . . . . . . . . . . . . . . . . . . . . 327
10 ACTUARIAL MATHEMATICS: CHAPTER 15 INSURANCE MODELS
INCLUDING EXPENSES 333
15.2 Expense Augmented Models . . . . . . . . . . . . . . . . . . . . . . . . . 333
15.4 More Expenses . . . . . . . . . . . . . . . . . . . . . . . . . . . . . . . . 336
15.6.1 Asset Shares . . . . . . . . . . . . . . . . . . . . . . . . . . . . . . . . . 339
CHAPTER 15 Formula Summary . . . . . . . . . . . . . . . . . . . . . . . . . 341
Asset Shares . . . . . . . . . . . . . . . . . . . . . . . . . . . . . . . . 341
Past SOA/CAS Exam Questions: . . . . . . . . . . . . . . . . . . . . . . . . . 341
SOLUTIONS to Past SOA-CAS Exam Problems: . . . . . . . . . . . . . . . . 345
11 DANIEL CHAPTER 1 - MULTI-STATE TRANSITION MODELS FOR
ACTUARIAL APPLICATIONS 349
1.1 Introduction . . . . . . . . . . . . . . . . . . . . . . . . . . . . . . . . . . . 349
1.2 Non-homogeneous Markov Chains . . . . . . . . . . . . . . . . . . . . . . 352
CHAPTER 2 CASH FLOWS AND THEIR ACTUARIAL PRESENT VALUES357
Section 2.1 Introduction . . . . . . . . . . . . . . . . . . . . . . . . . . 357
Cash Flows while in states . . . . . . . . . . . . . . . . . . . . . . . . . . . . . 357
Cash Flows upon transitions . . . . . . . . . . . . . . . . . . . . . . . . . . . . 362
Actuarial Present Values . . . . . . . . . . . . . . . . . . . . . . . . . . . . . . 362
ARCH Warm-up Questions: . . . . . . . . . . . . . . . . . . . . . . . . . . . . 367
Solutions . . . . . . . . . . . . . . . . . . . . . . . . . . . . . . . . . . . . . . 370
Past SOA/CAS Exam Questions: . . . . . . . . . . . . . . . . . . . . . . . . . 373
12 PROBABILITY MODELS: CHAPTER 5 THE EXPONENTIAL DISTRI-
BUTION AND THE POISSON PROCESS 383
5.3 The Poisson Process . . . . . . . . . . . . . . . . . . . . . . . . . . . . . . 383
5.3.1 Counting Processes . . . . . . . . . . . . . . . . . . . . . . . . . . 383
5.3.2 Denition of the Poisson Process . . . . . . . . . . . . . . . . . . 384
5.3.3 Interarrival and Waiting Time Distributions . . . . . . . . . . . . 385
5.3.4 Further Properties of Poisson Processes . . . . . . . . . . . . . . 387
5.3.5 Conditional Distribution of the Arrival Times . . . . . . . . . . . 390
5.4 Generalizations of the Poisson Process . . . . . . . . . . . . . . . . . . . . 392
5.4.1 Nonhomogeneous Poisson Process . . . . . . . . . . . . . . . . . 392
5.4.2 Compound Poisson Process . . . . . . . . . . . . . . . . . . . . . 393
5.4.3 Conditional or Mixed Poisson Processes: Gamma-Poisson Model 396
CHAPTER 5 Formula Summary . . . . . . . . . . . . . . . . . . . . . . . . . 399
ARCH Warm-up Problems: . . . . . . . . . . . . . . . . . . . . . . . . . . . . 400
Solutions: . . . . . . . . . . . . . . . . . . . . . . . . . . . . . . . . . . . . . . 401
Arch MLC, Fall 2007 c _Yufeng Guo 6
www.archactuarial.com www.guo.coursehost.com 7
Past SOA/CAS Exam Questions: . . . . . . . . . . . . . . . . . . . . . . . . . 402
13 ARCH Practice Exam 415
Answer Key for Practice Exam . . . . . . . . . . . . . . . . . . . . . . . . . . 428
14 DELUXE SOLUTION TO MAY 2007 MLC 447
Arch MLC, Fall 2007 c _Yufeng Guo 7
www.archactuarial.com www.guo.coursehost.com 8
Arch MLC, Fall 2007 c _Yufeng Guo 8
Chapter 0
INTRODUCTION
The origin of Arch Manual
The Arch Manual was originally written by two gifted actuaries, Nathan Hardiman and Robin
Cunningham.
In the late 90s, Nathan Hardiman and Robin Cunningham worked full-time at the former
Jeerson Pilot Financial. Nathan and Robin, like other exam candidates, faced the daunt-
ing challenge of plowing through dicult textbooks and mastering the fundamentals of life
contingency theories and probability models to pass the Course 3 exam, the most dicult of
the four preliminary actuarial exams and the exam with the highest failure rate.
The diculty of Course 3 was mainly due to its enormous scope. Candidates were required
to read encyclopedia-like textbooks such as Actuarial Mathematics and Probability Models,
gain sophisticated understanding of complex concepts such as multiple decrements, Markov
Chain, Brownian motion, and be ready to tackle tricky word problems on the exam.
Since Nathan and Robin both already had families and full-time jobs when they began study-
ing for exams, they created their own study framework and philosophy for quickly passing
Course 3. After passing the exam in one sitting using their unique study methods, Nathan
and Robin decided to jointly write a new study guide that would enable candidates to build a
core body of knowledge for Course 3 quickly. They wanted their manual to use straight talk
and down-to-earth examples to explain dicult fundamental concepts intuitively and simply.
Nathan and Robin published the rst edition of their study manual for Course 3 in 2001.
They named their study manual the Arch-3. Since its publication, Arch has been a popular
study manual for Course 3 and Exam M.
Archs power lies in its simplicity. While textbooks talk fancy, Arch talks simple. While
textbooks rigorously prove theorems, Arch explains the intuition. While textbooks demand
attention to everything, Arch separates the critical from the trivial.
While Arch sells well, Nathan and Robin climbed corporate ladder higher and higher. With
9
www.archactuarial.com www.guo.coursehost.com 10
each day passing, they have less and less time to keep up with the SOAs syllabus changes.
Finally, in fall, 2006, Nathan and Robin decided to withdraw from the Arch manual business
and passed on the copyright and ownership of Arch M manual to Yufeng Guo, who is the
author of Deeper Understanding manuals for Exam P, FM, and M.
Nathan and Robins contribution to actuarial education was not just the Arch manual but
more importantly the Archs eective teaching style. Before Arch was published, many
thought that learning dicult things such as Course 3 ought to be slow and painful. Archs
straight talk and down-to-earth examples showed the actuarial community that learning
dicult actuarial theories can indeed by fast-paced and enjoyable.
Praises of the Arch Manual originally written by Nathan and
Robin
-I am a huge fan of ARCH. It is by far the best study manual and I am recom-
mending it to all of my friends. I actually bought [several other manuals]. Now
I think I wasted a whole set of money on the others since they always end up
confusing me and I always have to come back to ARCH for clarication.
-I start a seminar on Friday, and I never would have been able to nish and
understand the material without your study guide.
-I want to personally thank both of you for the fantastic and brilliant work that
you did on ARCH. Seeing as its not my rst time tackling this exam, Ive had
the chance to use [several other manuals]; however this is by far superior to all
of those products. I have and will continue to recommend it to others in my
company.
-I would rst like to say that I am very happy with your manual so far. I feel
that I am progressing through the syllabus much faster than I would have without
it, and the depth of understanding that I am on my own giving up due to my not
using the texts themselves is more than compensated for by the excellent coverage
of the important topics in your manual.
About Yufeng Guo
Yufeng Guo was born in central China. After receiving his Bachelors degree in physics at
Zhengzhou University, he attended Beijing Law School and received his Masters of law. He
was an attorney and law school lecturer in China before immigrating to the United States.
He received his Masters of accounting at Indiana University. He has pursued a life actuarial
career and passed exams 1, 2, 3, 4, 5, 6, and 7 in rapid succession after discovering a successful
study strategy. Mr. Guos exam records are as follows:
Arch MLC, Fall 2007 c _Yufeng Guo 10
www.archactuarial.com www.guo.coursehost.com 11
Fall 2002 Passed Course 1
Spring 2003 Passed Course 2,3
Fall 2003 Passed Course 4
Spring 2004 Passed Course 6
Fall 2004 Passed Course 5
Spring 2005 Passed Course 7
Study guides by Mr. Guo:
Deeper Understanding, Faster Calc: P
Deeper Understanding, Faster Calc: FM
Deeper Understanding, Faster Calc: MLC
Deeper Understanding, Faster Calc: MFE
Arch MLC
If you have questions, you can email Mr. Guo at yufeng guo@msn.com.
FAQ
I notice you have two study guides for MLC: Arch MLC and Deeper Understanding MLC.
Whats the dierence? Which one should I buy?
Dierence: Arch MLC focuses on thoroughly explaining the core concepts. Deeper Under-
standing MLC focuses on teaching conceptual insights and calculation shortcuts.
Which one to buy: If money is not an issue, consider buying both. If you want to buy only
one guide, choose the one that better ts your need. For example, if you already have a
study guide and want to learn calculation shortcuts, buy Deeper Understanding: MLC. If
your goal, on the other hand, is to master basic concepts, buy Arch MLC.
How to use this manual
The ARCH manual is designed and written in such a way as to help you learn the material as
eciently as possible. The material for the course is broken down into dierent chapters from
the textbooks. The chapters are presented using down-to-earth explanations. In addition, I
point out the critical concepts and formulas most likely to be tested.
Each chapter of this manual contains plenty of examples with solutions. You are likely to
benet a great deal if every time you get to an example, you cover up the solution and
attempt to work it. You will get many of them wrong, especially the rst time you see them.
But the problem-solving experience will be extremely valuable!
On the exam, you will not be asked to explain anything. You will be asked to calculate
numerical answers. Therefore, much of our explanation of the material is done by way of
Arch MLC, Fall 2007 c _Yufeng Guo 11
www.archactuarial.com www.guo.coursehost.com 12
numerical examples and practice questions. Examples range from very simple ones (to make
sure you know the basic concepts), to thought provoking ones (to help you think about what
youve learned and really understand it), to exam questions from prior exams (to get you
ready for exam day).
I also suggest problems from the texts for you to work. Many of the problems in the
text are not transferable to the exam. Some, however, provide useful insight and prac-
tice. Solutions to these suggested problems are available on the Download Samples page at
www.archactuarial.com.
A formula summary for each chapter is included. These summaries are intended to serve as
a reference as you familiarize yourself with the syllabus material.
Finally, theres a full length practice exam of new questions. This practice exam is designed
to be used in conjunction with the prior Course 3 and Exam M problems in the SOA website
at www.soa.org. Make sure you work all of these exams!
All materials contained herein are copyrighted by Yufeng Guo. This PDF study manual is for
individual use for the sole purpose of taking Exam MLC. Reselling this manual is prohibited.
Redistribution of this manual in any form is prohibited.
You can purchase this manual at http://www.actuarialbookstore.com or http://www.
actexmadriver.com
Please check www.archactuarial.com for errata and answers to suggested text exercises.
Arch MLC, Fall 2007 c _Yufeng Guo 12
Chapter 1
ACTUARIAL MATHEMATICS:
CHAPTER 3 SURVIVAL
DISTRIBUTIONS AND LIFE
TABLES
Option A reference: Actuarial Mathematics Chapter 3
Option B reference: Models for Quantifying Risk Chapter 5,6
This text forms the heart and soul of the exam syllabus. The basic principles of life insurance
(and annuities) are explained throughout the book. You need to have a solid understanding
of this material in order to pass the exam. However, you do not need to understand the
majority of the underlying theory in this text. The key points that a student must learn
from this text are:
KEYPOINTS:
1. Notation much of this notation is new. While it can be confusing at rst, there is some
logic to it. It will help you to remember and understand the many symbols
if you regularly translate the notation into words as you read.
2. Basic ideas for example, chapter four introduces a variety of types of insurance. You
will want to make sure you have an understanding of these dierent products and their
benet designs. Another key point is that there are many parallels. Again in Chapter 4,
the rst part of the chapter considers products which pay a benet immediately upon
death. The second part of the chapter considers the same products except that the
benet is paid at the end of the year in which death occurs. It is helpful to realize that
you are really learning only one set of products, with a couple of benet options, rather
than two sets of products. These parallels run throughout the text (e.g., continuous vs
curtate functions).
13
www.archactuarial.com www.guo.coursehost.com 14
3. Learn key formulas there is no substitute for being able to recall the formula for,
say, a net level premium reserve for term insurance. If you can do this for most of the
formulas, you will be ready to answer questions quickly. This manual has tools to help
you learn these formulas, so dont feel overwhelmed!
To the text!!!
Arch MLC, Fall 2007 c _Yufeng Guo 14
www.archactuarial.com www.guo.coursehost.com 15
Chapter 3 is all about notation, denitions, and a few basic ideas that are essential to
life contingencies. If you can make yourself comfortable with the symbols and methods of
Chapter 3, the rest of Actuarial Mathematics will be easier to absorb.
3.2.1 The Survival Function
Option A reference: Actuarial Mathematics Chapter 3.2.1
Option B reference: Models for Quantifying Risk Chapter 5.1
Consider a newborn (i.e. a person whose attained age = 0).
Denitions
X = newborns age at death
You can also think of X as the future lifetime of a newborn.
Dene F(x) = Pr (X x), where x 0. Read as the probability that death will occur
prior to (or at) age x. In statistics, F(x) is the cumulative distribution function for the
future lifetime of a newborn. If y > x, it is always true that F(y) > F(x).
This makes sense. For a newborn, F(98), the probability of dying before age 98, is greater
than F(94), the probability of dying before age 94.
Dene s(x) = 1 F(X) = 1 Pr(X x). The function s(x) is a survival function. Read
it as the probability that death does not occur by age x or the probability of attaining
(surviving to) age x.
Pr(x < X z) = probability that a newborn dies between ages x and z
= F(z) F(x)
= [1 s(z)] [1 s(x)]
= s(x) s(z)
O
x z
F(x) s(x)
the pdf
y=f(x)
The gure shows the probability distribution function f(x) for death at age x. For any value
of x, F(x) is equal to the area under the curve y = f(x) and to the left of x. Similarly s(x)
is equal to the area under the curve and to the right of x.
By the way, you may have noticed that in our discussion, we dropped the subscript X in
F
X
(x) ... you can ignore it. I dont know if the authors realize it but they are being a little
Arch MLC, Fall 2007 c _Yufeng Guo 15
www.archactuarial.com www.guo.coursehost.com 16
inconsistent in their treatment of F and s! If two dierent random variables, say X and Y ,
referred to the future lifetimes of two dierent newborns, then you would need to keep the
F and s straight for each kid. Thats all the subscript is indicating.
3.2.2 Time-until-Death for a Person Age x
Option A reference: Actuarial Mathematics Chapter 3.2.2
Option B reference: Models for Quantifying Risk Chapter 5.3
Newborns are great, but if our pension and insurance companies are going to make money
we need to be able to deal with people who are older than 0. So ...
Consider a person with attained age = x.
The simple F(x) and s(x) functions no longer work, since we are now dealing with a person
who has already survived to age x. We are facing a conditional probability situation.
Pr(x < X z[X > x)
= probability that person living at age x will die between ages x and z
= the probability that an x-year-old will die before turning z
=
[F(z) F(x)]
[1 F(x)]
=
[s(x) s(z)]
[s(x)]
Why is this a conditional probability? Because it is the probability that a newborn will die
before age z given that the newborn survives to age x.
EXAMPLE:
1. Write two expressions (one with F only and one with s only) for the prob-
ability that a newborn dies between 17 and 40, assuming the newborn dies
between 10 and 40.
2. Interpret the following expression in English (or the language of your choice!).
S(20) S(35)
1 S(80)
SOLUTION:
1.
F(40) F(17)
F(40) F(10)
or
s(17) s(40)
s(10) s(40)
Arch MLC, Fall 2007 c _Yufeng Guo 16
www.archactuarial.com www.guo.coursehost.com 17
2. The probability of death between ages 20 and 35, given that the newborn
will not attain age 80.
Now, let the symbol (x) represent a person age x and let T(x) be the future lifetime of a
person age x. (So T(25) is the future lifetime of (25), a twenty-ve-year-old.)
Two basic probability functions exist regarding T(x):
t
q
x
= probability that person age x will die within t years
= Pr[T(x) t] where t 0
t
p
x
= probability that person age x will survive at least t years
= Pr[T(x) > t] where t 0
x
x+t
Age
t
p
x t x
q
In the gure,
t
q
x
is the probability that (x)s death will occur in the age-interval (x, x+t), and
t
p
x
is the probability that (x)s death will occur in the age interval (x +t, ). ( represents
the oldest possible age to which a person may survive.)
Useful notes:

t
p
0
is just s(t).
If t = 1, the convention is to drop the symbol 1, leaving us with either p
x
or q
x
.
Remember, these are the two basic functions. The formulas that follow are simply take-os
on
t
p
x
or
t
q
x
which you will learn with practice.
The symbol
t|u
q
x
represents the probability that (x) (that is, a person age x) survives at least t more years,
but dies before reaching age x+t +u. This is equal to each of the following expressions, each
of which you want to be able to put into words:
Pr[t < T(x) t +u]
t+u
q
x

t
q
x
t
p
x

t+u
p
x
(As with q
x
and p
x
, if u = 1, we drop it, leaving
t|
q
x
, the probability that (x) will survive t
years but not t + 1 years.)
Arch MLC, Fall 2007 c _Yufeng Guo 17
www.archactuarial.com www.guo.coursehost.com 18
Useful formulas:
t
p
x
=
x+t
p
0
x
p
0
=
s(x +t)
s(x)
t
q
x
= 1
s(x +t)
s(x)
t|u
q
x
=
s(x +t) s(x +t +u)
s(x)
=
s(x +t)
s(x)

s(x +t) s(x +t +u)
s(x +t)
=
t
p
x

u
q
x+t
This last equation makes sense. It says The probability of (x) dying between t and t + u
years from now (
t|u
q
x
) is equal to the probability that (x) will rst survive t years (
t
p
x
) and
then die within u years (
u
q
x+t
).
If you dont remember anything else from the above, remember the following!
t
p
x
=
s(x +t)
s(x)
CONCEPT REVIEW:
1. Write the symbol for the probability that (52) lives to at least age 77.
2. Write the symbol for the probability that a person age 74 dies before age 91.
3. Write the symbol for probability that (33) dies before age 34.
4. Write the symbol for probability that a person age 43 lives to age 50, but
doesnt survive to age 67.
5. Write
5|6
q
x
in terms of F and then in terms of p.
SOLUTIONS:
1.
25
p
52
2.
17
q
74
3. q
33
4.
7|17
q
43
5.
5|6
q
x
=
s(x + 5) s(x + 11)
s(x)
=
F(x + 11) F(x + 5)
1 F(x)
=
5
p
x
(1
6
p
x+5
) or =
5
p
x

11
p
x
.
To help memorize symbols, practice translating symbols into words and express words in
symbols. You can also make ash cards and quiz yourself.
Arch MLC, Fall 2007 c _Yufeng Guo 18
www.archactuarial.com www.guo.coursehost.com 19
3.2.3 Curtate-Future-Lifetime
Option A reference: Actuarial Mathematics Chapter 3.2.3
Option B reference: Models for Quantifying Risk Chapter 5.3.6
Suppose a person born on Jan 1, 1900 died on Sept 30, 1990. How old was he at death?
The true age was about 90.75 years old. The curtate age was 90. To nd the curtate age,
rst nd the true age. Next, throw away all the decimals and keep the integer. If theres no
decimal, then the curtate age is equal to the continuous (true) age. For example, if T(x) = 90,
then K(x) = 90. (This book and others use Curtate and Discrete interchangeably.)
Previously, we dened T(x) to be the future lifetime of (x). This is a continuous function.
Now we dene
K(x) = curtate future lifetime of (x)
= greatest integer in T(x)
= number of future years completed by (x) prior to death
= number of future birthdays (x) will have the opportunity to celebrate
A couple of formulas apply:
Pr(K(x) = k) = Pr(k T(x) < k + 1)
= Pr(k < T(x) k + 1)
=
k
p
x

k+1
p
x
=
k
p
x
q
x+k
=
k|
q
x
(Remember, the 1 in front of q has been dropped.)
EXAMPLE:
If s(x) =
100x
100
for every x, what is the probability that K = 19 for (18)?
SOLUTION:
Pr(K(18) = 19) =
19|
q
18
=
s(37) s(38)
s(18)
=
63 62
82
=
1
82
.
Arch MLC, Fall 2007 c _Yufeng Guo 19
www.archactuarial.com www.guo.coursehost.com 20
3.2.4 Force of Mortality
Option A reference: Actuarial Mathematics Chapter 3.2.4
Option B reference: Models for Quantifying Risk Chapter 5.1.4
The force of mortality can be thought of as the probability of death at a particular instant
given survival up to that time. This is an instantaneous measure, rather than an interval
measure. There is good bit of theory in this section, but the most important items are the
following formulas and the table of relationships.
(x) =
f(x)
1 F(x)
=
s

(x)
s(x)
(3.2.13)
It is very important to know the relationships and requirements given in Table 3.2.1. These
will probably be tested on the exam. Below is a summary of the useful information in this
table. Each row shows 4 ways to express the function in the left column.
F(x) s(x) f(x) (x)
F(x) F(x) 1 s(x)
_
x
0
f(u) du 1 e

_
x
0
(t) dt
s(x) 1 F(x) s(x)
_

x
f(u) du e

_
x
0
(t) dt
f(x) F

(x) s

(x) f(x) (x) e

_
x
0
(t) dt
(x)
F

(x)
1F(x)
s

(x)
s(x)
f(x)
s(x)
(x)
EXAMPLE: Constant Force of Mortality
If the force of mortality is a constant for every age x, show that
1. s(x) = e
x
2.
t
p
x
= e
t
SOLUTION:
1.
s(x) = e

_
x
0
dt
= e
x
.
2.
t
p
x
=
s(x +t)
s(x)
= e
t
.
Arch MLC, Fall 2007 c _Yufeng Guo 20
www.archactuarial.com www.guo.coursehost.com 21
3.3-3.5 Life Tables
Option A reference: Actuarial Mathematics Chapter 3.3-3.5
Option B reference: Models for Quantifying Risk Chapter 6
Life Table is widely used actuarial practice. Even today, Life Tables are often loaded into
systems for calculating reserves, premium rates, and the surrender cash value of an insurance
policy. Learning Life Tables will not only help you pass Exam MLC, it also helps you when
you become an actuary.
Denitions:
l
0
= number of people in cohort at age 0, also called the radix
l
i
= number of people in cohort at age i (those remaining from the original l
0
)
= limiting age at which probability of survival = 0 (s(x) = 0 for all x )
n
d
x
= number alive at age x who die by age x +n
Relationships:
l
x
= l
0
s(x)
q
x
=
l
x
l
x+1
l
x
n
q
x
=
l
x
l
x+n
l
x
n
p
x
=
l
x+n
l
x
n
d
x
= l
x
l
x+n
Illustrative Life Table: Basic Functions
Age l
x
d
x
1,000 q
x
0 100,000.0 2,042.2 20.4
1 97,957.8 131.6 1.4
2 97,826.3 119.7 1.2
3 97,706.6 109.8 1.1
.
.
.
.
.
.
.
.
.
.
.
.
40 93,131.6 259.0 2.8
41 92,872.6 276.9 3.0
42 92,595.7 296.5 3.2
43 92,299.2 317.8 3.4
Arch MLC, Fall 2007 c _Yufeng Guo 21
www.archactuarial.com www.guo.coursehost.com 22
EXAMPLE: Life Table Mortality
Above is an excerpt from the Illustrative Life Table in the book. The following
questions are all based on this excerpt.
1. Find s(42).
2. Find
40
d
2
.
3. Find
38
q
3
.
4. Find
2|
q
40
.
SOLUTION:
1. s(42) =
92,595.7
100,000
= 0.925957.
2.
40
d
2
= l
2
l
42
= 5230.6
3.
38
q
3
= 1
38
p
3
= 1
l
41
l
3
= 1
92,872.6
97,706.6
= 0.04947.
4.
2|
q
40
=
2
p
40
q
42
=
92,595.7
93,131.6
(0.0032) = 0.003182.
Concepts which follow from the Life Table:
Based on Equation (3.2.13) on an earlier page, we can determine that the probability density
function f(t) for T(x) is given by f(t) =
t
p
x
(x +t). This says that the probability that (x)
will die at age x + t, symbolized by f(t), is equal to the probability that (x) will survive t
years and then be hit at that instant by the force of mortality.
Among other things, this tells us that
_

0
t
p
x
(x +t)dt =
_

0
f(t)dt = 1.
The complete-expectation-of-life is the expected value of T(x) (or E[T(x)] for fans of
Statistics) and is denoted

e
x
. If you remember how to nd the expected value of a continuous
random variable, you can gure out that

e
x
= E[T(x)] =
_

0
t
p
x
dt
Var[T(x)] = 2
_

0
t
t
p
x
dt

e
2
x
(3.5.4)
The book shows how to gure both of these formulas out with integration by parts in Sec-
tion 3.5.1. I suggest that you memorize these two expressions.
The median future lifetime of (x) is denoted m(x) and simply represents the number m such
that
m
p
x
=
m
q
x
. In other words, it is the number of years that (x) is equally likely to survive
or not survive. It can be found by solving any of the following:
Pr[T(x) > m(x)] =
1
2
Arch MLC, Fall 2007 c _Yufeng Guo 22
www.archactuarial.com www.guo.coursehost.com 23
or
s[x +m(x)]
s(x)
=
1
2
or
m
p
x
=
1
2
.
The curtate-expectation-of-life is E[K(x)] and is denoted e
x
(no circle). To remember
the dierence between e
x
and

e
x
, remember life is a continuous circle. So a circle means
continuous.

e
x
= E[T(x)] and e
x
= E[K(x)].
Here are the formulas, note the Continuous/Curtate parallel:
e
x
= E[K(x)] =

1
k
p
x
Var[K(x)] =

1
(2k 1)
k
p
x
e
2
x
EXAMPLE: Constant Force of Mortality
Find

e
0
and

e
50
if the force of mortality is a constant .
SOLUTION:

e
0
=
_

0
t
p
0
dt =
_

0
e
t
dt =
_
1

e
t
_

0
=
1

e
50
=
_

0
t
p
50
dt =
_

0
e
t
dt =
_
1

e
t
_

0
=
1

If the force of mortality is constant, your future expected lifetime is the same
whether you are 0 (a newborn) or 50.
EXAMPLE: DeMoivres Law for Mortality
(Well learn DeMoivre later in this chapter.)
If
s(x) =
_
50x
50
0 < x < 50
0 Otherwise
for all x between 0 and 50, nd e
0
and e
45
.
SOLUTION:
e
0
=
50

1
t
p
0
=
50

1
50 t
50
= 50
1
50
50

1
t
Arch MLC, Fall 2007 c _Yufeng Guo 23
www.archactuarial.com www.guo.coursehost.com 24
= 50
1
50
(50)(51)
2
= 24.5
e
45
=
5

1
t
p
45
=
5

1
s(45 +t)
s(45)
=
5

1
5 t
5
=
4 + 3 + 2 + 1 + 0
5
= 2.
More Life Functions:
The expression L
x
denotes the total expected number of years, full or fractional, lived between
ages x and x +1 by survivors of the initial group of l
0
lives. Those who survive to x +1 will
live one year between x and x + 1, contributing one full year to L
x
. Those who die during
the year will contribute a fraction of a year to L
x
.
L
x
=
_
1
0
l
x+t
dt
The expression m
x
is the central death rate over the interval x to x +1. Make sure not to
confuse m
x
with m(x), the median future lifetime!
m
x
=
(l
x
l
x+1
)
L
x
L
x
and m
x
can be extended to time periods longer than a year:
n
L
x
=
_
n
0
l
x+t
dt
n
m
x
=
l
x
l
x+n
n
L
x
The remaining of Section 3.5.1 has obscure symbols T
x
and (x). They rarely show up in
the exam. Dont spend too much time on them.
Let T
x
be the total number of years lived beyond age x by the survivorship group with l
0
initial members (i.e. the l
x
people still alive at age x). Be careful with notation. This is not
T(x), the future lifetime of (x).
T
x
=
_

0
l
x+t
dt (3.5.16)
Note from the denitions that you can think of T
x
as

L
x
.
The nal symbol is (x). Its the expected death time given x dies next year.
Arch MLC, Fall 2007 c _Yufeng Guo 24
www.archactuarial.com www.guo.coursehost.com 25
(x) = E [T[T < 1] =
_
1
0
tf(t)dt
_
1
0
f(t)dt
=
_
1
0
ttpx (x+t)dt
_
1
0
tpx(x+t)dt
=
_
1
0
t
l
x+t
lx
(x+t)dt
_
1
0
l
x+t
lx
(x+t)dt
=
_
1
0
tl
x+t
(x+t)dt
_
1
0
l
x+t
(x+t)dt
If UDD, then f(t) = q
x
= c is a constant. Then (x) =
_
1
0
tcdt
_
1
0
cdt
=
_
1
0
tdt
_
1
0
dt
=
_
1
0
tdt =
1
2
.
EXAMPLE: Constant Force of Mortality
If l
0
= 1000 and the force of mortality is a constant = 0.1, nd
(A) L
5
(B) m
5
(C) T
5
Arch MLC, Fall 2007 c _Yufeng Guo 25
www.archactuarial.com www.guo.coursehost.com 26
SOLUTION:
(A)
L
5
=
_
1
0
l
5+t
dt =
_
1
0
t
p
5
l
5
.
Since
l
5
= l
0
e
5
= 1000e
0.5
= 606.5,
we have
L
5
= 606.5
_
1
0
e
0.1t
dt = 606.5
_
10e
0.1t
_
1
0
= 606.5
_
10(1 e
0.1
)
_
= 577.16.
(B)
m
5
=
l
5
l
6
L
5
=
606.5 548.8
577.16
= 0.10
This approximates the rate at which people were dying between the 5th and
6th years.
(C)
T
5
=
_

0
1000e
0.1(5+t)
dt = 606.5
_

0
e
0.1t
dt = 6065
So if we add up all of the time lived by each of the people alive at t = 5, we
expect to get a total of 6065 years, or 10 years per person.
Relationship:
T
x
l
x
=

e
x
This relationship makes sense. It says that the average number of years lived,

e
x
, by the
members of l
x
is equal to the total number of years lived by this group divided by l
x
.
We can determine the average number of years lived between x and x+n by the l
x
survivors
at age x as:
n
L
x
l
x
=
_
n
0
t
p
x
dt
n
L
x
l
x
= n-year temporary complete life expectancy of (x)
=

e
x:n
(p.71)
Arch MLC, Fall 2007 c _Yufeng Guo 26
www.archactuarial.com www.guo.coursehost.com 27
3.5.2 Recursion Formulas
Option A reference: Actuarial Mathematics Chapter 3.5.2
Option B reference: Models for Quantifying Risk Chapter 6
These are basically ways to avoid working integrals. They are based on the Trapezoid Rule
for integration maybe you remember the trapezoid rule from calculus.
Backward:
u(x) = c(x) +d(x) u(x + 1)
Forward:
u(x + 1) =
u(x) c(x)
d(x)
Note that the Forward Method is simply an algebraic recombination of the Backward Method.
Note also that this Forward formula is dierent from the book work out the formulas
yourself to convince yourself of their equivalence. Then, learn whichever form you nd more
straightforward.
The text shows how to use these formulas to compute e
x
and

e
x
starting with e

and

e

and
working backward. For e
x
, using the recursion once will produce e
1
, the second iteration
will produce e
2
, etc. until you get all the way back to e
0
, when you will have produced a
list of e
x
for every x between 0 and .
The formulas are: for e
x
,
u(x) = e
x
c(x) = p
x
d(x) = p
x
Starting Value = e

= u() = 0
So to start, set x + 1 = and the recursion will produce u(x) = u( 1).
For

e
x
,
u(x) =

e
x
c(x) =
_
1
0
s
p
x
ds
d(x) = p
x
Starting Value =

= u() = 0
Arch MLC, Fall 2007 c _Yufeng Guo 27
www.archactuarial.com www.guo.coursehost.com 28
3.6 Assumptions for Fractional Ages
Option A reference: Actuarial Mathematics Chapter 3.6
Option B reference: Models for Quantifying Risk Chapter 6.5
(OK, you can start paying attention again ....)
The random variable T is a continuous measure of remaining lifetime. The life table has been
developed as an approximation of T, using a curtate variable K. As weve discussed, K is
only dened at integers. So, we need some way to measure between two integer ages. Three
popular methods were developed.
For all of the methods that follow, let x be an integer and let 0 t 1. Suppose that we
know the value of s(x) for the two integers x and x + 1 and we want to approximate s at
values between x and x+1. In other words, we want to approximate s(x+t) where 0 t 1.
Method 1: Linear Interpolation:
s(x +t) = (1 t)s(x) +t s(x + 1)
This method is also known as Uniform Distribution of Deaths, or UDD. Under UDD,
s(x + t) and
t
p
x
are both straight lines between t = 0 to t = 1. This method assumes that
the deaths occurring between ages x and x + 1 are evenly spread out between the two ages.
As you might imagine, this is usually not quite correct, but is a pretty good approximation.
(Please note: the linearity of s(x +t) and
t
p
x
is only assumed to hold up to t = 1!)
One key formula for UDD you might want to memorize is:
f(t) = q
x
To see why, please note that the number of deaths from time zero to time t is a fraction of
the total deaths in a year
s(x) s(x +t) = t[s(x) s(x + 1)]
Here for convenience we interpret s(x + t) as the number of people alive at age x + t. For
example, if s(x + 0.5) = 0.9, we say that for each unit of people at age x, we have 0.9 unit
of people at age x + 0.5, with one unit being one billion, one million, or any other positive
constant.
t
q
x
=
s(x) s(x +t)
s(x)
= t
s(x) s(x + 1)
s(x)
= tq
x
f(t) =
d
dt
t
q
x
= q
x
Arch MLC, Fall 2007 c _Yufeng Guo 28
www.archactuarial.com www.guo.coursehost.com 29
You can also come up with f(t) = q
x
by intuitive thinking. Under UDD, death occurs at
a constant speed. If 12 people died in one year, then one person died each month. So f(t)
must be a constant. Then:
q
x
=
_
1
0
f(t)dt = f(t)
_
1
0
dt = f(t)
Method 2: Exponential Interpolation: Forget about the complex formula:
log s(x +t) = (1 t) log s(x) +t log s(x + 1)
All you need to know is that under the constant force of mortality, (x + t) = for
0 t 1.
Method 3: Harmonic Interpolation: This method is more commonly called the Balducci
assumption or the Hyperbolic assumption.
Forget about the complex formula
1
s(x +t)
=
1 t
s(x)
+
t
s(x + 1)
All you need to know about Balducci assumption is this:
1t
q
x+t
= (1 t)q
x
The above formula says that if you are x + t years old (where 0 t 1), then your chance
of dying in the remainder of the year is a fraction of your chance of dying in the whole year.
You can derive all the other formulas using
1t
q
x+t
= (1 t)q
x
. Later Ill show you how to
derive other formulas in Balducci assumptions.
Uniform Constant
Function Distribution Force Hyperbolic
t
q
x
tq
x
1 p
t
x
tqx
1(1t)qx
(x +t)
qx
1tqx
log p
x
qx
1(1t)qx
1t
q
x+t
(1t)qx
1tqx
1 p
1t
x
(1 t)q
x
y
q
x+t
yqx
1tqx
1 p
y
x
yqx
1(1yt)qx
t
p
x
1 tq
x
p
t
x
px
1(1t)qx
t
p
x
(x +t) q
x
p
t
x
log p
x
qxpx
[1(1t)qx]
2
Arch MLC, Fall 2007 c _Yufeng Guo 29
www.archactuarial.com www.guo.coursehost.com 30
Table 3.6.1
Table 3.6.1 summarizes UDD, constant force of mortality, and the Balducci assumption.
Dont try to memorize the whole table. Learn basic formulas and derive the rest on the spot.
Under UDD, for 0 t 1:
f(t) = q
x
= constant
t
q
x
=
_
t
0
f(t)dt =
_
t
0
q
x
dt = q
x
_
t
0
dt = tq
x
t
p
x
= 1
t
q
x
= 1 tq
x
(x +t) =
x
(t) =
f(t)
t
p
x
=
q
x
1 tq
x
t
p
x
(x +t) = f(t) = q
x
y
p
x+t
=
s(x +t +y)
s(x +t)
=
s(x)
t+y
p
x
s(x)
t
p
x
=
t+y
p
x
t
p
x
=
1 (t +y)q
x
1 tq
x
y
q
x+t
= 1
y
p
x+t
=
yq
x
1 tq
x
1t
q
x+t
=
(1 t)q
x
1 tq
x
Under constant force of mortality, for 0 t 1:

x
(t) =
t
p
x
= e

_
t
0
dt
= e
t
p
x
= e

= ln p
x
t
p
x
= e
t
= (e

)
t
= (p
x
)
t
y
p
x+t
=
t+y
p
x
t
p
x
=
(p
x
)
t
(p
x
)
(t+y)
= (p
x
)
y
Finally, lets derive log s(x +t) = (1 t) log s(x) +t log s(x + 1)
s(x +t) = s(x)e
t
log s(x +t) = log s(x) t
s(x + 1) = s(x)e

log s(x + 1) = s(x)


(1 t) log s(x) +t log s(x + 1) = (1 t) log s(x) +t log s(x) t = log s(x) t
log s(x +t) = (1 t) log s(x) +t log s(x + 1)
Arch MLC, Fall 2007 c _Yufeng Guo 30
www.archactuarial.com www.guo.coursehost.com 31
Under Balducci assumption: The starting point of Balducci assumption is
1t
q
x+t
= (1 t)q
x
You can derive all the other formulas from this starting point. For example, lets derive
the formula for
t
p
x
.
T(x) 0 t 1
Age x x +t x + 1
Number of people alive s(x) = 1 s(x +t) =
px
1(1t)qx
s(x + 1) = p
x
This is how to derive s(x +t) =
px
1(1t)qx
.
First, we set the starting population at s(x) = 1 for convenience. You can set it to any
positive constant and get the same answer. After setting s(x) = 1, well have s(x + 1) = p
x
.
This is because p
x
=
s(x+1)
s(x)
.
Next, lets nd s(x +t) using the formula
1t
q
x+t
= (1 t)q
x
.
1t
q
x+t
= 1
s(x + 1)
s(x +t)
= 1
p
x
s(x +t)
= (1 t)q
x
s(x +t) =
p
x
1 (1 t)q
x
However,
t
p
x
=
s(x+t)
s(x)
= s(x +t). This gives us:
t
p
x
=
px
1(1t)qx
.
Next, lets derive the formula f(t) =
t
p
x
(x +t) =
t
p
x

x
(t)
f(t) =
d
dt
t
p
x
=
d
dt
p
x
1 (1 t)q
x
=
q
x
p
x
[1 (1 t)q
x
]
2
Derive (x +t):
(x +t) =
f(t)
t
p
x
=
q
x
1 (1 t)q
x
Derive
y
q
x+t
:
y
q
x+t
= 1
s(x +t +y)
s(x +t)
Use the formula: s(x +t) =
px
1(1t)qx
Replace t with t +y, assuming 0 t +y 1:
s(x +t +y) =
p
x
1 (1 t y)q
x
Arch MLC, Fall 2007 c _Yufeng Guo 31
www.archactuarial.com www.guo.coursehost.com 32
y
q
x+t
= 1
s(x +t +y)
s(x +t)
= 1
1 (1 t)q
x
1 (1 t y)q
x
=
p
x
1 (1 t y)q
x
Finally, lets derive
1
s(x+t)
=
1t
s(x)
+
t
s(x+1)
s(x +t) =
p
x
1 (1 t)q
x
1
s(x +t)
=
1 (1 t)q
x
p
x
=
1 (1 t)(1 p
x
)
p
x
=
(1 t)p
x
+t
p
x
= (1 t) +
t
p
x
Since s(x) = 1 and s(x + 1) = p
x
, we have:
1 t
s(x)
+
t
s(x + 1)
= 1 t +
t
p
x

1
s(x +t)
=
1 t
s(x)
+
t
s(x + 1)
Now you see that you really dont need to memorize Table 3.6.1. Just memorize the following:
Under UDD, f(t) = q
x
is a constant.
Under constant force of mortality, (x +t) = .
Under Balducci,
1t
q
x+t
= (1 t)q
x
.
A couple of time-saving formulas that are valid under UDD only!!

e
x
= e
x
+
1
2
Var(T) = Var(K) +
1
12
EXAMPLE:
You are given that q
x
= 0.1. Find (A)
0.5
q
x
, (B)
0.5
q
x+0.5
under each of
Uniform Density of Deaths (UDD)
Exponentially distributed deaths (constant force)
Harmonic Interpolation (Balducci, hyperbolic)
SOLUTION:
(A) UDD:
0.5
q
x
= (0.5)(0.1) = 0.05
Arch MLC, Fall 2007 c _Yufeng Guo 32
www.archactuarial.com www.guo.coursehost.com 33
CF:
0.5
q
x
= 1 (0.9)
0.5
= 0.0513
Balducci:
0.5
q
x
=
(0.5)(0.1)
1 (0.5)(0.1)
= 0.0526.
(B) UDD:
0.5
q
x+0.5
=
(0.5)(0.1)
1 (0.5)(0.1)
= 0.0526
CF:
0.5
q
x+0.5
= 1 (0.9)
0.5
= 0.0513
Balducci:
0.5
q
x+0.5
= (0.5)(0.1) = 0.05.
3.7 Some Analytical Laws of Mortality
Option A reference: Actuarial Mathematics Chapter 3.7
Option B reference: Models for Quantifying Risk Chapter 5.2
Although computers have rendered Analytical Laws of Mortality less imperative to our pro-
fession, they are still important for understanding mortality and particularly for passing the
exam. The book describes four basic analytical laws/formulas:
De Moivres Law:
(x) = ( x)
1
and s(x) = 1
x

, where 0 x <
Gompertz Law:
(x) = Bc
x
and s(x) = exp[m(c
x
1)]
where B > 0, c > 1, m =
B
log c
, x 0
Makehams Law:
(x) = A+Bc
x
and s(x) = exp[Ax m(c
x
1)]
where B > 0, A B, c > 1, m =
B
log c
, x 0
Weibulls Law:
(x) = kx
n
and s(x) = exp(ux
n+1
)
where k > 0, n > 0, u =
k
(n + 1)
, x 0
Notes:
Arch MLC, Fall 2007 c _Yufeng Guo 33
www.archactuarial.com www.guo.coursehost.com 34
Gompertz is simply Makeham with A = 0.
If c = 1 in Gompertz or Makeham, the exponential/constant force of mortality results.
In Makehams law, A is the accident hazard while Bc
x
is the hazard of aging.
We will be seeing more of these laws later in the book. To be ready for the exam, you should
become intimately familiar with De Moivres Law. DeMoivres Law says that at age x, you
are equally likely to die in any year between x and .
Here are some key life-functions for DeMoivres Law in the form of an example. If you dont
read the proofs, still make an eort to understand the formulas and what they mean.
EXAMPLE:
Under DeMoivres law, show that
1.
t
p
x
=
x t
x
2.
t
q
x
=
t
x
3.

e
0
=

2
4.
e
0
=
1
2
5.

e
x
=
x
2
6.
e
x
=
x 1
2
Arch MLC, Fall 2007 c _Yufeng Guo 34
www.archactuarial.com www.guo.coursehost.com 35
SOLUTION:
1.
t
p
x
=
s(x +t)
s(x)
=
x t
x
2.
t
q
x
= 1
t
p
x
=
t
x
3.

e
0
=
_

0
t

dt =
_

( t)
2
2
_
=

2
4.
e
0
=

1
k
p
0
=

1
k

=
1

1
k =
1
2
5.

e
x
=
_
x
0
x t
x
dt =
x
2
6. Since mortality is uniform over all years under DeMoivres law, it is uniform
over each individual year, so UDD applies. Therefore
e
x
=

e
x

1
2
=
x 1
2
We could have done (4) this way also.
Modied DeMoivres Law
Often on the exam, a modied version of DeMoivres Law arises. This occurs, for example,
when
(x) =
c
x
where c is a positive constant. This gives rise to a set of formulas for each of the quantities
found for standard DeMoivres Law (c = 1) in the example above. All of the formulas for
Modied DeMoivres Law are listed in the formula summary at the end of this chapter.
3.8 Select and Ultimate Tables
Option A reference: Actuarial Mathematics Chapter 3.8
Option B reference: Models for Quantifying Risk Chapter 6.6
Arch MLC, Fall 2007 c _Yufeng Guo 35
www.archactuarial.com www.guo.coursehost.com 36
Suppose you are trying to issue life insurance policies and two 45 year-old women apply for
policies. You want to make sure you charge appropriate premiums for each one to cover the
cost of insuring them over time. One of the women is simply picked from the population
at large. The second women was picked from a group of women who recently passed a
comprehensive physical exam with ying colors signicantly healthier than the general
population.
Is it equitable to charge the same premium to the two women? No because you have
additional information about the second woman that would cause you to expect her to have
better mortality experience than the general population. Thus, to her premiums, you might
apply a select mortality table that reects better the mortality experience of very healthy
45 year olds. However, after 15 years, research might show that being very healthy at 45
does not indicate much of anything about health at age 60. So, you might want to go back
to using standard mortality rates at age 60 regardless of status at age 45. After all, 15 years
is plenty of time to take up smoking, eat lots of fried foods, etc.
This simple scenario illustrates the idea behind select and ultimate tables. For some period
of time, you expect mortality to be dierent than that for the general population the select
period. However, at some point, youre just not sure of this special status anymore, so those
folks fall back into the pack at some point the ultimate table.
Consider table 3.8.1. The symbol [x] signies an x-yr old with select status. Note that
for the rst two years (columns 1,2), select mortality applies with q
[x]
and q
[x]+1
. However,
at duration 3 (column 3), its back to standard mortality, q
x+2
. This table assumes the
selection eect wears o in just 2 years.
Excerpt from the AF80 Select-and-Ultimate Table in Bowers, et al.
(1) (2) (3) (4) (5) (6) (7)
[x] 1,000 q
[x]
1,000 q
[x]+1
1,000 q
x+2
l
[x]
l
[x]+1
l
x+2
x + 2
30 0.222 0.330 0.422 9,906.74 9,904.54 9,901.27 32
31 0.234 0.352 0.459 9,902.89 9,900.58 9,897.09 33
32 0.250 0.377 0.500 9,898.75 9,896.28 9,892.55 34
33 0.269 0.407 0.545 9,894.29 9,891.63 9,887.60 35
34 0.291 0.441 0.596 9,889.45 9,886.57 9,882.21 36
Table 3.8.1
Here are a few useful formulas and relationships. In general,
q
[x]
< q
[x1]+1
< q
x
Why would this hold? The expression q
x
represents a pick from the general population.
The expression q
[x]
indicates special knowledge about the situation for example, recently
passing a physical exam (This formula assumes we are trying to select out healthy people,
of course). The expression q
[x1]+1
indicates special knowledge about the situation as before
for example, recently passing a physical exam, but this time the applicant has had a year
Arch MLC, Fall 2007 c _Yufeng Guo 36
www.archactuarial.com www.guo.coursehost.com 37
for health to deteriorate since she was examined at age x 1 (one year ago) rather than at
age x.
EXAMPLE: Select and Ultimate Life Table
Using the select and ultimate life table shown above nd the value of
1000
_
3
q
32

3
q
[32]
_
.
Arch MLC, Fall 2007 c _Yufeng Guo 37
www.archactuarial.com www.guo.coursehost.com 38
SOLUTION:
3
q
32
deals only with the ultimate table so I am only interested in the values of
l
x
+ 2 in Column (6).
3
q
32
= 1
3
p
32
= 1
l
35
l
32
= 1
9,888
9,901
= 0.001313
For
3
q
[32]
, we need l
[32]
and l
[32]+3
which is just l
35
since the select period is only
2 years. So
3
q
[32]
= 1
l
35
l
[32]
= 1
9,888
9,899
= 0.001111
So the answer is 0.202.
Two important points regarding this example:
The probability that [32] will die in the next 3 years is lower if [32] is taken from a
select group. People you are sure are healthy should be less likely to die than someone
drawn from the general population.
To follow the people alive from the 9898.75 selected at age 32, you rst follow the
numbers to the right until you hit the ultimate column and then proceed down the
ultimate column. This is useful! You can quickly evaluate that
5
p
[31]
=
9882
9903
by counting o 5 years 2 to the right and then 3 down.
Conclusion
Chap. 3 introduces a lot of new concepts and notation. Make sure you understand the
notation in Table 3.9.1 this is the foundation for the rest of the text.
Chapter 3 Suggested Problems: 1(do rst row last), 5, 6, 7, 9, 12, 18abc, 20, 28,
30, 36, 39 There are lots for this chapter, some chapters in this book will have very few.
(Solutions are available at archactuarial.com on the Download Samples page.)
Arch MLC, Fall 2007 c _Yufeng Guo 38
www.archactuarial.com www.guo.coursehost.com 39
CHAPTER 3 Formula Summary
s(x) = 1 F(X) = 1 Pr(X x)
Pr(x < X z) = F(z) F(x) = s(x) s(z)
Pr(x < X z[X > x) =
[F(z) F(x)]
[1 F(x)]
=
[s(x) s(z)]
[s(x)]
t
p
x
= e
_

_
t
0
(x+s)ds
_
t
p
x
=
s(x +t)
s(x)
t
q
x
= 1
t
p
x
t|u
q
x
=
t
p
x

u
q
x+t t|u
q
x
=
t+u
q
x

t
q
x t|u
q
x
=
t
p
x

t+u
p
x
Pr(K(x) = k) =
k
p
x

k+1
p
x
=
k
p
x
q
x+k
=
k|
q
x
Life Tables:
l
x
= l
0
s(x)
n
d
x
= l
x
l
x+n
q
x
=
l
x
l
x+1
l
x
p
x
=
l
x+1
l
x
n
q
x
=
l
x
l
x+n
l
x
n
p
x
=
l
x+n
l
x
Constant Force of Mortality
If the force of mortality is a constant for every age x,
s(x) = e
x
t
p
x
= e
t

e
x
=
1

Var[T] =
1

2
Arch MLC, Fall 2007 c _Yufeng Guo 39
www.archactuarial.com www.guo.coursehost.com 40
Expected Future Lifetime

e
x
= E[T(x)] =
_

0
t
p
x
dt Var[T(x)] = 2
_

0
t
t
p
x
dt

e
2
x
e
x
= E[K(x)] =

1
k
p
x
Var[K(x)] =

1
(2k 1)
k
p
x
e
2
x
Under UDD only,

e
x
= e
x
+
1
2
Var(T) = Var(K) +
1
12
Median future lifetime
Pr[T(x) > m(x)] =
1
2
s[x +m(x)]
s(x)
=
1
2
m
p
x
=
1
2
Make sure not to confuse m
x
with m(x), the median future lifetime!
L
x
=
_
1
0
l
x+t
dt
n
L
x
=
_
n
0
l
x+t
dt
m
x
=
(l
x
l
x+1
)
L
x
n
m
x
=
l
x
l
x+n
n
L
x
T
x
=
_

0
l
x+t
dt
T
x
l
x
=

e
x
n
L
x
l
x
=
_
n
0
t
p
x
dt =

e
x:n
With the assumption of uniform distribution of deaths over the interval (x, x + 1),
a(x) = 1/2.
Without this assumption,
a(x) =
_
1
0
t l
x+t
(x +t)dt
_
1
0
l
x+t
(x +t)dt
=
_
1
0
t
t
p
x
(x +t)dt
_
1
0
t
p
x
(x +t)dt
Arch MLC, Fall 2007 c _Yufeng Guo 40
www.archactuarial.com www.guo.coursehost.com 41
De Moivres Law: (x) = ( x)
1
and s(x) = 1
x

, where 0 x <
Gompertz Law: (x) = Bc
x
and s(x) = exp[m(c
x
1)]
where B > 0, c > 1, m =
B
log c
, x 0
Makehams Law: (x) = A+Bc
x
and s(x) = exp[Ax m(c
x
1)]
where B > 0, A B, c > 1, m =
B
log c
, x 0
Weibulls Law: (x) = kx
n
and s(x) = exp(ux
n+1
)
where k > 0, n > 0, u =
k
(n + 1)
, x 0
DeMoivres Law and Modied DeMoivres Law:
If x is subject to DeMoivres Law with maximum age , then all of the relations on the left
below are true. The relations on the right are for Modied DeMoivres Law with c > 0.
DeMoivre Modied DeMoivre
(x) =
1
x
(x) =
c
x
s(x) =
x

s(x) =
_
x

_
c
l
x
= l
0

l
0
_
x

_
c

e
x
= E[T] =
x
2

e
x
=
x
c+1
Var[T] =
(x)
2
12
Var[T] =
(x)
2
c
(c+1)
2
(c+2)
t
p
x
=
xt
x
t
p
x
=
_
xt
x
_
c

x
(t) =
1
xt

x
(t) =
c
xt
Be careful on the exam - Modied DeMoivre problems are often disguised in a question that
starts something like
You are given
s(x) =
_
1
x
80
_
2

In cases like this, you have to recognize that the question is just a modied DeMoivre written
in a dierent algebraic form.
Arch MLC, Fall 2007 c _Yufeng Guo 41
www.archactuarial.com www.guo.coursehost.com 42
Note that in this table, the function listed at left are given in terms of the functions across
the top row!
F(x) s(x) f(x) (x)
F(x) F(x) 1 s(x)
_
x
0
f(u) du 1 e

_
x
0
(t) dt
s(x) 1 F(x) s(x)
_

x
f(u) du e

_
x
0
(t) dt
f(x) F

(x) s

(x) f(x) (x) e

_
x
0
(t) dt
(x)
F

(x)
1F(x)
s

(x)
s(x)
f(x)
s(x)
(x)
Assumptions for fractional ages.
Uniform Constant
Function Distribution Force Hyperbolic
t
q
x
tq
x
1 p
t
x
tqx
1(1t)qx
(x +t)
qx
1tqx
log p
x
qx
1(1t)qx
1t
q
x+t
(1t)qx
1tqx
1 p
1t
x
(1 t)q
x
y
q
x+t
yqx
1tqx
1 p
y
x
yqx
1(1yt)qx
t
p
x
1 tq
x
p
t
x
px
1(1t)qx
t
p
x
(x +t) q
x
p
t
x
log p
x
qxpx
[1(1t)qx]
2
Arch MLC, Fall 2007 c _Yufeng Guo 42
www.archactuarial.com www.guo.coursehost.com 43
Past SOA/CAS Exam Questions:
All of these questions have appeared on SOA/CAS exams between the years 2000 and 2005.
You will nd that they often involve some clever thinking in addition to knowledge of
actuarial math. These questions are used with permission.
1. Given:
(i)

e
0
= 25
(ii) l
x
= x, 0 x
(iii) T(x) is the future lifetime random variable.
Calculate Var[T(10)].
(A) 65 (B)93 (C) 133 (D) 178 (E) 333
Solution:

e
0
=
_

0
_
1
t

_
dt =

2
2
=

2
= 25 = 50

e
10
=
_
40
0
_
1
t
40
_
dt = 40
40
2
(2)(40)
= 20
Var [T(x)] = 2
_
40
0
t
_
1
t
40
_
dt (20)
2
= 2
_
t
2
2

t
3
3 40
_
40
0
(20)
2
= 133
Key: C
2. For a certain mortality table, you are given:
(i) (80.5) = 0.0202
(ii) (81.5) = 0.0408
(iii) (82.5) = 0.0619
(iv) Deaths are uniformly distributed between integral ages.
Calculate the probability that a person age 80.5 will die within two years.
(A) 0.0782 (B) 0.0785 (C) 0.0790 (D) 0.0796 (E) 0.0800
Arch MLC, Fall 2007 c _Yufeng Guo 43
www.archactuarial.com www.guo.coursehost.com 44
Solution:
0.0408 = (81.5) =
q
81
1 (1/2)q
81
q
81
= 0.0400
Similarly,
q
80
= 0.0200 and q
82
= 0.0600
2
q
80.5
=
1/2
q
80.5
+
1/2
p
80.5
_
q
81
+p
81

1/2
q
82
_
=
0.01
0.99
+
0.98
0.99
[0.04 + 0.96(0.03)] = 0.0782
Key: A
3. Mortality for Audra, age 25, follows De Moivres law with = 100. If she takes up hot
air ballooning for the coming year, her assumed mortality will be adjusted so that for
the coming year only, she will have a constant force of mortality of 0.1.
Calculate the decrease in the 11-year temporary complete life expectancy for Audra if
she takes up hot air ballooning.
(A) 0.10 (B) 0.35 (C) 0.60 (D) 0.80 (E) 1.00
Solution: STANDARD:

e
25:11
=
_
11
0
(1
t
75
)dt = t
t
2
2 75
[
11
0
= 10.1933
MODIFIED:
p
25
= e

_
1
0
0.1ds
= e
0.1
= 0.90484

e
25:11
=
_
1
0
t
p
25
dt +p
25
_
10
0
(1
t
74
)dt
=
_
1
0
e
0.1t
dt +e
0.1
_
10
0
(1
t
74
)dt
=
1 e
0.1
0.1
+e
0.1
_
t
t
2
2 74
_
[
10
0
= 0.95163 + 0.90484(9.32432) = 9.3886
The dierence is 0.8047. Key: D
Arch MLC, Fall 2007 c _Yufeng Guo 44
www.archactuarial.com www.guo.coursehost.com 45
4. You are given the following extract from a select-and-ultimate mortality table with a
2-year select period:
x l
[x]
l
[x]+1
l
x+2
x + 2
60 80,625 79,954 78,839 62
61 79,137 78,402 77,252 63
62 77,575 76,770 75,578 64
Assume that deaths are uniformly distributed between integral ages.
Calculate
0.9
q
[60]+0.6
.
(A) 0.0102 (B) 0.0103 (C) 0.0104 (D) 0.0105 (E) 0.0106
Solution:
l
[60]+0.6
= (0.6)(79,954) + (0.4)(80,625) = 80,222.4
l
[60]+1.5
= (0.5)(79,954) + (0.5)(78,839) = 79,396.5
0.9
q
[60]+0.6
=
80,222.4 79,396.5
80,222.4
= 0.0103
Key: B
5. Given:
(i) (x) = F +e
2x
, x 0
(ii)
0.4
p
0
= 0.50
Calculate F.
(A) -0.20 (B) -0.09 (C) 0.00 (D) 0.09 (E) 0.20
Arch MLC, Fall 2007 c _Yufeng Guo 45
www.archactuarial.com www.guo.coursehost.com 46
Solution:
0.4
p
0
= 0.5 = e

_
0.4
0
(F+e
2x
)dx
= e
0.4F
_
e
2x
2
_
0.4
0
= e
0.4F
_
e
0.8
1
2
_
0.5 = e
0.4F0.6128
ln(0.5) = 0.4F 0.6128
0.6931 = 0.4F 0.6128 F = 0.20
Key: E
6. An actuary is modeling the mortality of a group of 1000 people, each age 95, for the
next three years.
The actuary starts by calculating the expected number of survivors at each integral age
by
l
95+k
= 1000
k
p
95
, k = 1, 2, 3
The actuary subsequently calculates the expected number of survivors at the middle of
each year using the assumption that deaths are uniformly distributed over each year of
age.
This is the result of the actuarys model:
Age Survivors
95 1000
95.5 800
96 600
96.5 480
97
97.5 288
98
The actuary decides to change his assumption for mortality at fractional ages to the
constant force assumption. He retains his original assumption for each
k
p
95
.
Calculate the revised expected number of survivors at age 97.5.
(A) 270 (B) 273 (C) 276 (D) 279 (E) 282
Arch MLC, Fall 2007 c _Yufeng Guo 46
www.archactuarial.com www.guo.coursehost.com 47
Solution: From UDD, l
96.5
=
l
96
+l
97
2
.
480 =
600 +l
97
2
l
97
= 360
Likewise, from l
97
= 360 and l
97.5
= 288, we get l
98
= 216.
For constant force, e

=
l
98
l
97
=
216
360
= 0.6
0.5
p
x
= e
0.5
= (0.6)
1
2
= 0.7746
l
97.5
= (0.7746)l
97
= (0.7746)(360) = 278.86
Key: D
7. For a 4-year college, you are given the following probabilities for dropout from all causes:
q
0
= 0.15
q
1
= 0.10
q
2
= 0.05
q
3
= 0.01
Dropouts are uniformly distributed over each year.
Compute the temporary 1.5-year complete expected college lifetime of a student enter-
ing the second year,

e
1:1.5
.
(A) 1.25 (B) 1.3 (C) 1.35 (D) 1.4 (E) 1.45
Solution:

e
1:1.5
=
_
1.5
0
t
p
1
dt
=
_
1
0
t
p
1
dt +
1
p
1
_
0.5
0
x
p
2
dx
=
_
1
0
(1 0.1t)dt + 0.9
_
0.5
0
(1 0.05x)dx
=
_
t
0.1t
2
2
_
1
0
+ 0.9
_
x
0.05x
2
2
_
0.5
0
= 0.95 + 0.444 = 1.394 Key: D
Arch MLC, Fall 2007 c _Yufeng Guo 47
www.archactuarial.com www.guo.coursehost.com 48
8. For a given life age 30, it is estimated that an impact of a medical breakthrough will
be an increase of 4 years in

e
30
, the complete expectation of life.
Prior to the medical breakthrough, s(x) followed de Moivres law with = 100 as the
limiting age.
Assuming de Moivres law still applies after the medical breakthrough, calculate the
new limiting age.
(A) 104 (B) 105 (C) 106 (D) 107 (E)108
Solution: For deMoivres law,

e
30
=
30
2
.
Prior to medical breakthrough = 100

e
30
=
10030
2
= 35.
After medical breakthrough

e

30
=

e
30
+4 = 39.

30
= 39 =

30
2

= 108. Key E
9. For a select-and-ultimate mortality table with a 3-year select period:
(i)
x q
[x]
q
[x]+1
q
[x]+2
q
x+3
x + 3
60 0.09 0.11 0.13 0.15 63
61 0.10 0.12 0.14 0.16 64
62 0.11 0.13 0.15 0.17 65
63 0.12 0.14 0.16 0.18 66
64 0.13 0.15 0.17 0.19 67
(ii) White was a newly selected life on 01/01/2000.
(iii) Whites age on 01/01/2001 is 61.
(iv) P is the probability on 01/01/2001 that White will be alive on 01/01/2006.
Calculate P.
(A) 0 P < 0.43 (B) 0.43 P < 0.45 (C) 0.45 P < 0.47
(D) 0.47 P < 0.49 (E) 0.49 P 1.00
Arch MLC, Fall 2007 c _Yufeng Guo 48
www.archactuarial.com www.guo.coursehost.com 49
Solution:
5
p
[60]+1
=
_
1 q
[60]+1
_ _
1 q
[60]+2
_
(1 q
63
) (1 q
64
) (1 q
65
)
= (0.89)(0.87)(0.85)(0.84)(0.83) = 0.4589
Key: C
10. You are given:
(x) =
_
0.04, 0 < x < 40
0.05, x > 40
Calculate

e
25:25
.
(A) 14.0 (B) 14.4 (C) 14.8 (D) 15.2 (D) 15.6
Solution:

e
25:25
=
_
15
0
t
p
25
dt +
15
p
25
_
10
0
t
p
40
dt
=
_
15
0
e
0.04t
dt +
_
e

_
15
0
0.04 ds
__
10
0
e
0.05t
dt
=
1
0.04
_
1 e
0.60
_
+e
0.60
_
1
0.05
_
1 e
0.50
_
_
= 11.2797 + 4.3187 = 15.60
Key: E
11. T, the future lifetime of (0), has a spliced distribution.
(i) f
1
(t) follows the Illustrative Life Table.
(ii) f
2
(t) follows DeMoivres Law with = 100.
(iii)
f
T
(t) =
_
k f
1
(t), 0 t 50
1.2 f
2
(t), 50 < t
Calculate
10
p
40
.
(A) 0.81 (B) 0.85 (C) 0.88 (D) 0.92 (E) 0.96
Arch MLC, Fall 2007 c _Yufeng Guo 49
www.archactuarial.com www.guo.coursehost.com 50
SOLUTION: From the Illustrative Life Table:
l
50
l
0
= 0.8951
l
40
l
0
= 0.9313
1 =
_

0
f
T
(t) dt =
_
50
0
kf
1
(t) dt +
_

50
1.2 f
2
(t) dt
= kF
1
(50) + 1.2 (F
2
() F
2
(50))
= k (1
50
p
0
) + 1.2(1 0.5)
= k(1 0.8951) + 0.6
k =
1 0.6
1 08951
= 3.813
For x 50,
F
T
(x) =
_
x
0
3.813f
1
(t) dt = 3.813F
1
(x)
This gives us the following two results:
F
T
(40) = 3.813
_
1
l
40
l
0
_
= 0.262
F
T
(50) = 3.813
_
1
l
50
l
0
_
= 0.400
10
p
40
=
1 F
T
(50)
1 F
T
(40)
=
1 0.400
1 0.262
= 0.813
12. For a double decrement table, you are given:
(i)
(1)
x
(t) = 0.2
()
x
(t), t > 0
(ii)
()
x
(t) = kt
2
, t > 0
(iii) q

(1)
x
= 0.04
Calculate
2
q
(2)
x
.
(A) 0.45 (B) 0.53 (C) 0.58 (D) 0.64 (E) 0.73
Arch MLC, Fall 2007 c _Yufeng Guo 50
www.archactuarial.com www.guo.coursehost.com 51
SOLUTION:
We can use the exponential formulation for p

(x)
0.04 = q

(1)
x
= 1p

(1)
x
= 1e

_
1
0

(1)
x
(t) dt
= 1e

_
1
0
0.2
()
x
(t) dt
= 1e

_
1
0
0.2kt
2
dt
= 1e
0.2k/3
e
0.2k/3
= 0.96

(1)
x
(t) = 0.2
()
x
(t)
(2)
x
(t) = 0.8
()
x
(t)
2
q
(2)
x
=
_
2
0
t
p
()
x

(2)
x
(t)dt =
_
2
0
t
p
()
x
(0.8)
()
x
(t)dt = 0.8
2
q
()
x
To get
2
q
()
x
, we use
2
q
()
x
= 1
2
p
()
x
= 1 e

_
1
0

()
x
(t) dt
= 1 e

_
1
0
kt
2
dt
= 1 e
8k/3
= 1
_
e
0.2k/3
_
40
= (0.96)
40
= 0.1954

2
q
()
x
= 0.8046

2
q
(2)
x
= (0.8)(0.8046) = 0.644
13. You are given:
(i)

e
30:40
= 27.692
(ii) s(x) = 1
x

, x
(iii) T(x) is the future lifetime random variable for (x).
Calculate Var(T(30)).
(A) 332
(B) 352
(C) 372
(D) 392
(E) 412
SOLUTION:

e
30:40
=
_
40
0
t
p
30
dt
=
_
40
0
30t
30
dt
Arch MLC, Fall 2007 c _Yufeng Guo 51
www.archactuarial.com www.guo.coursehost.com 52
=
_
t
t
2
2(30)
_
[
40
0
= 40
800
30
= 27.692
= 95
t
p
30
=
65t
65
Now, realize (after getting = 95) that T(30) is uniformly on (0, 65), its variance is
just the variance of a continuous uniform random variable:
V ar =
(650)
2
12
= 352.08
Key: B
14. For a life table with a one-year select period, you are given:
(i)
x l
[x]
d
[x]
l
x+1

e
[x]
80 1000 90 8.5
81 920 90
(ii) Deaths are uniformly distributed over each year of age.
Calculate

e
[81]
.
(A) 8.0
(B) 8.1
(C) 8.2
(D) 8.3
(E) 8.4
SOLUTION:
Complete the table:
l
81
= l
[80]
d
[80]
= 910
l
82
= l
[81]
d
[81]
= 830
p
[80]
=
910
1000
= 0.91
p
[81]
=
830
920
= 0.902
p
81
=
830
910
= 0.912
e
[80]
=
1
2
q
[80]
+p
[80]
(1 + e
81
)
where q
[80]
contributes
1
2
since UDD
8.5 =
1
2
(1 0.91) + (0.91) (1 + e
81
)
Arch MLC, Fall 2007 c _Yufeng Guo 52
www.archactuarial.com www.guo.coursehost.com 53
e
81
= 8.291
e
81
=
1
2
q
81
+p
81
(1 + e
82
)
e
82
= 8.043
e
[81]
=
1
2
q
[81]
+p
[81]
(1 + e
82
)
=
1
2
(1 0.912) + (0.912)(1 + 8.043)
= 8.206
Key: C
15. You are given:
(x) =
_
0.05 50 x < 60
0.04 60 x < 70
Calculate
4|14
q
50
.
(A) 0.38
(B) 0.39
(C) 0.41
(D) 0.43
(E) 0.44
SOLUTION:
4
p
50
= e
(0.05)(4)
= 0.8187
10
p
50
= e
(0.05)(10)
= 0.6065
8
p
60
= e
(0.04)(8)
= 0.7261
18
p
50
= (
10
p
50
)(
8
p
60
) = 0.6065 0.7261 = 0.4404
4|14
q
50
=
4
p
50

18
p
50
= 0.8187 0.4404 = 0.3783
Key: A
Arch MLC, Fall 2007 c _Yufeng Guo 53
www.archactuarial.com www.guo.coursehost.com 54
16. For a population which contains equal numbers of males and females at birth:
(i) For males,
m
(x) = 0.10, x 0
(ii) For females,
f
(x) = 0.08, x 0
Calculate q
60
for this population.
(A) 0.076
(B) 0.081
(C) 0.086
(D) 0.091
(E) 0.096
SOLUTION:
s(60) =
e
(0.1)(60)
+e
(0.08)(60)
2
= 0.005354
s(61) =
e
(0.1)(61)
+e
(0.08)(61)
2
= 0.00492
q
60
= 1
0.00492
0.005354
= 0.081
Key: B
Arch MLC, Fall 2007 c _Yufeng Guo 54
www.archactuarial.com www.guo.coursehost.com 55
Problems from Pre-2000 SOA-CAS exams
1. You are given: (x) =

1
80 x
, 0 x < 80.
Calculate the median future lifetime of (20).
(A) 5.25 (B) 6.08 (C) 8.52 (D) 26.08 (E) 30.00
2. You are given:

t
p
x
= (0.8)
t
, t 0
l
x+2
= 6.4
Calculate T
x+1
.
(A) 4.5 (B) 7.2 (C) 28.7 (D) 35.9 (E) 44.8
Use the following information for the next 4 questions:
You are given:
T(x) is the random variable for the future lifetime of (x).
The p.d.f. of T is f
T
(t) = 2e
2t
, t 0.
3. Calculate

e
x
(A) 0.5 (B) 2.0 (C) 10.0 (D) 20.0 (E) 40.0
4. Calculate Var[T].
(A) 0.25 (B) 0.50 (C) 1.00 (D) 2.00 (E) 4.00
5. Calculate m(x), the median future lifetime of (x).
(A)
e
4
2
(B)
e
2
2
(C)
ln 2
2
(D)
ln 4
2
(E) 1
Arch MLC, Fall 2007 c _Yufeng Guo 55
www.archactuarial.com www.guo.coursehost.com 56
6. Calculate m
x
, the central-death-rate at age x.
(A)
e
2
2
(B) e
2
(C) 2e
2
(D) 1 (E) 2
7. You are given:
s(x) =
_
1
x

, 0 x < , where > 0 is a constant.


Calculate
x


e
x
.
(A)

+ 1
(B)

+ 1
(C)

2
+ 1
(D)

2
x
(E)
( x)
( + 1)
8. You are given:
q
60
= 0.3
q
61
= 0.4
f is the probability that (60) will die between ages 60.5 and 61.5 under the uniform
distribution of deaths assumption.
g is the probability that (60) will die between ages 60.5 and 61.5 under the Balducci
assumption.
Calculate 10,000(g f).
(A) 0 (B) 85 (C) 94 (D) 178 (E) 213
Arch MLC, Fall 2007 c _Yufeng Guo 56
www.archactuarial.com www.guo.coursehost.com 57
Solutions to Chapter 3
1. Key: A We need to nd t such that
t
p
20
= 0.5
0.5 = e

_
t
0
(20+s) ds
= e

_
t
0
(60s)
0.5
ds
= e
2[(60s)
0.5
]
t
0
= e
2[

60t

60]
ln 0.5 = 2
_

60 t

60
_

60 t = 7.40 t = 5.25
2. Key: D
T
x+1
=
_

0
l
x+1+t
dt
l
x+1
=
6.4
0.8
= 8 l
x+1+t
= 8(0.8)
t
T
x+1
= 8
_

0
(0.8)
t
dt =
8
ln 0.8
(0.8)
t
[

0
=
8
ln 0.8
= 35.9
3. Key: A This is constant force of mortality (CFM) with = 2.
E[T] =
1

= 0.5 =

e
x
4. Key: A (CFM) Var[T] =
1

2
= 0.25
5. Key: C
0.5 =
t
p
x
= e
t
= e
2t
t =
ln 2
2
Arch MLC, Fall 2007 c _Yufeng Guo 57
www.archactuarial.com www.guo.coursehost.com 58
6. Key: E
m
x
=
l
x
l
x+1
L
x
=
l
x
(1 e

)
_
1
0
l
x

t
p
x
dt
=
l
x
(1 e
2
)
l
x
_
1
0
e
t
dt
==
1 e
2
1
2
(1 e
2
)
= 2
7. Key: A This is a Modied DeMoivre problem with constant equal to . So
(x) =

x
,

e
x
=
x
+ 1
Multiplying these two quantities gives

+ 1
8. Key: B In both cases the probability is given by
0.5
p
60

0.5
q
60.5
+
1
p
60

0.5
q
61
UDD: [1 (0.5)(0.3)]
(0.5)(0.3)
1(0.5)(0.3)
+ (0.7) (0.5)(0.4) = 0.2900 = f
Balducci:
0.7
1(0.5)(0.3)
(0.5)(0.3)
1
+ (0.7)
(0.5)(0.4)
1(0.5)(0.4)
= 0.2985 = g
10,000(g f) = 85
Arch MLC, Fall 2007 c _Yufeng Guo 58
Chapter 2
ACTUARIAL MATHEMATICS
CHAPTER 4 LIFE INSURANCE
Option A reference: Actuarial Mathematics Chapter 4
Option B reference: Models for Quantifying Risk Chapter 9
This chapter introduces a variety of types of life insurance. Keep in mind that all of these
products have a basic structure of some benet paid at some point in the future if certain
conditions are met. Dont lose sight of the fact that all of the products mentioned have this
same structure they vary only by the conditions to be met.
The text considers products that pay a unit benet of 1. Sometimes this is easy to forget
because the 1 does not show up in multiplicative formulas. The text also considers products
that pay the benet at the moment of death. Clearly, barring a very annoying life insurance
agent, benets are not paid precisely at this point. The text then considers products paying
a benet at the end of the year of death. This too is not realistic, but products that pay o
at the moment of death or at the end of the year of death have the most convenient formulas
to work with. Also, they are what you need to know to pass the exam.
To the text!
59
www.archactuarial.com www.guo.coursehost.com 60
4.2 Insurances Payable at the Moment of Death
Option A reference: Actuarial Mathematics Chapter 4.2
Option B reference: Models for Quantifying Risk Chapter 9.3
Let b
t
be a benet function a benet payable at time t. Let v
t
be a discount function
that discounts the benet back to the time of policy issue. Unless stated otherwise, we will
assume the policy is issued at time t = 0.
The present value function for the benet is z
t
= b
t
v
t
. So, z
t
is the present value of the
benet payment at the time of policy issue.
issue
t=0
t - death
b
t
paid here
time
t
z is the value of
the benefit at
time 0.
Recall from Chapter 3 that for (x) (an individual age x), the future lifetime is a random
variable T = T(x), so associated with z
t
is a random variable Z
T
. Generally the text uses
the notation
Z = b
T
v
T
.
The expected value of Z is called the Actuarial Present Value of the benet this is a
critical concept. The usual present value that you are familiar with from Interest The-
ory discounts for interest only. Actuarial Present Value (APV) discounts for interest and
mortality. Mortality aects when and if the benet is received.
Consider the following two scenarios:
1. A payment of 10,000 is payable to you (or your heirs if you are dead) at the end of t
years from now.
2. A payment of 10,000 is payable to you at the end of t years from now only if you are
alive.
Actuarially, we would say the value of the payment in Scenario (1) is 10, 000v
t
, while the
value of the payment in Scenario (2) is 10, 000v
t
t
p
x
. That, in a nutshell, is the dierence
between interest theory from earlier actuarial courses and life contingencies in this course.
Arch MLC, Fall 2007 c _Yufeng Guo 60
www.archactuarial.com www.guo.coursehost.com 61
TYPES OF INSURANCE
4.2.1 Level Benet Insurance
Option A reference: Actuarial Mathematics Chapter 4.2.1
Option B reference: Models for Quantifying Risk none
Name: n-year Term Life Insurance
Description: A at benet is paid at death if the insured dies within an n-year term
starting at issue.
b
t
=
_
1 t n 1 if death within n years
0 t > n 0 otherwise
v
t
= v
t
t 0
Z =
_
v
T
T n Note this is b
t
v
t
in each case of T.
0 T > n
Since the random variable Z represents the present value of future benets, E(Z) repre-
sents the expected present value of the benets. This expected value takes into account the
probability of actually receiving benets. The expected value E(Z) is called the actuarial
present value since it contains a mortality factor. The following is important notation:
E(Z) =A
1
x:n
=
_
n
0
v
t
t
p
x

x
(t)dt
OK, there is a lot contained in that line that needs explaining so dont go anywhere for a
couple of minutes.
First, lets look at the components of the integral more carefully, then we will discuss the
A
1
x:n
notation.
A
1
x:n
=
_
n
0
interest discount
..
v
t

t
p
x
..
survival from 0 to t

prob of death between times t and t +dt


..

x
(t)dt
The expression inside the integral can be thought of as the (very small) value of a death
benet of 1 payable only if you die exactly at time t. What should such a benet be worth?
It should equal (1 discounted to time t)(the probability that you survive to time t) (the
probability that you then die at time t). That is v
t
t
p
x

x
(t). Now to get the value of A
1
x:n
we just have to integrate this benet over all the time that the insurance is in force; that is,
from t = 0 to t = n. I realize this may be a lot to take in right now, but it will become more
intuitive as we go through the dierent types of insurance.
Arch MLC, Fall 2007 c _Yufeng Guo 61
www.archactuarial.com www.guo.coursehost.com 62
Now, what about the symbol A
1
x:n
? Obviously this symbol was designed by a committee (no
kidding!). This odd-looking notation will become second nature well before test day.
1. The bar over the A tells us that the death benet is payable at the moment of death.
This is continuous insurance.
2. The x tells the age at which the expected value of the benet is calculated. Except in
the case of deferred insurance (which well see later), this is the age of the person when
coverage begins.
3. The n is the term of the policy. When the policy is whole life (i.e. when n = ), the
: n is omitted.
4. The 1 over the x tells us that death must occur for benets to be paid.
So much for E(Z), now for Var [Z]. Recall from Statistics that
Var [Z] = E(Z
2
) [E(Z)]
2
.
Throughout the book, E(Z
2
) will be calculated using the rule of moments. This rule says
that as long as the force of interest is deterministic (i.e. not a random variable or something
weird like that), E(Z
j
) at the force of interest is equal to E(Z) at the force of interest
j . In other words, to nd E(Z
j
), just multiply the interest at each point in time by j and
calculate the expected value. The symbol for E(Z) at 2 times the force of interest is
2
A
1
x:n
(as if the notation werent bad enough already). This leads to
Var(Z) =
2
A
1
x:n

_
A
1
x:n
_
2
.
Note: The rule of moments holds as long as the benet is always 0 or 1.
EXAMPLE: (try this by covering up the solution rst)
Let the force of interest be a constant = 0.03 and let the force of mortality for
(x) be a constant = 0.01 for all t,
1. Find the actuarial present value (APV) of a 5-year term insurance with a
death benet of 1.
2. Find Var(Z)
SOLUTION:
1. The APV equals
A
1
x:n
=
_
n
0
v
t
t
p
x

x
(t)dt =
_
5
0
e
t
e
t
dt .
Remember, we saw in Chapter 3 that in this situation
t
p
x
= e
t
. This
equals
(0.01)
_
5
0
e
0.04t
dt = 0.25
_
e
0.04t
_

5
0
= 0.25
_
1 e
0.2
_
= 0.0453.
Arch MLC, Fall 2007 c _Yufeng Guo 62
www.archactuarial.com www.guo.coursehost.com 63
2. The variance equals
2
A
1
x:n

_
A
1
x:n
_
2
. To nd
2
A
1
x:n
, we take the expression
for E(z) and double the force of interest every time it appears. This leads
to
2
A
1
x:n
=
_
5
0
e
2t
e
t
dt .
(0.01)
_
5
0
e
0.07t
dt =
1
7
_
1 e
0.35
_
= 0.0422.
Var(Z) = 0.0422 (0.0453)
2
= 0.0401
Name: Whole Life Insurance
Description: A at benet is paid at the moment of death at any time in the future (over
the whole life of the insured)
b
t
= 1 t 0 1 will denitely be paid at some point.
v
t
= v
t
t 0
Z = v
T
T 0 Note this equals b
t
v
t
.
The symbol A
x
is used to represent E(Z) and
A
x
= E(Z) =
_

0
v
t
t
p
x

x
(t)dt
Notes:
1. This is term insurance with n = .
2. The bar over the A indicates the benet is payable at the moment of death (continuous)
3. The x alone indicates whole life no limit to the term of coverage.
Finally, Var[Z] =
2
A
x

_
A
x
_
2
Before some examples, note that Example 4.2.1 in the text is a good example to study and
understand.
Note: For certain mortality and interest assumptions, A
x
takes a particularly simple form
that absolutely should be memorized. For example you can show for yourself (do a little
integration or read through Example 4.2.3 in the text.) that if the force of mortality is a
constant and the force of interest is a constant , then
A
x
=

+
You must know this formula and be comfortable using it on exam day. There will likely be
several problems that will require it.
Arch MLC, Fall 2007 c _Yufeng Guo 63
www.archactuarial.com www.guo.coursehost.com 64
EXAMPLE: Constant Force of Mortality
Assuming a constant force of mortality and constant force of interest , nd
Var[Z] in terms of and .
SOLUTION:
We know that Var[Z] =
2
A
x

_
A
x
_
2
. To nd
2
A
x
, we remember that it is just
the expression for E[Z] with the force of interest doubled every time it appears.
So
2
A
x
=

+ (2)
and
Var[Z] =

+ (2)

_

+
_
2
.
Dont bother memorizing this complicated formula. Instead, memorize the rule
of moments and the formula for E[Z].
EXAMPLE: DeMoivres Law
(x) is subject to DeMoivres law for mortality with = 100. If = 0.05, what
are 1. A
40
, and 2. A
1
40:25
?
(You should denitely try this one without looking at the solution!)
SOLUTION:
1.
A
x
=
_
60
0
v
t
t
p
x

x
(t)dt.
For DeMoivre,
t
p
x
=
x t
x
and
x
(t) =
1
x t
,

t
p
x

x
(t) =
1
60
.
A
x
=
_
60
0
e
0.05t
1
60
dt =
20
60
_
e
0.05t
_
60
0
=
1
3
_
1 e
3
_
= 0.3167.
2. For A
1
40:25
, we do exactly the same thing except the limits of integration are
0 and 25. The resulting answer is A
1
40:25
= 0.2378.
4.2.2 Endowment Insurance
Option A reference: Actuarial Mathematics Chapter 4.2
Option B reference: Models for Quantifying Risk Chapter 9.1.4
Arch MLC, Fall 2007 c _Yufeng Guo 64
www.archactuarial.com www.guo.coursehost.com 65
Unlike term and whole life insurance, endowment insurances pay you something if you live.
Name: n-year Pure Endowment
Description: Provides a benet at the end of n years if and only if the insured is still alive
at the end of the n-year period. (You must survive to get the benet.)
b
t
=
_
0 t n 0 if death within n years
1 t > n 1 if survive n years from issue
v
t
= v
n
t 0 If payment occurs, we know it will be at t = n.
Z =
_
0 T n Note this is b
t
v
t
in each case of T.
v
n
T > n
The symbol for E(Z) is A
x:
1
n
. Note where the 1 is. If 1 is over the n it indicates you get
paid only if you survive n years. If it is over the x, you only get paid if you die within n
years. Anyway,
A
x:
1
n
= E(Z) = v
n
n
p
x
and
Var[Z] =
2
A
x:
1
n

_
A
x:
1
n
_
2
.
Applying the rule of moments (and remembering that doubling the force of interest turns v
n
into v
(2n)
), we can see that
Var[Z] = v
(2n)
n
p
x
(v
n
n
p
x
)
2
= v
(2n)
n
p
x n
q
x
.
Again, I wouldnt memorize anything but the expression for A
x:
1
n
and the rule of moments.
One important note before we move on: When we get to annuities - there will be a new
symbol for pure endowments,
n
E
x
. For now just remember that
n
E
x
= A
x:
1
n
and they mean
the same thing: the present value of 1 paid to (x) in exactly n years if (x) survives n years.
Name: n-year Endowment Insurance
Description: The benet is paid at the earlier of death or n years. You receive the benet
either way, only the timing is aected by death or survival.
b
t
= 1 t 0 A benet of 1 is certain to be paid.
v
t
=
_
v
t
t n
v
n
t > n
Z =
_
v
T
T n
v
n
t > n
Arch MLC, Fall 2007 c _Yufeng Guo 65
www.archactuarial.com www.guo.coursehost.com 66
E(Z) =A
x:n
(no 1 over either the x or the n ),
Var[Z] =
2
A
x:n

_
A
x:n
_
2
.
Note: This insurance is exactly the same as an n-yr term policy plus an n-yr pure endowment.
Therefore,
A
x:n
=A
1
x:n
+A
x:
1
n
.
This is a good relationship for the SOA-CAS to use in a question. It comes up somewhere
on every SOA or CAS exam I have seen for this course. It also tells us that
A
x:n
=
_
n
0
v
t
t
p
x

x
(t)dt +v
n
n
p
x
.
EXAMPLE: DeMoivres Law
(x) is subject to DeMoivres law for mortality with = 100. If = 0.05, what is
A
40:25
?
SOLUTION:
We saw in an earlier example that A
1
40:25
= 0.2378 for this scenario, and since
A
40:25
=A
1
40:25
+A
40:
1
25
,
we only need to nd the value of
A
40:
1
25
= v
25
25
p
40
= e
(0.05)(25)
_
35
60
_
= 0.1671.
A
40:25
= 0.2378 + 0.1671 = 0.4049.
4.2.3 Deferred Insurance
Option A reference: Actuarial Mathematics Chapter 4.2.3
Option B reference: Models for Quantifying Risk none
Name: m-year Deferred Insurance
Description: Provides for a benet following death of the insured only if the insured dies
more than m years after issue. Basically, this is a whole life policy in which coverage
doesnt begin until m years have elapsed.
issue
t=0
t=m
Benefit paid if
death occurs in
this time period.
Arch MLC, Fall 2007 c _Yufeng Guo 66
www.archactuarial.com www.guo.coursehost.com 67
b
t
=
_
1 t > m
0 t m no benet unless insured lives at least m years.
v
t
= v
t
t 0
Z =
_
v
T
T > m Note this is b
t
v
t
in each case of T.
0 T m
The symbol for the APV of this insurance is
m|
A
x
(although this is lousy notation since the
omitted symbol to the right of the [ is instead of 1). Also
m|
A
x
= E(Z) =
_

m
v
t
t
p
x

x
(t)dt .
EXAMPLE: Constant Force of Mortality
1. If and are constant, nd
m|
A
x
in terms of , , and m.
2. Write
m|
A
x
in terms of two types of insurance that we have already seen.
SOLUTION:
1.
m|
A
x
=
_

m
v
t
t
p
x

x
(t)dt =
_

m
e
t
e
t
dt =

+
e
m(+)
Its up to you to decide whether to derive the formula on the spot or memorize
the formula. Robin thinks that he can derive the formula from scratch
quickly in the exam. Nathan wants to memorize it to avoid deriving the
formula in the exam. Yufeng will do both.
2.
m|
A
x
=A
x
A
1
x:m
This makes sense if you say it in English A whole life policy deferred for
m years is equal to a whole life policy starting now minus the rst m years
of coverage.
EXAMPLE: DeMoivres Law
(x) is subject to DeMoivres law for mortality with = 100. If = 0.05, what is
25|
A
40
?
SOLUTION:
Arch MLC, Fall 2007 c _Yufeng Guo 67
www.archactuarial.com www.guo.coursehost.com 68
We saw in an earlier example that A
1
40:25
= 0.2378 for this scenario, and we also
know that
25|
A
40
=A
40
A
1
40:25
.
Since we saw that A
40
= 0.3167, we have that
25|
A
40
= 0.3167 0.2378 = 0.0789.
Again, dont try to memorize the formulas, try to think logically why it must be
true that a whole life policy starting today is equal to a 25-year term policy plus
a 25-year deferred whole life policy.
Those are the basic level benet life insurance and endowment products covered in the
chapter. Now for the more esoteric stu.
4.2.4 Varying Benet Insurance
Option A reference: Actuarial Mathematics Chapter 4.2.4
Option B reference: Models for Quantifying Risk Chapter 9.4
Name: Annually Increasing Whole Life Insurance
Description: Pays 1 if you die in year 1, 2 in year 2, 3 in year 3, . . . . The later you die,
the more you receive upon death.
b
t
= t + 1| t 0 where | is the greatest integer function.
v
t
= v
t
t 0
Z = t + 1|v
t
T 0
The symbol for the APV of this insurance is (IA)
x
and
(IA)
x
= E(Z) =
_

0
t + 1|v
t
t
p
x

x
(t)dt .
Because of the discontinuous nature of the greatest integer function, this generally would be
a pretty nasty integral to have to integrate.
To get you used to some more notation, we might as well introduce some insurance that
increases in value more often, say m times per year.
Name: m-thly Increasing Whole Life Insurance
Description: Pays
1
m
at death in rst m-th of a year,
Pays
2
m
at death in second m-th of a year, etc.
Arch MLC, Fall 2007 c _Yufeng Guo 68
www.archactuarial.com www.guo.coursehost.com 69
Note that an m-th can be any period of time not necessarily a month, which naturally
comes to mind. Its useful to think of it as a month while learning the concept, but just
realize it can be any period of time less than a year. Also, dont be intimidated by this idea
its really the same idea as annually increasing whole life, except that the increases occur
more frequently than annually.
The symbol for the actuarial present value of this insurance is
_
I
(m)
A
_
x
. Letting m
in the above insurance results in an insurance that pays t at time t. That is:
b
t
= t,
v
t
= v
t
Z = Tv
T
.
This is called continuously increasing insurance and the symbol for its APV is
E(Z) = (IA)
x
.
Just one more type of continuous insurance before we are ready to put it all together with
some questions.
Name: Annually decreasing n-year term insurance
Description: During the n-year period from issue, provides a benet at the end of the year
of death equal to n k where k is the number of years completed since issue. For example,
if death occurs during the rst year, the benet is n. If death occurs during the second year,
benet is n 1, etc.
b
t
=
_
n t| t n
0 t > n (Its n-year term insurance.)
v
t
= v
t
t 0
Z =
_
v
T
(n t|) T n
0 T > n
The APV is
(DA)
1
x:n
= E(Z) =
_
n
0
v
T
(n t|)
t
p
x

x
(t)dt
Table 4.2.1 in the text is not very useful except for one very important fact that you wont
notice if you dont read the footnotes. It tells you exactly when the rule of moments applies.
It never applies for increasing or decreasing insurances - only for insurance where the benet
is always either 0 or 1.
EXAMPLE: (This is mostly for review so these are not too hard. Read the
directions carefully and dont look at the next page!)
Each of the following expressions is associated with a certain type of insurance or
endowment. For each symbol,
Arch MLC, Fall 2007 c _Yufeng Guo 69
www.archactuarial.com www.guo.coursehost.com 70
(A) Name the type of insurance associated.
(B) State what the benet would be under this insurance to a person dying at
t = 6.5
1.
8|
A
x
2. A
x:8
3. A
1
x:8
4. A
x:
1
8
5. (DA)
1
x:8
6. (IA)
1
x:8
7. (IA)
x
8.
_
I
(12)
A
_
x
9.
_
I
(3)
A
_
x
10. A
x
11. (IA)
1
x:4
(I know we havent seen this one but try anyway.)
12. A
x:4
13. A
1
x:4
14. A
x:
1
4
Arch MLC, Fall 2007 c _Yufeng Guo 70
www.archactuarial.com www.guo.coursehost.com 71
SOLUTION:
1. A) 8-year deferred whole life insurance; B) 0
2. A) 8-year endowment; B) 1
3. A) 8-year term insurance; B) 1
4. A) 8-year pure endowment; B) 0
5. A) 8-year annually decreasing term; B) 2
6. A) 8-year annually increasing term; B) 7
7. A) Continuously increasing whole life; B) 6.5
8. A) Monthly increasing whole life; B)
79
12
OK, I looked at the denition of benet for that one!
9. A) 3-thly increasing whole life; B)
20
3
10. A) Whole life; B) 1
11. A) Annually increasing 4-year term; B) 0 (the term has expired)
12. A) 4-year endowment; B) 0 (you were paid 1 at t = 4)
13. A) 4-year term; B) 0
14. A) 4-year pure endowment; B) 0 (you were paid 1 at t = 4)
4.3 Insurances Payable at the End of the Year of Death
Option A reference: Actuarial Mathematics Chapter 4.3
Option B reference: Models for Quantifying Risk Chapter 9.1
These are the curtate versions of the continuous functions we studied above. For continuous
insurance, benets and discounts depended on t the time since issue. In the curtate setting,
benets and discounts will only depend on k, the number of full years that have passed
since the policy was issued. This takes a little getting used to, but once you begin thinking
about them in the right way, curtate versions of insurance look just like their continuous
counterparts except that all the integrals are replaced by sums.
Name: n-year Term Life Insurance paying unit amount at end of the year of death
Description: Pays benet at the end of year of death if death occurs within an n-yr period
from issue.
b
k+1
=
_
1 k = 0, 1, 2, ..., n 1
0 k n
Note that we use the subscript k + 1 so that b
1
refers to the benet available at the end of
the rst year of coverage. If the insured dies in the rst year of coverage, k = 0 since no full
years have been lived. Therefore, b
1
= b
0+1
= b
k+1
.
v
k+1
= v
(k+1)
(discount benet to the end of year of death)
Arch MLC, Fall 2007 c _Yufeng Guo 71
www.archactuarial.com www.guo.coursehost.com 72
Z =
_
v
K+1
K = 0, 1, 2, ..., n 1
0 K n
The symbol for the actuarial present value of this insurance is A
1
x:n
and
E(Z) = A
1
x:n
=
n1

0
v
k+1
k
p
x
q
x+k
Recall that q
x+k
really signies
1
q
x+k
the 1 is dropped by convention. Just as we examined
the integral formula for A
1
x:n
it is worth looking at the stu inside the sum here more carefully.
A
1
x:n
=
n1

0
interest discount
..
v
k+1

k
p
x
..
survival from 0 to k

prob of death between times k and k + 1


..
q
x+k
For each year k after the year of issue, we are taking the present value of one dollar paid
at the end of that year and multiplying it times the probability of surviving to year k (
k
p
x
)
times the probability of dying in year k (q
x+k
). After all, to get paid at the end of the k-th
year of coverage, you have to survive to that year and then die before the year is over. Heres
the detailed formula for A
1
x:n
once again so you can see how closely connected the curtate
and continuous forms of insurance are.
A
1
x:n
=
_
n
0
interest discount
..
v
t

t
p
x
..
survival from 0 to t

prob of death between times t and t +dt


..

x
(t)dt
Remember, with curtate, we take sums instead of integrals. Also, the present value of a
benet for death at time t = 1.1 after issue is the same as for death at time t = 1.9 after
issue since both benets would be paid at the same time.
Var[Z] =
2
A
1
x:n

_
A
1
x:n
_
2
,
where, as before
2
A
1
x:n
is just E[Z] with the force of interest doubled. In other words,
2
A
1
x:n
=
n1

0
v
2(k+1)
k
p
x
q
x+k
.
EXAMPLE:
Write A
1
x:n
and A
x:
1
n
in terms of p, q, and v for n = 1, n = 2.
SOLUTION:
Arch MLC, Fall 2007 c _Yufeng Guo 72
www.archactuarial.com www.guo.coursehost.com 73
A
1
x:1
= vq
x
and A
x:
1
1
= vp
x
A
1
x:2
= vq
x
+v
2
p
x
q
x+1
and A
x:
1
2
= v
2
2
p
x
You should probably do at least up to n = 4 to make sure you have the pattern
down. You want to be able to reproduce this type of thing quickly on exam day.

Lets concentrate on A
1
x:2
for a minute. Rearranging things slightly, we can see that A
1
x:2
=
vq
x
+ vp
x
(vq
x+1
). The rst term is just one year of term insurance starting at age x and
the second term is vp
x
times one year of term insurance starting at age (x + 1). This is an
important way of looking at things. The equation says that a 2-year term insurance provides
one year of term insurance plus, if you survive, you get one year of term insurance starting
at age x + 1. The vp
x
term is discounting the second year of term insurance for interest (v)
and survivorship (p
x
). You have to live one year to get the second year of insurance.
More generally, we have our rst important recursion relation.
A
1
x:n
= vq
x
+vp
x
A
1
x+1:n1
You can learn recursions like this in two ways. First, know how to derive the recursive
formula rigorously (refer to the textbook). After knowing how to derive the formula, explain
the meaning of the formula in plain English.
This is what the formula means: an n-year term insurance on x is equal to a 1-year term
insurance plus, if the insured is still alive one year later, a brand new (n1)-year term
insurance on (x + 1). This makes lot of sense. However, remember to discount the if you
survive part for both interest and survivorship.
Name: Whole Life
Description: Pays benet at the end of year of death at any time during the future (over
the whole life of the insured)
b
k+1
= 1 k = 0, 1, 2, ...
v
k+1
= v
(k+1)
k = 0, 1, 2, ...
Z = v
K+1
K = 0, 1, 2, ...
E(Z) = A
x
=

0
v
(k+1)
k
p
x
q
x+k
After our careful recursion work above, the next formula should make sense:
A
x
= vq
x
+vp
x
A
x+1
Arch MLC, Fall 2007 c _Yufeng Guo 73
www.archactuarial.com www.guo.coursehost.com 74
It says that a whole life policy for (x) is the same thing as a 1-year term policy, plus, if (x)
survives one year, a whole life policy starting at age x + 1.
Hopefully, the pattern of conversion from continuous (payable at moment of death) to curtate
(payable at end of year of death) insurances can be seen from these two examples. For the
rest, well just list the expression for the APV and a formula. Note that Equation 4.3.13 in
the text has a typo, there should be no bar over the A. It should be
A
x:n
=
n1

0
v
k+1
k
p
x
q
x+k
+v
n
n
p
x
.
More insurances follow:
Name: APV
Increasing n-year term (IA)
1
x:n
Decreasing n-year term (DA)
1
x:n
Increasing Whole Life (IA)
x
m-year deferred n-year term
m|n
A
x
Table 4.3.1 is useful for learning these types of insurance. Remember, its the same idea as for
the payable at moment of death products. Except that now time is indicated by an integer
(k) rather than a continuous measure (t).
The list of recursion equations that begins on Page 119 of the text is useful and likely to
be tested one way or another. It is really unfortunate that the authors decided to confuse
matters by using (y x) instead of n for the number of years under the brackets! We have
already discussed recursions like (a), (b), and (c), so we will look at (d), (f), and(g).
I would rather write them as
m|n
A
x
= 0 +vp
x (m1)|n
A
x+1
(d)
(DA)
1
x:n
= nvq
x
+vp
x
(DA)
1
x+1:n1
(e)
(IA)
x
= A
x
+vp
x
(IA)
x+1
(f)
For each of these, you should convince yourself why they are logically correct. Recognize the
following similarity among the formulas. In each case,
PV of benet at age x = b
1
vq
x
+vp
x
(PV of benets at age x + 1).
EXAMPLE:
You are given: A
1
x:30
= 0.1, A
x+30
= 0.5, and A
x:30
= 0.7. Find the value of A
x
.
SOLUTION:
Arch MLC, Fall 2007 c _Yufeng Guo 74
www.archactuarial.com www.guo.coursehost.com 75
A whole life policy is equal to a 30-year term policy plus (if you survive) a whole
life policy starting at age x + 30, so
A
x
= A
1
x:30
+v
30
30
p
x
A
x+30
.
We have everything we need in this equation except for v
30
30
p
x
, which is just
A
x:
1
30
. But
A
x:30
= A
1
x:30
+A
x:
1
30
,
so A
x:
1
30
= 0.6.
A
x
= 0.1 + (0.6)(0.5) = 0.4.
EXAMPLE:
You are given:
i = 0.06
p
x
= 0.95
A
x
= 0.6.
Find the value of A
x+1
.
SOLUTION:
A
x
= A
1
x:1
+v p
x
A
x+1
= vq
x
+v p
x
A
x+1
.
v =
1
1.06
= 0.9434, and q
x
= 0.05
0.6 = (0.9434)(0.05) + (0.9434)(0.95)A
x+1
A
x+1
= 0.617
Note that A
x+1
> A
x
, as we would expect.
EXAMPLE: DeMoivres Law
Mortality is according to DeMoivres Law with = 80.
(A) Assuming i = 0.06, nd A
30:10
;
(B) Assuming i = 0, nd (IA)
30
.
SOLUTION:
Arch MLC, Fall 2007 c _Yufeng Guo 75
www.archactuarial.com www.guo.coursehost.com 76
(A)
A
x:n
= A
1
x:n
+A
x:
1
n
=
n1

0
_
v
k+1
k
p
x
q
x+k
_
+v
n
n
p
x
.
Under DeMoivres law,
k
p
x
=
x k
x
and
q
x+k
=
1
x k
=
1
# of yrs left to
.
So the sum reduces to
A
x:n
=
n1

0
_
v
k+1
1
x
_
+v
n
x n
x
A
30:10
=
1
50
9

0
_
1
1.06
_
k+1
+
_
1
1.06
_
10
_
40
50
_
= 0.02
10

1
_
1
1.06
_
k
+ 0.4467.
Now we use a fact for geometric series (this could come up on the test!) that
says, for r ,= 1,
n

1
(r
k
) =
r r
n+1
1 r
.

10

1
_
1
1.06
_
k
=
(0.9434) (0.9434)
11
1 (0.9434)
= 7.36.
A
30:10
= 0.02(7.36) + 0.4467 = 0.5939.
(B) This one is a little simpler than Part (A) because of the i = 0 assumption.
(IA)
30
=
49

0
(k + 1)v
k
k
p
x
q
x+k
Since i = 0, the vs drop out and
k
p
x
q
x+k
=
1
x
as in part (A). So
(IA)
30
=
1
50
49

0
(k + 1) =
1
50
50

1
k =
(50)(51)
2(50)
= 25.5
Whenever you are reading a problem and you see the assumption i = 0, it
could mean that they are simplifying some really messy computations for
you.
Arch MLC, Fall 2007 c _Yufeng Guo 76
www.archactuarial.com www.guo.coursehost.com 77
4.4 Relationships between Insurances Payable at the Moment
of death and the End of the Year of Death
Option A reference: Actuarial Mathematics Chapter 4.4
Option B reference: Models for Quantifying Risk Chapter 9.5
The lesson of this section is that there is NOT much of a relationship between continuous
insurance and curtate insurance unless you assume the Uniform Distribution of Deaths
(UDD) holds in the year of death. We saw UDD back in Chapter 3 as linear interpolation.
On the exam, SOA tends to describe this assumption as UDD.
Under UDD, the following key identity holds:
A
x
=
i

A
x
.
To remember that the multiplier is
i

, not

i
, remember that i > (the annual interest rate
should be greater than the instant interest rate)
1
and that A
x
> A
x
(in A
x
the death benet
is paid sooner). It is also true under UDD that
A
(m)
x
=
i
i
(m)
A
x
where A
(m)
x
is the APV of a whole life insurance that pays the death benet and the end of
the m-th of a year of death. (So A
(12)
x
is the APV of a whole life insurance that pays the
benet at the end of the month of death.) For completeness, well list some others:
A
1
x:n
=
i

A
1
x:n
(IA)
1
x:n
=
i

(IA)
1
x:n
Note that we have never used the symbol A
x:
1
n
. This is because a benet of 1, payable upon
survival for n years has the same value whether things are continuous or curtate. Therefore
A
x:
1
n
= A
x:
1
n
. This creates a problem since it means that
A
x:n
,=
i

A
x:n
.
Instead, we use
A
x:n
=A
1
x:n
+A
x:
1
n
=
i

A
1
x:n
+A
x:
1
n
.
EXAMPLE:
You are given:
1
1 + i = e

= 1 + +
1
2

2
+
1
3

3
+ .... This gives us i = +
1
2

2
+
1
3

3
+ .... So i > .
Arch MLC, Fall 2007 c _Yufeng Guo 77
www.archactuarial.com www.guo.coursehost.com 78
v
20
= 0.5

20
p
x
= 0.8
A
x
= 0.4
A
x+20
= 0.7
Deaths are uniformly distributed over the year of death.
Find A) A
x
, and B) A
x:20
.
SOLUTION:
(A)
A
x
=
i

A
x
.
v
20
= 0.5 i = 0.0353 = 0.0347.
A
x
=
0.0353
0.0347
(0.4) = 0.407.
(B) This one is a little harder.
A
x:20
=A
1
x:20
+A
x:
1
20
=
i

A
1
x:20
+v
20
20
p
x
= (1.0173)A
1
x:20
+ 0.4.
To nd A
1
x:20
, we use the fact that
A
x
= A
1
x:20
+v
20
20
p
x
A
x+20
.
A
1
x:20
= 0.4 (0.4)(0.7) = 0.12.
A
x:20
= (1.0173)(0.12) + 0.4 = 0.522.
EXAMPLE:
Step 1: Go to Page 123 of the text.
Step 2: Cover up the solution to Example 4.4.1!
Step 3: Work the example.
This is a very realistic example of the type of question that might show up on exam
day. It requires you to use the rule of moments as well as the UDD connection
between curtate and continuous functions.
Chapter 4 Suggested Problems: 6, 7, 10, 11ac, 14a, 16, 23, 26a
(Solutions are available at archactuarial.com)
Arch MLC, Fall 2007 c _Yufeng Guo 78
www.archactuarial.com www.guo.coursehost.com 79
CHAPTER 4 Formula Summary
CONTINUOUS INSURANCES
n-year term: E(Z) =A
1
x:n
=
_
n
0
v
t
t
p
x

x
(t)dt
Var(Z) =
2
A
1
x:n

_
A
1
x:n
_
2
Whole Life: A
x
= E(Z) =
_

0
v
t
t
p
x

x
(t)dt
Var(Z) =
2
A
x

_
A
x
_
2
If and are constant, then:
A
x
=

+
2
A
x
=

+ (2)
Pure Endowment: A
x:
1
n
= E(Z) = v
n
n
p
x
Var[Z] =
2
A
x:
1
n

_
A
x:
1
n
_
2
Endowment: A
x:n
=A
1
x:n
+A
x:
1
n
A
x:n
=
_
n
0
v
t
t
p
x

x
(t)dt +v
n
n
p
x
m-year deferred whole life:
m|
A
x
= E(Z) =
_

m
v
t
t
p
x

x
(t)dt
m|
A
x
=A
x
A
1
x:m
If and are constant, then:
m|
A
x
=
_

m
v
t
t
p
x

x
(t)dt =
_

m
e
t
e
t
dt =

+
e
m(+)
Arch MLC, Fall 2007 c _Yufeng Guo 79
www.archactuarial.com www.guo.coursehost.com 80
Increasing and Decreasing Insurances:
(IA)
x
=
_

0
t + 1|v
t
t
p
x

x
(t)dt
_
IA
_
x
=
_

0
tv
t
t
p
x

x
(t)dt
(DA)
1
x:n
= E(Z) =
_
n
0
v
T
(n t|)
t
p
x

x
(t)dt
DISCRETE INSURANCES (The usual formulas hold for Var(z).)
n-year term: E(Z) = A
1
x:n
=
n1

0
v
k+1
k
p
x
q
x+k
2
A
1
x:n
=
n1

0
v
2(k+1)
k
p
x
q
x+k
Whole Life: E(Z) = A
x
=

0
v
(k+1)
k
p
x
q
x+k
Endowment: A
x:n
=
n1

0
v
k+1
k
p
x
q
x+k
+v
n
n
p
x
Various Recursions: (Try to understand the logic behind each one.)
A
1
x:n
= vq
x
+vp
x
A
1
x+1:n1
A
x
= vq
x
+vp
x
A
x+1
(DA)
1
x:n
= nvq
x
+vp
x
(DA)
1
x+1:n1
(IA)
x
= A
x
+vp
x
(IA)
x+1
m|n
A
x
= 0 +vp
x (m1)|n
A
x+1
Under UDD:
A
x
=
i

A
x
A
(m)
x
=
i
i
(m)
A
x
A
1
x:n
=
i

A
1
x:n
(IA)
1
x:n
=
i

(IA)
1
x:n
BUT: A
x:n
,=
i

A
x:n
; Instead, we use A
x:n
=A
1
x:n
+A
x:
1
n
=
i

A
1
x:n
+A
x:
1
n
Arch MLC, Fall 2007 c _Yufeng Guo 80
www.archactuarial.com www.guo.coursehost.com 81
Past SOA/CAS Exam Questions:
1. An investment fund is established to provide benets on 400 independent lives age x.
(i) On January 1, 2001, each life is issued a 10-year deferred whole life insurance of
1000, payable at the moment of death.
(ii) Each life is subject to a constant force of mortality of 0.05.
(iii) The force of interest is 0.07.
Calculate the amount needed in the investment fund on January 1, 2001, so that the
probability, as determined by the normal approximation, is 0.95 that the fund will be
sucient to provide these benets.
(A) 55,300 (B) 56,400 (C) 58,500 (D) 59,300 (E) 60,100
Solution:
E(Z) = 1000

+
e
10(+)
= 1000
_
5
12
_
e
1.2
= 125.5
Var(Z) = (1000)
2
_

+ 2
e
10(+2)

_
5
12
_
2
e
2.4
_
= 23, 610.16
E(S) = 400 E(Z) = 50, 200
Var(S) = 400 Var(Z) = 9, 444, 064
0.95 = Pr
_
S E(S)
_
Var(S)

k 50, 200

9, 444, 064
_
k = 1.645
_
9, 444, 064 + 50, 200 = 55, 255
Key: A
Arch MLC, Fall 2007 c _Yufeng Guo 81
www.archactuarial.com www.guo.coursehost.com 82
2. A risky investment with a constant rate of default will pay:
(i) principal and accumulated interest at 16% compounded annually at the end of 20
years if it does not default; and
(ii) zero if it defaults.
A risk-free investment will pay principal and accumulated interest at 10% compounded
annually at the end of 20 years.
The principal amounts of the two investments are equal.
The actuarial present values of the two investments are equal at time zero.
Calculate the median time until default or maturity of the risky investment.
(A) 9 (B) 10 (C) 11 (D) 12 (E) 13
Solution:
The expected values at the end of 20 years are (1.1)
20
and (1.16)
20
e
20
. Actuarial
present values at time zero are (1.1)
20

20
and (1.16)
20
e
20

20
.
So 1.1 = 1.16e

and = log
1.16
1.1
= 0.0531.
S(x
0.5
) = e
x
0.5
= 0.5 x
0.5
=
log 0.5

=
0.6931
0.0531
= 13.05
Key: E
3. A decreasing term life insurance on (80) pays (20 k) at the end of the year of death
if (80)dies in year k + 1, for k = 0, 1, 2, . . . , 19.
(i) i = 0.06
(ii) For a certain mortality table with q
80
= 0.2, the single benet premium for this
insurance is 13.
(iii) For this same mortality table, except that q
80
= 0.1, the single benet premium
for this insurance is P.
Calculate P.
(NOTE: Single Benet Premium means the same thing as Actuarial Present
Value)
(A) 11.1 (B) 11.4 (C) 11.7 (D) 12.0 (E) 12.3
Arch MLC, Fall 2007 c _Yufeng Guo 82
www.archactuarial.com www.guo.coursehost.com 83
Solution:
(DA)
1
80:20
= 20v q
80
+v p
80
_
(DA)
1
81:19
_
For the case when q
80
= 0.2,
13 =
20(0.2)
1.06
+
0.8
1.06
(DA)
1
81:19
(DA)
1
81:19
=
13(1.06) 4
0.8
= 12.225
And for the case when q
80
= 0.1,
(DA)
1
80:20
= 20v(0.1) +v(0.9)(12.225)
=
2 + 0.9(12.225)
1.06
= 12.267
Key: E
4. For a special 3-year term insurance on (x) , you are given:
(i) Z is the present-value random variable for the death benets.
(ii) q
x+k
= 0.02(k + 1) k = 0, 1, 2
(iii) The following death benets, payable at the end of the year of death:
k b
k+1
0 300,000
1 350,000
2 400,000
(iv) i = 0.06
Calculate E(Z).
(A) 36,800 (B) 39,100 (C) 41,400 (D) 43,700 (E) 46,000
SOLUTION:
APV (xs benet) =
2

k=0
v
k+1
b
k+1 k
p
x
q
x+k
= 1000
_
300v(0.02) + 350v
2
(0.98)(0.04) + 400v
3
(0.98)(0.96)(0.06)
_
= 36,829
Key: A
Arch MLC, Fall 2007 c _Yufeng Guo 83
www.archactuarial.com www.guo.coursehost.com 84
5. Lee, age 63, considers the purchase of a single premium whole life insurance of 10,000
with death benet payable at the end of the year of death.
The company calculates benet premiums using:
(i) mortality based on the Illustrative Life Table,
(ii) i = 0.05
The company calculates contract premiums as 112% of benet premiums.
The single contract premium at age 63 is 5233.
Lee decides to delay the purchase for two years and invests the 5233.
Calculate the minimum annual rate of return that the investment must earn to accu-
mulate to an amount equal to the single contract premium at age 65.
(A) 0.030 (B) 0.035 (C) 0.040 (D) 0.045 (E) 0.050
SOLUTION:
10,000 A
63
(1.12) = 5233
A
63
= 0.4672
A
x+1
=
A
x
(1 +i) q
x
p
x
A
64
=
(0.4672)(1.05) 0.01788
1 0.01788
= 0.4813
A
65
=
(0.4813)(1.05) 0.01952
1 0.01952
= 0.4955
Single contract premium at 65 = (1.12)(10,000)(0.4955) = 5550
(1 +i)
2
=
5550
5233
= i =
_
5550
5233
1 = 0.02984
Key A
Arch MLC, Fall 2007 c _Yufeng Guo 84
www.archactuarial.com www.guo.coursehost.com 85
6. For a whole life insurance of 1 on (41) with death benet payable at the end of year of
death, you are given:
(i) i = 0.05
(ii) p
40
= 0.9972
(iii) A
41
A
40
= 0.00822
(iv)
2
A
41

2
A
40
= 0.00433
(v) Z is the present-value random variable for this insurance.
Calculate Var(Z).
(A) 0.023 (B) 0.024 (C) 0.025 (D) 0.026 (E) 0.027
SOLUTION:
Var(Z) =
2
A
41
(A
41
)
2
A
41
A
40
0.00822 = A
41
(vq
40
+vp
40
A
41
)
= A
41

_
0.0028
1.05
+
0.9972
1.05
A
41
_
A
41
= 0.21650
2
A
41

2
A
40
= 0.00433 =
2
A
41

_
v
2
q
40
+v
2
p
40
2
A
41
_
=
2
A
41

_
0.0028
1.05
2
+
0.9972
1.05
2

2
A
41
_

2
A
41
= 0.07193
Var(Z) = 0.07193 (0.2165)
2
= 0.02544
Arch MLC, Fall 2007 c _Yufeng Guo 85
www.archactuarial.com www.guo.coursehost.com 86
7. For an increasing 10-year term insurance, you are given:
(i) b
k+1
= 100, 000(1 +k), k = 0, 1, . . . , 9
(ii) Benets are payable at the end of the year of death.
(iii) Mortality follows the Illustrative Life Table.
(iv) i = 0.06
(v) The single benet premium for this insurance on (41) is 16,736.
Calculate the single benet premium for this insurance on (40).
(A) 12,700
(B) 13,600
(C) 14,500
(D) 15,500
(E) 16,300
SOLUTION:
100, 000(IA)
1
40:10|
= 100, 000p
40
[(IA)
1
41:10|
10
10
9
p
41
q
50
] +A
1
40:10|
(100, 000) [see com-
ment]
= 100, 000
0.99722
1.06
_
0.16736
10
_
8,950,901
9,287,264
_
1.06
10
(0.00592)
_
+ (0.02766 100, 000)
= 15, 513.
Where A
1
40:10|
= A
40

10
E
40
A
50
= 0.16132 (0.53667)(0.24905)
= 0.02766.
Comment: the rst line comes from comparing the benets of the two insurances. At
each of age 40, 41, 42, . . . , 49 (IA)
1
40:10|
provides a death benet 1 greater than (IA)
1
41:10|
.
Hence the A
1
40:10|
term. But (IA)
1
41:10|
provides a death benet at 50 of 10, while
(IA)
1
40:10|
provides 0. Hence a term involving
9|
q
41
=
9
p
41
q
50
. The various s and ps
just get all actuarial present values at age 40.
Key: D
Arch MLC, Fall 2007 c _Yufeng Guo 86
www.archactuarial.com www.guo.coursehost.com 87
8. A group of 1000 lives each age 30 sets up a fund to pay 1000 at the end of the rst year
for each member who dies in the rst year, and 500 at the end of the second year for
each member who dies in the second year. Each member pays into the fund an amount
equal to the single benet premium for a special 2-year term insurance, with:
(i) Benets:
k b
k+1
0 1000
1 500
(ii) Mortality follows the Illustrative Life Table.
(iii) i = 0.06
The actual experience of the fund is as follows:
k Interest Rate Earned Number of Deaths
0 0.070 1
1 0.069 1
Calculate the dierence, at the end of the second year, between the expected size of
the fund as projected at time 0 and the actual fund.
(A) 840
(B) 870
(C) 900
(D) 930
(E) 960
SOLUTION:
A
1
30:2|
= 1000q
30
+ 500
2
1|
q
30
= 1000
_
1
1.06
_
(0.00153) + 500
_
1
1.06
_
2
(0.99847)(0.00161) = 2.15875
Initial fund = 2.15875 1000 participants = 2158.75
Let F
n
denote the size of Fund 1 at the end of year n.
F
1
= 2158.75(1.07) 1000 = 1309.86
F
2
= 1309.86(1.065) 500 = 895.00
Expected size of Fund 2 at end of year 2 = 0 (since the amount paid was the single
benet premium). Dierence is 895.
Key: C
Arch MLC, Fall 2007 c _Yufeng Guo 87
www.archactuarial.com www.guo.coursehost.com 88
9. For a special whole life insurance on (x), payable at the moment of death:
(i)
x
(t) = 0.05, t > 0
(ii) = 0.08
(iii) The death benet at time t is b
t
= e
0.06t
, t > 0.
(iv) Z is the present value random variable for this insurance at issue.
Calculate V ar(Z).
(A) 0.038
(B) 0.041
(C) 0.043
(D) 0.045
(E) 0.048
SOLUTION:
E[Z] =
_

0
b
t
v
t

t
p
x
(x +t)dt =
_

0
e
0.06t
e
0.08t
e
0.05t 1
20
dt
=
1
20
_
100
7
_
_
e
0.07t

0
=
5
7
E[Z
2
] =
_

0
_
b
t
v
t
_
2

t
p
x
(x +t)dt =
_

0
e
0.12t
e
0.16t
e
0.05t 1
20
dt =
1
20
_

0
e
0.09t
dt
=
1
20
_
100
9
_
_
e
0.09t

0
=
5
9
V ar[Z] =
5
9

_
5
7
_
2
= 0.04535.
Key: D
Arch MLC, Fall 2007 c _Yufeng Guo 88
www.archactuarial.com www.guo.coursehost.com 89
10. For a group of individuals all age x, you are given:
(i) 25% are smokers (s); 75% are nonsmokers (ns).
(ii)
k q
s
x+k
q
ns
x+k
0 0.10 0.05
1 0.20 0.10
2 0.30 0.15
(iii) i = 0.02.
Calculate 10, 000A
1
x:2
for an individual chosen at random from this group.
(A) 1690
(B) 1710
(C) 1730
(D) 1750
(E) 1770
SOLUTION:
Let ns = nonsmoker and s = smoker
k = q
(ns)
x+k
p
(ns)
x+k
q
(s)
x+k
p
(s)
x+k
0 .05 0.95 0.10 0.90
1 .10 0.90 0.20 0.80
2 .15 0.85 0.30 0.70
A
1(ns)
x:2
= q
(ns)
x
+
2
p
(ns)
x
q
(ns)
x+1
=
1
1.02
(0.05) +
1
1.02
2
0.95 0.10 = 0.1403
A
1(s)
x:2
= q
(s)
x
+
2
p
(s)
x
q
(s)
x+1
=
1
1.02
(0.10) +
1
1.02
2
0.90 0.20 = 0.2710
A
1
x:2
= weighted average = (0.75)(0.1403) + (0.25)(0.2710) = 0.1730
Key: C
Arch MLC, Fall 2007 c _Yufeng Guo 89
www.archactuarial.com www.guo.coursehost.com 90
11. Z is the present value random variable for a 15-year pure endowment of 1 on (x):
(i) The force of mortality is constant over the 15-year period.
(ii) = 0.9
(iii) Var(Z) = 0.065E[Z]
Calculate q
x
.
(A) 0.020
(B) 0.025
(C) 0.030
(D) 0.035
(E) 0.040
SOLUTION:
Variance =
30

15
p
x

15
q
x
Expected value =
15

15
p
x

30

15
p
x

15
q
x
= 0.065
15

15
p
x

15

15
q
x
= 0.065
15
q
x
= 0.3157
Since is constant
15
q
x
=
_
1 (p
x
)
15
_
(p
x
)
15
= 0.6843
p
x
= 0.975, q
x
= 0.025
Key: B
Arch MLC, Fall 2007 c _Yufeng Guo 90
www.archactuarial.com www.guo.coursehost.com 91
Problems from Pre-2000 SOA-CAS exams
1. For a special whole life insurance on (x), you are given:

x
(t) = , t 0

t
= , t 0
the death benet, payable at the moment of death, is 1 for the rst 10 years and
0.5 thereafter.
The single benet premium is 0.3324.
Z is the present-value random variable at issue of the death benet.
Calculate Var(Z).
(A) less than 0.07
(B) greater than or equal to 0.07 but less than 0.08
(C) greater than or equal to 0.08 but less than 0.09
(D) greater than or equal to 0.09 but less than 0.10
(E) greater than or equal to 0.10
2. You are given:
A
35:1
= 0.9434
A
35
= 0.1300
p
35
= 0.9964
(IA)
35
= 3.71
Calculate (IA)
36
.
(A) 3.81 (B) 3.88 (C) 3.94 (D) 4.01 (E) 4.08
3. You are given:
q
x
= 0.10
q
x+1
= 0.20
i = 0.12
Deaths are uniformly distributed over each year of age.
Calculate
2
A
1
x:2
.
(A) 0.206 (B) 0.209 (C) 0.218 (D) 0.224 (E) 0.232
Arch MLC, Fall 2007 c _Yufeng Guo 91
www.archactuarial.com www.guo.coursehost.com 92
4. You are given:
Deaths are uniformly distributed over each year of age.
i = 0.10
q
x
= 0.05
q
x+1
= 0.08
Calculate A
1
x:2
(A) 0.103 (B) 0.108 (C) 0.111 (D) 0.114 (E) 0.119
5. You are given:
s(40) = 0.500
s(41) = 0.475
i = 0.06
A
41
= 0.54
Deaths are uniformly distributed over each year of age.
Calculate A
40
.
(A) 0.483 (B) 0.517 (C) 0.523 (D) 0.531 (E) 0.645
Arch MLC, Fall 2007 c _Yufeng Guo 92
www.archactuarial.com www.guo.coursehost.com 93
Solutions
1. Key: C
Var(Z) = E[Z
2
] (E[Z])
2
E[Z] =A
x:10
+ 0.5
10|
A
x
=A
x

10
E
x
A
x+10
+ 0.5
_
10|
E
x
A
x+10
_
=

+
e
10
e
10


+
+ 0.5
_
e
10
e
10


+
_
=
1
2

1
4
e
20
= 0.3324 = 0.02 =
For E[Z
2
], we cant use the rule of moments since the benet is not always 1 or zero.
E[Z
2
] =
_
10
0
v
2t
t
p
x
dt +
_

10
1
4
v
2t
t
p
x
dt
= (0.02)
_
10
0
e
0.06t
dt +
0.02
4
_

10
e
0.06t
dt =
1
3
_
e
0.06t
_
10
0

1
12
_
e
0.06t
_

10
=
1
3

3
12
e
0.6
= 0.1961
Var[Z] = 0.1961 (0.3324)
2
= 0.0856
2. Key: A
(IA)
35
= vq
35
+vp
35
(A
36
+ (IA)
36
)
A
35
= vq
35
+vp
35
A
36
(IA)
35
A
35
= vp
35
(IA)
36
(IA)
36
=
[(IA)
35
A
35
]
vp
35
A
35:1
= A
1
35:1
+vp
35
= vq
35
+vp
35
= v = 0.9434
(IA)
36
=
3.71 0.13
(0.9434)(0.9964)
= 3.808
Arch MLC, Fall 2007 c _Yufeng Guo 93
www.archactuarial.com www.guo.coursehost.com 94
3. Key: C
= ln 1.12 = 0.1133287 2 = 0.2266574
The i corresponding to 2 is i = e
2
1 = 0.2544
2
A
1
x:2
=
0.10
1.2544
+
0.9(0.20)
(1.2544)
2
= 0.194113
2
A
1
x:2
=
0.2544
0.2266574
(0.194113) = 0.217872
4. Key: D
A
1
x:2
=
i

A
1
x:2
= ln 1.1 = 0.0953 and v =
1
1.1
= 0.9091
i

A
1
x:2
=
i

_
vq
x
+v
2
p
x
q
x+1
_
=
0.1
0.0953
_
0.9091(0.05) + (0.9091)
2
(0.95)(0.08)
_
= 0.114
5. Key: B
A
41
=
i

A
41
A
41
=

i
A
41
=
ln 1.06
0.06
(0.54) = 0.5244202
p
40
=
s(41)
s(40)
=
475
500
= 0.95 q
40
= 0.05
A
x
= vq
x
+vp
x
A
x+1
A
40
= vq
40
+vp
x
A
41
=
1
1.06
(0.05) +
_
1
1.06
_
0.95 (0.5244202) = 0.517
Arch MLC, Fall 2007 c _Yufeng Guo 94
Chapter 3
ACTUARIAL MATHEMATICS:
CHAPTER 5 LIFE ANNUITIES
Option A reference: Actuarial Mathematics Chapter 5
Option B reference: Models for Quantifying Risk Chapter 10
The previous chapter discussed life insurances which provide payment contingent upon death.
We now turn to a lighter topic annuities. Annuities provide benet payments contingent
upon survival of the insured. There are several dierent annuity variations to learn in this
chapter. Again, the critical items from this chapter are to understand the idea of an annuity
and to learn the notation.
Usually when we think of a life annuity, we think of giving an insurance company a pile of
money, then receiving payments for some specied period of time. That is a classic annuity
you purchase from a nancial institution. We will be thinking of annuities in additional ways.
For example:
1. Consider a 30 year xed mortgage. In a sense, the mortgage company has purchased
from the homeowner an annuity with a monthly payment interval.
2. Consider the premiums you pay after being issued a life insurance policy. The insured
is paying an annuity with annual, semi-annual, or monthly payments to the insurance
company. The insured is exchanging an annuity of premium payments for insurance
coverage.
5.2 Continuous Life Annuities
Option A reference: Actuarial Mathematics Chapter 5.2
Option B reference: Models for Qualifying Risk Chapter 10.1.3
In all of these examples, the payment is a unit benet of 1 per year. It will be easy to
generalize to dierent benet amounts once we have the basic concepts.
Name: Whole Life Annuity
95
www.archactuarial.com www.guo.coursehost.com 96
Description: Provides a payment continuously throughout the life of the annuitant. (Even
though the payment is continuous, the rate of payment is 1 per year.)
Let Y = (Present value of future payments) = a
T
. Then the actuarial present value for
this annuity is represented by a
x
, and
E(Y ) = a
x
=
_

0
a
t
t
p
x
(x +t)dt .
This is the messier formula for a
x
and were going to simplify it. But rst, lets look at
the integrand and convince ourselves that this is the right expression. The rst part a
t
represents the present value of this annuity for a person that lives exactly t years. The
second part
t
p
x
(x + t) represents the probability of living exactly t years and then dying.
Integrating over all possible times t produces the expected value.
The text slips in some new notation at this point. Recall the actuarial present value symbol for
an n-yr pure endowment, A
x:
1
n
. That is the notation for a pure endowment in an insurance
context. The notation for this same idea in an annuity context is
n
E
x
. You need to be equally
comfortable with either symbol. Now we have what we need to simplify the expression for
a
x
. The book shows how to use the chain rule to prove
a
x
=
_

0
v
t
t
p
x
dt
_
=
_

0
t
E
x
dt
_
This is the form we nd easiest to remember (actually the middle expression is the best). For
once the book does a good job describing how to think of this integral so I will just quote
the books explanation of it (page 135 of Bowers):
This integral can be considered as involving a momentary payment of 1dt made
at time t, discounted at interest back to time 0 by multiplying by v
t
and further
multiplied by
t
p
x
to reect the probability that a payment is made at time t.
Basically, as long as you live (
t
p
x
) you will continue to receive a payment of 1dt and for APV,
we have to discount the payment back to time 0. Must have been a dierent person writing
that part of the book!
In general, the APV for a continuous annuity is
_

0
v
t
Pr[payments are being made at time t] [payment rate at time t]dt .
Just as with insurance, there is an natural recursion relationship with annuities:
a
x
= a
x:1
+vp
x
a
x+1
.
In this expression a
x:1
represents the APV of a continuous annuity that pays at the rate of
1 per year for up to a year, or until you die, whichever comes rst. It is called an n-year
temporary life annuity (with n = 1 in this case!). The equation says that a lifetime
continuous annuity on (x) is equal to a 1-year temporary life annuity plus, if you survive,
another life annuity starting at age x + 1. Note that the annuity starting at age x + 1 is
discounted for both interest and survivorship.
Arch MLC, Fall 2007 c _Yufeng Guo 96
www.archactuarial.com www.guo.coursehost.com 97
EXAMPLE:
You are given:

10
p
50
= 0.9; = 0.05
a
60
= 8; a
50
= 10
Find a
50:10
.
SOLUTION:
a
50
= a
50:10
+
10
E
50
a
60
.
10 = a
50:10
+e
10(0.05)
(0.9)(8)
a
50:10
= 5.63.
EXAMPLE: DeMoivres Law
Mortality is subject to DeMoivres Law with = 80 and = 0.05. Find a
30
.
SOLUTION: Well do this with integration by parts.
a
30
=
_
x
0
v
t
t
p
x
dt =
_
50
0
e
t
x t
x
dt
=
1
50
_
50
0
e
0.05t
(50 t) dt.
Integration by Parts:
_
udv = uv
_
v du
u = 50 t du = dt
v = 20e
0.05t
dv = e
0.05t
dt
So the integral equals
1
50
_
20e
0.05t
(50 t) 20
_
50
0
e
0.05t
dt
_
=
20
50
_
e
0.05t
(t 50) + 20e
0.05t
_

50
0
=
20
50
_
0 + 20e
2.5
+ 50 20
_
= 12.66
A quick check is to see that the annuity is denitely less than 1/, the value of a
perpetuity.
Arch MLC, Fall 2007 c _Yufeng Guo 97
www.archactuarial.com www.guo.coursehost.com 98
The most important equation so far(!)
The following relation is key to life contingencies and if you are comfortable with it, you will
go far on the exam! It ties together insurance and annuities.
1 = a
x
+A
x
(5.2.8)
The book derives this using a relation familiar from interest theory. It says that whatever
the future lifetime T of (x),
1 = a
T
+v
T
.
You might remember this formula in the form
a
T
=
1 v
T

from Interest Theory. Along with this formula, two very important facts are used to derive
equation (5.2.8):
1. E[v
T
] =A
x
2. E[a
T
] = a
x
We should put each of these equations into words so you wont forget them. Be familiar with
these and ready to use them without prompting!
1. The expected value of 1 discounted over the total future lifetime of (x) is equal to the
APV of a benet of 1 paid at the death of (x).
2. To me this is just the denition of a
x
.
Dierent forms of equation (5.2.8) will denitely be required on the exam so its a good one
to know backwards, forwards, and all rearranged. The most important form might be
a
x
=
1 A
x

. (Remember This One!)


You might nd this form the easiest to remember because it looks just like
a
T
=
1 v
T

from Interest Theory (And not by coincidence!).


We know that E[a
T
] = a
x
and we can use our new equations to nd Var[a
T
].
Var[a
T
] = Var
_
1 v
T

_
=
1

2
Var
_
1 v
T
_
=
Var[v
T
]

2
And we know from Chapter 4 that Var[v
T
] =
2
A
x
(A
x
)
2
, so
Var[a
T
] =
2
A
x
(A
x
)
2

2
Arch MLC, Fall 2007 c _Yufeng Guo 98
www.archactuarial.com www.guo.coursehost.com 99
EXAMPLE: Constant Force of Mortality
1. Assuming constant force of mortality, nd an expression for a
x
in terms of
and .
2. Assuming constant force of mortality, nd an expression for Var[a
T
].
3. Find Pr
_
a
T
> a
x
_
.
(These are the questions of Example 5.2.1 in the book, I hope our approach is
simpler for you than the texts.)
Arch MLC, Fall 2007 c _Yufeng Guo 99
www.archactuarial.com www.guo.coursehost.com 100
SOLUTION:
1.
a
x
=
1 A
x

=
1

+

=
1
+
2.
Var[a
T
] =
2
A
x
(A
x
)
2

2
=
1

2
_

+ 2

_

+
_
2
_
3. Well use the books solution for this one but try to explain what they are
doing. This kind of question makes an appearance now and then because
it separates prepared students from unprepared students. It is the kind of
question that, if you are not ready for it, can leave you kind of ustered.
Lets start by just rewriting the only part of the expression with a variable
in it.
Pr
_
a
T
> a
x
_
= Pr
_
1 v
T

> a
x
_
Although there are lots of constants, there is still only one variable in this
expression. We will get this variable by itself on one side of the inequality.
= Pr
_
v
T
< 1 a
x
_
= Pr
_
e
T
< 1

+
_
= Pr
_
T < log
_

+
__
= Pr
_
T >
1

log
_

+
__
Now all that stu on the right of the inequality is just some constant a
number; call it t
0
. And Pr[T > t
0
] is just the probability that (x) lives past
time t
0
. We know this probability as
t
0
p
x
and we know from Chapter 3, that
in the presence of constant mortality,
t
0
p
x
= e
t
0
, so
Pr
_
a
T
> a
x
_
= e
t
0
=
_

+
_

(Be sure you can work out the last step.)


I realize that this problem was long with lots of messy algebra, but it is
worth going over since many of the questions of this type are similar. First,
isolate the random variable in the inequality, then look at what you have
and interpret it in terms of
t
p
x
.
Useful notes to remember:
Under constant forces of interest and mortality,
a
x
=
1
+
A
x
=

+
2
A
x
=

2+
Arch MLC, Fall 2007 c _Yufeng Guo 100
www.archactuarial.com www.guo.coursehost.com 101
EXAMPLE: Constant Force of Mortality
You are given: = 0.025, = 0.06.
Find a
x:10
.
SOLUTION:
a
x
= a
x:10
+
10
E
x
a
x+10
Since we have constant force of mortality, both a
x
and a
x+10
are equal to
1
+
=
1
0.085
= 11.76.
Also
10
E
x
= e
0.085(10)
= 0.427.
a
x:10
= a
x

10
E
x
a
x+10
= 6.738.
Name: n-yr Temporary Life Annuity
Description: Payment of benets until the earlier of death or end of n-year period.
Again, Y is the present value of future benets.
Y =
_
a
T
0 T < n
a
n
T n
E(Y ) =
_
n
0
v
t
t
p
x
dt = a
x:n
=
1 A
x:n

One tricky point: notice that the APV of an n-year temporary annuity is not written in
terms of an n-year temporary insurance but is written in terms of an n-year endowment. As
long as the subscripts of the annuity and insurance match, you are OK.
Var(Y ) =
2
A
x:n

_
A
x:n
_
2

2
.
EXAMPLE:
For an n-year temporary annuity, write Var[Y ] solely in terms of annuity symbols.
SOLUTION:
We know that A
x:n
= 1a
x:n
but what about
2
A
x:n
? Remember that it is just
the APV of the same insurance at twice the force of interest. So
2
A
x:n
= 1 (2)
2
a
x:n
where
2
a
x:n
is the n-year temporary life annuity at twice the force of interest. So
our answer is
Var[Y ] =
1 (2)
2
a
x:n

_
1 a
x:n
_
2

2

Arch MLC, Fall 2007 c _Yufeng Guo 101
www.archactuarial.com www.guo.coursehost.com 102
Name: n-yr Deferred Whole Life Annuity
Description: Payment of benets commencing at the end of an n-yr period and continuing
for the annuitants life.
Y =
_
0 0 T < n Nothing until n-yr period is over,
v
n
a
Tn
T n then payments begin.
E(Y ) =
n|
a
x
=
_

n
v
t
t
p
x
dt =
_

n
t
E
x
dt
Note that if you die before age x+n, you get nothing with this annuity. So this is really just
a life annuity starting at age x + n and discounted n years for interest and survivorship. In
other words
n|
a
x
= v
n
n
p
x
a
x+n
=
n
E
x
a
x+n
.
The formula for Var(Y ) is ugly:
Var(Y ) =
2

v
2n
n
p
x
_
a
x+n

2
a
x+n
_

_
n|
a
x
_
2
Dont bother memorizing this formula. If SOA writes a question on this variance formula,
chances are that the problem can be solved using basic principles instead of a memorized
formula.
EXAMPLE:
Simplify the following expression: a
x:n
+
n|
a
x
SOLUTION: This is an n-year temporary annuity plus, if you survive, a life
annuity starting at age x+n. This sounds a lot like a life annuity starting at age
x. So
a
x:n
+
n|
a
x
= a
x
You should remember this one.
Name: n-yr certain and life annuity
Description: Regardless of survival or death, payments will be made for at least n years.
Then, if the annuitant is still alive at x+n, payments will continue for the life of the annuitant.
(Not that it matters for the exam, but this is a very commonly sold product.) The present
value of the benets is
Y =
_
a
n
T n
a
T
T n
The APV is represented by a
x:n
. The bar over the x : n indicates that the annuity
continues until the later of the two events shown (x for death, n for the passing of an n-year
interval). Thinking about it, it is clear that this annuity consists of an n-year annuity certain
(a pure Interest Theory entity) followed by an n-year deferred life annuity. So
a
x:n
= a
n
+
n|
a
x
= a
n
+
n
E
x
a
x+n
Arch MLC, Fall 2007 c _Yufeng Guo 102
www.archactuarial.com www.guo.coursehost.com 103
Now for the variance of an n-year certain and life annuity.
Think about the dierence between an n-year certain and life annuity and an n-year deferred
life annuity. The only dierence is that one pays 1 per year during the rst n years and the
other pays 0 per year for the rst n years. In both cases, the payments for the rst n years
are certain so they dont aect the variance of either product. Therefore the two products
should have the same variance!
Var(Y ) =
2

v
2n
n
p
x
_
a
x+n

2
a
x+n
_

_
n|
a
x
_
2
Again, although it is dicult to remember this formula, it might be useful to remember why
these two products have the same variance.
Accumulation with interest and survivorship:
Recall the following accumulation function from interest theory,
s
n
=
_
n
0
(1 +i)
nt
dt
It represents the accumulated value of a continuous annuity of 1 per year at the end of n
years. Note that in this formula, benets only accumulate with interest. In this course, we
have a similar function that accumulates benets with interest and survivorship.
s
x:n
=
a
x:n
n
E
x
=
_
n
0
1
nt
E
x+t
dt
You might want to bookmark this spot. I remember having to lookup the denition of s
x:n
several times when I was studying for Life Contingencies!
The symbol s
x:n
is a dicult one to describe intuitively. The books explanation at the top
of page 141 is denitely wrong! The quantity they describe is not equal to s
x:n
but is equal
to s
n
. You should memorize the symbol and its denition
1
:
s
x:n
=
a
x:n
n
E
x
Finally, Table 5.2.1 is useful to review
1
The symbol a
x:n
is the net single premium youll charge each of 1000 people at t = 0 to provide each of
them with an annuity of 1 per year until they die, up to n years. You could make the following alternative
payment arrangement: instead of charging each one a net single premium at t = 0, you provide the annuity
to the 1000 people and not charge them for it until n years have elapsed. Of course some of the 1000 will
have died so you will only have survivors to charge. s
x:n
is the net single premium youll charge each of
the survivors at t = n to retroactively fund the benet. From the insureds viewpoint, to buy an n-year life
annuity, each of the 1,000 insureds can send you a check of a
x:n
at t = 0; if they dont want to pay you at
t = 0, each of the survivors can send you a check of s
x:n
at t = n. Each of the two payment methods will
fund the annuity. The only bad thing about collecting s
x:n
at t = n is that an insured can cancel his annuity
policy before t = n and wont pay you anything.
Arch MLC, Fall 2007 c _Yufeng Guo 103
www.archactuarial.com www.guo.coursehost.com 104
5.3 Discrete Life Annuities
Option A reference: Actuarial Mathematics Chapter 5.3
Option B reference: Models for Qualifying Risk Chapter 10.1.1, 10.1.2
Remember that throughout the text, we deal with continuous functions, then their discrete
analogs. The ideas are the same integrals become sums, a few notational changes, etc.
However, there is a new concept when dealing with discrete annuities Due vs. Immediate.
Due signies that payment occurs at the beginning of a period. For example, as you pay
your annual term life insurance premium, your payment on Jan 1, 2001 covers the period
through Dec. 31, 2001 the payment occurred at the beginning of the period. Immediate
annuities pay at the end of the period.
Name: Whole Life Annuity-Due
Description: Pays benet at the beginning of each year the annuitant is alive.
The present value random variable is
Y = a
K+1
=
1 v
K+1
d
where, as in earlier chapters, K is the curtate future lifetime of (x).
E(Y ) = a
x
=

0
v
k
k
p
x
.
Recursion formula: a
x
= 1 +v p
x
a
x+1
(A whole life annuity due for (x) is equal to 1 payable right now plus a whole life annuity
due starting at age x + 1 if (x) survives one year.)
EXAMPLE:
An (unlucky) individual, (20), is subject to the following survival probabilities:
x p
x
20 0.95
21 0.80
22 0.50
23 0.00
Assuming i = 0.06, nd a
20
for this individual.
SOLUTION:
a
x
=

0
v
k
k
p
x
= v
0
0
p
x
+v
1
1
p
x
+
Arch MLC, Fall 2007 c _Yufeng Guo 104
www.archactuarial.com www.guo.coursehost.com 105
We should note that
0
p
x
= 1, since it is the probability of surviving 0 years if you
are alive at age x. We will need
k
p
x
for k = 0, 1, 2, 3, 4. We will use that fact that
2
p
x
= p
x
p
x+1
, etc.
k
k
p
20
0 1
1 0.95
2 0.76
3 0.38
4 0
a
20
= v
0
0
p
x
+v
1
1
p
x
+v
2
2
p
x
+v
3
3
p
x
+v
4
4
p
x
= 1 +
1
1.06
(0.95) +
_
1
1.06
_
2
(0.76) +
_
1
1.06
_
3
(0.38) +
_
1
1.06
_
4
(0)
= 1 + 0.896 + 0.676 + 0.319 + 0 = 2.891.
Just as for continuous annuities, there is a fundamental life-annuity relation for discrete
annuities.
a
x
=
1 A
x
d
Recall that d = (1 v) =
_
1
1
1+i
_
.
Var(Y ) = Var
_
1 v
K+1
d
_
=
1
d
2
Var
_
v
K+1
_
=
2
A
x
(A
x
)
2
d
2
.
EXAMPLE:
You are given:
a
x
= 10

1
E
x
= 0.94
A
1
x:1
= 0.02
Find A
x
.
SOLUTION:
This problem has a
x
and A
x
in it, so there is a good chance it involves the
fundamental relation,
a
x
=
1 A
x
d
.
Arch MLC, Fall 2007 c _Yufeng Guo 105
www.archactuarial.com www.guo.coursehost.com 106
To nd d, we note that
1
E
x
= vp
x
and A
1
x:1
= vq
x
. Since p
x
+q
x
= 1,
v =
1
E
x
+A
1
x:1
= 0.96.
d = 1 v = 0.04.
Therefore,
10 =
1 A
x
0.04
A
x
= 0.6 .
EXAMPLE: Illustrative Life Table
Y represents the present value of a whole life annuity-due to be paid to (35).
According to the illustrative life table at the back of the text, what is the variance
of Y ?
SOLUTION:
Var(Y ) =
2
A
35
(A
35
)
2
d
2
,
and according to the table
A
35
= 0.1287
2
A
35
= 0.03488 d = 1
1
1.06
= 0.0566
Var(Y ) =
0.03488 0.01656
0.0032
= 5.725.
If you get a negative number for the variance, it usually means that you forgot to
square A
x
.
Name: n-yr Temporary Life Annuity-Due (Ill let you work this one out through the following
example. The idea is that you dont have to memorize every detail about every one of these
products, just the general pattern and how to set up the formulas using this pattern and the
particular type of insurance)
Description: Pays a benet for the the annuitants remaining life or an n-yr period,
whichever is shorter.
EXAMPLE:
(You denitely want to cover up the solutions for this one.)
1. Find an expression for Y the present value random variable.
2. Find a sum equal to a
x:n
the APV of this benet.
3. Following the logic of the annuities we have seen so far, write an appropriate
expression for Var(Y ).
4. Write the Fundamental life-annuity relation for this annuity.
SOLUTIONS:
1. Y =
_
a
K+1
0 K < n
a
n
K n Payments stop at n even if still alive.
Arch MLC, Fall 2007 c _Yufeng Guo 106
www.archactuarial.com www.guo.coursehost.com 107
2.
E(Y ) = a
x:n
=
n1

0
v
k
k
p
x
Note the limits of summation, there are n payments in all.
3.
Var(Y ) =
2
A
x:n

_
A
x:n
_
2
d
2
4.
a
x:n
=
1 A
x:n
d
You also get full credit for 1 = d a
x:n
+A
x:n
.
Recursion formula: Same idea as all previous recursions, it just looks more complicated!
a
x:n
= 1 +vp
x
a
x+1:n1
EXAMPLE: Illustrative Life Table
According to the illustrative life table at the back of the text, what is the value
of a
40:25
?
SOLUTION:
a
40
= a
40:25
+
25
E
40
a
65
.
From the table,
14.817 = a
40:25
+
_
1
1.06
_
25
25
p
40
(9.897) = a
40:25
+ (2.306)
l
65
l
40
a
40:25
= 14.817 (2.306)
75,340
93,132
= 12.952.
EXAMPLE:
You are given: A
x:10
= 0.6, i = 0.06,
10
p
x
= 0.8.
Find s
x:10
.
SOLUTION:
s
x:10
=
a
x:10
10
E
x
a
x:10
=
1 A
x:10
d
=
0.4
0.0566
= 7.07.
10
E
x
=
_
1
1.06
_
10

10
p
x
= 0.447.
s
x:10
=
7.07
0.447
= 15.81.
Arch MLC, Fall 2007 c _Yufeng Guo 107
www.archactuarial.com www.guo.coursehost.com 108
If you like, you can try to work out Y and E[Y ] for the next two products. Read the
description; try to answer questions 1 and 2 from the n-year Temporary Life Annuity-due
example above.
Name: n-yr Deferred Whole Life Annuity Due
Description: After an n-yr period with no payments, benets will be received at the begin-
ning of each year for the remainder of the annuitants life.
Y =
_
0 0 K < n
n|
a
K+1n
K n
E[Y ] =
n|
a
x
=

k=n
v
k
k
p
x
= a
x
a
x:n
;
Var[Y ] =
2
d
v
2n
n
p
x
_
a
x+n

2
a
x+n
_
+
2
n|
a
x

_
n|
a
x
_
2
Note that
n|
a
x
=
n
E
x
a
x+n
and
n|
a
x
= a
x
a
x:n
.
Name: n-yr Certain and Life Annuity Due
Description: Payments for an n-yr period regardless of death/survival, with payments
continuing for life of annuitant if they are indeed alive after n-yr period.
Y =
_
a
n
0 K < n
a
K+1
K n
E(Y ) = a
x:n
= a
n
+

k=n
v
k
k
p
x
Notice that
a
x:n
= a
n
+
n|
a
x
.
IMMEDIATE ANNUITIES:
We now turn to the immediate annuities. These are just like annuities due except that
payment is at the end of the year (and we see i a lot more than d).
Name: Whole Life Annuity Immediate
Description: Pays benet at the END of each year the annuitant is alive.
For this annuity, Y = a
K
, and
a
x
= E(Y ) =

k=1
v
k
k
p
x
Arch MLC, Fall 2007 c _Yufeng Guo 108
www.archactuarial.com www.guo.coursehost.com 109
Notice that the summation starts at k = 1; you have to live a year to even get that rst
payment. Now life would be really nice if it were true that a
x
=
1Ax
i
. Unfortunately, the
end of year of death (for A
x
) and the end of last full year of life (for a
x
) dont match up,
so the identity is instead
a
x
=
1 (1 +i)A
x
i
You might think of this as the Unfortunate Fundamental Relation. Try to remember it.
Compare these formulas to those for the Whole Life Annuity Due. Note that the only real
changes are as follows:
(A) The dots above a are gone indicates an annuity immediate,
(B) The annuity period changes from K + 1 to K
(C) Summations are from 1 to rather than 0 to .
These dierences all relate to the time a payment is made. They are pretty easy to see and
not too much trouble to remember. Unfortunately, they are easy to mix up in the heat of
the battle during the exam. Work some problems with these two types of annuities to insure
that you can recognize and analyze them appropriately.
EXAMPLE:
Recall our unlucky individual, (20), subject to the following survival probabilities:
x p
x
20 0.95
21 0.80
22 0.50
23 0.00
Assuming i = 0.06, nd: A) a
20
, B) a
20:2
.
SOLUTION:
(A)
a
x
=

1
v
k
k
p
x
= v
1
1
p
x
+v
1
1
p
x
+
We will need
k
p
x
for k = 1, 2, 3, 4.
k
k
p
20
1 0.95
2 0.76
3 0.38
4 0
a
20
= v
1
1
p
x
+v
2
2
p
x
+v
3
3
p
x
+v
4
4
p
x
Arch MLC, Fall 2007 c _Yufeng Guo 109
www.archactuarial.com www.guo.coursehost.com 110
=
1
1.06
(0.95) +
_
1
1.06
_
2
(0.76) +
_
1
1.06
_
3
(0.38) +
_
1
1.06
_
4
(0)
= 0.896 + 0.676 + 0.319 + 0 = 1.891.
A quicker solution would have been to notice that a
x
= a
x
1.
(B)
a
20:2
= vp
x
+v
2
2
p
x
= 0.896 + 0.676 = 1.572.
5.4 Life Annuities with m-thly Payments
Option A reference: Actuarial Mathematics Chapter 5.4
Option B reference: Models for Qualifying Risk Chapter 10.4
In real life, annuities generally pay more frequently than once per year for example, monthly,
quarterly, etc. This will seem complicated, but if you understand the previous sections of
Chapter 5, there is no new theory here. Payments simply come more frequently.
We know that K is the number of complete years lived. Consider now a new random variable
J which represents the number of complete m-ths of a year lived in the year of death.
Then
J = (T K)m|
where | represents the greatest integer function. (So for example, if we are talking about
months, m = 12, and T = 3.51, the person completed 6 months of life in the year of death
(3.51 3)12| = 6.)
For an m-thly annuity due, there will be payments of 1/m at the beginning of each m-th
of a year that the annuitant is alive. So there will be (mK) m-thly payments made for the
completed years, plus J + 1 payments of 1/m in the year of death. The symbol for the
actuarial present value of this annuity is a
(m)
x
. The present value random variable is
Y =
mK+J

j=0
1
m
v
j/m
.
a pretty ugly expression. The expected value is given by
E(Y ) =

h=0
v
h/m
h/m
p
x
.
Note that the fundamental relation between annuities and insurances holds:
E[Y ] = a
(m)
x
=
1 A
(m)
x
d
(m)
.
Finally, the variance is given, as we would expect, by
Var(Y ) =
2
A
(m)
x

_
A
(m)
x
_
2
_
d
(m)
_
2
.
Arch MLC, Fall 2007 c _Yufeng Guo 110
www.archactuarial.com www.guo.coursehost.com 111
In the formulas above, A
(m)
x
represents a discrete insurance that is payable at the end of
the m-th of a year of death. For example, if m = 12, the death benet is payable at the
end of the month of death. We saw these in Chapter 4 so you may recall that there is a
connection between A
(m)
x
and A
x
that comes up occasionally on the exam. The important
thing to remember is that this relation only holds under UDD
A
(m)
x
=
i
i
(m)
A
x
(UNDER UDD ONLY!)
Relating a
(m)
x
to a
x
is a little trickier. It makes sense that a
(m)
x
< a
x
. Since the m-thly benet
only pays
1
m
per m-th of a year up until you die in the year of death, while the annually
payable annuity pays the whole 1 up front. In fact, you might guess that
a
(2)
x
= a
x

1
4
since, on average you would expect to get one of the two semi-annual payments of
1
2
in the
year of death for the annuity payable semi-annually. It turns out that this answer is very
close to being correct. And in general it is very nearly true that
a
(m)
x
= a
x

m1
2m
.
On most questions the SOA might give you on the exam, this approximation will be plenty
accurate to provide the right answer. The only time it might not suce is if the answer
choices have lots of decimal places in them and are not far dierent. (See Example 5.4.1 in
the text.) In those questions where lots of decimals are in the answers, you will have to resort
to the exact relation,
a
(m)
x
= (m) a
x
(m) .
Where
(m) =
i d
i
(m)
d
(m)
, (m) =
i i
(m)
i
(m)
d
(m)
.
Memorizing these will be a little bit of a pain but will probably be worth it. Especially if you
are asked to nd directly. (However, if you are asked to nd and only one of the answer
choices is close to
1
2
, choose that one! Also, is usually pretty close to 1.)
Finally, just as
a
x:n
= a
x

n
E
x
a
x+n
,
it is also true that
a
(m)
x:n
= a
(m)
x

n
E
x
a
(m)
x+n
.
EXAMPLE: Illustrative Life Table
According to the Illustrative Life Table, what is the value of a
(4)
40
?
Arch MLC, Fall 2007 c _Yufeng Guo 111
www.archactuarial.com www.guo.coursehost.com 112
A) 13.9 B) 14.4 C) 14.9 D) 15.4 E) 15.9
SOLUTION:
Since there are not lots of decimal places in the options, we can be condent that
the answer will be very close to a
40

3
8
= 14.816 0.375 = 14.441. So the correct
choice is B. (Notice that even just approximating with a
40

1
2
would have gotten
you close enough to the correct answer, and the higher m is, the better
1
2
is as an
approximation.)
EXAMPLE: Illustrative Life Table
According to the Illustrative Life Table, what is the value of a
(4)
40
? (Note, the
wording here is exactly the same as the previous example on purpose!)
A) 14.0 B) 14.1 C) 14.2 D) 14.3 E) 14.4
SOLUTION:
OK, these options are close enough that we have to work it out. (Actually the
approximation still works (!) but it is so close that I would be nervous using it.)
a
(4)
40
= (4) a
40
(4)
We could work out the formulas for (4) and (4) but you will be given a sheet
the day of the exam that will give you several values of (m) and (m) and any
that you will need will almost certainly be on that sheet. So I will just tell you
that (4) = 1.00057 and (4) = 0.3889 (very close to
3
8
.) If you want to work
them out, remember that
_
1 +
i
(4)
4
_
4
= 1 +i
_
1
d
(4)
4
_
4
= 1 d
Back to the problem!
(1.00057)(14.8166) 0.3889 = 14.436
The correct choice is E.
EXAMPLE: Illustrative Life Table
You are given
Mortality and interest are as given by the Illustrative Life Table.
Deaths are uniformly distributed over the year of death.
Find the actuarial present value of a whole life insurance that pays 1000 at the
end of the month of death of (40).
Arch MLC, Fall 2007 c _Yufeng Guo 112
www.archactuarial.com www.guo.coursehost.com 113
SOLUTION: We want 1000A
(12)
40
. And since we have UDD, we know that
A
(12)
40
=
i
i
(12)
A
40
=
i
i
(12)
(0.16132).
i = 0.06 and using
_
1 +
i
(12)
12
_
12
= 1 +i,
we obtain i
(12)
= 0.0584. Therefore,
A
(12)
40
=
0.06
0.0584
(0.16132),
and the answer is 165.74.
Finally, the book briey mentions m-thly annuity immediates. The odds are small that they
will come up on the exam since annuities due are so much more natural for premium payment,
mortgage payment, etc. To be safe, you might want to briey review this material before
exam day. A couple of extra suggested problems have been added to give you some familiarity
with immediate annuities.
Chapter 5 Suggested Problems: 1, 6a, 33, 45, 51, 53(age 13 only), 56(age 13
only),
Additional Problem:
(A) Find an expression for a
x
a
x
.
(B) Find an expression for a
x:n
a
x:n
.
(C) i = 0.08, a
66:10
= 6, p
65
= 0.95. Find a
65:10
.
(D) Use the illustrative life table to nd a
30:20
.
Arch MLC, Fall 2007 c _Yufeng Guo 113
www.archactuarial.com www.guo.coursehost.com 114
CHAPTER 5 Formula Summary
Continuous Annuities:
a
x
=
_

0
a
t
t
p
x
(x +t)dt
a
x
= a
x:1
+vp
x
a
x+1
Var[a
T
] =
2
A
x
(A
x
)
2

2
1 = a
x
+A
x
a
x
=
1 A
x

. (Remember This One!)


Assuming constant force of mortality, :
a
x
=
1 A
x

=
1

+

=
1
+
Var[a
T
] =
2
A
x
(A
x
)
2

2
=
1

2
_

+ 2

_

+
_
2
_
Temporary life annuities:
a
x:n
=
_
n
0
v
t
t
p
x
dt =
1 A
x:n

2
A
x:n
= 1 (2)
2
a
x:n
where
2
a
x:n
is the n-year temporary life annuity at twice the force of interest.
Deferred life annuities:
n|
a
x
=
_

n
v
t
t
p
x
dt =
_

n
t
E
x
dt
n|
a
x
= v
n
n
p
x
a
x+n
=
n
E
x
a
x+n
For which, Var(Y ) =
2

v
2n
n
p
x
_
a
x+n

2
a
x+n
_

_
n|
a
x
_
2
a
x:n
+
n|
a
x
= a
x
a
x:n
= a
n
+
n|
a
x
= a
n
+
n
E
x
a
x+n
Accumulation function:
s
x:n
=
a
x:n
n
E
x
=
_
n
0
1
nt
E
x+t
Arch MLC, Fall 2007 c _Yufeng Guo 114
www.archactuarial.com www.guo.coursehost.com 115
Discrete annuities:
a
x
=

0
v
k
k
p
x
Var(Y ) =
2
A
x
(A
x
)
2
d
2
Recursion formula: a
x
= 1 +v p
x
a
x+1
Recall that d = (1 v) =
_
1
1
1+i
_
.
a
x
=
1 A
x
d
, a
x:n
=
1 A
x:n
d
, 1 = d a
x:n
+A
x:n
a
x:n
= 1 +vp
x
a
x+1:n1
n|
a
x
=
n
E
x
a
x+n
,
n|
a
x
= a
x
a
x:n
a
x:n
= a
n
+

k=n
v
k
k
p
x
= a
n
+
n|
a
x
a
x
=
1 (1 +i)A
x
i
m-thly annuities:
a
(m)
x
=
1 A
(m)
x
d
(m)
Var(Y ) =
2
A
(m)
x

_
A
(m)
x
_
2
_
d
(m)
_
2
Under UDD only:
A
(m)
x
=
i
i
(m)
A
x
a
(m)
x
a
x

m1
2m
The exact relation for the annuity is
a
(m)
x
= (m) a
x
(m) .
Where
(m) =
i d
i
(m)
d
(m)
, and (m) =
i i
(m)
i
(m)
d
(m)
.
a
(m)
x:n
= a
(m)
x

n
E
x
a
(m)
x+n
.
Arch MLC, Fall 2007 c _Yufeng Guo 115
www.archactuarial.com www.guo.coursehost.com 116
Past SOA/CAS Exam Questions:
1. An insurance company has agreed to make payments to a worker age x who was injured
at work.
(i) The payments are 150,000 per year, paid annually, starting immediately and con-
tinuing for the remainder of the workers life.
(ii) After the rst 500,000 is paid by the insurance company, the remainder will be
paid by a reinsurance company.
(iii)
t
p
x
=
_
(0.7)
t
, 0 t 5.5
0, 5.5 < t
(iv) i = 0.05
Calculate the actuarial present value of the payments to be made by the reinsurer.
(A) Less than 50,000
(B) At least 50,000, but less than 100,000
(C) At least 100,000, but less than 150,000
(D) At least 150,000, but less than 200,000
(E) At least 200,000
Solution:
If the worker survives for three years, the reinsurance will pay 100,000 at t = 3, and
everything after that. So the actuarial present value of the reinsurers portion of the
claim
= 100,000
_
0.7
1.05
_
3
+ 150,000
_
_
0.7
1.05
_
4
+
_
0.7
1.05
_
5
_
= 79,012
Key: B
2. For a continuous whole life annuity of 1 on (x):
(i) T(x), the future lifetime of (x), follows a constant force of mortality 0.06.
(ii) The force of interest is 0.04.
Calculate Pr( a
T(x)
> a
x
).
(A) 0.40 (B) 0.44 (C) 0.46 (D) 0.48 (E) 0.50
Arch MLC, Fall 2007 c _Yufeng Guo 116
www.archactuarial.com www.guo.coursehost.com 117
Solution:
Pr
_
a
T
> a
x
_
= Pr
_
1
T

> a
x
_
= Pr
_
T >
1

log
_

+
__
=
t
0
p
x
where t
0
=
1

log
_

+
_
= e
t
0
=
_

+
_

=
_
6
10
_3
2
= 0.4648
Key: C
3. For a whole life annuity-due of 1 on (x), payable annually:
(i) q
x
= 0.01
(ii) q
x+1
= 0.05
(iii) i = 0.05
(iv) a
x+1
= 6.951
Calculate the change in the actuarial present value of this annuity-due if p
x+1
is in-
creased by 0.03.
(A) 0.16 (B) 0.17 (C) 0.18 (D) 0.19 (E) 0.20
Solution:
a
x
= 1 +p
x
+
2
p
x
p
x+1
a
x+2
Let y denote the change in p
x+1
.
a(with increase) = 1 +p
x
+
2
p
x
(p
x+1
+y) a
x+2
= a(without) +y
2
p
x
a
x+2
y
2
p
x
a
x+2
= change in actuarial present value
a
x+1
= 6.951 = 1 +p
x+1
a
x+2
= 1 +
1
1.05
(1 0.05) a
x+2
Therefore, a
x+2
= 6.577.
0.03
_
1
1.05
_
2
0.99 6.577 = 0.177
Key: C
Arch MLC, Fall 2007 c _Yufeng Guo 117
www.archactuarial.com www.guo.coursehost.com 118
4. A person age 40 wins 10,000 in the actuarial lottery. Rather than receiving the money
at once, the winner is oered the actuarially equivalent option of receiving an annual
payment of K (at the beginning of each year) guaranteed for 10 years and continuing
thereafter for life.
You are given:
(i) i = 0.04
(ii) A
40
= 0.30
(iii) A
50
= 0.35
(iv) A
1
40:10
= 0.09
Calculate K.
(A) 538 (B) 541 (C) 545 (D) 548 (E) 551
Solution:
The person receives K per year guaranteed for 10 years. Therefore, K a
10
= 8.4353K.
The person received K per years alive starting 10 years from now. So the relevant APV
is
10|
a
40
K.
Hence we have 10,000 = (8.4353 +
10
E
40
a
50
)K.
Derive
10
E
40
:
A
40
= A
1
40:10
+ (
10
E
40
)A
50
10
E
40
=
A
40
A
1
40:10
A
50
=
0.30 0.09
0.35
= 0.60
a
50
=
1 A
50
d
=
1 0.35
0.04
1.04
= 16.90
Plug in values:
10,000 = (8.4353 + (0.60)(16.90))K = 18.5753K
K = 538.35
Key: A
Arch MLC, Fall 2007 c _Yufeng Guo 118
www.archactuarial.com www.guo.coursehost.com 119
5. Y is the present-value random variable for a special 3-year temporary life annuity-due
on (x). You are given:
(i)
t
p
x
= 0.9
t
, t 0
(ii) K is the curtate-future-lifetime random variable for (x).
(iii) Y =
_

_
1.00, K = 0
1.87, K = 1
2.72, K = 2, 3, . . .
Calculate Var(Y ).
(A) 0.19 (B) 0.30 (C) 0.37 (D) 0.46 (E) 0.55
Solution:
Pr(K = 0) = 1 p
x
= 0.1
Pr(K = 1) =
1
p
x

2
p
x
= 0.9 0.81 = 0.09
Pr(K > 1) =
2
p
x
= 0.81
E(Y ) = 0.1 1 + 0.09 1.87 + 0.81 2.72 = 2.4715
E(Y
2
) = 0.1 1
2
+ 0.09 1.87
2
+ 0.81 2.72
2
= 6.407
Var(Y ) = 6.407 2.4715
2
= 0.299
Key: B
6. A fund is established to pay annuities to 100 independent lives age x. Each annuitant
will receive 10,000 per year continuously until death. You are given:
(i) = 0.06
(ii)

A
x
= 0.40
(iii)
2

A
x
= 0.25
Calculate the amount (in millions) needed in the fund so that the probability, using the
normal approximation, is 0.90 that the fund will be sucient to provide the payments.
(A) 9.74 (B) 9.96 (C) 10.30 (D) 10.64 (E) 11.10
Arch MLC, Fall 2007 c _Yufeng Guo 119
www.archactuarial.com www.guo.coursehost.com 120
Solution:
E[Y
AGG
] = 100 E[Y ] = 100(10,000)a
x
= 100(10,000)
_
1 A
x

_
= 10,000,000

Y
=
_
Var[Y ] =
_
(10,000)
2
1

2
_
2
A
x
A
2
x
_
=
(10,000)

_
(0.25) (0.16) = 50,000

AGG
=

100
Y
= 10(50,000) = 500,000
0.9 = Pr
_
F E[Y
AGG
]

AGG
> 0
_
1.282 =
F E[Y
AGG
]

AGG
F = 1.282
AGG
+ E[Y
AGG
]
F = 1.282(500,000) + 10,000,000 = 10,641,000
Key: D
7. For a special 30-year deferred annual whole life annuity-due of 1 on (35):
(i) If death occurs during the deferral period, the single benet premium is refunded
without interest at the end of the year of death.
(ii) a
65
= 9.90
(iii) A
35:30
= 0.21
(iv) A
1
35:30
= 0.07
Calculate the single benet premium for this special deferred annuity.
(A) 1.3 (B) 1.4 (C) 1.5 (D) 1.6 (E) 1.7
Solution:
Let denote the single benet premium.
=
30|
a
35
+A
1
35:30
=
30|
a
35
1 A
1
35:30
=
_
A
35:30
A
1
35:30
_
a
65
1 A
1
35:30
=
(0.21 0.07)9.9
1 0.07
=
1.386
0.93
= 1.49
Key: C
Arch MLC, Fall 2007 c _Yufeng Guo 120
www.archactuarial.com www.guo.coursehost.com 121
8. A government creates a fund to pay this years lottery winners.
You are given:
(i) There are 100 winners each age 40.
(ii) Each winner receives payments of 10 per year for life, payable annually, beginning
immediately.
(iii) Mortality follows the Illustrative Life Table.
(iv) The lifetimes are independent.
(v) i = 0.06
(vi) The amount of the fund is determined, using the normal approximation, such that
the probability that the fund is sucient to make all payments is 95%.
Calculate the initial amount of the fund.
(A) 14,800 (B) 14,900 (C) 15,050 (D) 15,150 (E) 15,250
SOLUTION:
Let Y = present value random variable for payments on one life.
S =

Y = present value random variable for all payments.
E[Y ] = 10 a
40
= 148.166
Var[Y ] = 10
2
_
2
A
40
A
2
40
_
d
2
= 100(0.04863 0.16132
2
)(1.06/0.06)
2
= 705.55
E[S] = 100E[Y ] = 14,816.6
Var[S] = 100 Var[Y ] = 70,555 Std Dev [S] =

70555 = 265.62
By normal approximation, need
E[S] + 1.645 Standard Deviations = 14,816.6 + (1.645)(265.62) = 15,254
Key: E
9. For a continuous whole life annuity of 1 on (x):
(i) The force of interest and force of mortality are constant and equal.
(ii) a
x
= 12.50
Calculate the standard deviation of a
T(x)
.
(A) 1.67 (B) 2.50 (C) 2.89 (D) 6.25 (E) 7.22
Arch MLC, Fall 2007 c _Yufeng Guo 121
www.archactuarial.com www.guo.coursehost.com 122
SOLUTION:
12.50 = a
x
=
1
+
+ = 0.08 = = 0.04
A
x
=

+
= 0.5
2
A
x
=

+ 2
=
1
3
Var
_
a
T
_
=
2
A
x
A
2
x

2
=
1
3

1
4
0.0016
= 52.083
S.D. =

52.083 = 7.217 Key: E


10. For a group of individuals all age x, you are given:
(i) 30% are smokers and 70% are non-smokers.
(ii) The constant force of mortality for smokers is 0.06.
(iii) The constant force of mortality for non-smokers is 0.03.
(iv) = 0.08
Calculate Var
_
a
T(x)
_
for an individual chosen at random from this group.
(A) 13.0 (B) 13.3 (C)13.8 (D) 14.1 (E) 14.6
SOLUTION:
A
x
= E
_
v
T(x)
_
= E
_
v
T(x)
[NS
_
Prob(NS) + E
_
v
T(x)
[S
_
Prob(S)
=
_
0.03
0.03 + 0.08
_
0.70 +
_
0.6
0.06 + 0.08
_
0.30 = 0.3195
Similarly,
2
A
x
=
_
0.03
0.03 + 0.16
_
0.70 +
_
0.06
0.06 + 0.16
_
0.30 = 0.1923
Var
_
a
T(x)
_
=
2
A
x
A
2
x

2
=
0.1923 0.3195
2
0.08
2
= 14.1
Key: D
Arch MLC, Fall 2007 c _Yufeng Guo 122
www.archactuarial.com www.guo.coursehost.com 123
11. You are given:
(i) A
x
= 0.28
(ii) A
x+20
= 0.40
(iii) A
x:
1
20
= 0.25
(iv) i = 0.05
Calculate a
x:20
.
(A) 11.0 (B) 11.2 (C) 11.7 (D) 12.0 (E) 12.3
SOLUTION:
(i) a
x:20
= a
x:20
1 +
20
E
x
(ii) a
x:20
=
1 A
x:20
d
(iii) A
x:20
= A
1
x:20
+A
x:
1
20
(iv) A
x
= A
1
x:20
+
20
E
x
A
x+20
0.28 = A
1
x:20
+ (0.25)(0.40) A
1
x:20
= 0.18
Now plug into (iii): A
x:20
= 0.18 + 0.25 = 0.43
Now plug into (ii): a
x:20
=
1 0.43
_
0.05
1.05
_ = 11.97
Now plug into (i): a
x:20
= 11.97 1 + 0.25 = 11.22 Key: B
12. For an annuity payable semiannually, you are given:
(i) Deaths are uniformly distributed over each year of age.
(ii) q
69
= 0.03
(iii) i = 0.06
(iv) 1000A
70
= 530
Calculate a
(2)
69
(A) 8.35 (B) 8.47 (C) 8.59 (D) 8.72 (E) 8.85
Arch MLC, Fall 2007 c _Yufeng Guo 123
www.archactuarial.com www.guo.coursehost.com 124
SOLUTION:
A
70
=

i
A
70
=
ln(1.06)
0.06
(0.53) = 0.5147
a
70
=
1 A
70
d
=
1 0.5147
0.06
1.06
= 8.5736
a
69
= 1 +vp
69
a
70
= 1 +
_
0.97
1.06
_
(8.5736) = 8.8457
a
(2)
69
= (2) a
69
(2) = (1.00021)(8.8457) 0.25739 = 8.5902
Note that the approximation a
(m)
x
a
x

(m1)
2m
works well (closer to the key than to
any other option and is o from the correct answer by less than 0.01). Since m = 2,
this estimate gives
8.8457
1
4
= 8.5957
13. Your company currently oers a whole life annuity product that pays the annuitant
12,000 at the beginning of each year. A member of your product development team
suggests enhancing the product by adding a death benet that will be paid at the end
of the year of death.
Using a discount rate, d, of 8%, calculate the death benet that minimizes the variance
of the present value random variable of the new product.
(A) 0 (B) 50,000 (C) 100,000 (D) 150,000 (E) 200,000
SOLUTION:
Annuity Benet: Z
1
= 12,000
1v
K+1
d
K = 0, 1, 2, . . .
Death Benet: Z
2
= Bv
K+1
K = 0, 1, 2, . . .
New Benet: Z = Z
1
+Z
2
= 12,000
1v
K+1
d
+Bv
K+1
=
12,000
d
+
_
B
12,000
d
_
v
K+1
Var(Z) =
_
B
12, 000
d
_
2
Var
_
v
K+1
_
Var(Z) = 0 when B =
12,000
0.08
= 150,000
Arch MLC, Fall 2007 c _Yufeng Guo 124
www.archactuarial.com www.guo.coursehost.com 125
14. You are given:
(i)
x
(t) = 0.03, t 0
(ii) = 0.05
(iii) T(x) is the future lifetime random variable.
(iv) g is the standard deviation of a
T(x)
.
Calculate Pr
_
a
T(x)
> a
x
g
_
.
(A) 0.53
(B) 0.56
(C) 0.63
(D) 0.68
(E) 0.79
SOLUTION:
a
x
=
_

0
e
0.08t
dt = 12.5

A
x
=
_

0
e
0.08t
(0.03)dt = 3/8 = 0.375
2

A
x
=
_

0
e
0.13t
(0.03)dt = 3/13 = 0.23077
( a
T|
) =
_
V ar[ a
T|
] =
_
1

2
_
2
A
x
(

A
x
)
2

=
_
400 [0.23077 (0.375)
2
] = 6.0048
Pr
_
a
T|
> a
x
( a
T|
)
_
= Pr
_
a
T|
> 12.5 6.0048
_
= Pr
_
1
T
0.05
> 6.4952
_
= Pr
_
0.67524 > e
0.05T
_
= Pr
_
T >
ln 0.67524
0.05
_
= Pr [T > 7.85374]
= e
0.037.85374
= 0.79
Key: E
Arch MLC, Fall 2007 c _Yufeng Guo 125
www.archactuarial.com www.guo.coursehost.com 126
15. Your company is competing to sell a life annuity-due with an actuarial present value of
500, 000 to a 50-year old individual.
Based on your companys experience, typical 50-year old annuitants have a complete life
expectancy of 25 years. However, this individual is not as healthy as your companys
typical annuitant, and your medical experts estimate that his complete life expectancy
is only 15 years.
You decide to price the benet using the issue age that produces a complete life ex-
pectancy of 15 years. You also assume:
(i) For typical annuitants of all ages, mortality follows De Moivres Law with the
same limiting age, .
(ii) i = 0.06
Calculate the annual benet that your company can oer to this individual.
(A) 38, 000
(B) 41, 000
(C) 46, 000
(D) 49, 000
(E) 52, 000
SOLUTION:
For De Moivres Law:

e
x
=
x
2
k|
q
x
=
1
x
A
x
=
x1

k=b

k+1
k|
q
x
=
1
x
x1

k=b

k+1
A
x
=
a
x
x
a
x
=
1Ax
d

e
50
= 25 = 100 for typical annuitants
e
y
= 15 y = Assumed age = 70
A
70
=
a
30
30
= 0.45883
a
70
= 9.5607
500000 = b a
70
b = 52, 297
Key: E
Arch MLC, Fall 2007 c _Yufeng Guo 126
www.archactuarial.com www.guo.coursehost.com 127
16. At interest rate i:
(i) a
x
= 5.6
(ii) The actuarial present value of a 2-year certain and life annuity-due of 1 on (x) is
a
x:2
= 5.6459.
(iii) e
x
= 8.83
(iv) e
x+1
= 8.29
Calculate i.
(A) 0.077
(B) 0.079
(C) 0.081
(D) 0.083
(E) 0.084
SOLUTION:
e
x
= p
x
+p
x
e
x+1
p
x
=
ex
1+e
x+1
=
8.83
9.29
= 0.95048
a
x
= 1 +p
x
+
2

2
p
x
+. . .
a
x:2
= 1 + +
2

2
p
x
+. . .
a
x:2
a
x
= q
x
= 5.6459 5.60 = 0.0459
(1 0.95048) = 0.0459
= 0.9269
i =
1

1 = 0.0789
Key: B
Arch MLC, Fall 2007 c _Yufeng Guo 127
www.archactuarial.com www.guo.coursehost.com 128
17. For a special 3-year temporary life annuity-due on (x), you are given:
(i)
t Annuity Payment p
x+t
0 15 0.95
1 20 0.90
2 25 0.85
(ii) i = 0.06
Calculate the variance of the present value random variable for this annuity.
(A) 91
(B) 102
(C) 114
(D) 127
(E) 139
SOLUTION:
Event Prob Present Value
x = 0 (0.05) 15
x = 1 (0.95)(0.10) = 0.095 15 + 20/1.06 = 33.87
x = 2 (0.95)(0.90) = 0.855 15 + 20/1.06 + 25/1.06
2
= 56.12
E[X] = (0.05)(15) + (0.095)(33.87) + (0.855)(56.12) = 51.95
E[X
2
] = (0.05)(15)
2
+ (0.095)(33.87)
2
+ (0.855)(56.12)
2
= 2813.01
V ar[X] = E(X
2
) E(X)
2
= 2813.01 (51.95)
2
= 114.2
Key: C
Arch MLC, Fall 2007 c _Yufeng Guo 128
www.archactuarial.com www.guo.coursehost.com 129
Problems from Pre-2000 SOA-CAS exams
1. For a 3-year temporary life annuity-due on (30), you are given:
s(x) = 1
x
80
, 0 x 80
i = 0.05
Y =
_
a
K+1
, K = 0, 1, 2
a
3
, K = 3, 4, 5, ...
Calculate Var(Y ).
(A) 0.08 (B) 0.29 (C) 0.36 (D) 0.60 (E) 0.93
2. You are given:
a
(4)

= 17.287
A
x
= 0.1025
Deaths are uniformly distributed over each year of age.
Calculate a
(4)
x
.
(A) 15.48 (B) 15.51 (C) 15.75 (D) 15.82 (E) 15.86
3. You are given:
T(x) is the random variable for the future lifetime of (x).

x+t
= , t 0 is the constant force of mortality.
=
Calculate Var
_
a
T
_
.
(A)
1
12
(B) + (C)
1
6
(D)
1

2
_

4 + 1
_
(E)
_
1
6
_
2
Arch MLC, Fall 2007 c _Yufeng Guo 129
www.archactuarial.com www.guo.coursehost.com 130
Use the following information for the rest of the questions:
For (x), you are given:
T(x) is the random variable for the future lifetime of (x).

x+t
= 0.04, t 0
= 0.06
4. Determine
d
dn
(
n
E
x
)
(A) 0.1 ne
0.1n
(B) 0.1 e
0.1n
(C) 0.1 e
0.1
(D) 0.1 e
0.1n
(E) 0.1 ne
0.1n
5. Calculate the standard deviation of a
T
.
(A) 5.0 (B) 6.0 (C) 6.5 (D) 25.0 (E) 42.3
6. Determine a
x:n
.
(A) 10
_
1 e
0.1
_
(B) 10
_
1 e
0.1n
_
(C) 10
_
1 +e
0.1n
_
(D) 10
_
1 0.67 e
0.1n
_
(E) 10
_
1 + 0.67 e
0.1n
_
7. Determine
d
dx
_
a
x:n
_
.
(A) 2 e
0.1n
(B) 0.33 e
0.1n
(C) 0 (D) 0.33 e
0.1n
(E) 2 e
0.1n
Arch MLC, Fall 2007 c _Yufeng Guo 130
www.archactuarial.com www.guo.coursehost.com 131
Solutions to Pre-2000 Problems: Chapter 5
1. Key: A This is a Demoivres Law problem. Death is uniformly distributed evenly over
the next 50 years. Lets do this the way we might do it in a Statistics course. Here is
the distribution for Y
y P(Y = y)
a
1
= 1 0.02 (only if (30) dies in rst year)
a
2
= 1 +v = 1.952 0.02 (only if (30) dies in 2nd year)
a
3
= 1 +v +v
2
= 2.859 0.96
E[Y ] = (0.02) 1 + (0.02)(1.952) + (0.96)(2.859) = 2.804
E[Y
2
] = (0.02) 1
2
+ (0.02)(1.952)
2
+ (0.96)(2.859)
2
= 7.943
Var[Y ] = 7.943 (2.804)
2
= 0.0806
2. Key: A
a
(4)

=
1
d
(4)
= 17.287 d
(4)
= 0.05785
1 +i =
_
1
1
d
(4)
4
_
4
= 1.06 i = 0.06
Under UDD,
A
(4)
x
=
i
i
(4)
A
x
i
(4)
= 4
_
(1 +i)
0.25
1
_
= 0.0587 A
(4)
x
= 0.10477
a
(4)
x
=
1 A
(4)
x
d
(4)
= 15.476
3. Key: A
Var[a
T
] =
2
A
x
(A
x
)
2

2
This is constant force, so
A
x
=

+
=
1
2
and
2
A
x
=

+ 2
=
1
3
Var[a
T
] =
1
3

1
4

2
=
1
12
2
=
1
12
Arch MLC, Fall 2007 c _Yufeng Guo 131
www.archactuarial.com www.guo.coursehost.com 132
4. Key: B
n
E
x
= v
n
n
p
x
= e
n
e
n
= e
0.1n
d
dn
n
E
x
= 0.1e
0.1n
5. Key: A
A
x
=

+
= 0.4
2
A
x
=

+ 2
= 0.25
Var[a
T
] =
2
A
x
(A
x
)
2

2
=
0.25 0.16
0.06
2
= 25
So the standard deviation is 5.
6. Key: B
a
x:n
= a
x

n
E
x
a
x+n
=
1
+
e
0.1n
1
+
= 10 10e
0.1n
7. Key: C
d
dx
a
x:n
=
d
dx
(10 10e
0.1n
) = 0
There is no x in the expression for a
x:n
. You could also answer this one intuitively.
Since constant force applies, the value of this annuity has to be the same at every age.
Therefore the derivative with respect to age must be zero.
Arch MLC, Fall 2007 c _Yufeng Guo 132
Chapter 4
ACTUARIAL MATHEMATICS:
CHAPTER 6 BENEFIT
PREMIUMS
Option A reference: Actuarial Mathematics Chapter 6
Option B reference: Models for Quantifying Risk Chapter 11
Consider a simple market with yourself as an insurer. You would like to sell benets to
customers like the death benets and endowments we learned about in Chapter 4. For
oering this coverage, you will receive a stream of premiums. This stream of premiums will
be exactly like one of the annuities that we studied in Chapter 5. Also, issues of prot and
expenses are not dealt with in this chapter. We will be assuming that the APV of the benet
exactly equals the APV of the annuity of contract premiums used to purchase the benet.
Even if the expected values of the benet and the contract premiums are the same, you as
the insurer are still facing the risk of a loss. Suppose, for example, you sell a discrete death
benet of 1000 in return for annual premium payments of 100 per year. Although this may
be extremely protable, the insured may die one month after issue leaving you with a huge
loss.
To analyze this situation, we will look the Loss function:
Loss = PV of Benets PV of Premiums.
In this expression, the loss and PVs are random variables (not expected values) whose values
will depend on when death actually occurs.
Suppose you as the insurer want your expected inows to equal your expected outows, so
that:
E(Loss) = E(PV of Benets) E(PV of Premiums) = 0.
In other words, you want E(PV of Benets) = E(PV of Payments). Given a set of benets,
the premium that satises this equation is called the benet premium. The equation itself
133
www.archactuarial.com www.guo.coursehost.com 134
is called the equivalence principle. If you keep the equivalence principle in mind, nding
a benet premium can be pretty straightforward.
6.2 Fully Continuous Premiums
Option A reference: Actuarial Mathematics Chapter 6.2
Option B reference: Models for Quantifying Risk Chapter 11.1.2, 11.1.3, 11.1.4
Consider a whole life policy with a benet of 1 payable immediately upon death. For any
continuously paid premium (explained below), P, we have the loss function:
L = v
T
Pa
T
This loss equals the present value of the benet (b
t
v
t
with b
t
= 1) minus the present value of
the stream of premiums. In chapter 5, we considered only annuities with annual payments
of size 1. Pa
T
represents the expected value of a continuous stream of payments of P per
year for life beginning at age x. The premium is simply a dollar amount just like what you
send your auto insurer.
If the Equivalence Principle (E[L] = 0) is used to determine the continuous premiums for
this whole life policy, the premium is denoted
P
_
A
x
_
and is called the equivalence premium. To nd the equivalence premium for this insurance,
we would take expected values of the equation for L above to get
E(L) =A
x
P
_
A
x
_
a
x
= 0.
Rearranging algebraically,
P
_
A
x
_
=
A
x
a
x
.
So, the benet premium based on the equivalence principle is simply the ratio of expected
present value of benets to the expected value of a corresponding annuity. Remember that
and you basically have the most important concept in this chapter.
As an insurer, you will be interested not only in the expected value of the loss on a product
but also in the variance of the Loss Function.
Var(L) = Var
_
v
t
Pa
T
_
= Var
_
v
t
P
1 v
T

_
= Var
_
v
T
_
1 +
P

_
=
_
1 +
P

_
2
Var
_
v
T
_
.
Arch MLC, Fall 2007 c _Yufeng Guo 134
www.archactuarial.com www.guo.coursehost.com 135
But v
T
is just the present value of 1 dollar received at the moment of death (i.e., continuous
whole life insurance), so we can simplify this expression further using the fact from Chapter 4
that
Var
_
v
T
_
=
2
A
x

_
A
x
_
2
.
Var(L) =
_
1 +
P

_
2 _
2
A
x

_
A
x
_
2
_
.
EXAMPLE: Constant Force of Mortality
If the force of interest and force of mortality are constants, and , nd an
expression for the continuous benet premium P for a continuous whole life policy
paying a benet of 1 per year.
SOLUTION:
Under constant interest and mortality assumptions,
A
x
=

+
and a
x
=
1
+
.
The equivalence principle tells us that the APV of future premiums must equal
the APV of future benets, so:
P
_
A
x
_
a
x
=A
x
,
which reduces, after some algebra to
P(A
x
) =
This tells us that the annual premium (paid continuously!) to fund this whole life
policy is equal to per year. This assumes that premiums are paid during the
entire period that the insurance is in force.
This way of nding the benet premium works for all types of insurance you will encounter
on this exam. The key is to match up the Insurance benet with the corresponding annuity.
For example, the annuity stream of premium payments for ten-year term insurance is dierent
than for whole life insurance.
EXAMPLE: Constant Force of Mortality
If, in the previous example, = 0.01, and = 0.04, nd Var(L).
SOLUTION:
Var(L) =
_
1 +
P

_
2 _
2
A
x

_
A
x
_
2
_
Arch MLC, Fall 2007 c _Yufeng Guo 135
www.archactuarial.com www.guo.coursehost.com 136
We need
A
x
= 0.2, a
x
= 20, P(A
x
) = 0.01,
2
A
x
=

+ 2
= 0.11.
Var(L) = [0.11 0.04]
_
1 +
0.01
0.04
_
2
= (0.07)(1.56) = 0.109.
EXAMPLE: Illustrative Life Table
You are given:
Deaths are uniformly distributed over the year of death.
Mortality and interest are as described in the illustrative life table in the
text.
Find the continuous annual premium for a whole life policy on (40) that pays
10,000 at the moment of death.
Arch MLC, Fall 2007 c _Yufeng Guo 136
www.archactuarial.com www.guo.coursehost.com 137
SOLUTION:
We are looking for
10,000P(A
40
) = 10,000
A
40
a
40
.
The illustrative life table only provides values of A
x
, but since we have UDD, we
can use
A
40
=
i

A
40
=
0.06
log (1 + 0.06)
(0.163242) = 0.1661.
a
40
=
1 A
40

= 14.31,
10,000P(A
40
) =
(10,000)(0.1661)
14.31
= 116.06 .
EXAMPLE: Illustrative Life Table
Find the variance of the loss function for the insurance and premium in the
previous example.
SOLUTION:
First we nd the variance as if the benet were 1.
Var(L) =
_
1 +
P

_
2 _
2
A
x

_
A
x
_
2
_
=
_
1 +
0.01161
0.05827
_
2
_
2i +i
2
2

2
A
40
(0.1661)
2
_
= 1.438 [(1.0606)(0.048633) (0.0276)] = 0.0345.
Since, our policy has death benet and premium multiplied by 10,000, the variance
will be (10,000)
2
Var(L). So the variance we want is
(10,000)
2
(0.0345) = 3,450,000.
Arch MLC, Fall 2007 c _Yufeng Guo 137
www.archactuarial.com www.guo.coursehost.com 138
Table 6.2.1 is worth some discussion.
(Part of Table 6.2.1, Page 173 of Bowers et al.)
Fully Continuous Benet Premiums
Loss Components Premium Formula
Plan b
T
v
T
Y
P =
E[b
T
v
T
]
E[Y ]
Whole life
insurance 1 v
T
a
T
P(A
x
) =
A
ax
n-Year term
insurance
1 v
T
0
a
T
, T n
a
n
, T > n
P(A
1
x:n
) =
A
1
x:n
a
x:n
n-Year
endowment
insurance
1 v
T
1 v
n
a
T
, T n
a
n
, T > n
P(A
x:n
) =
A
x:n
a
x:n
h-Payment
whole life
insurance
1 v
T
1 v
T
a
T
, T h
a
h
, T > h
h
P(A
x
) =
Ax
a
x:h
h-Payment, n-year
endowment
insurance
1 v
T
1 v
T
1 v
n
a
T
, T h
a
h
, h < T n
a
h
, T > n
h
P(A
x:n
) =
A
x:n
a
x:h
n-Year pure
endowment
0
1 v
n
a
T
, T n
a
n
, T > n
P(A
x:
1
n
) =
A
x:
1
n
a
x:n
Column 1 is pretty self-explanatory: the name of the type of insurance and payment plan
(more on the payment plans in a minute). Column 2 shows the present value of the benet
that will be paid, and Column 3 shows the present value of the premiums that will pay for
the benets. Finally, the most important column is Column 4, which shows the formula and
notation for the benet premium. You want to be able to cover up Column 4 and reproduce
it quickly looking only at Column 1. Notice that all of the premiums follow the same general
scheme; only the forms of the annuity and the death benet change.
The most important payment scheme shown is the one having h payments. For these plans,
the insured pays premiums for the rst h years that the insurance is in force, at which point
the insurance is paid up and continues in force for the rest of the coverage period without
further premiums.
Arch MLC, Fall 2007 c _Yufeng Guo 138
www.archactuarial.com www.guo.coursehost.com 139
EXAMPLE:
Fred is 50 years old today and is interested in buying some continuously payable
10-pay whole life insurance. That is, he pays his premium continuously for the
next 10 years or until he dies, whichever comes rst. At the moment of death, he
will receive 1000. Using the following assumptions, nd the benet premium for
this insurance.
a
50
= 13.3 , a
60
= 11.1
A
50
= 0.249

10
p
50
= 0.92
= 0.04
SOLUTION:
It looks like there might be too much information here and it is not obvious
from the start how everything is going to t together, so we should begin by
writing down the equivalence principle and then search for the quantities we
need. Having carefully studied Table 6.2.1 above we are able to correctly use the
notation to write:
h
P
_
A
x
_
a
x:h
= 1000A
x
,
10
P
_
A
50
_
a
50:10
= 1000A
50
.
We know that
a
50:10
= a
50

10
E
50
a
60
,
and
10
E
50
= e
(0.04)(10)
10
p
50
.
a
50:10
= 13.3 (0.67032)(0.92)(11.1) = 6.455.
Therefore,
10
P
_
A
50
_
=
249
6.455
= 38.58.
So the premium for this insurance is 38.58 per year, payable continuously.
What does it mean to pay 38.58 per year continuously?
Suppose the annual premium is 38.58. This idea of continuously paid can be thought of
if the insurance company has a direct tap into your checking account. It deducts money at
a continuous rate such that over a years time, 38.58 has been removed from your account.
This is pretty absurd but it ts nicely into the mathematical theory of actuarial mathematics.
Furthermore, some insurances where premiums are collected weekly approach this idea.
So far we have only looked at premiums paid continuously. Most people are more comfortable
with the idea of paying their premiums on a discrete basis, so you can guess whats coming
next ....
Arch MLC, Fall 2007 c _Yufeng Guo 139
www.archactuarial.com www.guo.coursehost.com 140
6.3 Fully Discrete Premiums
Option A reference: Actuarial Mathematics Chapter 6.3
Option B reference: Models for Quantifying Risk Chapter 11.1.1, 11.1.2
These are the discrete analogs of the continuous cases.
Consider a unit whole life policy with benet payable at the end of the year of death. (See
gure)
x x+1 x+2
. . .
x+K x+T x+(K+1)
Benefit payable at
time (K+1)
We have the following loss function:
L = v
K+1
P
x
a
K+1
.
Note: There is no bar over the P. The symbol P
x
denotes a whole life premium paid
annually. a
K+1
is an annuity with payments due at the beginning of each year starting at
policy issue and including the year of death. (See gure)
x x+1 x+2
. . .
x+K x+T x+(K+1)
P
x
P
x
P
x
P
x
P
x
Taking the expected value gives us
E(L) = E
_
v
K+1
_
P
x
E
_
a
K+1
_
.
Just as with continuous premiums, we use the Equivalence Relation, E(L) = 0, to obtain
0 = A
x
P
x
a
x
So, we come back to the basic premium structure:
P
x
=
A
x
a
x
.
Arch MLC, Fall 2007 c _Yufeng Guo 140
www.archactuarial.com www.guo.coursehost.com 141
Once again, the benet premium is just the death benet divided by the appropriate annuity
of premiums. We also have (as before)
Var(L) =
_
1 +
P
d
_
2 _
2
A
x
(A
x
)
2
_
.
There is another form of the variance that might be necessary on the exam.
Var(L) =
2
A
x
(A
x
)
2
(d a
x
)
2
It is not obvious that these two expressions are the same, but you should be able to prove
it using P
x
a
x
= A
x
, and a
x
=
1Ax
d
. Either form of the variance might well be needed on
the exam, and I would memorize both of them if you have time. If you dont have time, just
memorize the rst one and practice the derivation.
This structure can be extended to a variety of discrete insurance products. Table 6.3.1 is an
excellent source for these formulas.
(Part of Table 6.3.1, Page 173 of Bowers et al.)
Fully Discrete Benet Premiums
Loss Components Premium Formula
Plan b
K+1
v
K+1
Y
P =
E[b
K+1
v
K+1
]
E[Y ]
Whole life
insurance 1 v
K+1
a
K+1
, K = 0, 1, 2,
P
x
=
Ax
ax
n-Year term
insurance
1 v
K+1
0
a
K+1
, K = 0, 1, , n 1
a
n
, K = n, n + 1,
P
1
x:n
=
A
1
x:n
a
x:n
n-Year
endowment
insurance
1 v
K+1
1 v
n
a
K+1
, K = 0, 1, , n 1
a
n
, K = n, n + 1,
P
x:n
=
A
x:n
a
x:n
h-Payment
whole life
insurance
1 v
K+1
1 v
K+1
a
K+1
, K = 0, 1, , h 1
a
h
, K = h, h + 1,
h
P
x
=
Ax
a
x:h
h-Payment, n-year
endowment
insurance
1 v
K+1
1 v
K+1
1 v
n
a
K+1
, K = 0, 1, , h 1
a
h
, K = h, , n 1
a
h
, K = n, n + 1,
h
P
x:n
=
A
x:n
a
x:h
n-Year pure
endowment
0
1 v
n
a
K+1
, K = 0, 1, , n 1
a
n
, K +n, n + 1,
P
x:
1
n
=
A
x:
1
n
a
x:n
Arch MLC, Fall 2007 c _Yufeng Guo 141
www.archactuarial.com www.guo.coursehost.com 142
EXAMPLE:
A professional cli diver, age 30, wants to buy a 2-year term life policy with a benet of
100,000 payable at the end of year of death. Interest is i = 0.08, and the mortality for the
diver is shown in the table:
x p
x
30 0.95
31 0.93
32 0.90
1. Find the equivalence premium for this insurance.
2. Find the standard deviation of the loss function.
SOLUTION:
1. P
30
_
a
30:2
_
= A
1
x:2
and P(1 +v p
30
) = v q
30
+v
2
p
30
q
31
.
P =
0.0463 + 0.057
1.8796
= 0.0550 100,000P
30
= 5,500.
2. Since the special variance formulas only hold for endowment insurance and whole life,
we have to use elementary probabilities and the fact that
Var(L) = E[L
2
] (E[L])
2
.
This will work well especially since we already know that E[L] = 0, since that is how
P was chosen.
Event L Pr
dies 1st yr 100,000v 5,500 = 87,093 0.05
dies 2nd yr (100,000)v
2
5,500(1 +v) = 75,141 0.0665
lives 2 yrs 5,500(1 +v) = 10,593 0.8835
note: 0.0665 = (0.95)(0.07), and 0.8835 = (0.95)(0.93).
Var(L) = E[L
2
] = (0.05)(87,093)
2
+ (0.0665)(75,141)
2
+ (0.8835)(10,593)
2
= 845,400,000.
So the standard deviation of the loss function is
_
Var(L) = 29,076.
Semi-Continuous Premiums
So far in this chapter, we have seen two types of insurance plans
1. Death benets are paid at the moment of death and premiums are paid continuously.
2. Death benets are paid at the end of the year of death and premiums are paid annually
at the beginning of the year.
Arch MLC, Fall 2007 c _Yufeng Guo 142
www.archactuarial.com www.guo.coursehost.com 143
The setup that comes closest to reality is a combination of these two: Death benets payable
at the moment of death while premiums are payable at the beginning of the year. To capture
this idea, semicontinuous premiums were developed. Semicontinuous premiums are denoted
P
_
A
x
_
which indicates discrete premiums and continuous benets. As you might expect
P
_
A
x
_
=
A
x
a
x
.
EXAMPLE: You are given:
Deaths are uniformly distributed over the year of death
Interest and mortality are as given by the Illustrative Life Table.
Find P
_
A
65
_
.
SOLUTION:
P
_
A
65
_
=
A
65
a
65
.
UDD A
65
=
i

A
65
=
0.06
log 1.06
(0.4398) = 0.4529 .
P
_
A
65
_
=
0.4529
9.90
= 0.04575.
The 3-premium Principle:
The next few examples deal with a premium concept that frequently makes an appearance
on the exam.
EXAMPLE: (These are just warm-ups to the main point.)
1. What premium, payable annually, is required to provide 25 years of term
insurance to (30) with a death benet of 1 ?
2. What premium, payable annually, is required to both provide 25 years of
term coverage to (30) and provide (30) with a benet of 1 upon attainment
of age 55?
3. What premium, payable annually, is required to provide (30) with a benet
of w upon attainment of age 55?
4. What premium, payable annually for 25 years, will provide (30) with a whole
life policy?
SOLUTION:
1. P
1
30:25
2. P
30:25
3. wP
30:
1
25
4.
25
P
30

Arch MLC, Fall 2007 c _Yufeng Guo 143
www.archactuarial.com www.guo.coursehost.com 144
EXAMPLE:(more warmup)
1. Rank the following in order of size from largest to smallest and justify your
answer in words (no mathematical proofs, please!).
h
P
x
, P
x:h
P
1
x:h
2. Xavier, Yuri, and Zelda each made a deal with an insurance company.
Xavier agreed to pay P
1
30:25
each year for 25 years and received the
corresponding coverage from the company.
Yuri agreed to pay P
30:25
each year for 25 years and received the corre-
sponding coverage from the company.
Zelda agreed to pay
25
P
30
each year for 25 years and received the corre-
sponding coverage from the company.
If each of these three survived to age 55, describe what remained of each
policy at that age.
SOLUTION:
1. P
x:h
>
h
P
x
> P
1
x:h
. To see why, note that all three premiums buy the same
benet for the rst h years and then provide $1, A
x+h
, and $0, respectively,
upon attainment of age x +h.
2. Xavier will have nothing at age 55 (except the satisfaction of knowing
that had he died in the previous 25 years, his beneciary would be
somewhat richer).
Yuri will have $1.
Zelda will have a whole life policy whose value is A
55
.
EXAMPLE: (This one is the main point!)
Find an expression for
P
x:n

n
P
x
P
x:
1
n
.
SOLUTION:
Well approach this one conceptually, which is just how you should approach it
on the exam.
Recall that P
x:n
is the premium required to provide n years of $1 term coverage
followed by a payment of $1 at age x+n should the insured survive to that point.
Similarly, paying a premium of
n
P
x
for n years will provide n years of $1 term
coverage and then will provide the insured with a whole life policy with value
A
x+n
should the insured survive to age x +n.
Arch MLC, Fall 2007 c _Yufeng Guo 144
www.archactuarial.com www.guo.coursehost.com 145
Therefore P
x:n

n
P
x
is the premium required to provide 1 1 = 0 dollars of
term coverage for n years followed by a payment of 1 A
x+n
upon attainment of
age x + n. But this just describes an n-year pure endowment with a payment of
1 A
x+n
, and we know the premium for this is (1 A
x+n
) P
x:
1
n
. Therefore,
P
x:n

n
P
x
= (1 A
x+n
) P
x:
1
n
,
and
P
x:n

n
P
x
P
x:
1
n
= 1 A
x+n
.
Reread this argument a couple of times if necessary and then you should be able
to do the next couple of examples quickly!
EXAMPLE:
1. What is the value of
P
x:n
P
1
x:n
P
x:
1
n
?
2. What is the value of
n
P
x
P
1
x:n
P
x:
1
n
?
SOLUTION:
1. 1 2. A
x+n

6.4 True m-thly Payment Premiums
Option A reference: Actuarial Mathematics Chapter 6.4
Option B reference: Models for Quantifying Risk Chapter 11.4
Sometimes premiums are paid more frequently than annually (say 12, 4, or 2 times per year).
Let P
(m)
x
denote a true level annual benet premium payable in m-thly installments at the
beginning of each m-thly period for a unit of whole life insurance payable at the end of the
year of death. Note that only one m-th of P
(m)
x
is paid at each installment. So if you are
buying a whole-life policy with a death benet of 100, 000 and P
(12)
x
= 120, then you will be
paying 10 each month.
Table 6.4.1 shows notation and formulas for both discrete and continuous cases. Dont be
intimidated! You already know these functions and the basic premium structure.
Just learn the new notation for m-thly payment and youll be set. It is always
P
(m)
#
=
A
#
a
(m)
#
.
Arch MLC, Fall 2007 c _Yufeng Guo 145
www.archactuarial.com www.guo.coursehost.com 146
There is really nothing new here, the premium is the benet divided by the annuity of
payments.
EXAMPLE: Illustrative Life Table
You are given: (2) = 1.00021 and (2) = 0.2574.
Use the Illustrative Life Table to nd P
(2)
_
A
1
40:20
_
.
SOLUTION:
P
(2)
_
A
1
40:20
_
=
A
1
40:20
a
(2)
40:20
.
A
1
40:20
= A
40

20
E
40
A
60
= 0.16132
1
(1.06)
20

l
60
l
40
(0.36913)
= 0.16132 (0.2741)(0.36913) = 0.06013.
a
(2)
40:20
= a
(2)
40

20
E
40
a
(2)
60
,
where
a
(2)
40
= (2) a
40
(2) = (1.00021)(14.817) 0.2574 = 14.56,
a
(2)
60
= (2) a
60
(2) = (1.00021)(11.145) 0.2574 = 10.89.
a
(2)
40:20
= (14.56) (0.2741)(10.89) = 11.57.
P
(2)
_
A
1
40:20
_
=
0.06013
11.57
= 0.005197
So, for example, the biannual payment for this insurance with a benet of 10,000
would be (0.5)(51.97) = 25.99.
On the test, you may be required to relate an m-thly premium to the corresponding annually
payable premium. The connection is always through the insurance provided.
EXAMPLE:
Write
h
P
(m)
(A
x
) as a multiple of
h
P(A
x
).
SOLUTION: Use the fact that both premiums are used to provide A
x
so
h
P
(m)
(A
x
) a
(m)
x:h
=A
x
=
h
P(A
x
) a
x:h
.

h
P
(m)
(A
x
) =
a
x:h
a
(m)
x:h
h
P(A
x
).
Arch MLC, Fall 2007 c _Yufeng Guo 146
www.archactuarial.com www.guo.coursehost.com 147
Percentile Premiums:
This is one more topic that shows up several times in Chapter 6 and has made appearances
on exams in the past.
Denition: L() is the loss function for a given insurance, assuming that the premium charged
is , (not necessarily the equivalence premium). So for a discrete, whole life policy,
L() = v
K+1
a
K+1
.
Before doing the following two examples, it is a good idea to start by doing example 6.3.3 b)
in the text.
EXAMPLE:
Consider a fully continuous whole life insurance on (30). Mortality and interest
are constants = 0.02, = 0.05, respectively.
Find the smallest premium such that the probability of loss is less than 0.5.
SOLUTION:
How big the loss L is depends only on how long you live. The longer you live
the better it is for the insurance company (smaller loss). The way to solve this
problem is to rst nd the age that the insured has probability 0.5 of living
beyond. Then nd the premium that makes the loss for that age equal to zero.
The resulting premium will produce a loss half of the time.
L = v
T
a
T
Assume 0.5 =
t
p
x
= e
t
. This implies
t =
log 0.5
0.02
= 34.66.
The premium that produces a loss of 0 when t = 34.66 is the one for which
0 = v
34.66
(a
34.66
) = e
0.05(34.66)

_
1 e
0.05(34.66)
0.05
_
0 = 0.1768 (16.46)
= 0.0107
EXAMPLE:
Consider a fully discrete whole life insurance on (30) where mortality and interest
are as given by the Illustrative Life Table.
Find the smallest premium such that the probability of a positive loss on the
insurance is less than 0.25.
Arch MLC, Fall 2007 c _Yufeng Guo 147
www.archactuarial.com www.guo.coursehost.com 148
SOLUTION:
Remember, the longer you live, the lower the loss is. So we want to nd t so that
75% of the time, the insured lives longer than t. We need to solve
0.75
k
p
30
.
Since l
30
= 95,014, we need to nd the age a such that l
a
= (0.75)(95014) = 71261.
The correct age is somewhere between 67 and 68. Since losses are larger for earlier
deaths, we play it safe by going with 67, so K = 37. Now the loss function is
given by
L() = v
K+1
a
K+1
=
_
1
1.06
_
38
a
38
.
So we want to solve
0 = 0.1092
1 v
38
d
,
= 0.1092
0.0566
0.8908
= 0.00694
One last note on this chapter. In some later chapters and in many exam questions, the phrase
single benet premium is used in the place of actuarial present value. If you pay for
your insurance all in one premium at the beginning, you should expect to pay the APV (if
no prot is involved!).
Chapter 6 Suggested Problems: 4, 10(long but good practice! Do the 1st and
last rows only), 13, 16, 17, 27, 29, 31, 32
(Solutions at archactuarial.com)
Arch MLC, Fall 2007 c _Yufeng Guo 148
www.archactuarial.com www.guo.coursehost.com 149
CHAPTER 6 Formula Summary
Fully Continuous Equivalence Premium:
P
_
A
x
_
=
A
x
a
x
Var(L) =
_
1 +
P

_
2 _
2
A
x

_
A
x
_
2
_
Var(L) =
2
A
x

_
A
x
_
2
(a
x
)
2
Under constant interest and mortality assumptions,
A
x
=

+
a
x
=
1
+
P(A
x
) =
Fully discrete:
P
x
=
A
x
a
x
Var(L) =
_
1 +
P
d
_
2 _
2
A
x
(A
x
)
2
_
Var(L) =
2
A
x
(A
x
)
2
(d a
x
)
2
Semicontinous:
P
_
A
x
_
=
A
x
a
x
Sample 3-premium relation:
P
x:n

n
P
x
P
x:
1
n
= 1 A
x+n
Be able to reason out this relation and the similar ones.
m-thly premiums:
P
(m)
#
=
A
#
a
(m)
#
.
h-payment insurance:
h
P(A
x
) =
A
x
a
x:h
h
P(A
x:n
) =
A
x:n
a
x:h
h
P
x
=
A
x
a
x:h
h
P
x:n
=
A
x:n
a
x:h
Be familiar with the two big tables in this chapter summarizing the benets and premiums
for various types of continuous and discrete insurance.
Arch MLC, Fall 2007 c _Yufeng Guo 149
www.archactuarial.com www.guo.coursehost.com 150
Past SOA/CAS Exam Questions:
1. The pricing actuary at Company XYZ sets the premium for a fully continuous whole life
insurance of 1000 on (80) using the equivalence principle and the following assumptions:
(i) The force of mortality is 0.15.
(ii) i = 0.06
The pricing actuarys supervisor believes that the Illustrative Life Table with deaths
uniformly distributed over each year of age is a better mortality assumption.
Calculate the insurers expected loss at issue if the premium is not changed and the
supervisor is right.
(A) 124 (B) 26 (C) 0 (D) 37 (E) 220
Solution:
=
1000
_

0
e
(+)t
dt
_

0
e
(+)t
dt
=
1000

+
1
+
= 1000 = 150
Expected loss = 1000A
80
a
80
.
1000A
80
= 1000
i

A
80
= (1.0297)(665.75) = 685.53
a
80
=
1 0.68553

= 5.3969
Expected loss = 685.53 150(5.3969) = 124.
Key: A
2. For a fully discrete whole life insurance of 10,000 on (30):
(i) denotes the annual premium and L() denotes the loss-at-issue random variable
for this insurance.
(ii) Mortality follows the Illustrative Life Table
(iii) i = 0.06
Calculate the lowest premium,

, such that the probability is less than 0.5 that the


loss L(

) is positive.
(A) 34.6 (B) 36.6 (C) 36.8 (D) 39.0 (E) 39.1
Arch MLC, Fall 2007 c _Yufeng Guo 150
www.archactuarial.com www.guo.coursehost.com 151
Solution:
Pr[L(

) > 0] < 0.5 Pr[10,000v


K+1

a
K+1
> 0] < 0.5
From Illustrative Life Table,
47
p
30
= 0.50816 and
48
p
30
= 0.47681
Since L is a decreasing function of K, to have Pr [L(

) > 0] < 0.5 means we must have


L(

) 0 for K 47.
Highest value of L(

) for K 47 is at K = 47.
L(

)[at K = 47] = 10,000v


47+1

a
47+1
= 609.98 16.589

L(

) 0 =(609.98 16.589

) 0 =

>
609.98
16.589
= 36.77 Key: C
3. Company ABC issued a fully discrete three-year term insurance of 1000 on Pat whose
stated age at issue was 30. You are given:
(i)
x q
x
30 0.01
31 0.02
32 0.03
33 0.04
(ii) i = 0.04
(iii) Premiums are determined using the equivalence principle.
During year 3, Company ABC discovers that Pat was really age 31 when the insurance
was issued. Using the equivalence principle, Company ABC adjusts the death benet
to the level death benet it should have been at issue, given the premium charged.
Calculate the adjusted death benet.
(A) 646 (B) 664 (C) 712 (D) 750 (E) 963
Solution:
A
1
30:3
=
0.01
1.04
+
0.99(0.020)
1.04
2
+
(0.99)(0.98)(0.03)
1.04
3
= 0.053796725
a
30:3
= 1 +
0.99
1.04
+
(0.99)(0.98)
1.04
2
= 2.848927515
1000P
1
30:3
= 1000
0.053796725
2.848927515
= 18.88315
A
1
31:3
=
0.02
1.04
+
0.98(0.03)
1.04
2
+
(0.98)(0.97)(0.04)
1.04
3
= 0.080215919
Arch MLC, Fall 2007 c _Yufeng Guo 151
www.archactuarial.com www.guo.coursehost.com 152
a
31:3
= 1 +
0.98
1.04
+
(0.98)(0.97)
1.04
2
= 2.82119
1000P
1
30:3
a
31:3
= BA
1
31:3
53.2792954 = B(0.08215919) B = 664 Key: B
4. For a fully continuous whole life insurance of 1 on (x):
(i) is the benet premium.
(ii) L is the loss-at-issue random variable with the premium equal to .
(iii) L

is the loss-at-issue random variable with the premium equal to 1.25 .


(iv) a
x
= 5.0
(v) = 0.08
(vi) Var(L) = 0.5625
Calculate the sum of the expected value and the standard deviation of L

.
(A) 0.59 (B) 0.71 (C) 0.86 (D) 0.89 (E) 1.01
Solution:
In general Var(L) =
_
1 +
p

_
2
_
2
A
x
A
2
x
_
.
P(A
x
) =
1
a
x
=
1
5
0.08 = 0.12.
Var(L) =
_
1 +
0.12
0.08
_
2 _
2
A
x
A
2
x
_
= 0.5625,
Var(L

) =
_
1 +
5
4
(0.12)
0.08
_
2
_
2
A
x
A
2
x
_
.
Var(L

) =
_
1 +
15
8
_
2
_
1 +
12
8
_
2
(0.5625) = 0.744
E[L

] =A
x
0.15a
x
= 1 a
x
( + 0.15) = 1 5(0.23) = 0.15
E[L

] +
_
Var(L

) = 0.7125 Key: B
Arch MLC, Fall 2007 c _Yufeng Guo 152
www.archactuarial.com www.guo.coursehost.com 153
5. On January 1, 2002, Pat, age 40, purchases a 5-payment, 10-year term insurance of
100,000:
Death benets are payable at the moment of death.
Contract premiums of 4000 are payable annually at the beginning of each year for
5 years.
i = 0.05
L is the loss random variable at time of issue.
Calculate the value of L if Pat dies on June 30, 2004.
(A) 77,100 (B) 80,700 (C) 82,700 (D) 85,900 (E) 88,000
Solution:
0
L = 100,000v
2.5
4,000 a
3
at i = 0.05 = 77,079 Key: A
6. For a special fully discrete 35-payment whole life insurance on (30):
(i) The death benet is 1 for the rst 20 years and is 5 thereafter.
(ii) The initial benet premium paid during the each of the rst 20 years is one fth
of the benet premium paid during each of the 15 subsequent years.
(iii) Mortality follows the Illustrative Life Table.
(iv) i = 0.06
(v) A
30:20
= 0.32307
(vi) a
30:35
= 14.835
Calculate the initial annual benet premium.
(A) 0.010 (B) 0.015 (C) 0.020 (D) 0.025 (E) 0.030
SOLUTION:
Initial Benet Prem =
5A
30
4
_
A
1
30:20
_
5 a
30:35
4 a
30:20
=
5(0.10248) 4(0.02933)
5(14.835) 4(11.959)
=
0.5124 0.11732
74.175 47.836
=
0.39508
26.339
= 0.015
where
A
1
30:20
=
_
A
30:20
A
30:
1
20
_
= 0.32307 0.29374 = 0.02933
Arch MLC, Fall 2007 c _Yufeng Guo 153
www.archactuarial.com www.guo.coursehost.com 154
and
a
30:20
=
1 A
30:20
d
=
1 0.32307
_
0.06
1.06
_ = 11.959
Comment: The numerator could equally have been calculated as
A
30
+ 4
20
E
30
A
50
= 0.10248 + (4)(0.29374)(0.24905) = 0.39510 Key B
7. For a special fully continuous whole life insurance on (x):
(i) The level premium is determined using the equivalence principle.
(ii) Death benets are given by b
t
= (1 +i)
t
where i is the interest rate.
(iii) L is the loss random variable at t = 0 for the insurance.
(iv) T is the future lifetime random variable of (x).
Which of the following expressions is equal to L?
(A)
(v
T
A
x
)
(1 A
x
)
(B) (v
T
A
x
)(1 +A
x
)
(C)
(v
T
A
x
)
(1 +A
x
)
(D) (v
T
A
x
)(1 A
x
)
(E)
(v
T
+A
x
)
(1 +A
x
)
SOLUTION:
Let denote the premium.
L = b
T
v
T
a
T
= (1 +i)
T
v
T
a
T
= 1 a
T
E[L] = 1 a
x
= 0 =
1
a
x
L = 1 a
T
= 1
a
T
a
x
=
a
x
(1 v
T
)
a
x
=
v
T
(1 a
x
)
a
x
=
v
T
A
x
1 A
x
Key A
Arch MLC, Fall 2007 c _Yufeng Guo 154
www.archactuarial.com www.guo.coursehost.com 155
8. For a fully discrete 2-year term insurance of 1 on (x):
(i) 0.95 is the lowest premium such that there is a 0% chance of loss in year 1.
(ii) p
x
= 0.75
(iii) p
x+1
= 0.80
(iv) Z is the random variable for the present value at issue of future benets.
Calculate Var(Z).
(A) 0.15 (B) 0.17 (C) 0.19 (D) 0.21 (E) 0.23
SOLUTION:
v is the lowest premium to ensure a zero % chance of loss in year 1 (The present value
of the payment upon death is v, so you must collect at least v to avoid a loss should
death occur). Thus v = 0.95.
E(Z) = vq
x
+v
2
p
x
q
x+1
= 0.95 0.25 + (0.95)
2
0.75 0.2 = 0.3729
E
_
Z
2
_
= v
2
q
x
+v
4
p
x
q
x+1
= (0.95)
2
0.25 + (0.95)
4
0.75 0.2 = 0.3478
Var(Z) = E
_
Z
2
_
[E(Z)]
2
= 0.3478 (0.3729)
2
= 0.21 Key: D
9. For a special 2-payment whole life insurance on (80):
(i) Premiums of are paid at the beginning of years 1 and 3.
(ii) The death benet is paid at the end of the year of death.
(iii) There is a partial refund of premium feature:
If (80) dies in either year 1 or year 3, the death benet is 1000 +

2
.
Otherwise, the death benet is 1000.
(iv) Mortality follows the Illustrative Life Table.
(v) i = 0.06
Calculate , using the equivalence principle.
(A) 369 (B) 381 (C) 397 (D) 409 (E) 425
Arch MLC, Fall 2007 c _Yufeng Guo 155
www.archactuarial.com www.guo.coursehost.com 156
SOLUTION:

_
1 +
2
p
80
v
2
_
= 1000A
80
+
vq
80
2
+
v
3
2
p
80
q
82
2

_
1 +
0.83910
1.06
2
_
= 665.75 +
_
0.08030
2(1.06)
+
0.83910 0.09561
2(1.06)
3
_
(1.74680) = 665.75 +(0.07156) = 397.41
Where
2
p
80
=
3,284,542
3,914,365
= 0.83910
Or
2
p
80
= (1 0.08030)(1 0.08764) = 0.83910 Key: C
10. For a special 3-year deferred whole life annuity-due on (x):
(i) i = 0.04
(ii) The rst annual payment is 1000.
(iii) Payments in the following years increase by 4% per year.
(iv) There is no death benet during the three year deferral period.
(v) Level benet premiums are payable at the beginning of each of the rst three
years.
(vi) e
x
= 11.05 is the curtate expectation of life for (x).
(vii)
k
k
p
x
1 0.99
2 0.98
3 0.97
Calculate the annual benet premium.
(A) 2625 (B) 2825 (C) 3025 (D) 3225 (E) 3425
SOLUTION:
e
x
= p
x
+
2
p
x
+
3
p
x
+. . . = 11.05
Annuity = v
3
3
p
x
1000 +v
4
4
p
x
1000 (1.04) +. . .
=

k=3
1000(1.04)
k3
v
k
k
p
x
= 1000v
3

k=3
k
p
x
= 1000v
3
(e
x
0.99 0.98) = 1000
_
1
1.04
_
3
9.08 = 8072
Let = benet premium:
_
1 + 0.99v + 0.98v
2
_
= 8072
2.8580 = 8072 = 2824 Key: B
Arch MLC, Fall 2007 c _Yufeng Guo 156
www.archactuarial.com www.guo.coursehost.com 157
11. For a special fully discrete 10-payment whole life insurance on (30) with level annual
benet premium :
(i) The death benet is equal to 1000 plus the refund, without interest, of the benet
premiums paid.
(ii) A
30
= 0.102
(iii)
10|
A
30
= 0.088
(iv) (IA)
1
30:10
= 0.078
(v) a
30:10
= 7.747
Calculate .
(A) 14.9 (B) 15.0 (C) 15.1 (D) 15.2 (E) 15.3
SOLUTION: a
30:10
= 1000A
30
+P(IA)
1
30:10
+ (10)
_
10|
A
30
_
=
1000A
30
a
30:10
(IA)
1
30:10
10
_
10|
A
30
_
=
1000(0.102)
7.747 0.078 10(0.088)
=
102
6.789
= 15.024 Key: B
12. (Well see more or this normal approximation idea in later chapters. The idea is to nd
the mean and variance for all of the lives and then treat the problem as if it is a normal
distribution with that mean and variance.)
Each of 100 independent lives purchase a single premium 5-year deferred whole life
insurance of 10 payable at the moment of death. You are given:
(i) = 0.04
(ii) = 0.06
(iii) F is the aggregate amount the insurer receives from the 100 lives.
Using the normal approximation, calculate F such that the probability the insurer has
sucient funds to pay all claims is 0.95.
(A) 280 (B) 390 (C) 500 (D) 610 (E) 720
Arch MLC, Fall 2007 c _Yufeng Guo 157
www.archactuarial.com www.guo.coursehost.com 158
SOLUTION:
Let Z be the present value random variable for one life.
Let S be the present value random variable for the 100 lives.
E(Z) = 10
_

5
e
t
e
t
dt = 10

+
e
(+)5
= 2.426
E
_
Z
2
_
= 10
2
_

2 +
_
e
(2+)5
= 10
2
_
0.04
0.16
_
_
e
0.8
_
= 11.233
Var(Z) = E
_
Z
2
_
[E(Z)]
2
= 11.233 2.426
2
= 5.348
E(S) = 100 E(Z) = 242.6
Var(S) = 100 Var(Z) = 534.8
F 242.6

534.8
= 1.645 F = 281 Key: A
13. For a whole life insurance of 1000 on (x) with benets payable at the moment of death:
(i)

t
=
_
0.04, 0 < t 10
0.05, 10 < t
(ii)

x
(t) =
_
0.06, 0 < t 10
0.07, 10 < t
Calculate the single benet premium for this insurance.
(A) 379 (B) 411 (C) 444 (D) 519 (E) 594
SOLUTION:
Since this is CFM, the solution is faster if we use the facts that
A
x
=

+
, A
1
x:10
=

+
(1
10
E
x
)
and
10
E
x
= e
10(+)
Then the solution is
1000A
x
= 1000
_
A
1
x:10
+
10
E
x
A
x+10
_
Arch MLC, Fall 2007 c _Yufeng Guo 158
www.archactuarial.com www.guo.coursehost.com 159
= 1000
__
0.06
0.06 + 0.04
_
_
1 e
10(0.06+0.04)
_
+e
10(0.06+0.04)
_
0.07
0.07 + 0.05
__
= 1000
_
(0.6)
_
1 e
1
_
+ 0.5833e
1
_
= 593.86
14. Two actuaries use the same mortality table to price a fully discrete 2-year endowment
insurance of 1000 on (x).
(i) Kevin calculates non-level benet premiums of 608 for the rst year and 350 for
the second year.
(ii) Kira calculates level annual benet premiums of .
(iii) d = 0.05
Calculate .
(A) 482 (B) 489 (C) 497 (D) 508 (E) 517
SOLUTION:
d = 0.05 v = 0.95
From Kevins work:
608 + 350vp
x
= 1000vq
x
+ 1000v
2
p
x
(p
x+1
+q
x+1
)
608 + 350(0.95)p
x
= 1000(0.95)(1 p
x
) + 1000(0.9025)p
x
(1)
608 + 332.5p
x
= 950(1 p
x
) + 902.5p
x
p
x
=
342
380
= 0.9
Now to arrive at Kiras value for 1000P
x:2
:
608 + 350(0.95)(0.9) = 1000P
x:2
[1 + (0.95)(0.9)]
1000P
x:2
=
299.25 + 608
1.855
= 489.08
Arch MLC, Fall 2007 c _Yufeng Guo 159
www.archactuarial.com www.guo.coursehost.com 160
15. Z is the present-value random variable for a whole life insurance of b payable at the
moment of death of (x). You are given:
(i) = 0.04.
(ii)
x
(t) = 0.02, t 0
(iii) The single benet premium for this insurance is equal to Var(Z).
Calculate b.
(A) 2.75
(B) 3.00
(C) 3.25
(D) 3.50
(E) 3.75
SOLUTION:
E[Z] = b

A
x
since constant force

A
x
= /( +)
E(Z) =
b
+
=
b(0.02)
(0.06)
= b/3
V ar[Z] = V ar[b
T
] = b
2
V ar[
T
] = b
2
(
2

A
x


A
2
x
)
= b
2
_

+2

_

+
_
2
_
= b
2
_
2
10

1
9
_
= b
2
_
4
45
_
V ar(Z) = E(Z)
b
2
_
4
45
_
=
b
3
b
_
4
45
_
=
1
3
b = 3.75
Key: E
Arch MLC, Fall 2007 c _Yufeng Guo 160
www.archactuarial.com www.guo.coursehost.com 161
16. For a special 3-year term insurance on (30), you are given:
(i) Premiums are payable semiannually.
(ii) Premiums are payable only in the rst year.
(iii) Benets, payable at the end of the year of death, are:
k b
k+1
0 1000
1 500
2 250
(iv) Mortality follows the Illustrative Life Table.
(v) Deaths are uniformly distributed within each year of age.
(vi) i = 0.06
Calculate the amount of each semiannual benet premium for this insurance.
(A) 1.3
(B) 1.4
(C) 1.5
(D) 1.6
(E) 1.7
SOLUTION:
A
1
30:3|
= 1000q
30
+ 500
2
1|
q
30
+ 250
3
2|
q
30
= 1000
_
1
1.06
_ _
1.53
1000
_
+500
_
1
1.06
_
2
(0.99847)
_
1.61
1000
_
+250
_
1
1.06
_
3
(0.99847) (0.99839)
_
1.70
1000
_
= 1.4434 + 0.71535 + 0.35572 = 2.51447
a
(2)
30:1|
= 1/2 + 1/2
_
1
1.06
_
1/2
(1 (1/2)q
30
) = 1/2 + 1/2 (0.97129) (1 (1/2)(0.00153))
= 1/2 + (1/2)(0.97129)(0.999235)
= 0.985273.
Annualized premium =
2.51447
0.985273
= 2.552
Each semiannual premium =
2.552
2
= 1.28.
Key: A
Arch MLC, Fall 2007 c _Yufeng Guo 161
www.archactuarial.com www.guo.coursehost.com 162
17. Company ABC sets the contract premium for a continuous life annuity of 1 per year
on (x) equal to the single benet premium calculated using:
(i) = 0.03
(ii)
x
(t) = 0.02, t 0
However, a revised mortality assumption reects future mortality improvement and is
given by

x
(t) =
_
0.02 for t 10
0.01 for t > 10
Calculate the expected loss at issue for ABC (using the revised mortality assumption)
as a percentage of the contract premium.
(A) 2%
(B) 8%
(C) 15%
(D) 20%
(E) 23%
SOLUTION:
Let P denote the contract premium.
P = a
x
=
_

0
e
t
e
t
dt =
_

0
e
0.05t
dt = 20
E[L] = a
IMP
x
P
a
IMP
x
=
_
10
0
e
0.03t
e
0.02t
dt +e
0.03(10)
e
0.02(10)
_

0
e
0.03t
e
0.02t
dt
=
1e
0.05
0.05
+
e
0.05
0.04
= 23
E[L] = 23 20 = 3
E[L]
P
=
3
20
= 15%
Key: C
Arch MLC, Fall 2007 c _Yufeng Guo 162
www.archactuarial.com www.guo.coursehost.com 163
18. For a fully continuous whole life insurance of 1 on (x), you are given:
(i) The forces of mortality and interest are constant.
(ii)
2

A
x
= 0.20
(iii)

P(

A
x
) = 0.03
(iv)
0
L is the loss-at-issue random variable based on the benet premium.
Calculate Var(
0
L).
(A) 0.20
(B) 0.21
(C) 0.22
(D) 0.23
(E) 0.24
SOLUTION:

P(A
x
) = = 0.03
2
A
x
= 0.20 =

2+
=
0.03
2+0.03
= 0.06
V ar(
0
L) =
2
Ax(Ax)
2
(a)
2
=
0.20(
1
3
)
2
(
0.06
0.09
)
2
= 0.20
where A =

+
=
0.03
0.09
=
1
3
a =
1
+
=
1
0.09
Key: A
19. For a fully discrete whole life insurance of 100,000 on each of 10,000 lives age 60, you
are given:
(i) The future lifetimes are independent.
(ii) Mortality follows the Illustrative Life Table.
(iii) i = 0.06
(iv) is the premium for each insurance of 100,000.
Using the normal approximation, calculate , such that the probability of a positive
total loss is 1%.
(A) 3340
(B) 3360
(C) 3380
(D) 3390
(E) 3400
Arch MLC, Fall 2007 c _Yufeng Guo 163
www.archactuarial.com www.guo.coursehost.com 164
SOLUTION:
A
60
= 0.36913 d = 0.05660
2
A
60
= 0.17741
and
_
2
A
60
A
2
60
= 0.202862.
Expected Loss on one policy is E(L()) =
_
100, 000 +

d
_
A
60


d
Variance on one policy is V ar(L()) =
_
100, 000 +

d
_
2
_
2
A
60
A
2
60
_
On the 10000 lives,
E[S] = 10, 000E[L()] and V ar[S] = 10, 000V ar[L()]
The is such that 0 E[S]/V ar[S] = 2.326 since (2.326) = 0.99
10, 000
_

d

_
100, 000 +

d
_
A
60
_
100
_
100, 000 +

d
_
_
2
A
60
A
2
60
= 2.326
100
_

d

_
100, 000 +

d
_
0.36913
_
_
100, 000 +

d
_
0.202862
= 2.326
63.087

d
3691300
100, 000 +

d
= 0.4719
63.087

d
3691300 = 47190 + 0.4719

d
=
3691300 + 47190
63.087 0.4719
= 59705.885
= 59706d = 3379
Key: C
Arch MLC, Fall 2007 c _Yufeng Guo 164
www.archactuarial.com www.guo.coursehost.com 165
20. For a special fully discrete 3-year term insurance on (x):
(i)
b
k+1
=
_
0 for k = 0
1, 000(11 k) for k = 1, 2
(ii)
k q
x+k
0 0.200
1 0.100
2 0.097
(iii) i = 0.06
Calculate the level annual benet premium for this insurance.
(A) 518
(B) 549
(C) 638
(D) 732
(E) 799
SOLUTION:
Actuarial present value Benets equals
=
(0.8)(0.1)(10, 000)
1.06
2
+
(0.8)(0.9)(0.097)(9, 000)
1.06
3
= 1,239.75
1,239.75 = P
_
1 +
(0.8)
1.06
2
+
(0.8)(0.9)
1.06
2
_
= P(2.3955)
P = 517.53 518
Key: A
Arch MLC, Fall 2007 c _Yufeng Guo 165
www.archactuarial.com www.guo.coursehost.com 166
Problems from Pre-2000 SOA-CAS exams
1. For a fully continuous whole life insurance of 1 on (x), you are given:
Z is the present-value random variable at issue of the death benet.
L is the loss-at-issue random variable.
Premiums are determined using the equivalence principle.

Var(Z)
Var(L)
= 0.36
a
x
= 10
Calculate P
_
A
x
_
.
(A) 0.03 (B) 0.04 (C) 0.05 (D) 0.06 (E) 0.07
Arch MLC, Fall 2007 c _Yufeng Guo 166
www.archactuarial.com www.guo.coursehost.com 167
2. You are an actuary reviewing a tentative pricing calculation for a fully discrete whole
life insurance of 1000 on (60). The tentative benet premium seems surprisingly high.
You discover that an incorrect value for q
60
was used.
You are given:
i = 0.06
The incorrect value q
60
= 0.10 was used.
Based on that incorrect value, 1000 A
60
= 405.54.
The correct value is q
60
= 0.01.
For all other ages, the correct mortality rates were used.
L

denotes the insurers loss-at-issue random variable, assuming the tentative ben-
et premium is not corrected.
Calculate: E[L

]
(A) -110 (B) -105 (C) -100 (D) -95 (E) -90
3. For a 5-year endowment insurance of 1 on (35), you are given:
The death benet is payable at the moment of death.
Premiums, payable continuously, are determined using the equivalence principle.

35
(t) = 0.05, t 0
= 0.04

t
L is the prospective loss at time t.
Calculate 1000 P
_
A
35:5
_
.
(A) 151 (B) 173 (C) 198 (D) 208 (E) 226
Arch MLC, Fall 2007 c _Yufeng Guo 167
www.archactuarial.com www.guo.coursehost.com 168
Solutions to Pre-2000 Exam Questions: Chapter 6
1. Key: B
Var[Z] =
2
A
x
(A
x
)
2
Var[L] =
_
1 +
P(A
x
)

_
2
_
2
A
x
(A
x
)
2
_
=
_
1 +
P

_
2
Var[Z]

_
1 +
P

_
2
=
1
0.36

_
1 +
P

_
=
5
3
P(A
x
) =
2
3

Also
P(A
x
) =
A
x
a
x
=
1 a
x
a
x

2
3
a
x
= 1 a
x
a
x
=
3
5
P(A
x
) =
1 a
x
a
x
=
0.4
10
= 0.04
2. Key: E
E[L

] = 1000A
60
P a
60
To calculate A and a, we use the corrected q. To get P, we use the incorrect q. First
P:
P = 1000
A
60
a
60
= 1000
A
60
1A
60
d
= 1000
0.40554
10.502127
= 38.615
A
61
is the same before and after the correction:
0.40554 = v(0.1) +v(0.9) A
61
A
61
=
(1.06)(0.40554) 0.1
0.9
= 0.3665
Now for the corrected A and a.
A
60
= vq
60
+vp
60
A
61
=
1
1.06
(0.01) +
1
1.06
(0.99)(0.3665) = 0.3517
a
60
=
1 A
60
d
= 11.45
E[L

] = 1000A
60
P a
60
Where A and a are from the correct q; and P is from the incorrect q.
E[L

] = 1000(0.35175) 38.615(11.45) 90
Arch MLC, Fall 2007 c _Yufeng Guo 168
www.archactuarial.com www.guo.coursehost.com 169
3. Key: D
1000P
_
A
35:5
_
=
1000A
35:5
a
35:5
a
35:5
= a
35

5
E
35
a
40
Since this is constant force, this equals
1
+
e
0.45
1
+
= 11.11 e
0.45
(11.11) = 4.026
A
35:5
= 1 a
35:5
= 0.8390
1000P
_
A
35:5
_
=
839.0
4.026
208
Arch MLC, Fall 2007 c _Yufeng Guo 169
www.archactuarial.com www.guo.coursehost.com 170
Arch MLC, Fall 2007 c _Yufeng Guo 170
Chapter 5
ACTUARIAL MATHEMATICS:
CHAPTER 7 BENEFIT
RESERVES
Option A reference: Actuarial Mathematics Chapter 7
Option B reference: Models for Quantifying Risk Chapter 12
As we calculated premiums in the previous chapter, we depended upon the equivalence prin-
ciple. That is, at the time of policy issue,
E (PV of Future Benets) = E (PV of Future Premiums) .
This is true at issue. But consider a person who is now 95 years old with a 100,000 whole life
policy with annual premiums of 300 that she has been paying every year since age 35. The
expected value of the benets and premiums were equal at issue, but now,
E (PV of Future Benets) >>>> E (PV of Future Premiums) .
Intuitively, we know that 100,000 will be paid at some point in the next 10 years or so.
However, there are only 3,000 worth of premiums coming in over that time. Hopefully,
the insurer has been saving up some of the premiums received over the years to cover this
upcoming benet. In fact, the insurer is legally required to do so! This accumulation is
called a reserve and is one reason for the existence of the actuarial profession. In the rest
of this chapter, you can think of the reserve, at time t after issue, as the amount of money
the insurance company must have saved up to be able to provide for the future benets of
the policy. It makes sense that this amount should equal the APV of future benets minus
the APV of future premiums.
This chapter covers the calculation of benet reserves. A benet reserve is the dierence
between the expected value of future benets and the expected value of future premiums:
t
V = E(PV of future benets) E(PV of future premiums)
171
www.archactuarial.com www.guo.coursehost.com 172
Read
t
V as the benet reserve at time t.
Question: Why must it be true that the benet reserve is 0 at time t = 0?
7.2 Fully Continuous Benet Reserves
Option A reference: Actuarial Mathematics Chapter 7.2
Option B reference: Models for Quantifying Risk Chapter 12.3.2, 12.3.3
Consider a unit whole life policy on a fully continuous basis with an annual continuous benet
premium rate of P
_
A
[x]
_
.
Let the Prospective Loss at time t be:
t
L = v
T(x)t
P
_
A
[x]
_
a
T(x)t
This is a familiar idea in a new setting. In plain English, the loss function at time t,
t
L,
equals the present value of future benets, v
T(x)t
minus the present value of future premi-
ums, P
_
A
[x]
_
a
T(x)t
. (Note again the importance of understanding the notation. The loss
formula above is for a whole life policy you will need to be able to transform this formula
for other types of insurance.)
The Benet Reserve,
t
V is the expected value of
t
L:
t
V = E[
t
L].
This reserve takes into account mortality probabilities as well as interest.
t
V
_
A
[x]
_
=A
[x]+t
P
_
A
[x]
_
a
[x]+t
.
Remember, the [x] signies a person selected at age x. If no special selection occurred,
simply drop the [ ] notation. In fact, you might nd the look of the benet reserve more
natural without the [x] notation
t
V
_
A
x
_
=A
x+t
P
_
A
x
_
a
x+t
.
EXAMPLE:
You are given
l
x
= 100 x, for 0 < x < 100.
i = 0.08
Find
20
V
40
.
SOLUTION:
Arch MLC, Fall 2007 c _Yufeng Guo 172
www.archactuarial.com www.guo.coursehost.com 173
This is just a DeMoivres Law question in disguise (with = 100). At the end of
the example, well talk about other ways to recognize DeMoivre.
20
V
40
=A
60
P
_
A
40
_
a
60
(I know this is getting repetitive, but once again, this is just PVFB - PVFP for
a person now age 60, who started the policy at age 40.)
First well ndA
60
, and this will give us a
60
as well using the fundamental identity.
A
60
=
_
60
0
v
t
t
p
x

x+t
dt.
At this point you can remember that for DeMoivre, f(x+t) =
t
p
x

x+t
is uniform
over (60, 100), or you can work it out using
t
p
x
=
x t
x
and
x+t
=
1
x t
.
Either way the integral equals
1
40
_
40
0
e
t
dt =
1
40
_
40
0
e
0.07696t
dt
=
1
(0.07696)(40)
_
e
0.07696t
_
40
0
=
1 e
40(0.07696)
3.0784
= 0.3099 =A
60
.
a
60
=
1 A
60

=
0.6901
0.07696
= 8.97
Similarly,
A
40
=
1 e
60(0.07696)
60(0.07696)
= 0.2144
Finally, we can use the fundamental identity to say that
P
_
A
40
_
=
A
40
a
40
=
A
40
1 A
40
=
(0.07696)(0.2144)
0.786
= 0.02099 .
Putting it all together, we have
20
V
40
= 0.3099 (0.02099)(8.97) = 0.122
As you have just seen, they can test lots of chapters with a single reserve question.
Often the SOA will slip you a DeMoivres Law question without telling you it is DeMoivre
(as in this example). You can recognize a DeMoivre question if you are given any of the
following as being true for all x between 0 and . (You might want to bookmark this page.)
l
x
= x (The can be replaced by any number.)
Arch MLC, Fall 2007 c _Yufeng Guo 173
www.archactuarial.com www.guo.coursehost.com 174
s(x) = 1
x

=
x

(x) =
1
x
, or
x
(t) =
1
xt
F(x) =
x

Table 7.2.1 is a good one to understand. Make sure you can recognize the dierent types of
insurance presented. Read the subscripts of the As and as carefully!
Fully Continuous Benet Reserves: Age at Issue x, Duration t, Unit Benet
Actuarial
Plan Notation Prospective Formula
Whole life insurance
t
V (A
x
) A
x+t
P(A
x
)a
x+t
n-Year term insurance
t
V (A
1
x:n
)
_
_
_
A
1
x+t:nt
P
_
A
1
x:n
_
a
x+t:nt
0
t < n
t = n
n-Year endowment
insurance
t
V (A
x:n
)
_
A
x+t:nt
P(A
x:n
)a
x+t:nt
1
t < n
t = n
h-Payment years, whole
life insurance
h
t
V (A
x
)
_
A
x+t

h
P(A
x
)a
x+t:ht
A
x+t
t h
t > h
h-Payment years, n-year
endowment insurance
h
t
V (A
x:n
)
_

_
A
x+t:nt

h
P(A
x:n
)a
x+t:ht
A
x+t:nt
1
t h < n
h < t < n
t = n
n-Year pure endowment
t
V (A
x:
1
n
)
_
_
_
A
x+t:
1
nt
P(A
x:
1
n
)a
x+t:nt
1
t < n
t = n
n-Year Deferred
whole life annuity
t
V (
n|
a
x
)
_
nt|
a
x+t
P(
n|
a
x
)a
x+t:nt
a
x+t
t n
t > n
Notes:
1. The reserve always has the same format:
Reserve at time t = APV of benet (Premium) APV of annuity.
Simply change the notation to reect dierent types of benet and payment streams
(annuities).
2. It is critical to have a good handle on the earlier chapters:
Reserve at time t
. .
Chapter 7
= APV of benet
. .
Chapter 4
(Premium) (APV of annuity)
. .
(Chap. 6)(Chap. 5)
Arch MLC, Fall 2007 c _Yufeng Guo 174
www.archactuarial.com www.guo.coursehost.com 175
7.3 Other Methods for Calculating the Benet Reserve
Option A reference: Actuarial Mathematics Chapter 7.3
Option B reference: Models for Quantifying Risk Chapter 12.1
1) Prospective Formula
So far, we have only thought of the benet reserve in one way: as the dierence between the
APV of future benets and the APV of future premiums at some time t.
t
V
_
A
x
_
=A
x+t
P
_
A
x
_
a
x+t
This is one of many formulas for the reserve and is called the Prospective Method for
nding the reserve. The name prospective is used because the reserve is calculated by
looking at future (prospective) benets and premiums. There are several other methods for
reserve calculation which the SOA will expect you to know. One important thing to keep in
mind is that all of the formulas represent exactly the same quantity. For simplicity, we will
present all of these formulas for a whole life insurance. Then well look at some examples for
various types of insurance.
2) Retrospective Formula
After the Prospective Formula, the second most natural way to look at the reserve is Ret-
rospectively, i.e. to look at the APV of past benets and premiums. Before we do this
we need to remind ourselves how to accumulate benets and premiums with interest and
survivorship. Recall that if you are standing at time 0, the value of 5 that you will be paid
if you are alive at time t, is 5
t
E
x
, where the discounting includes interest and survivorship.
Now assume you are standing at time t and trying to determine the APV of what, at time 0
was 5. If it has been accumulating with the benet of interest and survivorship, then, to be
consistent, its value must be
5
t
E
x
.
EXAMPLE:
Assume i = 0.06 and a
x:8
= 6.9, and
8
p
x
= 0.9. What is the APV (accumulated
with interest and survivorship) at time t = 8 of an 8-year temporary life annuity
payable to (x)?
SOLUTION:
s
x:8
=
a
x:8
8
E
x
=
6.9
v
8
8
p
x
=
(6.9)(1.06)
8
0.9
12.22.
Now we are ready to talk about the retrospective reserve. Assume you are an insurance
company that at time 0 issued a fully continuous whole life policy to (x). It is now time t
and we can calculate prospectively that
t
V =A
x+t
P
_
A
x
_
a
x+t
Arch MLC, Fall 2007 c _Yufeng Guo 175
www.archactuarial.com www.guo.coursehost.com 176
as given above. But where did the money for this reserve come from? Past premiums and
interest are the only source. In fact, by looking retrospectively, we can see that this reserve
must exactly equal the accumulated value of past premiums (with interest and survivorship)
minus the accumulated value of past benets. I.e.,
Reserve = (APV at t of past premiums) (APV at t of past benets).
But what is the APV of past premiums? Premiums have been paid for t years as a continuous
life annuity on (x), and at time 0, the APV of these premiums was
P(A
x
)a
x:t
.
So at time t, the APV of these premiums must be
1
t
E
x
P(A
x
)a
x:t
= P(A
x
)s
x:t
.
Now for the APV of past benets. If we only look at the expected benets of the policy
between time 0 and time t, we are looking at a t-year term policy whose APV at time 0 was
A
1
x:t
. The APV of past benets at time t is this value accumulated t years for interest and
survivorship, or
A
1
x:t
t
E
x
.
Therefore, the retrospective form of the reserve at time t is (nally!)
t
V = P(A
x
)s
x:t

A
1
x:t
t
E
x
. (Retrospective Reserve Formula)
EXAMPLE:
You are given
(x) =
1
100x
, for 0 < x < 100.
i = 0.08
Find
20
V
40
using the Retrospective Reserve Formula.
SOLUTION:
This is the exact same DeMoivres Law setup that we saw earlier, so we will use
some things from that solution. But rst,
20
V
40
= P
_
A
40
_
s
40:20

A
1
40:20
20
E
40
= P
_
A
40
_
a
40:20
20
E
40

A
1
40:20
20
E
40
Arch MLC, Fall 2007 c _Yufeng Guo 176
www.archactuarial.com www.guo.coursehost.com 177
From the work we did earlier nding this reserve prospectively, we know
P
_
A
40
_
= 0.02099, A
40
= 0.2144, A
60
= 0.3099,
a
60
= 8.97, a
40
=
1 A
40

=
1 A
40
0.07696
= 10.21.
Next we need
20
E
40
and well be ready to nd the reserve
20
E
40
= v
20
20
p
40
=
_
1
1.08
_
20
_
x t
x
_
= (0.2145)
_
100 40 20
100 40
_
= 0.143.
OK - we are ready to start (!)
a
40:20
= a
40

20
E
40
a
60
= 10.21 (0.143)(8.97) = 8.93
A
1
40:20
=A
40

20
E
40
A
60
= 0.2144 (0.143)(0.3099) = 0.170
So the reserve is
20
V
40
= (0.02099)
8.93
0.143

0.170
0.143
= 0.122
Heres the most important part Note that we got the same answer (except for
some rounding) that we got using the prospective reserve formula. All of these
reserve formulas are dierent ways to arrive at the same quantity.
EXAMPLE: Constant Force of Mortality
The insured (40) is subject to a constant force of mortality, and interest at a
rate .
Find an expression for
t
V
x
.
SOLUTION:
t
V
x
=A
x+t
P(A
x
)a
x+t
For any age y,
A
y
=

+
, a
y
=
1
+
.
And we saw earlier that P(A
x
) = , so
t
V
x
=

+

_
1
+
_
= 0.
This means that if the insured is subject to a constant force of mortality and
premiums are paid continuously, no reserve is necessary! The insurance is self-
supporting at every age.
Arch MLC, Fall 2007 c _Yufeng Guo 177
www.archactuarial.com www.guo.coursehost.com 178
3) Premium Dierence Formula
t
V
_
A
x
_
=
_
P
_
A
x+t
_
P
_
A
x
__
a
x+t
To see that this one is true, think of us as the insurance company standing at time t. Our
client (x +t) has been paying premiums since she was (x). Her premium rate since then and
for the rest of her life has been P
_
A
x
_
, so the APV of future premiums is clearly
P
_
A
x
_
a
x+t
.
[It is a
x+t
instead of a
x
because we are standing here at t and can only expect a life annuity
of premiums on the life of (x + t).] However the future benets that we are oering from
this point forward are the equivalent of a whole life policy on (x + t), so the APV of future
benets is
A
x+t
= P
_
A
x+t
_
a
x+t
.
Now using
t
V = (APV future benets) (APV future prems)
gives the Premium Dierence Formula. Make sure you see how this formula is just a substi-
tution and recombination of the prospective method.
EXAMPLE:
Write the general premium dierence reserve formula for n-yr endowment insur-
ance.
SOLUTION:
t
V
x:n
=
_
P
_
A
x+t:nt
_
P
_
A
x:n
__
a
x+t:nt
.
4) Paid-Up Insurance Formula
t
V
_
A
x
_
=
_
_
1
P
_
A
x
_
P
_
A
x+t
_
_
_
A
x+t
You can obtain this formula quickly from the Premium Dierence Formula by noticing that
a
x+t
=
A
x+t
P
_
A
x+t
_.
Either this formula or the Premium Dierence Formula could prove useful if you are given
a couple of premiums and need to come up with a reserve. But this formula used to have
another, more practical use. Notice that it can be looked at as a simple multiple of a whole
life policy. If, after paying premium for t years on a whole life policy, you were to decide to
stop paying premiums, this formula shows that the reserve can be given to you as a whole
life policy with face value equal to
_
_
1
P
_
A
x
_
P
_
A
x+t
_
_
_
.
Arch MLC, Fall 2007 c _Yufeng Guo 178
www.archactuarial.com www.guo.coursehost.com 179
It tells, in terms of a paid-up (no more premiums) whole life policy, just how big a reserve
the insurance company is holding to provide for future benets for you. Thats how this
formula got the name Paid-Up Insurance Formula. You could use it for a question like the
following.
EXAMPLE:
Joan has been paying annual premiums for a continuous whole life policy since
she was 25 years old. The face value of the policy is 100,000. Joan is now 65,
is retiring, and would like to stop paying premiums and take a reduced, paid-up
whole life policy. Use the following to determine the appropriate face value of the
paid-up policy.
a
25
= 16,
A
25
= 0.09,
P
65
= 0.044.
SOLUTION:
The value of the policy should match the value of the reserve. Since the value of
the policy will be F A
65
, where F is the face amount of the policy, we want to
solve:
40
V
25
= F A
65
.
We can do this using the Paid-Up Insurance Formula, which says
40
V
25
=
_
_
1
P
_
A
25
_
P
_
A
65
_
_
_
A
65
.
P(A
25
) =
A
25
a
25
=
0.09
16
= 0.005625,

40
V
25
=
_
1
0.005625
0.044
_
A
65
= 0.87216.
So the face amount of the paid up policy will be 87,216. Note that we are able to
do this without ever knowing what the reserve is.
5) Other Reserve Formulas
I would absolutely memorize and be able to produce quickly all four of the above reserve
formulas. Here are some others that could appear on the exam but are less likely. You might
want to be sure you can arrive at each one using the fundamental identity a =
1A

and some
algebra. Then if it looks like you need one on the exam, youll have a ghting chance. If
you nd yourself with extra preparation time close to the exam, you could commit these to
memory as well.
(If you want to make sure you understand these, you could quickly use each one to show that
t
V
x
= 0 when the force of mortality is a constant.)
Arch MLC, Fall 2007 c _Yufeng Guo 179
www.archactuarial.com www.guo.coursehost.com 180
5a) The Annuity Reserve Formula,
t
V
_
A
x
_
= 1
a
x+t
a
x
,
uses only annuity functions.
5b) The Premium Reserve Formula,
t
V
_
A
x
_
=
P
_
A
x+t
_
P
_
A
x
_
P
_
A
x+t
_
+
uses only premium and interest information.
5c) The Death Benet Only Formula,
t
V
_
A
x
_
=
A
x+t
A
x
1 A
x
uses only APV of benet information.
With these and the basic reserve formulas, you need to be able to quickly recognize which
actuarial quantities you are given and use that to decide which formula you need to use.
EXAMPLE:
Write four formulas for
30
15
V (A
40
).
SOLUTION: (I just wanted to include at least one example with the funky
notation for an h-pay life reserve.)
We want the reserve at year 15 of a 30-Pay whole life policy.
1. (Prospective)
30
15
V (A
40
) =A
55

30
P(A
40
) a
55:15
2. (Retrospective)
30
15
V (A
40
) =
30
P(A
40
) s
40:15

A
1
x:15
15
E
40
3. (Premium Dierence)
30
15
V (A
40
) =
_
15
P(A
55
)
30
P(A
40
)
_
a
55:15
4. (Reduced Paid-up)
30
15
V (A
40
) =
_
1
30
P(A
40
)
15
P(A
55
)
_
A
55
.
Arch MLC, Fall 2007 c _Yufeng Guo 180
www.archactuarial.com www.guo.coursehost.com 181
7.4 Fully Discrete Reserves
Option A reference: Actuarial Mathematics Chapter 7.4
Option B reference: Models for Quantifying Risk Chapter 12.2
An insurance product is fully discrete if it has annual premium payments and benets are
paid at the end of the year of death. The loss function in this case is:
k
L = v
(K(x)k)+1
P
x
a
(K(x)k)+1
.
Taking the expected value gives
k
V
x
= E [
k
L[K(x) = k, k + 1, k + 2, . . .] .
The Prospective Reserve formula is
k
V
x
= A
x+k
P
k
a
x+k
As you may have already noticed, this formula is very closely related to the formula for
fully continuous reserves with only a few standard notational changes when moving from
continuous to discrete. It is critical to know this notation, so well introduce the remaining
formulas in a way that might help make some of the distinctions between continuous and
discrete seem more natural. Remember, all of these formulas are describing the same quantity:
the amount the insurance company should have saved up by time t after issue to fund the
rest of the policy (allowing for any remaining premiums).
Consider a continuous n-yr term policy of unit benet.
Premium Dierence Formula
Continuous:
t
V
_
A
1
x:n
_
=
_
P
_
A
1
x+t:nt
_
P
_
A
1
x:n
__
a
x+t:nt
.
Discrete:
k
V
1
x:n
=
_
P
1
x+k:nk
P
1
x:n
_
a
x+k:nk
.
Note the convention of dropping the accompanying A notation (in the expressions for
premium) in the discrete versions of the formulas.
Paid Up Insurance Formula
Continuous:
t
V
_
A
1
x:n
_
=
_

_
1
P
_
A
1
x:n
_
P
_
A
1
x+t:nt
_
_

_
A
1
x+t:nt
.
(This formula assumes t < n. If t > n, the reserve is 0 since the term of insurance has
expired.)
Arch MLC, Fall 2007 c _Yufeng Guo 181
www.archactuarial.com www.guo.coursehost.com 182
Discrete:
k
V
1
x:n
=
_

_1
P
1
x:n
P
1
x+k:nk
_

_A
1
x+k:nk
.
Retrospective Formula for t < n
Continuous:
t
V
_
A
1
x:n
_
= P
_
A
1
x:n
_
s
x:t

t
k
x
,
where
t
k
x
=
A
1
x:t
t
E
x
.
The symbol
t
k
x
is new and is called the accumulated cost of insurance. It has no
meaning other than that shown above which is just the cost of t-year term insurance on (x)
accumulated (with interest and survivorship) to the end of the t years.
Discrete:
t
V
x:n
= P
x:n
s
x:t

t
k
x
,
where
t
k
x
=
A
1
x:t
t
E
x
.
Other Discrete Formulas for the reserve: (You have seen all of these already in the
continuous form.)
k
V
x
= 1
a
x+k
a
x
,
k
V
x
=
P
x+k
P
x
P
x+k
+d
,
k
V
x
=
A
x+k
A
x
1 A
x
.
EXAMPLE: Illustrative Life Table
A 20-year term policy is issued to (35) and mortality and interest are assumed to
follow the Illustrative Life Table (ILT). Find the benet reserve at the end of the
10-th year.
SOLUTION:
10
V
1
35:20
= A
1
45:10
P
1
35:20
a
45:10
.
Using the ILT, we can determine the following:
A
1
45:10
= A
45

10
E
45
A
55
= 0.2012 (0.5265)(0.3051) = 0.0406,
Arch MLC, Fall 2007 c _Yufeng Guo 182
www.archactuarial.com www.guo.coursehost.com 183
a
45:10
= a
45

10
E
45
a
55
= 14.11 (0.5265)(12.28) = 7.64.
Similarly, we can nd (you should do it!) that
P
1
35:20
= 0.0035.

10
V
1
35:20
= 0.0406 (0.0035)(7.64) = 0.0138.
EXAMPLE: Illustrative Life Table
Based on the illustrative life table, what is the benet reserve at the end of the
20th year for a 10-pay, 30-year endowment on (25)?
SOLUTION:
This is
10
20
V
25:30
.
We could look up a messy formula, but lets just think about it instead. We know
the reserve at year 20 is
(PV FB
yr20
- PV FP
yr20
).
The second part of this equation is easy to deal with since this is a 10-pay policy,
there are no premiums left! so PV FP
yr20
= 0. Therefore, all we need to nd is
PV FB
yr20
= A
45:10
= A
1
45:10
+A
45:
1
10
Using the ILT,
A
1
45:10
= A
45

10
E
45
A
55
= 0.0406,
A
45:
1
10
=
10
E
45
= 0.5265.

10
20
V
25:30
= 0.5265 + 0.0406 = 0.5671.
EXAMPLE:
Recall our cli diver, age 30, who bought a 2-year term life policy with a benet
of 100,000 payable at the end of year of death. Interest is i = 0.08, and the
mortality for the diver is shown in the table:
x p
x
30 0.95
31 0.93
32 0.90
In Chapter 6, we determined that the benet premium for this insurance is P =
5500.
Arch MLC, Fall 2007 c _Yufeng Guo 183
www.archactuarial.com www.guo.coursehost.com 184
Find the benet reserve for this policy at the end of year 1.
SOLUTION:
There is no telling with this type of question whether prospective or retrospective
will be faster, so I usually start with the prospective method and then if I get
stuck, I try the other.
1
V
1
30:2
= 100,000 A
1
31:1
P a
31:1
= 100,000 vq
31
(5500)(1) = 981.48.
EXAMPLE:
Given:
P
x:30
= 0.03

20
V
x:30
= 0.6

20
k
x
= 0.3

20
E
x
= 0.2
Find a
x:20
.
SOLUTION: Seeing the k is a strong indication that we might be interested in
the retrospective reserve formula:
20
V
x:30
= P
x:30
s
x:20

20
k
x
.
0.6 = 0.03
a
x:20
20
E
x
0.3
0.9 = 0.15 a
x:20
,
a
x:20
= 6.
7.5 Benet Reserves on a Semi-Continuous Basis
Option A reference: Actuarial Mathematics Chapter 7.5
Option B reference: Models for Quantifying Risk Chapter none
In Chapter 6, the idea of a semi-continuous insurance product was introduced. This product
has an annual premium, but the benet is paid at the moment of death. This idea is now
carried into the reserve formulas. In general, it boils down to using fully discrete formulas,
but replacing A with A, and P with P(A). Also, the reserve notation is V (A); no bar above
the V , but there is a bar above the A to indicate the continuous nature of the death benet.
Arch MLC, Fall 2007 c _Yufeng Guo 184
www.archactuarial.com www.guo.coursehost.com 185
Well take this opportunity to look at an h-payment, n-yr endowment in the case that t
h < n (note this is not always the case, but were using it for illustrative purposes). Compare
the notation for the three dierent types of insurance we now know.
Continuous:
h
t
V
_
A
x:n
_
=A
x+t:nt

h
P
_
A
x:n
_
a
x+t:ht
(t h < n),
Discrete:
h
k
V
x:n
= A
x+k:nk

h
P
x:n
a
x+k:hk
(k < h < n),
Semi-Continuous:
h
k
V
_
A
x:n
_
=A
x+k:nk

h
P
_
A
x:n
_
a
x+k:hk
(k < h < n).
EXAMPLE:
Assuming Uniform Distribution of Deaths over each year of age, use the Illustra-
tive Life Table to nd
20
V (A
25
).
SOLUTION:
20
V (A
25
) =A
45
P(A
25
) a
45
=
i

A
45

A
25
a
25
a
45
=
0.06
0.05827
(0.2012)
0.06
0.05827
(0.0817)
16.22
(14.11) = 0.134.

7.6 Benet Reserves Based on True m-thly Benet Premiums


Option A reference: Actuarial Mathematics Chapter 7.6
Option B reference: Models for Quantifying Risk Chapter 12.4
Remember true m-thly premiums? We now need to be able to incorporate those into
reserve formulas.
h
k
V
(m)
x:n
= A
x+k:nk

h
P
(m)
x:n
a
(m)
x+k:hk
k < h
It is a pain to even write that formula down, but note that the only real change from the
analogous fully discrete formula is the addition of the (m) notation. It does make the point
though, that you need to be comfortable with the idea of m-thly premiums, annuities, and
reserves.
Chapter 7 Suggested Problems: 8, 14, 16, 19, 23(Middle row and last column),
24, 26, 29, 30, 31
Arch MLC, Fall 2007 c _Yufeng Guo 185
www.archactuarial.com www.guo.coursehost.com 186
CHAPTER 7 Formula Summary
Continuous Reserve Formulas:
Prospective Reserve Formula:
t
V
_
A
x
_
=A
x+t
P
_
A
x
_
a
x+t
Retrospective Reserve Formula:
t
V = P(A
x
)s
x:t

A
1
x:t
t
E
x
Premium Dierence Formula:
t
V
_
A
x
_
=
_
P
_
A
x+t
_
P
_
A
x
__
a
x+t
Paid-Up Insurance Formula:
t
V
_
A
x
_
=
_
_
1
P
_
A
x
_
P
_
A
x+t
_
_
_
A
x+t
The Annuity Reserve Formula (uses only annuity functions):
t
V
_
A
x
_
= 1
a
x+t
a
x
,
The Premium Reserve Formula:
t
V
_
A
x
_
=
P
_
A
x+t
_
P
_
A
x
_
P
_
A
x+t
_
+
The Death Benet Only Formula (uses only APV of benet information):
t
V
_
A
x
_
=
A
x+t
A
x
1 A
x
Discrete Reserves:
k
V
x
= A
x+k
P
k
a
x+k
k
V
x:n
=
_
P
x+k:nk
P
x:n
_
a
x+k:nk
.
k
V
x:n
=
_
1
P
x:n
P
x+k:nk
_
A
x+k:nk
.
Retrospective Formula for t < n
t
V
x:n
= P
x:n
s
x:t

t
k
x
, where
t
k
x
=
A
1
x:t
t
E
x
.
Arch MLC, Fall 2007 c _Yufeng Guo 186
www.archactuarial.com www.guo.coursehost.com 187
Other Discrete Formulas for the reserve:
k
V
x
= 1
a
x+k
a
x
,
k
V
x
=
P
x+k
P
x
P
x+k
+d
,
k
V
x
=
A
x+k
A
x
1 A
x
.
Comparison for h-payment reserves:
Continuous:
h
t
V
_
A
x:n
_
=A
x+t:nt

h
P
_
A
x:n
_
a
x+t:ht
(t h < n),
Discrete:
h
k
V
x:n
= A
x+k:nk

h
P
x:n
a
x+k:hk
(k < h < n),
Semi-Continuous:
h
k
V
_
A
x:n
_
=A
x+k:nk

h
P
_
A
x:n
_
a
x+k:hk
(k < h < n).
h
k
V
(m)
1
x:n
= A
x+k:nk

h
P
(m)
x:n
a
(m)
x+k:hk
k < h
Arch MLC, Fall 2007 c _Yufeng Guo 187
www.archactuarial.com www.guo.coursehost.com 188
Past SOA/CAS Exam Questions:
1. For a 10-year deferred whole life annuity of 1 on (35) payable continuously:
(i) Mortality follows De Moivres law with = 85.
(ii) i = 0
(iii) Level benet premiums are payable continuously for 10 years.
Calculate the benet reserve at the end of ve years.
(A) 9.38 (B) 9.67 (C) 10.00 (D) 10.36 (E) 10.54
Solution:
There are no benets in the rst 5 years, so the retrospective reserve gives
10
5
V
_
10|
a
35
_
= P
_
10|
a
35
_
s
35:5
P
_
10|
a
35
_
=
a
45
s
35:10
Since i = 0, a
45
=

e
45
. And since this is DeMoivre,
a
45
=

e
45
=
45
2
= 20
s
35:10
=
a
35:10
10
E
35
Since i = 0,
10
E
35
=
10
p
35
=
4
5
, and
a
35:10
=
_
10
0
v
t
t
p
x
dt =
_
10
0
t
p
x
dt =
_
10
0
50 t
50
dt = 9
P
_
10|
a
35
_
= (20)
4
45
= 1.778

10
5
V = P
_
10|
a
35
_
s
35:5
=
1.778
50
_
5
0
(50 t)dt
l
35
l
40
=
1
50
(1.778)
_
50
45
_
_
50t
t
2
2
_
5
0
= 9.38
Key: A
Arch MLC, Fall 2007 c _Yufeng Guo 188
www.archactuarial.com www.guo.coursehost.com 189
2. For a fully discrete 20-payment whole life insurance of 1000 on (x), you are given:
(i) i = 0.06
(ii) q
x+19
= 0.01254
(iii) The level annual benet premium is 13.72.
(iv) The benet reserve at the end of year 19 is 342.03.
Calculate 1000P
x+20
, the level annual benet premium for a fully discrete whole life
insurance of 1000 on (x + 20).
(A) 27 (B) 29 (C) 31 (D) 33 (E) 35
SOLUTION:
1000
20
20
V
x
= 1000A
x+20
=
1000(
20
19
V
x
+
20
P
x
)(1.06) q
x+19
(1000)
p
x+19
=
(342.03 + 13.72)(1.06) (0.01254)(1000)
0.98746
= 369.18
a
x+20
=
1 0.36918
0.06
1.06
= 11.1445
so, 1000P
x+20
= 1000
A
x+20
a
x+20
=
369.18
11.1445
= 33.1 Key: D
3. You are given:
(i) Mortality follows De Moivres law with = 100.
(ii) i = 0.05
(iii) The following annuity-certain values:
a
40
= 17.58
a
50
= 18.71
a
60
= 19.40
Calculate
10
V
_
A
40
_
.
(A) 0.075 (B) 0.077 (C) 0.079 (D) 0.081 (E) 0.083
Arch MLC, Fall 2007 c _Yufeng Guo 189
www.archactuarial.com www.guo.coursehost.com 190
SOLUTION: = ln(1.05) = 0.04879
Since this is DeMoivres law:
A
x
=
_
x
0
t
p
x

x
(t)e
t
dt =
_
x
0
1
x
e
t
dt =
1
x
a
x
From here, many formulas for
10
V
_
A
40
_
could be used. One approach is:
A
50
=
a
50
50
=
18.71
50
= 0.3742, so a
50
=
_
1 A
50

_
= 12.83
A
40
=
a
60
60
=
19.40
60
= 0.3233, so a
40
=
_
1 A
40

_
= 13.87
P
_
A
40
_
=
0.3233
13.87
= 0.02331
10
V
_
A
40
_
=
_
A
50
P
_
A
40
_
a
50
_
= [0.3742 (0.02331)(12.83)] = 0.0751. Key: A
4. You are given:
(i) P
x
= 0.090
(ii)
n
V
x
= 0.563
(iii) P
x:
1
n
= 0.00864
Calculate P
1
x:n
.
(A) 0.008 (B) 0.024 (C) 0.040 (D) 0.065 (E) 0.085
SOLUTION:
Apply the 3-premium principle.
_
P
x
P
1
x:n
_
P
x:
1
n
=
n
V
x
This is true since the payment of P
1
x:n
for n years entitles you to nothing from the
insurance company if you live to time n. Paying P
x
for n years entitles you to the
reserve if you live.
0.563 =
0.090 P
1
x:n
0.00864
P
1
x:n
= 0.090 (0.00864)(0.563) = 0.0851 Key: E
Arch MLC, Fall 2007 c _Yufeng Guo 190
www.archactuarial.com www.guo.coursehost.com 191
5. For a fully discrete 3-year endowment insurance of 1000 on (x), you are given:
(i)
k
L is the prospective loss random variable at time k.
(ii) i = 0.10
(iii) a
x:3
= 2.70182
(iv) Premiums are determined by the equivalence principle.
Calculate
1
L, given that (x) dies in the second year from issue.
(A) 540
(B) 630
(C) 655
(D) 720
(E) 910
SOLUTION:
Let K be the curtate future lifetime of (x +k)
k
L = 1000
K+1
1000P
x:3
a
K+1|
When (as given in the problem), (x) dies in the second year from issue, the curtate
future lifetime of (x + 1) is 0, so
1
L = 1000 1000P
x:3
a
1|
=
1000
1.1
279.21
= 629.88 630
The premium came from:
P
x:3
=
A
x:3
a
x:3
A
x:3
= 1 d a
x:3
P
x:3
= 279.21 =
1 d a
x:3
a
x:3
=
1
a
x:3
d Key: B
Arch MLC, Fall 2007 c _Yufeng Guo 191
www.archactuarial.com www.guo.coursehost.com 192
Problems from Pre-2000 SOA-CAS exams
1. You are given:
P
_
A
50
_
= 0.02781

20
V
_
A
50
_
= 0.21512

20
E
50
= 0.18072

20
k
50
= 1.28895
Calculate a
50:20
.
(A) 8.77 (B) 9.27 (C) 9.77 (D) 10.27 (E) 10.77
2. For a fully continuous whole life insurance of 1 on (x), you are given:

x+k+t
=

t
=
P
_
A
x
_
= 0.03
+P
_
A
x
_
= 0.07
Calculate
k
V
_
A
x
_
.
(A) 0.0 (B) 0.1 (C) 0.2 (D) 0.3 (E) 0.4
3. A fully discrete 3-year endowment insurance of 1000 is issued on (x).
You are given:
i = 0.06
The following extract from the mortality table:
y l
y
x 1000
x + 1 900
x + 2 810
1000P
x:3
= 332.51
Calculate: 1000
_
2
V
x:3

1
V
x:3
_
(A) 330.38 (B) 332.86 (C) 334.51 (D) 337.22 (E) 340.74
Arch MLC, Fall 2007 c _Yufeng Guo 192
www.archactuarial.com www.guo.coursehost.com 193
Arch MLC, Fall 2007 c _Yufeng Guo 193
www.archactuarial.com www.guo.coursehost.com 194
Solutions to Pre-2000 Questions: Chapter 7
1. Key: C
20
V
_
A
50
_
= P
_
A
50
_
20
s
50

20
k
50
20
s
50
=
20
V
_
A
50
_
+
20
k
50
P
_
A
50
_ =
0.21512 + 1.28895
0.02781
a
50:20
=
20
s
50

20
E
50
= 54.084 0.1807 = 9.774
2. Key: D
k
V (A
x
) =A
x+k
P(A
x
)a
x+k
Since after time k, the force of mortality is constant at , this equals

+
P(A
x
)
1
+
+ = P(A
x
) + +P(A
x
) = 0.03 + 0.07 = 0.1

k
V (A
x
) =
P(A
x
)
0.1
= 0.3
3. Key: A
The premium paid by all l
x
people alive at time t = 0 must pay death benets during
the year plus provide a reserve at the end of the year for those who are alive.
l
x
(1000 P
x:3
) = 1000 v d
x
+ 1000v l
x+1

1
V
x:3
(1000)(332.51) =
1000
1.06
(100) +
1000
1.06
(900)
1
V
x:3
1000
1
V
x:3
= 280.51
Using the same recursion idea
1000l
x+1
_
1
V
x:3
+P
x:3
_
= 1000v
_
d
x+1
+l
x+2

2
V
x:3
_
(900)(332.51 + 280.51) =
1000
1.06
_
90 + (810)
2
V
x:3
_
1000
2
V
x:3
= 610.89
1000
_
2
V
x:3

1
V
x:3
_
= 610.89 280.51 = 330.38
Note that many concepts from this chapter are tested in questions that better t into Chap-
ter 8.
Arch MLC, Fall 2007 c _Yufeng Guo 194
Chapter 6
ACTUARIAL MATHEMATICS:
CHAPTER 8 ANALYSIS OF
BENEFIT RESERVES
Option A reference: Actuarial Mathematics Chapter 8
Option B reference: Models for Quantifying Risk Chapter 12.2, 12.7
In Chapter 7, we calculated Benet Reserves assuming level premiums and level benets. In
addition, we calculated the reserves only at policy anniversaries. In this chapter, we move
to a more general framework which can include non-level premiums and non-level benets,
along with calculation between policy anniversaries. So, you already know the underlying
principles for this chapter were simply increasing the utility of these ideas to include more
general cases.
8.2 Benet Reserves for General Insurances
Option A reference: Actuarial Mathematics Chapter 8.2
Option B reference: Models for Qualifying Risk Chapter 12.7
Consider a fully discrete insurance on (x). Unlike Chapter 7, we now assume that benet
and premium amounts can change during the period of coverage.
Let b
j
equal the death benet payable at the end of year of death for the j
th
policy
year, where j = 1, 2, 3.... So if b
j
= 1000j, then the death benet is 1000 if death occurs in
the rst year, 2000 if death occurs in the second year, and so on.
Let
j1
equal the benet premium paid at the beginning of each policy year for the
j
th
policy year. So
0
is the premium for the rst year (payable at time 0),
1
is the premium
for the second year (payable at time 1), and so on.
Now we need to recall from chapter 7 what a Loss Function is: the present value at time
h of future benets minus the present value of future premiums. Since were dealing with
195
www.archactuarial.com www.guo.coursehost.com 196
possibly varying premiums and benets, you have to keep in mind that b
j
is not necessarily
equal to b
i
when i ,= j. This holds true for premiums at dierent times also. So we have a
revised loss function formula:
h
L =
_

_
0 K(x) = 0, 1, . . . h 1
b
K(x)+1
v
K(x)+1h

K(x)

j=h

j
v
jh
K(x) = h, h + 1, . . .
Note that
h
L is zero for K(x) < h because that means (x) died in the rst h years. Since we
are considering future losses while standing at time t = h, they are zero in this case since (x)
has already died and we have already taken our loss!
As before, the reserve formula is the expected value of the loss function, conditional on
K(x) > h.
h
V = E [
h
L [ K(x) h]
=

j=0
b
h+j+1
v
j+1
j
p
x+h
q
x+h+j

j=0

h+j
v
j
j
p
x+h
This is the Prospective Formula for a benet reserve. It is very messy but is easier to
reproduce if you keep two ideas in mind.
1. Recall that the Reserve equals the APV of benets (benets discounted for interest and
mortality) minus the APV of premiums (again, discounted for interest and mortality).
The format of the reserve above ts this denition.
2. All of the ps and qs start at x + h because we are looking at the situation h years
after policy issue. Therefore, the insured, who was (x) at policy issue, is now (x + h).
Similarly, we are only concerned with bs and s for time h and beyond.
EXAMPLE:
A fully discrete whole life insurance on (40) features premiums that will be level
in years 1-10 and then will double and remain xed. The death benet for all
years will be 10,000.
You are given:
A
40
= 0.15, A
50
= 0.24,
i = 0.07,

10
p
40
= 0.96.
Find: 1)
0
, and 2)
10
V for this insurance.
SOLUTION:
Arch MLC, Fall 2007 c _Yufeng Guo 196
www.archactuarial.com www.guo.coursehost.com 197
1. The APV of premiums has to equal the APV of the benets.

0
a
40
+
10
E
40

0
a
50
= 10,000A
40
.
The left-hand side is correct since we can think of the premium structure as
paying
0
forever and then paying an extra
0
starting 10 years from now.
a
40
=
1 A
40
d
=
0.85
0.0654
= 12.99,
a
50
=
1 A
50
d
=
0.76
0.0654
= 11.62,
10
E
40
=
_
1
1.07
_
10
(0.96) = 0.488.

0
(12.99) + (0.488)
0
(11.62) = 10,000(0.15)

0
= 80.38
2.
10
V equals PVFB - PVFP:
10
V = 10,000 A
50
2
0
a
50
= 2400 2(80.38)(11.62) = 532.
Note that if we made the premium jump too big at year 10 (say a multiple of 4),
we could even achieve a negative reserve.
EXAMPLE: Illustrative Life Table
A fully discrete whole life policy is issued to (50) with some special features. The
death benet is 1000 for all years. The premium for the rst year will be set
equal to the APV of the rst years benet and the remaining premiums will all
be level. Mortality and interest are according to the ILT.
Find
(i)
0
,
(ii)
1
,
(iii)
1
V .
SOLUTION:
(i)

0
= 1000 A
1
50:1
= 1000 vq
50
= 1000
1
1.06
_
1
l
51
l
50
_
= (943.4)
_
1
88,979
89,509
_
= 5.59
Arch MLC, Fall 2007 c _Yufeng Guo 197
www.archactuarial.com www.guo.coursehost.com 198
(ii) We can set up the equivalence equation to nd
1
since all the premiums
after the rst one will equal
1
.
5.59 +
1
E
50

1
a
51
= 1000A
50
5.59 +v p
50

1
(13.08) = 249.05

1
= 19.85.
(iii) Since it is just one year, I will go with the retrospective approach:
1
V = 5.59 s
50:1
1000
1
k
50
= 5.59
a
50:1
1
E
x
1000
A
1
50:1
1
E
x
= 5.59
a
50:1
vp
50
1000
A
1
50:1
vp
50
= 5.59
1
(0.9434)(0.994)

5.59
(0.9434)(0.994)
= 0.
This may be surprising but it makes sense. If the rst year of premium
exactly covers expected deaths in the rst year, there is nothing left over to
provide a reserve for the survivors. (For a more general example of this type
of problem see Example 8.2.1 in the text.)
Now, lets look at the continuous analog of the discrete case. Consider a fully continuous
insurance on (x). What does fully continuous imply with respect to premium and benet
payment?
Let
b
t
= benet payable at the moment of death.

t
= annual rate of benet premiums payable continuously at t.
As in the discrete case, the benets and premiums can change over the life of the contract.
So b
s
is not necessarily equal to b
t
and
s
is not necessarily equal to
t
when s ,= t.
For the continuous case, the loss function at time t is given by
t
L =
_
_
_
0 T(x) t
b
T(x)
v
T(x)t

_
T(x)
t

u
v
ut
du T(x) > t
where, as in the discrete case,
t
L = 0 for T(x) t since that means (x) died prior to the
time at which future losses are being considered.
The reserve at time t can be expressed as
t
V =
_

0
b
t+u
v
u
u
p
x+t

x
(t +u) du
_

0

t+r
v
r
r
p
x+t
dr.
Arch MLC, Fall 2007 c _Yufeng Guo 198
www.archactuarial.com www.guo.coursehost.com 199
Note the similarities and dierences between the discrete and continuous cases. In both cases,
t
L = PV (future benets) PV (future premiums) ,
and
t
V = APV (future benets) APV (future premiums) .
And in both cases,
t
L is a random variable and
t
V = E[
t
L[T(x) > t], but there are some
dierences:
Discrete Continuous

_
K(x) T(x)
Use of p, q Use of p, .
Make sure you recognize the signicance of the each of these dierences.
EXAMPLE:
For a continuous whole life policy of 1 on (x), premiums will remain constant
over the life of the contract but benets will grow at a rate of 5 percent per year
(compounded continuously). If mortality and interest are constants = 0.025
and = 0.06, respectively, nd the benet premium for this insurance.
SOLUTION:
The death benet at time t is b
t
= e
0.05t
so the equivalence equation becomes
a
x
=
_

0
v
t
b
t t
p
x
dt =
_

0
e
0.06t
e
0.05t
e
0.025t
(0.025)dt

1
+
= (0.025)
_

0
e
0.035t
dt
(11.76) =
0.025
0.035
_
e
0.035t
_

0
= (0.714)(1)
= 0.061.
The insured has to live quite a while for the company to break even on this
insurance!
Arch MLC, Fall 2007 c _Yufeng Guo 199
www.archactuarial.com www.guo.coursehost.com 200
8.3 Recursion Relations for Fully Discrete Benet Reserves
Option A reference: Actuarial Mathematics Chapter 8.3
Option B reference: Models for Qualifying Risk Chapter 12.2.2
Luckily, the rst reserve recursion is intuitive! Lets consider what
h
V represents. It is the
amount of money the insurance company must have saved at time h to have enough (along
with future premiums) to provide future benets.
Looking only one year ahead, we need to start the year with the quantity
h
V +
h
to have enough to pay a death benet if the insured dies plus provide a reserve at time h+1
if the insured lives. In other words
h
V +
h
= v b
h+1
q
x+h
+v
h+1
V p
x+h
.
Make sure this equation in some form is completely familiar and logical to you. You will
almost certainly nd it useful on the exam in some form. Remember - Those who die get
the benet, those who live get the reserve.
Multiplying both sides of this equation by (1 +i) gives
(
h
V +
h
)(1 +i) = b
h+1
q
x+h
+
h+1
V p
x+h
.
This says that if we start the year with
(
h
V +
h
)
and let it accumulate interest for a year, we will have enough to pay o those who died and
have a reserve in place for those who live. This is very intuitive! I dont care which form you
learn, but learn this equation!
Arch MLC, Fall 2007 c _Yufeng Guo 200
www.archactuarial.com www.guo.coursehost.com 201
Some Terminology from the text:
The policy year from time t = h to time t = (h +1) is called policy year h +1. This seems
screwy at rst but it is accurate since the year from t = 0 to t = 1 ought to be policy year 1.
For policy year h + 1, the following terminology applies:
h
V +
h
initial benet reserve for policy year h + 1
h
V terminal benet reserve for year h
h+1
V terminal benet reserve for year h + 1
These names are logical if you think about them and you need to be able to instantly write
down the correct corresponding expression whether a question uses initial reserves or ter-
minal reserves.
The last idea from this section on reserves is that of Net Amount at Risk. This refers
to the amount of money the insurance company could have to pay the insured above and
beyond the reserve accumulated. In other words, the net amount at risk for policy year
h + 1 is
(b
h+1

h+1
V ) .
In the insurance business, this is an important quantity. It is the amount of money the
company will have to produce from sources other than the insureds benet reserve if the
insured dies in policy year h + 1.
OK that is a lot of theory, here are a couple of examples.
EXAMPLE:
Mortality and interest are as given in the Illustrative Life Table. For a fully
discrete whole life policy on (40) with a benet of 1000, nd
(A) the terminal benet reserve for policy year 10,
(B) the initial benet reserve for policy year 11,
(C) the terminal net amount at risk for policy year 10.
SOLUTION:
(A) This is just
10
V
40
= A
50
P
40
a
50
. According to the ILT,
A
50
= 0.249, a
50
= 13.27,
P
40
=
A
40
a
40
=
0.161
14.82
= 0.0109.
10
V
40
= 0.249 (0.0109)(13.27) = 0.104.
So the reserve for this policy is 1000
10
V
40
= 104.
(B) This is 1000
10
V
40
+ 1000P
40
= 114.9.
(C) b
10

10
V
40
= 1000 104 = 896.
Arch MLC, Fall 2007 c _Yufeng Guo 201
www.archactuarial.com www.guo.coursehost.com 202
EXAMPLE:
The insured (x) has purchased a 20-year term policy which pays a at benet of
100,000 at the end of the year of death plus, if the insured lives for 20 years, all
premiums will be returned (without interest).
You are given:

20
E
x
= 0.286,
10
E
x
= 0.543,
a
x:20
= 11.88, a
x:10
= 7.73,
A
x
= 0.129 A
x+10
= 0.201 A
x+20
= 0.305
Find
(A) the benet premium,
(B) the terminal reserve for the 10th policy year.
SOLUTION:
(A) We should start by setting the APV of premiums equal to the APV of
benets. Since this is a non-standard insurance, I will use for the premium.
a
x:20
= 100,000A
1
x:20
+ (20)
20
E
x
A
1
x:20
= A
x

20
E
x
A
x+20
= 0.129 (0.286)(0.305) = 0.042
(11.88) = 4200 + (5.72)
= 681.82
(B) Since the premiums are constant but the benets are complicated, the ret-
rospective method might work best:
100,000
10
V
x
= s
x:10
100,000
A
1
x:10
10
E
x
= (681.82)
a
x:10
10
E
x
100,000
A
x

10
E
x
A
x+10
10
E
x
= (681.82)
7.73
0.543
100,000
0.129 (0.543)(0.201)
0.543
= 6049.
Note that since we carried the premium through at its full value, we also had
to carry the death benet through. You have to be careful when doing these
to see that all parts of the formula are assuming the same death benet.
(Leaving out the 100,000 would have been assuming a death benet of 1 for
past benets, while the premium assumes a death benet of 100,000.)
Arch MLC, Fall 2007 c _Yufeng Guo 202
www.archactuarial.com www.guo.coursehost.com 203
8.4 Benet Reserves at Fractional Durations
Option A reference: Actuarial Mathematics Chapter 8.4
Option B reference: Models for Qualifying Risk Chapter 12.6
Consider a fully discrete insurance with benets b
j+1
payable at the end of the year. This
insurance is purchased by annual benet premiums of
j
, where j = 0, 1, 2, . . .. Note that by
this time, saying end of year benet and annual premiums should seem redundant once
weve said we are working with fully discrete insurance.
In this section, we pursue the Interim Benet Reserve, which sounds like what it is
the reserve at some point in time between policy anniversaries. The notation for this reserve
is:
h+s
V for h = 0, 1, 2, . . . and 0 < s < 1.
So, s represents a fraction of a year.
h h+1 h+2 h+3
h+s
For fractional year reserves (reserves at time h +s where 0 < s < 1), we basically want to be
able to do two things.
1. Write and understand the messy formula that gets us from
h+s
V to
h+1
V .
2. Write and understand a very simple formula that relates
h+s
V to both
h
V and
h+1
V
First recall the recursive formula for
h
V :
h
V +
h
= vb
h+1
q
x+h
+v
h+1
V p
x+h
.
For
h+s
V , we are standing at time (h+s) so
h
is in the past and will not be in the formula.
In addition all discounting will be for (1 s) years instead of 1 year. Finally, any survival
functions will start at age (x +h +s) and involve survival for (1 s) years. The result is
h+s
V = v
1s
b
h+1 1s
q
x+h+s
+ v
1s
h+1
V
1s
p
x+h+s
.
This is a pretty messy formula, but if you think about what it means, it is possible to
remember. In plain English - The reserve at time h + s must be enough to provide the
benet b
h+1
for those who die in the next (1 s) years plus provide
h+1
V at the end of the
year for those who survive the next (1 s) years.
By the way, the text uses the name interim benet reserve for the fractional year reserve
above, so that is the name to expect to see on the exam.
It would be nice if the interim benet reserve (at time h + s) were the linear interpolation
of the beginning-of-year reserve and end-of-year reserve for the same year. Then it would be
true that
h+s
V = (1 s) (
h
V +
h
) + (s) (
h+1
V ) (UDD)
Arch MLC, Fall 2007 c _Yufeng Guo 203
www.archactuarial.com www.guo.coursehost.com 204
In fact this is only true under Uniform Distribution of Deaths - as you may have guessed from
the UDD written next to the equation! It is a formula worth remembering since the exam
writers often give you UDD as a simplifying assumption, and the formula is nearly correct
even when UDD does not apply.
There is a useful way of rewriting the UDD linear interpolation:
h+s
V = (1 s) (
h
V )
. .
terminal reserve at h
+(s) (
h+1
V )
. .
terminal reserve at h + 1
+ (1 s)
h
. .
unearned premium
So, in the UDD case, we have linear interpolation between the two end point terminal
reserves, plus the unearned benet premium. The unearned benet premium can be
thought of as follows: A premium was paid at the beginning of the year to cover the whole
year. At time s, where (0 < s < 1), only a part of the premium has been earned by the
insurance company. If the policy were canceled, a pro rata portion of the premium might be
returned to the policyholder since it has not been earned. In other words, a premium is fully
earned only when the time-period for which it was intended to provide coverage has passed.
h
h+1 h+s
.
s (ann prem) is
earned before
h+s.
(1-s) (ann prem)
is earned after
h+s.
.
EXAMPLE:
Suppose a premium of 100 was paid on a fully discrete insurance policy. As of
April 27th of the year (not a leap year), how much premium has been earned?
How much is unearned? In this case, what are
h
and s? (Work it out before
looking at the solution!)
SOLUTION:

h
= 100
Jan + Feb + March + (27 days of April ) = 31 + 28 + 31 + 27 = 117 days.
s =
117
365
= 0.3205,
(1 s)
h
= (1 0.3205) 100 = 67.95 of unearned premium.
Then there must be 100 67.95 = 32.05 of earned premium. Unearned premium
is a simple concept that might seem a little dicult at rst, but is easier to
understand if you think about it logically.
Arch MLC, Fall 2007 c _Yufeng Guo 204
www.archactuarial.com www.guo.coursehost.com 205
EXAMPLE:
A non-standard insurance was bought on (x) 10.5 years ago and you are asked to
estimate the reserve as of today.
You are given
b
11
= 10

11
V = 43
= 0.06
q
x+10
= 0.1
(A) Find
10.5
V assuming constant force of mortality.
(B) Find
10.5
V assuming uniform density of deaths in the year of death.
SOLUTION:
This one is denitely hard. We have to remember some stu from all the way back
in Chapter 3. For both (A) and (B), the reserve we need at t = 10.5 is enough to
pay a death benet at the end of the year for those who die, plus enough to fund
11
V = 43 for those who live. (Or you can memorize that messy formula!)
10.5
V = v
0.5
b
11 0.5
q
x+10.5
+v
0.5
11
V
0.5
p
x+10.5
(This is messy, think about why each part is right.)
(A) Assuming constant force of mortality,
y
q
x+t
= 1 p
y
x

0.5
q
x+10.5
= 1 (0.9)
0.5
= 0.0513
0.5
p
x+10.5
= 1
0.5
q
x+10.5
= 0.9487
10.5
V = e
(0.06)(0.5)
(10)(0.0513) +e
(0.06)(0.5)
(43)(0.9487)
= 0.498 + 39.59 = 40.09.
(B) Assuming UDD,
y
q
x+t
=
yq
x
1 tq
x

0.5
q
x+10.5
=
(0.5)q
x+10
1 (0.5)q
x+10
=
0.05
0.95
= 0.0526
Note: I used the Chapter 3 formula here but I tend to go right to the last step
by saying, the probability of dying in the last half of the year is the number
that die in the last half (0.05 since UDD and 0.1 die over entire year) divided
by the number alive at midyear (0.95 out of 1 alive at beginning of year).
0.5
p
x+10.5
= 1
0.5
q
x+10.5
= 0.9474
10.5
V = e
(0.06)(0.5)
(10)(0.0526) +e
(0.06)(0.5)
(43)(0.9474)
= 0.510 + 39.53 = 40.04
Arch MLC, Fall 2007 c _Yufeng Guo 205
www.archactuarial.com www.guo.coursehost.com 206
EXAMPLE:
For a non-standard insurance policy, you are given:
Deaths are uniformly distributed over the year of death,

h
= 25 for all values of h,

20
V = 250,
21
V = 265.
Find
20.25
V .
SOLUTION:
We are almost always happy to see UDD and this question is no exception: The
reserve at time t = 10.25 will be the weighted average of the beginning of year
reserve and the end of year reserve. We should be sure that we put the heavier
weight (0.75 in this case) on the reserve that is closer in time.
20.25
V = (0.75)(
20
V +
20
) + (0.25)
21
V
= (0.75)(275) + (0.25)(265) = 272.50.
8.5 The Hattendorf Theorem
Option A reference: Study Note MLC-25-05 Section 8.5 from the second printing of
Actuarial Mathematics, Second Edition
Option B reference: Models for Qualifying Risk Chapter 12.2.3
Recall from Section 8.2 that the loss function
h
L is the present value at time h of all
future losses.
h
L =
_

_
0 K(x) = 0, 1, . . . h 1
b
K(x)+1
v
K(x)+1h

K(x)

j=h

j
v
jh
K(x) = h, h + 1, . . .
In this section, we are interested in the present value of losses that occur in the next year
only. That is, we are standing at time t = h and we are interested in the present value of
losses that occur in the next year. How do we measure those losses? They are the present
value of any benets paid plus any increase in reserve minus any premiums paid. The symbol
for the present value at time h of losses incurred between times h and h + 1 is
h
. (That is
a Greek capital L. I guess L was already taken by
h
L and I have no idea why Bowers feels
the h belongs after the symbol rather than before it as with
h
L.
We will give the formula for
h
and then explain why each of the terms is correct.

h
=
_

_
0 K(x) = 0, 1, . . . h 1
v b
h+1

h
+ (V
h
) K(x) = h
(
h
) + (v
h+1
V
h
V ) K(x) = h + 1, h + 2, . . .
Arch MLC, Fall 2007 c _Yufeng Guo 206
www.archactuarial.com www.guo.coursehost.com 207
First, if K(x) < h then the insured person is already dead and no loss will be incurred
on their policy next year (seems clear enough!) so V
h
= 0.
Second, if the insured dies during the next year (i.e, if K(x) = h), then a loss equal to
the death benet will be suered but it will oset by the release of the reserve. Since
the person has died by year-end, we no longer need to hold a reserve for that policy.
Third, if the insured is still alive at the end of next year, then the loss amount is equal
to any increase in the reserve minus premium paid at the beginning of the year.
If we assume that the insured dies after time t = h (otherwise the losses for next year are
not very interesting), we have the random variable
[
h
[K(x) h]
which has the following probability distribution:
Pr [
h
= [K(x) h]
v b
h+1

h
V
h
q
x+h

h
+v
h+1
V
h
V p
x+h
Using this, we can quickly calculate that
E[
h
[K(x) h] = (v b
h+1

h
V
h
) q
x+h
+ (
h
+v
h+1
V
h
V ) p
x+h
= vb
h+1
q
x+h
+v
h+1
V p
x+h
(
h
+
h
V )
Now you can use the standard recursion relation for reserves (earlier in this chapter) to show
that this equals zero. Therefore,
E[
h
[K(x) h] = 0
The expected loss for next year equals zero given survival to today. Similar techniques
(elementary statistics) can be used to show that
Var [
h
[K(x) h] = v
2
(b
h+1

h+1
V )
2
p
x+h
q
x+h
You are required to be able to use this formula for the exam.
Earlier in this chapter, we saw that E[
h
L[K(x) h] =
h
V . The point to the Hattendorf
Theorem is that the variance of the loss function
h
L can be calculated using a recursion that
involves the variance of
h
. Sparing you the derivation (see Bowers pages 244-245), we have
Var[
h
L[K(x) h] = Var[
h
[K(x) h] +v
2
p
x+h
Var [
h+1
L[K(x) h + 1]
This is a messy formula but it is a little easier to remember to think about what it says. The
left-hand-side is the variance in present value of all future losses and the equation says that
that quantity is equal to the variance in present value of next years losses plus the variance
in present value for years after next year if you live till next year. Like the formula above,
you want to remember this one for the exam.
Arch MLC, Fall 2007 c _Yufeng Guo 207
www.archactuarial.com www.guo.coursehost.com 208
EXAMPLE:
A fully discrete 5-year term insurance on (50) with a face amount of 1000, you
have calculated the following:

h q
50+h h
V
0 0.005890 0.00
1 0.006459 1.04
2 0.007017 1.64
3 0.007633 1.73
4 0.008309 1.21
i = 0.06
For h = 4, 3, 2, nd Var [
h
L[K(50) h]
SOLUTION:
h = 4: Since the policy only lasts for 5 years, it is clear that the variance of
the loss function for the rest of the lifetime of the policy at time t = 4 (
4
L) is
equal to the variance in the loss function for next year only (
4
). Therefore
Var[
4
L[K(x) 4] = Var[
4
[K(x) 4] = v
2
(b
5

5
V )
2
p
54
q
54
b
5
= 1000 and since the policy expires at t = 5, the necessary reserve at that
time (after benets are paid) is
5
V = 0. Finally,
v =
1
1.06
= 0.9434
So we have
Var[
4
[K(x) 4] = (0.9434)
2
(1000 0)
2
(0.991691) (0.008309) = 7334
h = 3:
Var[
3
L[K(x) 3] = Var[
3
[K(x) 3] +v
2
p
53
Var [
4
L[K(x) 4]
= Var[
3
[K(x) 3] + (0.9434)
2
(0.992367)(7334)
= v
2
(b
4

4
V )
2
p
53
q
53
+ 6477
= (0.890)(1000 1.21)
2
(0.992367)(0.007633) + 6477 = 13,202
h = 2:
Var[
2
L[K(x) 2] = Var[
2
[K(x) 2] +v
2
p
52
Var [
3
L[K(x) 3]
= v
2
(b
3

3
V )
2
p
52
q
52
+v
2
p
52
(13,202) = 17,847
Two nal notes:
Arch MLC, Fall 2007 c _Yufeng Guo 208
www.archactuarial.com www.guo.coursehost.com 209
Be sure to do Problem 18 in the suggested exercises, it is good practice with topics
from earlier chapters as well as with Hattendorfs Theorem.
Dont ignore this Hattendorf material! Its on the syllabus.
Chapter 8 Suggested Problems: 14, 30, 35ab
(Solutions at archactuarial.com)
Arch MLC, Fall 2007 c _Yufeng Guo 209
www.archactuarial.com www.guo.coursehost.com 210
CHAPTER 8 Formula Summary
h
L =
_

_
0 K(x) = 0, 1, . . . h 1
b
K(x)+1
v
K(x)+1h

K(x)

j=h

j
v
jh
K(x) = h, h + 1, . . .
h
V = E [
h
L [ K(x) h] =

j=0
b
h+j+1
v
j+1
j
p
x+h
q
x+h+j

j=0

h+j
v
j
j
p
x+h
For the continuous case, the loss function at time t is given by
t
L =
_
_
_
0 T(x) t
b
T(x)
v
T(x)t

_
T(x)
t

u
v
ut
du T(x) > t
t
V =
_

0
b
t+u
v
u
u
p
x+t

x
(t +u) du
_

0

t+r
v
r
r
p
x+t
dr.
t
L = PV (future benets) PV (future premiums) ,
t
V = APV (future benets) APV (future premiums) .
It is essential to understand and remember the next two!
h
V +
h
= v b
h+1
q
x+h
+v
h+1
V p
x+h
.
(
h
V +
h
)(1 +i) = b
h+1
q
x+h
+
h+1
V p
x+h
.
Net Amount at Risk for policy year h + 1: (b
h+1

h+1
V )
h+s
V = v
1s
b
h+1 1s
q
x+h+s
+ v
1s
h+1
V
1s
p
x+h+s
.
h+s
V = (1 s) (
h
V +
h
) + (s) (
h+1
V ) (UDD)
There is a useful way of rewriting the UDD linear interpolation:
h+s
V = (1 s) (
h
V )
. .
terminal reserve at h
+(s) (
h+1
V )
. .
terminal reserve at h + 1
+ (1 s)
h
. .
unearned premium
Arch MLC, Fall 2007 c _Yufeng Guo 210
www.archactuarial.com www.guo.coursehost.com 211
Chapter 8 More Formulas
At time t = h, the variance in the present value of losses to be suered in the next year equals
Var [
h
[K(x) h] = v
2
(b
h+1

h+1
V )
2
p
x+h
q
x+h
At time t = h, the variance in the present value of losses to be suered in all future years
equals
Var[
h
L[K(x) h] = Var[
h
[K(x) h] +v
2
p
x+h
Var [
h+1
L[K(x) h + 1]
Arch MLC, Fall 2007 c _Yufeng Guo 211
www.archactuarial.com www.guo.coursehost.com 212
Past SOA/CAS Exam Questions:
1. For a fully discrete two-year term insurance of 400 on (x):
(i) i = 0.1
(ii) 400 P
1
x:2
= 74.33
(iii) 400
1
V
1
x:2
= 16.58
(iv) The contract premium equals the benet premium.
Calculate the variance of the loss at issue.
(A) 21,615 (B) 23,125 (C) 27,450 (D) 31,175 (E) 34,150
Solution:
Need to determine q
x
and q
x+1
.
Formula 7.23: 16.58 =
1
V =
400q
x+1
1.1
74.33 q
x+1
= 0.25
Formula 8.39
0
V = 0 = 400q
x
+
1
V p
x
q
x
=
(1.1)
1
V
400
1
V
= 0.17
Loss (Example 8.51):
0 400
_
1
1.1
_
74.33 = 289.30
1 400
_
1
1.1
_
2
74.33
_
1 +
1
1.1
_
= 188.68
2 74.33
_
1 +
1
1.1
_
= 141.90
Since E[L] = 0,
Var = (289.30)
2
(0.17)+(188.68)
2
(0.25)(10.17)+(141.90)
2
(10.17)(10.25) = 34, 150
Key: E
Arch MLC, Fall 2007 c _Yufeng Guo 212
www.archactuarial.com www.guo.coursehost.com 213
2. For a fully discrete whole life insurance with non-level benets on (70):
(i) The level benet premium for this insurance is equal to P
50
.
(ii) q
70+k
= q
50+k
+ 0.01, k = 0, 1, . . . , 19
(iii) q
60
= 0.01368
(iv)
k
V =
k
V
50
, k = 0, 1, . . . , 19
(v)
11
V
50
= 0.16637
Calculate b
11
, the death benet in year 11.
(A) 0.482 (B) 0.624 (C) 0.636 (D) 0.648 (E) 0.834
Solution:
11
V = (
10
V +P
50
) (1 +i) (b
11

11
V ) q
80
(1)
But by the traditional formula:
11
V
50
= (
10
V
50
+P
50
) (1 +i) (1
11
V
50
) q
60
and since
10
V =
10
V
50
and
11
V =
11
V
50
,
11
V = (
10
V +P
50
) (1 +i) (1
11
V ) q
60
(2)
(1) (2) (b
11

11
V ) q
80
= (1
11
V ) q
60
b
11
=
(1
11
V ) q
60
q
80
+
11
V
=
(1 0.16637)(0.01368)
0.02368
+ 0.16637 = 0.64796 0.648
Key: D
3. For a fully discrete 3-year endowment insurance of 1000 on (x):
i) q
x
= q
x+1
= 0.20, ii) i = 0.06, iii) 1000P
x:3
= 373.63
Calculate 1000
_
2
V
x:3

1
V
x:3
_
.
(A) 320 (B) 325 (C) 330 (D) 335 (E) 340
Arch MLC, Fall 2007 c _Yufeng Guo 213
www.archactuarial.com www.guo.coursehost.com 214
Solution:
1000
1
V
x:3
=
_
_
_
1000P
x:3
_
(1 +i) (1000q
x
)
p
x
_
_
=
_
(373.63)(1.06) (1000)(0.2)
0.8
_
= 245.06
1000
2
V
x:3
=
_
_
_
1000P
x:3
+
1
V
x:3
_
(1 +i) (1000q
x+1
)
p
x+1
_
_
=
_
(373.63 + 245.06)(1.06) (1000)(0.2)
0.8
_
= 569.76
1000
_
2
V
x:3

1
V
x:3
_
= 569.76 245.06 = 324.70
Key: B
4. For a fully discrete 20-year endowment insurance on (55):
(i) Death benets in year k are given by b
k
= (21 k), k = 1, 2, . . . , 20.
(ii) The maturity benet is 1.
(iii) Annual benet premiums are level.
(iv)
k
V denotes the benet reserve at the end of year k, k = 1, 2, . . . , 20.
(v)
10
V = 5.0
(vi)
19
V = 0.6
(vii) q
65
= 0.10
(viii) i = 0.08
Calculate
11
V .
(A) 4.5 (B) 4.6 (C) 4.8 (D) 5.1 (E) 5.3
SOLUTION:
denotes benet premium.
19
V = APV Future benets - APV future premiums
0.6 =
1
1.08
= = 0.326
11
V =
(
10
V +)(1.08) (q
65
)(10)
p
65
Arch MLC, Fall 2007 c _Yufeng Guo 214
www.archactuarial.com www.guo.coursehost.com 215
=
(5.0 + 0.326)(1.08) (0.10)(10)
1 0.10
= 5.28
Key E
5. For a 5-year fully continuous term insurance on (x):
(i) = 0.10
(ii) All the graphs below are to the same scale.
(iii) All the graphs show
x
(t) on the vertical axis and t on the horizontal axis.
Which of the following mortality assumptions would produce the highest benet reserve
at the end of year 2?
(A)
0 1 2 3 4 5
0
0.02
0.04
0.06
0.08
(B)
0 1 2 3 4 5
0
0.02
0.04
0.06
0.08
(C)
0 1 2 3 4 5
0
0.02
0.04
0.06
0.08
(D)
0 1 2 3 4 5
0
0.02
0.04
0.06
0.08
(E)
0 1 2 3 4 5
0
0.02
0.04
0.06
0.08
Arch MLC, Fall 2007 c _Yufeng Guo 215
www.archactuarial.com www.guo.coursehost.com 216
Solution:
Comparing B to D: Prospectively at time 2, they have the same future benets. At
issue, B has the lower benet premium. Thus, by formula 7.2.2, B has the higher
reserve.
Comparing A to B: Use formula 7.3.5. At issue, B has the higher benet premium.
Until time 2, they have had the same benets, so B has the higher reserve.
Comparing B to C: Visualize a graph C* that matches graph B on one side of t=2 and
matches graph C on the other side. By using the logic of the two preceding paragraphs,
Cs reserve is lower than C*s which is lower than Bs.
Comparing B to E: Reserve on E are constant at 0.
Key: B
6. For a fully discrete three-year endowment insurance of 10,000 on (50), you are given:
(i) i = 0.03
(ii) 1000q
50
= 8.32
(iii) 1000q
51
= 9.11
(iv) 10,000
1
V
50:3
= 3209
(v) 10,000
2
V
50:3
= 6539
(vi)
0
L is the prospective loss random variable at issue, based on the benet premium.
Calculate the variance of
0
L.
(A) 277,000 (B) 303,000 (C) 357,000 (D) 403,000 (E) 454,000
Solution: = 10,000v
s
V
50:3
= 9708.74 6539 = 3169.74
0
L =
_

_
10,000v a
1
= 6539 for K = 0
10,000v
2
a
2
= 3178.80 for K = 1
10,000v
3
a
3
= 83.52 for K > 1
Pr(K = 0) = q
50
= 0.00832
Pr(K = 1) = p
50
q
51
= (0.99168)(0.00911) = 0.0090342
Pr(K > 1) = 1 Pr(K = 0) Pr(K = 1) = 0.98265
Var(
0
L) = E
_
0
L
2
_
(E [
0
l])
2
= E
_
0
L
2
_
since is the benet premium,
= 0.00832 (6539)
2
+0.00903 (3178.80)
2
+0.98265 (83.52)
2
= 453,895 Key: E
Arch MLC, Fall 2007 c _Yufeng Guo 216
www.archactuarial.com www.guo.coursehost.com 217
7. For a special fully discrete 3-year term insurance on (x):
(i) Level benet premiums are paid at the beginning of each year.
(ii)
k b
k+1
q
x+k
0 200,000 0.03
1 150,000 0.06
2 100,000 0.09
(iii) i = 0.06
Calculate the initial benet reserve for year 2.
(A) 6,500 (B) 7,500 (C) 8,100 (D) 9,400 (E) 10,300
SOLUTION:
Let = benet premium.
Actuarial Present Value of benets =
= (0.03)(200,000)v + (0.97)(0.06)(150,000)v
2
+ (0.97)(0.94)(0.09)(100,000)v
3
= 5660.38 + 7769.67 + 6890.08 = 20,320.13
Actuarial Present Value of benet premiums
= a
x:3
=
_
1 + 0.97v + (0.97)(0.94)v
2
_
= 2.7266
=
20,320.13
2.7266
= 7452.55
1
V =
(7452.55)(1.06) (200,000)(0.03)
1 0.03
= 1958.46
Initial reserve, year two =
1
V + = 1958.56 + 7452.55 = 9411.01 Key:D
Arch MLC, Fall 2007 c _Yufeng Guo 217
www.archactuarial.com www.guo.coursehost.com 218
8. Lottery Life issues a special fully discrete whole life insurance on (25):
(i) At the end of the year of death there is a random drawing. With probability 0.2,
the death benet is 1000. With probability 0.8, the death benet is 0.
(ii) At the start of each year, including the rst, while (25) is alive, there is a random
drawing. With probability 0.8, the level premium is paid. With probability 0.2,
no premium is paid.
(iii) The random drawings are independent.
(iv) Mortality follows the Illustrative Life Table.
(v) i = 0.06
(vi) is determined using the equivalence principle.
Calculate the benet reserve at the end of year 10.
(A) 10.25 (B) 20.50 (C) 30.75 (D) 41.00 (E) 51.25
SOLUTION: At age x:
APV of future benets =
_
1
5
A
x
_
1000; APV of future premiums =
_
4
5
a
x
_

1000
5
A
25
=
4
5
a
25
by equivalence principle
1000
4
A
25
a
25
= =
1
4

81.65
16.2242
= 1.258
10
V = APV(Future Benets) - APV(Future Premiums)
=
1000
5
A
35

4
5
a
35
=
1
5
(128.72)
4
5
(1.258)(15.3926) = 10.25 Key: A
9. For a special fully continuous whole life insurance on (65):
(i) The death benet at time t is b
t
= 1000e
0.04t
, t 0.
(ii) Level benet premiums are payable for life.
(iii)
65
(t) = 0.02, t 0
(iv) = 0.04
Calculate
2
V , the benet reserve at the end of year 2.
(A) 0 (B) 29 (C) 37 (D) 61 (E) 83
Arch MLC, Fall 2007 c _Yufeng Guo 218
www.archactuarial.com www.guo.coursehost.com 219
SOLUTION:
At issue, actuarial present value (APV) of benets
=
_

0
b
t
v
t
t
p
65

65
(t) dt =
_

0
1000
_
e
0.04t
_ _
e
0.04t
_
t
p
65

65
(t) dt
= 1000
_

0
t
p
65

65
(t) dt = 1000

q
65
= 1000
APV of premiums= a
65
=
_
1
0.04 + 0.02
_
= 16.667
Benet premium =
1000
16.667
= 60
2
V =
_

0
b
2+u
v
u
u
p
67

65
(2 +u) du a
67
=
_

0
1000e
0.04(2+u)
e
0.04u
u
p
67

65
(2 +u) du (60)(16.667)
= 1000e
0.08
_

0
u
p
67

65
(2 +u) du 1000
= 1083.29

q
67
1000 = 1083.29 1000 = 83.29 Key: E
10. For a special fully discrete whole life insurance on (x):
(i) The death benet is 0 in the rst year and 5000 thereafter.
(ii) Level benet premiums are payable for life.
(iii) q
x
= 0.05
(iv) v = 0.90
(v) a
x
= 5.00
(vi)
10
V
x
= 0.20
(vii)
10
V is the benet reserve at the end of year 10 for this insurance.
Calculate
10
V .
(A) 795 (B) 1000 (C) 1090 (D) 1180 (E) 1225
Arch MLC, Fall 2007 c _Yufeng Guo 219
www.archactuarial.com www.guo.coursehost.com 220
SOLUTION:
v = 0.90 d = 0.10 A
x
= 1 d a
x
= 1 (0.10)(5) = 0.5
Benet premium =
5000A
x
5000vq
x
a
x
=
(5000)(0.5) 5000(0.90)(0.05)
5
= 455
10
V
x
= 1
a
x+10
a
x
0.2 = 1
a
x+10
5
a
x+10
= 4
A
x+10
= 1 d a
x+10
= 1 (0.10)(4) = 0.6
10
V = 5000A
x+10
a
x+10
= (5000)(0.6) (455)(4) = 1180 Key: D
11. For a special fully discrete whole life insurance of 1000 on (40):
(i) The level benet premium for the rst 20 years is .
(ii) The benet premium payable thereafter at age x is 1000v q
x
, x = 60, 61, . . .
(iii) Mortality follows the Illustrative Life Table.
(iv) i = 0.06
Calculate .
(A) 4.79 (B) 5.11 (C) 5.34 (D) 5.75 (E) 6.07
SOLUTION:
APV Benets = 1000A
1
40:20
+

k=20
k
E
40
1000v q
40+k
APV Premiums = a
40:20
+

k=20
k
E
40
1000v q
40+k
The equivalence principle gives us
1000A
1
40:20
+

k=20
k
E
40
1000v q
40+k
= a
40:20
+

k=20
k
E
40
1000v q
40+k
= 1000
A
1
40:20
a
40:20
=
161.32 (0.27414)(369.13)
14.8166 (0.27414)(11.1454)
= 5.11
Arch MLC, Fall 2007 c _Yufeng Guo 220
www.archactuarial.com www.guo.coursehost.com 221
12. For a special fully discrete whole life insurance on (40):
(i) The death benet is 1000 for the rst 20 years; 5000 for the next 5 years; 1000
thereafter.
(ii) The annual benet premium is 1000P
40
for the rst 20 years; 5000P
40
for the next
5 years; thereafter.
(iii) Mortality follows the Illustrative Life Table.
(iv) i = 0.06
Calculate
21
V , the benet reserve at the end of year 21 for this insurance.
(A) 255 (B) 259 (C) 263 (D) 267 (E) 271
SOLUTION:
1000P
40
=
A
40
a
40
=
161.32
14.8166
= 10.89
1000
20
V
40
= 1000
_
1
a
60
a
40
_
= 1000
_
1
11.1454
14.8166
_
= 247.78
21
V =
(
20
V + 5000P
40
) (1 +i) 5000q
60
P
60
=
(247.78 + 5(10.89)) (1.06) 5000(0.01376)
1 0.01376
= 255
Note: For this insurance,
20
V = 1000
20
V
40
because retrospectively, this is identical to
a usual whole life policy.
13. Michel, age 45, is expected to experience higher than standard mortality only at age 64.
For a special fully discrete whole life insurance of 1 on Michel, you are given:
(i) The benet premiums are not level
(ii) The benet premium for year 20,
19
, exceeds P
45
for a standard risk by 0.010.
(iii) Benet reserves on his insurance are the same as benet reserves for a fully dis-
crete whole life insurance of 1 on (45) with standard mortality and level benet
premiums.
(iv) i = 0.03
(v)
20
V
45
= 0.427
Calculate the excess q
64
for Michel over the standard q
64
.
(A) 0.012 (B) 0.014 (C) 0.016 (D) 0.018 (E) 0.020
Arch MLC, Fall 2007 c _Yufeng Guo 221
www.archactuarial.com www.guo.coursehost.com 222
SOLUTION:
Let q
64
for Michel equal the standard q
64
plus c. We need to solve for c. The recursion
formula for standard insurance can be written as
(
19
V
45
+P
45
) (1.03) = q
64
+ (1 q
64
)
20
V
45
The recursion formula for Michels insurance is the same except that 0.01 is added to
the premium and c is added to q
64
:
(
19
V
45
+P
45
+ 0.01) (1.03) = (q
64
+c) + (1 q
64
c)
20
V
45
The values of
19
V
45
and
20
V
45
are the same in the two equations since we are told that
Michels benet reserves are the same as for standard insurance.
Subtracting the second equation from the rst and rearranging terms gives:
0 = (1.03)(0.01) +c (1
20
V
45
)
c =
(1.03)(0.01)
1
20
V
45
=
0.0103
1 0.427
= 0.018
14. For a fully discrete 10-payment whole life insurance of 100,000 on (x), you are given:
(i) i = 0.05
(ii) q
x+9
= 0.011
(iii) q
x+10
= 0.012
(iv) q
x+11
= 0.014
(v) The level annual benet premium is 2078.
(vi) The benet reserve at the end of year 9 is 32,535.
Calculate 100,000A
x+11
.
(A) 34,100 (B) 34,300 (C) 35,500 (D) 36,500 (E) 36,700
Arch MLC, Fall 2007 c _Yufeng Guo 222
www.archactuarial.com www.guo.coursehost.com 223
SOLUTION:
Because no premiums are paid after year 10 for (x), we know that
11
V = A
x+11
The usual reserve recursion formula is:
(
h
V +
h
)(1 +i) = b
h+1
q
x+h
+
h+1
V p
x+h
Rearranging the terms we have
h+1
V =
(
h
V +
h
)(1 +i) b
h+1
q
x+h
p
x+h

10
V =
(32,535+2078)(1.05) 100,000(0.011)
0.989
= 35,635.642

11
V =
(35,635.642+0)(1.05) 100,000(0.012)
0.988
= 36,657.31 = A
x+11
15. For a fully discrete whole life insurance of b on (x), you are given:
(i) q
x+9
= 0.02904
(ii) i = 0.03
(iii) The initial benet reserve for policy year 10 is 343.
(iv) The net amount at risk for policy year 10 is 872.
(v) a
x
= 14.65976
Calculate the terminal benet reserve for policy year 9.
(A) 280
(B) 288
(C) 296
(D) 304
(E) 312
SOLUTION:
(
9
V +P)(1.03) = q
x+9
b + (1 q
x+9
)
10
V
= q
x+9
(b
10
V ) +
10
V
(343)(1.03) = 0.02904(872) +
10
V
10
V = 327.97
b = (b
10
V ) +
10
V = 872 + 327.97 = 1199.97
Arch MLC, Fall 2007 c _Yufeng Guo 223
www.archactuarial.com www.guo.coursehost.com 224
P = b
_
1
ax
d
_
= 1200
_
1
14.65976

0.03
1.03
_
= 46.92
9
V = initial reserve P = 343 46.92 = 296.08
Key: C
16. For a special fully discrete 2-year endowment insurance of 1000 on (x), you are given:
(i) The rst year benet premium is 668.
(ii) The second year benet premium is 258.
(iii) d = 0.06
Calculate the level annual premium using the equivalence principle.
(A) 469
(B) 479
(C) 489
(D) 499
(E) 509
SOLUTION:
d = 0.06 V = 0.94
Step 1 Determine p
x
668 + 258p
x
= 1000q
x
+ 1000
2
p
x
(p
x+1
+q
x+1
)
668 + 258(0.94)p
x
= 1000(0.94)(1 p
x
) + 1000(0.8836)p
x
(1)
668 + 242.52p
x
= 940(1 p
x
) + 883.6p
x
p
x
= 272/298.92 = 0.91
Step 2 Determine 1000P
x:2|
668 + 258(0.94)(0.91) = 1000P
x:2|
[1 + (0.94)(0.91)]
1000P
x:2|
=
[220.69+668]
1.8554
= 479
Key: B
Arch MLC, Fall 2007 c _Yufeng Guo 224
www.archactuarial.com www.guo.coursehost.com 225
17. For a fully discrete 5-payment 10-year decreasing term insurance on (60), you are given:
(i) b
k+1
= 1000(10 k), k = 0, 1, 2, . . . , 9
(ii) Level benet premiums are payable for ve years and equal 218.15 each.
(iii) q
60+k
= 0.02 + 0.001k, k = 0, 1, 2, . . . , 9
(iv) i = 0.06
Calculate
2
V , the benet reserve at the end of year 2.
(A) 70
(B) 72
(C) 74
(D) 76
(E) 78
SOLUTION:
1
V =
218.15(1.06)10,000(0.02)
10.02
= 31.88
2
V =
(31.88+218.15)(1.06)(9,000)(0.021)
10.021
= 77.66
18. For a fully discrete 3-year endowment insurance of 1000 on (x):
(i) i = 0.05
(ii) p
x
= p
x+1
= 0.7
Calculate the second year terminal benet reserve.
(A) 526
(B) 632
(C) 739
(D) 845
(E) 952
Arch MLC, Fall 2007 c _Yufeng Guo 225
www.archactuarial.com www.guo.coursehost.com 226
SOLUTION:
t p
x+t t
p
x

t

tt
p
x
0 0.7 1 1 1
1 0.7 0.7 0.95238 0.6667
2 - 0.49 0.90703 0.4444
3 - - - -
From above a
x:3
=
2

t=0

t
t
p
x
= 2.1111
1000
2
V
x:3
= 1000
_
1
a
x+2:1
a
x:3
_
= 1000
_
1
1
2.1111
_
= 526
Alternatively,
P
x:3
=
1
a
x:3
d
1000
2
V
x:3
= 1000( P
x:3
)
= 1000(0.95238 0.4261)
= 526
You could also calculate A
x:3
and use it to calculate P
x:3
.
Key: A
19. For a special fully discrete 5-year deferred whole life insurance of 100,000 on (40), you
are given:
(i) The death benet during the 5-year deferral period is return of benet premiums
paid without interest.
(ii) Annual benet premiums are payable only during the deferral period.
(iii) Mortality follows the Illustrative Life Table.
(iv) i = 0.06
(v) (IA)
1
40:5
= 0.04042
Calculate the annual benet premiums.
(A) 3300
(B) 3320
(C) 3340
(D) 3360
(E) 3380
Arch MLC, Fall 2007 c _Yufeng Guo 226
www.archactuarial.com www.guo.coursehost.com 227
SOLUTION:
a
40:5
= a
40

5
E
40
a
40
= 14.8166 (0.73529)(14.1121)
= 4.4401
a
40:5
= 100, 000A
45

5
5
p
40
+ +(IA)
1
40:5
= 100, 000A
45

5
E
40
/
_
a
40:5
(IA)
1
40:5
_
= 100, 000(0.20120)(0.73529)/(4.4401 0.04042)
= 3363
Key: D
20. For a special fully discrete whole life insurance of 1000 on (42):
(i) The contract premium for the rst 4 years is equal to the level benet premium
for a fully discrete whole life insurance of 1000 on (40).
(ii) The contract premium after the fourth year is equal to the level benet premium
for a fully discrete whole life insurance of 1000 on (42).
(iii) Mortality follows the Illustrative Life Table.
(iv) i = 0.06
(v)
3
L is the prospective loss random variable at time 3, based on the contract pre-
mium.
(vi) K(42) is the curtate future lifetime of (42).
Calculate E[
3
L[K(42) 3].
(A) 27
(B) 31
(C) 44
(D) 48
(E) 52
Arch MLC, Fall 2007 c _Yufeng Guo 227
www.archactuarial.com www.guo.coursehost.com 228
SOLUTION:
P
40
= A
40
/ a
40
= 0.16132/14.8166 = 0.0108878
P
42
= A
42
/ a
42
= 0.17636/14.5510 = 0.0121201
a
45
= a
45
1 = 13.1121
E[
3
L[K(42) 3] = 1000A
45
1000P
40
1000P
42
a
45
= 201.20 10.89 (12.12)(13.1121)
= 31.39
Key: B
Many similar formulas would work equally well. One possibility would be 1000
3
V
42
+
1000P
42
1000P
40
, because prospectively after duration 3, this diers from the normal
benet reserve in that in the next year you collect 1000P
40
instead of 1000P
42
.
21. For a special fully discrete 3-year endowment insurance on (75), you are given:
(i) The maturity value is 1000.
(ii) The death benet is 1000 plus the benet reserve at the end of the year of death.
(iii) Mortality follows the Illustrative Life Table.
(iv) i = 0.05
Calculate the level benet premium for this insurance.
(A) 321
(B) 339
(C) 356
(D) 364
(E) 373
SOLUTION:
1
V = (
0
V +)(1 +i) (1000 +
1
V
1
V ) q
75
= 1.05 1000q
75
.
Similarly,
2
V = (
1
V +) 1.05 1000q
76
.
3
V = (
2
V +) 1.05 1000q
77
.
1000 =
3
V = (1.05
3
+1.05
2
+1.05) (1000)1.05
2
q
75
(1000)1.05q
76
(1000)q

77
Arch MLC, Fall 2007 c _Yufeng Guo 228
www.archactuarial.com www.guo.coursehost.com 229
=
1000 + 1000(1.05
2
q
75
+ 1.05q
76
+q
77
)
(1.05)
3
+ (1.05)
2
+ 1.05
=
1000(1 + 1.05
2
(0.05169) + 1.05(0.05647) + 0.06168)
3.310125
=
1000 1.17796
3.310125
= 355.87
Key: C
This equation is algebraic manipulation of the three equations in three unknowns
(
1
V,
2
V, ). One method - usually eective in problems where benet = stated amount
plus reserve, is to multiply the
1
V equation by 1.05
2
, the
2
V equation by 1.05, and add
those two to the
3
V equation: in the result, you can cancel out the
1
V , and
2
V terms.
Or you can substitute the
1
V equation into the
2
V equation, giving
2
V in terms of ,
and then substitute that into the
3
V equation.
22. For a deferred whole life annuity-due on (25) with annual payment of 1 commencing at
age 60, you are given:
(i) Level benet premiums are payable at the beginning of each year during the de-
ferral period.
(ii) During the deferral period, a death benet equal to the benet reserve is payable
at the end of the year of death.
Which of the following is a correct expression for the benet reserve at the end of the
20
th
year?
(A) ( a
60
/ s
35
) s
20
(B) ( a
60
/ s
20
) s
35
(C) ( s
20
/ a
60
) s
35
(D) ( s
35
/ a
60
) s
20
(E) ( a
60
/ s
35
)
SOLUTION:
Let be the benet premium
Let
k
V denote the benet reserve a the end of year k.
For any n, (
n
V +)(1 +i) = (q
25+n

n+1
V +p
25+n

n+1
V ) =
n+1
V
Thus
1
V = (
0
V +)(1 +i)
2
V = (
1
V +)(1 +i) = ((1 +i) +)(1 +i) = s
2
Arch MLC, Fall 2007 c _Yufeng Guo 229
www.archactuarial.com www.guo.coursehost.com 230
3
V = (
2
V +)(1 +i) = ( s
2
+)(1 +i) = s
3
By induction (proof omitted)
n
V = s
n
For n = 35,
n
V = a
60
(actuarial present value of future benets; there are no future
premiums)
a
60
= s
35
=
a
60
s
35
For n = 20,
20
V = s
20
=
_
a
60
s
35
_
s
20
The technique, for situations where the death benet is a specied amount (here, 0)
plus the benet reserve is discussed in section 8.3 of Bowers. This specic problem is
Example 8.3.1.
Key: A
23. You are given:
(i)
k
V
A
is the benet reserve at the end of year k for type A insurance, which is a
fully discrete 10-payment whole life insurance of 1000 on (x).
(ii)
k
V
B
is the benet reserve at the end of year k for type B insurance, which is a
fully discrete whole life insurance of 1000 on (x).
(iii) q
x+10
= 0.004
(iv) The annual benet premium for type B is 8.36.
(v)
10
V
A

10
V
B
= 101.35
(vi) i = 0.06
Calculate
11
V
A

11
V
B
.
(A) 91
(B) 93
(C) 95
(D) 97
(E) 99
Arch MLC, Fall 2007 c _Yufeng Guo 230
www.archactuarial.com www.guo.coursehost.com 231
SOLUTION:
11
V
A
=
_
10
V
A
+ 0
_
(1 +i)
p
x+10

q
x+10
p
x+10
1000 (1)
11
V
B
=
_
10
V
B
+
B
_
(1 +i)
p
x+10

q
x+10
p
x+10
1000 (2)
Taking Equation (1) minus Equation (2) implies that
11
V
A

11
V
B
=
_
10
V
A

10
V
B

B
_
(1 +i)
p
x+10
= (101.35 8.36)
(1.06)
1 0.004
= 98.97
Key: E
Arch MLC, Fall 2007 c _Yufeng Guo 231
www.archactuarial.com www.guo.coursehost.com 232
Problems from Pre-2000 SOA-CAS exams
1. For a special fully discrete whole life insurance on (40), you are given:
The net premium for this insurance is equal to P
20
.

k
V =
k
V
20
, k = 0, 1, . . . , 19

11
V =
11
V
20
= 0.08154
q
40+k
= q
20+k
+ 0.01, k = 0, 1, . . . , 19
q
30
= 0.008427
Calculate b
11
, the death benet in year 11.
(A) 0.457 (B) 0.468 (C) 0.480 (D) 0.491 (E) 0.502
2. For a deferred temporary life annuity on (57), you are given:
= 0.04
= 0.06
The premiums are payable continuously for the rst two years at the rate of P.
Annuity benets are paid at the beginning of the year.
The following annuity payment schedule
Year 1 2 3 4 5 6 7 8 9+
Annuity Benet 0 0 0 10 8 6 4 2 0
Calculate the reserve at the end of year 3.
(A) 23.95 (B) 24.95 (C) 25.45 (D) 25.95 (E) 26.45
3. For a fully discrete 10-payment whole life insurance of 1000 on (x), you are given:
i = 0.06
q
x+9
= 0.01262
The annual benet premium is 32.88.
The benet reserve at the end of year 9 is 322.87.
Calculate 1000 P
x+10
.
(A) 31.52 (B) 31.92 (C) 32.32 (D) 32.72 (E) 33.12
Arch MLC, Fall 2007 c _Yufeng Guo 232
www.archactuarial.com www.guo.coursehost.com 233
Use the following information for the next 3 questions (note we have revised this
question to make interest assumptions consistent with the current version of the exam):
For a special fully discrete 20-year endowment insurance on (68), you are given:
Mortality follows the Illustrative Life Table.
i = 0.06
Net premiums, for k ,= 5, are given by

k
= 1000P
68
, k = 0, 1, 2, 3, 4, 6, 7, . . . , 19

5
> 0
Death benets, for k + 1 ,= 16, are given by
b
k+1
= 1000, k + 1 = 1, 2, . . . , 14, 15, 17, 18, 19, 20
b
16
> 0
The endowment benet is 1000.
The following values, based on the Illustrative Life Table at 6% interest:
1000P
68
= 53.21 a
68:20
= 8.8325
1000A
68:20
= 500.05 a
85:3
= 2.501
4. Calculate
17
V .
(A) 489 (B) 509 (C) 587 (D) 715 (E) 725
5. Calculate
2
V .
(A) 59 (B) 67 (C) 80 (D) 91 (E) 99
6. For this question only, you are given additional information:
5
V = 148.00 and
6
V = 282.00
Calculate
5
.
(A) 147 (B) 152 (C) 157 (D) 162 (E) 167
Arch MLC, Fall 2007 c _Yufeng Guo 233
www.archactuarial.com www.guo.coursehost.com 234
Use the following information for the next 4 questions:
The random variable
k
L is the prospective loss at time k for a fully discrete 3-year
endowment insurance of 3 on (x).
You are given:
i = 0.10
q
x
= 0.009
The premium is 3P
x:3
= 0.834

k 3
k
V
x:3
1 0.898
2 1.893
3 3.000
7. Calculate q
x+1
.
(A) 0.007 (B) 0.011 (C) 0.015 (D) 0.019 (E) 0.023
8. Calculate
0
L, given that (x) dies in the rst year.
(A) 0.08 (B) 0.23 (C) 0.63 (D) 1.25 (E) 1.89
9. Calculate Var [
1
L].
(A) 0.011 (B) 0.016 (C) 0.021 (D) 0.026 (E) 0.031
10. Calculate Var [
0
].
(A) 0.016 (B) 0.024 (C) 0.033 (D) 0.042 (E) 0.054
11. For a fully discrete whole life insurance of 1 on (30), you are given:
Var [
10
L[K(x) 10] = y +v
2
p
40
Var [
11
L[K(x) 11]
Mortality follows the illustrative live table.
i = 0.06
Calculate y.
(A) 0.0022 (B) 0.0023 (C) 0.0024 (D) 0.0025 (E) 0.0026
Arch MLC, Fall 2007 c _Yufeng Guo 234
www.archactuarial.com www.guo.coursehost.com 235
Solutions to Pre-2000 Problems: Chapter 8
1. Key: E
11
V = (
10
V +P
20
) (1 +i) (b
11

11
V ) q
50
(1)
11
V
20
= (
10
V
20
+P
20
) (1 +i) (1
11
V
20
) q
30
Also,
10
V =
10
V
20
and
11
V =
11
V
20
So,
11
V = (
10
V +P
20
) (1 +i) (1
11
V ) q
30
(2)
Subtracting equations (2) from equation (1) gives
(b
11

11
V ) q
50
= (1
11
V ) q
30
b
11
=
(1
11
V ) q
30
q
50
+
11
V
=
(1 0.08154)(0.008427)
0.018427
+ 0.08154 = 0.502
2. Key: E
Reserve = PV[Future Benets] - PV[Future Premiums]
There are no future premiums at time 3, so the reserve equals
10 + 8vp
60
+ 6v
2
2
p
60
+ 4v
3
3
p
60
+ 2v
4
4
p
60
= 10 + 8e
0.06
e
0.04
+ 6
_
e
0.06
e
0.04
_
2
+ 4
_
e
0.06
e
0.04
_
3
+ 2
_
e
0.06
e
0.04
_
4
= 10 + 8e
0.1
+ 6e
0.2
+ 4e
0.3
+ 2e
0.4
= 10 + 7.239 + 4.912 + 2.963 + 1.341 = 26.45
Arch MLC, Fall 2007 c _Yufeng Guo 235
www.archactuarial.com www.guo.coursehost.com 236
3. Key: E
10
V = 1000 A
x+10
=
(
9
V +P)(1.06) q
x+9
(1000)
p
x+9
=
(322.87 + 32.88)(1.06) 12.62
0.98738
= 369.13
a
x+10
=
1 0.36913
0.06
1.06
= 11.14537
1000 P
x+10
=
1000 A
x+10
a
x+10
=
369.13
11.14537
= 33.1196
4. Key: E
17
V = 1000
_
A
85:3
P
68
a
85:3
_
A
85:3
= 1 d a
85:3
= 1 (0.0566)(2.501) = 0.85844

17
V = 858.44 53.21(2.501) = 725.36
5. Key: A
Since the premiums and benets for the rst 2 years are the same as for a whole life
insurance on (68), the reserve at the end of year should be the same.
2
V = 1000
2
V
68
= 1000
_
1
a
70
a
68
_
From the illustrative life table, this equals
= 1000
_
1
8.5693
9.1066
_
= 59.00
6. Key: A
(
5
V +
5
) (1 +i) = p
73
(
6
V ) + 1000 q
73

5
=
p
73 6
V + 1000 q
73
1 +i

5
V
=
(0.9567)(282) + 1000(0.0433)
1.06
148 = 147.37
Arch MLC, Fall 2007 c _Yufeng Guo 236
www.archactuarial.com www.guo.coursehost.com 237
7. Key: B
1
V +P = 3v q
x+1
+
2
V v p
x+1
(0.898 + 0.834) =
3
1.1
q
x+1
+
1.893
1.1
(1 q
x+1
)
1.732
1.893
1.1
=
3 1.893
1.1
q
x+1
q
x+1
= 0.0110
8. Key: E
0
L = 3v
1
P
x:3
a
1
=
3
1.1
0.834 = 1.89
9. Key: A
Var[
1
L[K(x) 1] = Var[
1
[K(x) 1] +v
2
p
x+1
Var [
2
L[K(x) 2]
But since this is endowment insurance, a person alive at time t = 2 will receive the
same payment at the end of the year whether they live or die. Therefore,
Var [
2
L[K(x) 2] = 0
and
Var[
1
L[K(x) 1] = Var[
1
[K(x) 1] = v
2
(b
2

2
V )
2
p
x+1
q
x+1
Two problems earlier, we saw that q
x+1
= 0.011 so that p
x+1
= 0.989 and
Var[
1
L[K(x) 1] =
_
1
1.1
_
2
(3 1.893)
2
(0.989)(0.011) = 0.011
10. Key: C
Since survival to time zero is guaranteed:
Var[
0
] = Var[
0
[K(x) 0] = v
2
(b
1

1
V )
2
p
x
q
x
=
_
1
1.1
_
2
(3 0.898)
2
(0.991)(0.009) = 0.033
Arch MLC, Fall 2007 c _Yufeng Guo 237
www.archactuarial.com www.guo.coursehost.com 238
11. Key: C
Var[
10
L[K(x) 10] = Var[
10
[K(x) 10] +v
2
p
40
Var [
11
L[K(x) 11]
So
y = Var [
10
[K(x) 10] = v
2
(b
11

11
V
30
)
2
p
40
q
40
11
V
30
= 1
a
41
a
30
= 0.0885
y =
_
1
1.06
(1 0.0885)
_
2
(0.99722)(0.00278) = 0.0024
Arch MLC, Fall 2007 c _Yufeng Guo 238
Chapter 7
ACTUARIAL MATHEMATICS:
CHAPTER 9 MULTIPLE LIFE
FUNCTIONS
Option A reference: Actuarial Mathematics Chapter 9
Option B reference: Models for Quantifying Risk Chapter 13
So far, we have developed the basic ideas of actuarial mathematics in the context of just one
insured life. We now expand that theory to cover multiple lives. In this chapter there are
no new actuarial concepts to learn. However, you have to understand the single life material
covered in earlier chapters in order to grasp the multiple life functions. A lot of the multiple
life material parallels the single life material very closely, so it shouldnt be too strenuous a
chapter, though it is long!
An important idea to remember as we go through Chapter 9 is that of independence among
lives. For the most part, this chapter considers multiple lives to be independent. While this
is not likely to be true for a group of people buying a policy together, it has historically been
assumed for the sake of convenience. As a student, be thankful for this assumption! It saves
you tons of headaches.
Instead of discussing a life, we now consider a status. Think of two people on a desert
island. We can dene two types of common status. First, a joint life status survives as
long as both people survive on the island. As soon as one person dies (or is voted o the
island), the joint life status terminates. Alternatively, a last survivor status survives as
long as at least one of the people remains alive on the island. We will come back to this idea,
but be familiar with the thought of a status continuing or terminating, as opposed to a life
continuing or terminating.
9.2 Joint Distributions of Future Lifetimes
Option A reference: Actuarial Mathematics Chapter 9.2
239
www.archactuarial.com www.guo.coursehost.com 240
Option B reference: Models for Quantifying Risk Chapter 13.1
We are going to bypass the theory behind joint distributions and just work a couple of
examples. The solutions will involve enough explanation so that you should have the idea
after working through them.
One important concept from this section is that of the joint survival function. Recall that,
in the single life case, the survival function is
s
T(x)
(t) = s(t) = Pr [T(x) > t] =
t
p
x
.
Similarly, joint survival function is
s
T(x)T(y)
(s, t) = Pr [T(x) > t and T(y) > s].
Most of the time, when we encounter this function, we will actually be interested in the
probability that both (x), and (y) survive t years, that is
t
p
xy
= s
T(x)T(y)
(t, t) = Pr [T(x) > t and T(y) > t].
EXAMPLE:
(x) and (y) are independent lives. The force of mortality for each is
x
(t) =
1
50t
for 0 < t < 50. Find the joint survival function for T(x) and T(y): s
T(x)T(y)
(t, t).
SOLUTION:
First nd s
T(x)
(t) and s
T(y)
(t) and then we can just multiply since the lives are
independent. We know from Chapter 3 that s(t) = e

_
(t)dt
,
s
T(x)
(t) = e

_
t
0
1
50
d
= e
[ln(50)]
t
0
= e
ln[
50t
50
]
=
50 t
50
s
T(x)T(y)
(t, t) = s
T(x)
(t) s
T(y)
(t) =
_
50 t
50
_
2
.
EXAMPLE:
Lives (x) and (x +3) are independent. If
3
q
x
= 0.08 and
6
q
x
= 0.2, nd the value
of
3
p
xy
.
SOLUTION:
Since the lives are independent,
3
p
xy
= Pr [T(x) > t and T(y) > t] = Pr[T(x) > t] Pr[T(y) > t] =
3
p
x 3
p
x+3
.
3
p
x
= 1
3
q
x
= 0.92 and
6
p
x
= 0.8.
We also need
3
p
x+3
. To get it we notice that
6
p
x
=
3
p
x

3
p
x+3
.
So
3
p
x+3
= 0.87, and
3
p
xy
= (0.92)(0.87) = 0.80.
Arch MLC, Fall 2007 c _Yufeng Guo 240
www.archactuarial.com www.guo.coursehost.com 241
9.3 Joint Life Status
Option A reference: Actuarial Mathematics Chapter 9.3
Option B reference: Models for Quantifying Risk Chapter 13.1
A joint life status survives as long as all members of a set survive, then terminates upon
the rst death. The notation that follows parallels the single life case: for (x), we have
t
p
x
,
while for (xy), a two life status, we have
t
p
xy
. The quantity
t
p
xy
represents the probability
that the joint-life status (x, y) will survive for at least t years. In less mathematical and more
intuitive terms,
t
p
xy
is the probability that both (x) and (y) will survive for at least the next
t years.
Joint-Life: Cumulative Distribution Function
The distribution function for T, the time to failure of the joint-life status, is
F
T
(t) = Pr min [T(x), T(y)] t
=
t
q
xy
= Pr (T t)
= 1 s
T(x)T(y)
(t, t).
When T(x) and T(y) are independent, (and on the exam, this will almost always be
the case), we can express the above relations in terms of single life functions:
F
T
(t) =
t
q
xy
= 1
t
p
x t
p
y
=
t
q
x
+
t
q
y

t
q
x t
q
y
This tells us that when (x) and (y) are independent,
t
q
xy
=
t
q
x
+
t
q
y

t
q
x t
q
y
.
This says that the probability that at least one of (x) and (y) will die in the next t years
is equal to the probability that (x) will die plus the probability that (y) will die minus the
probability that both will die. (We have to subtract out the probability that both will die
because it has been counted in both
t
q
x
and
t
q
y
.)
The following is important!
So we have established that if (x) and (y) are independent lives then
t
p
xy
=
t
p
x t
p
y
and
t
q
xy
=
t
q
x
+
t
q
y

t
q
x t
q
y
.
Both of these relations are extremely important and you must be ready to produce them at
any moment! Because the one for ps is simpler, you could just remember that one and then
get the other if you need it using
t
q
xy
= 1
t
p
xy
= 1
t
p
x t
p
y
= 1 (1
t
q
x
)(1
t
q
y
) =
t
q
x
+
t
q
y

t
q
x t
q
y
.
If that doesnt seem quick and easy to you, denitely remember both relations. They can be
intuitive if you think about them.
Arch MLC, Fall 2007 c _Yufeng Guo 241
www.archactuarial.com www.guo.coursehost.com 242
Note the basic idea we have discovered so far:
GENERAL (POSSIBLY DEPENDENT) CASE = DIFFICULT!
INDEPENDENT CASE = MUCH EASIER!
Fortunately, questions on the exam tend to be of the easier variety.
EXAMPLE:
x and y are independent lives with p
x
= 0.9, and q
xy
= 0.15.
Find q
y
.
SOLUTION:
t
q
xy
=
t
q
x
+
t
q
y

t
q
x t
q
y
=0.15 = 0.1 +
t
q
y
(0.1)
t
q
y

t
q
y
= 0.056.
EXAMPLE:
x and y are independent lives:
n
p
x
= 0.75,
n
p
y
= 0.60.
1. What is the probability that exactly one of (x) and (y) will be alive at the
end of n years?
2. What is the probability that at least one of (x) and (y) will die in the next
n years?
3. What is the probability that (x) will survive the next n years and y will not?
4. What is the probability that both lives will die in the next n years?
SOLUTION:
1. Either x lives and y dies or y lives and x dies. The probability of this is
n
p
x n
q
y
+
n
q
x n
p
y
= (0.75)(0.4) + (0.25)(0.6) = 0.45.
2.
n
q
xy
=
n
q
x
+
n
q
y

n
q
x n
q
y
= (0.25) + (0.4) (0.25)(0.4) = 0.55.
3.
n
p
x

n
q
y
= (0.75)(0.4) = 0.3.
4.
n
q
x

n
q
y
= (0.25)(0.4) = 0.1.
Useful Formula: Just as

e
x
, the expected future lifetime of (x), is given by

e
x
=
_

0
t
p
x
dt,
the complete expectation for the joint-life status,

e
xy
, is given by

e
xy
=
_

0
t
p
xy
dt.
Arch MLC, Fall 2007 c _Yufeng Guo 242
www.archactuarial.com www.guo.coursehost.com 243
EXAMPLE:
x and y are independent lives that have constant forces of mortality equal to

x
= 0.03 and
y
= 0.05, respectively.
Find

e
xy
.
SOLUTION:

e
xy
=
_

0
t
p
xy
dt
Since the lives are independent, this equals
_

0
t
p
x t
p
y
dt =
_

0
e
(0.03)t
e
(0.05)t
dt =
_

0
e
(0.08)t
dt =
1
0.08
= 12.5
This means that the expected future time before at least one of x and y dies is
12.5 years.
EXAMPLE:
(50) and (60) are independent lives that are both subject to DeMoivres Law for
mortality with = 100.
Find

e
50:60
.
SOLUTION:

e
xy
=
_

0
t
p
x

t
p
x
dt
For DeMoivres Law,
t
p
x
=
x t
x
.

e
50:60
=
_

0
t
p
50

t
p
60
dt.
Since they both can stay alive for at most 40 years ( = 100) we can put 40 for
the upper limit of integration. Along with substituting for
t
p
x
and
t
p
y
, this gives

e
50:60
=
_
40
0
_
50 t
50
__
40 t
40
_
dt
There is a tendency to shy away from a problem like this once you see it has an
integral, but most of the integrals they give on the exam are polynomials or a
simple exponential. We denitely want to do these!
=
1
2000
_
40
0
_
2000 90t +t
2
_
dt =
1
2000
_
2000t 45t
2
+
1
3
t
3
_
40
0
=
1
2000
[80,000 72,000 + 21,333] = 14.66.
Arch MLC, Fall 2007 c _Yufeng Guo 243
www.archactuarial.com www.guo.coursehost.com 244
Joint-Life: Probability Distribution Function and Force of Failure
INDEPENDENT CASE: Recall from Chapter 3 that, in the single life case,
f
T(x)
(t) = f(t) =
t
p
x

x
(t) =
t
p
x
(x +t).
Similarly,
f
T(xy)
(t) =
t
p
xy

xy
(t) =
t
p
x t
p
y

xy
(t).
It just so happens, that when (x) and (y) are independent,

xy
(t) = (x +t) +(y +t) (Remember This!).
This says that the Force of Failure for the joint-life status (x, y) is equal to the sum of the
forces of mortality for (x) and (y). Therefore,
f
T(xy)
(t) =
t
p
x t
p
y
((x +t) +(y +t)) .
That takes care of the PDF and force of failure in the independent case. In the general case,
the force of failure is given by
GENERAL CASE: The PDF is very messy! (I.e., Forgetaboutit!)
Force of Failure see below.

xy
(t) =
f
T(xy)
(t)
1 F
T(xy)
(t)
This is just like

x
(t) =
f(t)
t
p
x
from Chapter 3. This form doesnt simplify any further so a question of this type for joint
life distributions would have to be somewhat straightforward.
EXAMPLE:
(50) and (60) are independent lives that are both subject to DeMoivres Law for
mortality with = 100.
Find
xy
(t).
SOLUTION:

xy
(t) = (x +t) +(y +t)
Now we can either remember for DeMoivres law that
(x +t) =
1
x t
or we can derive it using
t
p
x
=
x t
x
and (x +t) =

d
dt
t
p
x
t
p
x
.
In any case,
xy
(t) =
1
50 t
+
1
40 t
=
90 2t
(50 t)(40 t)
.
Arch MLC, Fall 2007 c _Yufeng Guo 244
www.archactuarial.com www.guo.coursehost.com 245
EXAMPLE:
The lives (x) and (y) have constant forces of mortality. You are given that

e
x
= 20
and

e
y
= 40.
Find

e
xy
.
SOLUTION:
20 =

e
x
=
1

x
(since constant force of mortality)

x
= 0.05.
Similarly,
y
= 0.025. Now, the joint status (xy) also has a constant force of
mortality equal to
xy
= 0.05 + 0.025 = 0.075

e
xy
=
1
0.075
= 13.33.
This approach is a lot faster than using integrals (see the constant force of mor-
tality question above, which you ought to be able to more quickly now!)
Curtate Life Functions
The text just includes a few blurbs about curtate life functions and there doesnt seem to
be anything new that you dont already know as long as you remember that for independent
lives,
k
p
xy
=
k
p
x k
p
y
,
k
q
xy
=
k
q
x
+
k
q
y

k
q
x k
q
y
.
The probability that a joint-life status terminates during the period (k, k+1) can be expressed
as follows:
Pr(k < T k + 1) = Pr(T k + 1) Pr(T k)
=
k
p
xy

k+1
p
xy
=
k
p
xy
q
x+k:y+k
.
(This is nothing new, and dont let the colon in x+k : y +k disturb you! The authors should
really have included it in the xy notation as well. You can ignore it.) As always, keep in
mind the English translation of each line. The second line of the equation says that for the
rst death to occur in the period (k, k + 1), both (x) and (y) rst have to survive k years
(
k
p
xy
) but they cannot both survive k +1 years. The third line can be interpreted similarly:
For the rst death to occur in (k, k +1) both must survive k years and then at least one of
the two must die in the k + 1st year.
Again, the independent assumption is very nice in that
q
x+k:y+k
= q
x+k
+q
y+k
q
x+k
q
y+k
.
Arch MLC, Fall 2007 c _Yufeng Guo 245
www.archactuarial.com www.guo.coursehost.com 246
Recall that K is the number of complete years completed prior to failure for the status. The
probability function (pf ) is as follows:
Pr(K = k) =
k
p
xy
q
x+k:y+k
=
k|
q
xy
.
And nally, the curtate complete expectation of life e
x
has the expected formula based on
that from Chapter 3 for single lives:
e
xy
= E[K(xy)] =

k=0
k+1
p
xy
.
The text gives very little attention to the curtate functions, so understanding these formulas
should be all you need to know as long as you remember what you already know for single
life functions.
9.4 Last Survivor Status
Option A reference: Actuarial Mathematics Chapter 9.4
Option B reference: Models for Quantifying Risk Chapter 13.2
This type of status exists as long as at least one member of the set is alive. The status
terminates upon the nal death of the set of members.
The Following is Important!
Here is a key point to this chapter that is new to the joint life functions: suppose you have
two individual lives (x) and (y) represented by remaining life functions T(x) and T(y). Then
T(xy) and T(xy) represent both lives. The quantity T(xy) represents the remaining time
until the rst death, while T(xy) represents the remaining time until the last death. No
matter how things turn out for these two lives, T(xy) will be the time until one of the deaths
and T(xy) will be the time until the other death. However, the exact same statement can be
made of T(x) and T(y)! Therefore, it must be true that
T(xy) +T(xy) = T(x) +T(y) ((A) A key Relation!).
All of the following must be true for exactly the same reasons:
T(xy) T(xy) = T(x) T(y), (B)
c
T(xy)
+c
T(xy)
= c
T(x)
+c
T(y)
for c > 0, (C)
F
T(xy)
(t) +F
T(xy)
(t) = F
T(x)
(t) +F
T(y)
(t), (D)
f
T(xy)
(t) +f
T(xy)
(t) = f
T(x)
(t) +f
T(y)
(t), (E)
t
p
xy
+
t
p
xy
=
t
p
x
+
t
p
y
. (F)
Arch MLC, Fall 2007 c _Yufeng Guo 246
www.archactuarial.com www.guo.coursehost.com 247
EXAMPLE:
You are given:
(40) and (50) are independent lives
Mortality is as given in the Illustrative Life Table.
Find
20
p
40:50
.
SOLUTION:
t
p
xy
=
t
p
x
+
t
p
y

t
p
xy
=
t
p
x
+
t
p
y

t
p
x t
p
y
20
p
40:50
=
20
p
40
+
20
p
50
(
20
p
40
) (
20
p
50
)
20
p
40
=
l
60
l
40
=
81,881
93,132
= 0.879, similarly,
20
p
50
= 0.739.

20
p
40:50
= 0.879 + 0.739 (0.879)(0.739) = 0.968.
Arch MLC, Fall 2007 c _Yufeng Guo 247
www.archactuarial.com www.guo.coursehost.com 248
EXAMPLE:
1. Assuming the future lifetimes of (x) and (y) are independent, write
k
p
xy
in
terms of
k
p
x
and
k
p
y
.
2. Assuming the future lifetimes of (x) and (y) are independent, write
k
q
xy
in
terms of
k
q
x
and
k
q
y
.
SOLUTION:
1. The probability that at least one of (x) and (y) will be alive after k years
is the probability that (x) will be alive plus the probability that (y) will be
alive minus the probability that both will be alive. Or,
k
p
xy
=
k
p
x
+
k
p
y

k
p
x k
p
y
.
2. Both have to die,
t
q
xy
=
k
q
x k
q
y
.
9.5 More Probabilities and Expectations
Option A reference: Actuarial Mathematics Chapter 9.5
Option B reference: Models for Quantifying Risk Chapter 13.1.7, 13.2.2
In this section, the text introduces a few new formulas. No big new ideas here just some good
formulas that the SOA could put on the exam (Remember from Chapter 3 that

e
x
= E[T(x)]):
Continuous Curtate

e
xy
=
_

0
t
p
xy
dt e
xy
=

1
k
p
xy

e
xy
=
_

0
t
p
xy
dt e
xy
=

1
k
p
xy

e
xy
=

e
x
+

e
y


e
xy
e
xy
= e
x
+e
y
e
xy
Formulas for the Variance of the future lifetime (in the continuous case) are
Var[T(xy)] = E[T(xy)
2
] E[T(xy)]
2
= 2
_

0
t
t
p
xy
dt (

e
xy
)
2
,
Var[T(xy)] = E[T(xy)
2
] E[T(xy)]
2
= 2
_

0
t
t
p
xy
dt (

e
xy
)
2
.
EXAMPLE: Mr. and Mrs. Moneybags
Mr. and Mrs. Moneybags have promised to donate their entire fortune to the
national home for unwanted cats after they both have died. They are ages 50 and
Arch MLC, Fall 2007 c _Yufeng Guo 248
www.archactuarial.com www.guo.coursehost.com 249
60 respectively, and their mortality is given by Demoivres Law with = 100.
How long can the home for unwanted cats expect to wait before receiving their
donation? (Hint: we saw in an earlier problem that, for this pair,

e
50:60
= 14.66.)
SOLUTION:
We are looking for

e
50:60
, the expected future lifetime of the status (xy). We know
that

e
50:60
=

e
50
+

e
60


e
50:60
.

e
50
=
_
50
0
t
p
50
dt =
_
50
0
50 t
50
dt =
_
50
0
50 t
50
dt = 25
(Or we can use the fact that under DeMoivre,

e
x
=
x
2
.) Similarly,

e
60
= 20.

e
50:60
= 20 + 25 14.66 = 30.34.
EXAMPLE: Mr. and Mrs. Moneybags
For Mr. and Mrs. Moneybags, write an integral expression for Var[T(xy)]. The
integrand should be a function of t only.
SOLUTION:
Var[T(xy)] = 2
_

0
t
t
p
xy
dt (

e
xy
)
2
= 2
_
40
0
t
_
50 t
50
__
40 t
40
_
dt (14.66)
2
That will do it - note that there is no reason we are unable to do this integral, it
is just a polynomial if we multiply it out.
Later in the chapter it will be useful to notice that many of these formulas are exactly the
same for a single-life status (Chapters 3 and 4), joint-life status, and last survivor status,
except that the status (x, xy, xy) changes. It is useful to recognize that, in general, a status
can be denoted u. Then, write whatever the function is (A
x
, Var[T(x)], etc.) with u as the
single-life status. Then depending on which case youre using, plug in either u = xy or
u = xy into the formula. So, if
Var[T(u)] = 2
_

0
t
t
p
u
dt
_

e
u
_
2
,
then
Var[T(xy)] = 2
_

0
t
t
p
xy
dt (

e
xy
)
2
.
We will see more of this later.
Arch MLC, Fall 2007 c _Yufeng Guo 249
www.archactuarial.com www.guo.coursehost.com 250
A sneaky question the SOA could pose deals with the covariance of T(xy) and T(xy). Here
is what you need to know:
Cov[T(xy), T(xy)] = Cov[T(x), T(y)] +
_
E[T(x)] E[T(xy)]
_

_
E[T(y)] E[T(xy)]
_
= Cov[T(x), T(y)] + (

e
x


e
xy
)(

e
y


e
xy
).
This formula applies to both dependent lives and independent lives. Hopefully, any exam
question will be in the case where (x) and (y) are independent lives, in which case the
covariance term on the right-hand-side is 0. The formula for this is:
Cov[T(xy), T(xy)] = (

e
x


e
xy
)(

e
y


e
xy
).
A question using this formula is more likely than the one above.
EXAMPLE: Mr. and Mrs. Moneybags
For Mr. and Mrs. Moneybags, let T
1
be the random variable representing the
time of the rst death, and let T
2
be the random variable representing the time
of the second death. Find Cov [T
1
, T
2
].
SOLUTION:
We want
Cov[T(xy), T(xy)] = (

e
x


e
xy
)(

e
y


e
xy
) = (25 14.66)(20 14.66) = 55.22
Note that this means T(xy) and T(xy) are positively correlated. This makes
sense, if T(xy) is small (meaning the rst death occurred quickly), it increases
the chance that T(xy) will be small.
EXAMPLE: Mr. and Mrs. Moneybags
Mr. and Mrs. Moneybags, have amended their will such that a donation will be
made to the national home for cats only if the rst death occurs between 10 and
20 years from now.
What is the probability that the national home for cats will get a donation from
the Moneybags?
SOLUTION:
We want the probability that the rst death does not occur in the next ten years
but does occur in the next twenty years. This is
10
p
u

20
p
u
,
Arch MLC, Fall 2007 c _Yufeng Guo 250
www.archactuarial.com www.guo.coursehost.com 251
where u = xy. So the answer is
10
p
50:60

20
p
50:60
=
10
p
50 10
p
60

20
p
50 20
p
60
.
By recalling that for DeMoivre,
t
p
x
=
xt
x
, we get that
10
p
50:60

20
p
50:60
= (0.8)(0.75) (0.6)(0.5) = 0.3.
Note that we used the fact that the lives were independent. Thats ok because
each persons mortality is given by DeMoivre regardless of when the other dies.
So the lives are independent.
EXAMPLE: Mr. and Mrs. Moneybags
Mr. and Mrs. Moneybags, have amended their will yet again such that a donation
will be made to the national home for cats only if the second death occurs between
10 and 20 years from now.
What is the probability that the national home for cats will get a donation from
the Moneybags?
SOLUTION:
We want the probability that the second death does not occur in the next 10 years
but does occur in the next 20 years. Again this is
10
p
u

20
p
u
.
But this time, u = xy. So we want
10
p
50:60

10
p
50:60
= (
10
p
50
+
10
p
60

10
p
50 10
p
60
) (
20
p
50
+
20
p
60

20
p
50 20
p
60
)
= [0.8 + 0.75 (0.8)(0.75)] [0.6 + 0.5 (0.6)(0.5)] = 0.15
9.6 Dependent Lifetime Models
9.6.1 Common Shock (Non-Theoretical Version)
Option A reference: Actuarial Mathematics Chapter 9.6
Option B reference: Models for Quantifying Risk Chapter 13.6
Well skip the theoretical aspects of this section and concentrate on what you are likely to
need for the exam.
Arch MLC, Fall 2007 c _Yufeng Guo 251
www.archactuarial.com www.guo.coursehost.com 252
Symbols Meaning
T

(x), T

(y) What the future lifetimes of (x) and (y)


would be if there were no common peril.
These two are independent.
Z What the future lifetime of both
would be if the only way to die was
the common peril
T(x), T(y) The actual future lifetimes of (x) and (y)
with both the common peril and
everything else. These two are not
independent of each other.
T

(x) and T

(y) refer to what the future lifetimes of (x) and (y) would be if they were
completely independent and there were no common perils (i.e., just like everything we have
looked at before this section!) The symbol Z represents what either of their lifetimes would
be if the only peril they faced in life was the common peril. Since each of (x) and (y) must
in fact face the common peril and everything else in life that could kill them, the resulting
survival function is the product of the survival functions for 1) the common peril, and 2)
everything else. The result is
s
T(x)
(t) = s
T

(x)
(t) s
Z
(t) = s
T

(x)
(t) e
t
,
s
T(y)
(t) = s
T

(y)
(t) s
Z
(t) = s
T

(y)
(t) e
t
,
s
T(x)T(y)
= s
T

(x)
(t) s
T

(y)
(t) s
Z
(t) = s
T

(x)
(t) s
T

(y)
(t) e
t
.
But wait a minute!, you may say, I know full well that
s
T(x)T(y)
= s
T(x)
(t) s
T(y)
(t)
since that is just
t
p
xy
=
t
p
x t
p
y
!!
Not Quite! That is true only when (x) and (y) are independent lives. The common
shock assumption makes (x) and (y) have dependent future lifetimes because they are exposed
to a common peril.
EXAMPLE:
In the presence of a common shock peril, write s
T(xy)
(t) in terms of s
T

(x)
(t),
s
T

(y)
(t), and s
Z
(t).
SOLUTION:
Arch MLC, Fall 2007 c _Yufeng Guo 252
www.archactuarial.com www.guo.coursehost.com 253
We know that
s
T(xy)
(t) = s
T(x)
(t) +s
T(y)
(t) s
T(xy)
(t).
s
T(xy)
(t) = s
T

(x)
(t) e
t
+s
T

(y)
(t) e
t
s
T

(x)T

(y)
(t) e
t
= [s
T

(x)
(t) +s
T

(y)
(t) s
T

(x)T

(y)
(t)]e
t
.
Recall from Section 9.4 that if T(x) and T(y) are independent, then

xy
(t) = (x +t) +(y +t).
Since, in the context of common shock, T

(x), T

(y), and Z are all independent, it makes


sense that

xy
(t) = (x +t) +(y +t) +.
(Note that =
Z
(t).) So at each moment, the joint life status is threatened by 3 dierent
forces of mortality.
Notational Comment: Make sure you recognize that, throughout this section, the normal
random variables i.e., with no common shock associated are designated with a *. This
seems a little backwards, since in previous chapters there was no * in use.
EXAMPLE:
(x) and (y) are subject to constant forces of mortality
x
= 0.02 and
y
= 0.03
respectively. In addition, both lives are subject to a common shock peril with
= 0.01.
Find 1.
8
p
xy
, and 2.

e
xy
.
SOLUTION:
1.
xy
(t) = 0.02 + 0.03 + 0.01 = 0.06, a constant. Therefore,
8
p
xy
= e
0.06(8)
= 0.619.
2. Since
xy
is a constant,

e
xy
=
1
0.06
= 16.67
EXAMPLE:
(x) and (y) are subject to constant forces of mortality
x
= 0.02 and
y
= 0.03
respectively. In addition, both lives are subject to a common shock peril with
= 0.01.
Find the probability that the common peril will end the pair of lives.
SOLUTION:
For the common peril to cause the failure of the joint life status at time t. The pair
must survive all perils up to time t and then get hit by the force of mortality .
The probability that this will happen at some time t is
_

0
t
p
xy
dt =
_

0
e
(x+y+)
dt =

x
+
y
+
= 0.167.
Arch MLC, Fall 2007 c _Yufeng Guo 253
www.archactuarial.com www.guo.coursehost.com 254
9.7 Insurance and Annuity Benets
9.7.1 Survival Statuses
Option A reference: Actuarial Mathematics Chapter 9.7.1
Option B reference: Models for Quantifying Risk Chapter 13.6
In this section, the text basically repeats many formulas from chapters 4 and 5, so most
explanations will be brief. However, dont underestimate the importance of this section! In
many ways, it is some of the most testable stu for multiple lives.
CURTATE INSURANCE AND ANNUITY FUNCTIONS:
Consider a whole life insurance with unit benet payable at the end of year of death of the
survival status u (u can be x, xy, or xy) As before, let K be the curtate-future-lifetime of
u. So if u = x, then K is the number of future complete years lived by x, whereas if u = xy,
then K is the number of complete years lived by both x and y. (Get the idea? If so, go
ahead and say to yourself what K is if u = xy.) Now, using what you know for single-life
functions, you can write down most simple actuarial functions for the multiple-life case.
For example, we know that
A
u
=

k=0
v
k+1
k
p
u
q
u+k
,
so
A
xy
=

k=0
v
k+1
k
p
xy
q
x+k:y+k
.
When doing this translation process, you have to be careful dealing with the last survivor
status. It happened with the rst-to-die status that we could write
k
p
xy
q
x+k:y+k
by essentially
just replacing u with xy. With last survivor status, you have to be a lot more careful. As we
will see in the next EXAMPLE below. The key way to think about it is that
A
u
=

k=0
v
k+1
Pr[K = k],
and then you have to think to gure out logically what is the probability that the last
survivor status fails in year k + 1. I.e., what is Pr[K = k]?
If desired, you can go further with this translation when (x) and (y) are independent lives
by translating the entire sum into one containing only single life functions, using the single
life translations of
k
p
xy
and q
x+k:y+k
. For the joint-life status, the messy result would be
A
xy
=

k=0
v
k+1
k
p
x k
p
y
(q
x+k
+q
y+k
q
x+k
q
y+k
).
Just as in Chapter 4, the random variable Z = v
K+1
represents the present value at the time
the policy is issued of the benet payment. Also,
A
u
= E[Z] and Var[Z] =
2
A
u
(A
u
)
2
.
Arch MLC, Fall 2007 c _Yufeng Guo 254
www.archactuarial.com www.guo.coursehost.com 255
Therefore, if u = xy,
Var[Z] =
2
A
xy
(A
xy
)
2
.
Nothing really new here; just making sure you know what you know!
EXAMPLE:
(x) and (y) bought a last survivor curtate life insurance policy together with a
benet of 1,000,000 payable on the last death of the two. Write expressions for
expected value of the benet and the variance of the present value of the death
benet.
SOLUTION:
1. 1,000,000 A
xy
Ok, that was a pretty cheap answer. You probably wrote
something more complex, like
A
xy
=

k=0
v
k+1
Pr[K = k].
Then you might have gotten muddled over what to do with Pr[K = k]. The
problem is that
k
p
xy
q
x+k:y+k
is not the right expression since q
x+k:y+k
is the probability of the status
failing in year k +1 if both are alive at the beginning of the year, but that
might not be the case even if the last survivor status survives k years (i.e.,
it may be that only one of x and y is alive at the start of the last year).
This is a hard one! You have to recognize that
Pr[K = k] = f
K(xy)
(k)
and use the relation
f
K(xy)
(k) = f
K(x)
(k) +f
K(y)
(k) f
K(xy)
(k)
=
k
p
x
q
x+k
+
k
p
y
q
y+k

k
p
xy
q
x+k:y+k
.
So
A
xy
=

k=0
v
k+1
_
k
p
x
q
x+k
+
k
p
y
q
y+k

k
p
xy
q
x+k:y+k
_
.
and our answer is 1,000,000 times that.
This is a messy answer to a messy example, but it contains lots of useful
review for me, the hardest part is remembering that Pr[K = k] = f
K(xy)
(k)!
Review this example.
2. If the benet were 1, then
Var[Z] =
2
A
xy
(A
xy
)
2
.
Since we want Var[1,000,000 Z] the variance of the present value of the benet
is
(1,000,000)
2
_
2
A
xy
(A
xy
)
2
_
.
Well leave it at that!
Arch MLC, Fall 2007 c _Yufeng Guo 255
www.archactuarial.com www.guo.coursehost.com 256
EXAMPLE:
Given the following information, what is the actuarial present value of a con-
tinuous life annuity that pays 25,000 per year until the last death of (65) and
(70)?
a
65
= 10.2
a
65:5
= 4.6
a
65:70
= 6.2

5
E
65
= 0.82
SOLUTION:
We want to nd a
65:70
and we know that
a
65:70
= a
65
+a
70
a
65:70
.
We have all of those quantities except for a
70
, for that we need to use
a
65
= a
65:5
+
5
p
65
v
5
a
70
.
a
70
=
10.2 4.6
0.82
= 6.83,
and
a
65:70
= 10.2 + 6.83 6.2 = 10.83.
So our answer is (25,000)(10.83).
It is worth noting in this example that a
65:70
< a
70
< a
65
< a
65:70
. You should reason out
why this ought to be the case.
CONTINUOUS INSURANCE AND ANNUITY FUNCTIONS:
Since the last example had to do with continuous multiple-life functions, this is a good time
to mention that the same process that we use to get from single-life to multiple-life cases
for curtate insurance and annuities also works for continuous functions. Since we gave a
specic life insurance example in the Curtate case, we will look at an annuity example for
the continuous case.
Recall from Chapter 5 that for a continuous annuity of 1 per year until failure of the survival
status u, the present value of future benets is given by the random variable
Y = a
T
,
a
u
=
_

0
v
t
t
p
u
dt,
and
Var[Y ] =
2
A
u

_
A
u
_
2

2
.
Arch MLC, Fall 2007 c _Yufeng Guo 256
www.archactuarial.com www.guo.coursehost.com 257
EXAMPLE:
Write out the corresponding formula for a
xy
and Var[Y ] where Y is the present
value random variable for a joint-life continuous life annuity.
SOLUTION:
a
xy
=
_

0
v
t
t
p
xy
dt,
Var[Y ] =
2
A
xy

_
A
xy
_
2

2
.
Again, we could take this further and get everything in terms of single life func-
tions if we are told that (x) and (y) are independent lives.
EXAMPLE:
(x) and (y) are subject to constant forces of mortality
x
= 0.03 and
y
= 0.01,
respectively. The force of interest is = 0.06. The pair have purchased a whole
life policy that pays 1000 upon the rst death of the two.
(A) Find the actuarial present value of this insurance.
(B) Find the standard deviation of the present value of this insurance.
SOLUTION:
(A) We are looking for A
50:60
. The force of mortality for this joint life status
will be

xy
=
x
+
y
= 0.04.
Since this is a constant, we know the APV of the benet is just
1000 A
50:60
= 1000

50:60

50:60
+
= 1000
0.04
0.1
= 400.
(B)
Var
_
v
T
_
=
2
A
50:60

_
A
50:60
_
2
2
A
50:60
=

50:60

50:60
+ 2
=
0.04
0.16
= 0.25
Var
_
v
T
_
= 0.25 (0.4)
2
= 0.09.
The variance of our insurance is
Var
_
1000v
T
_
= (1000)
2
(0.09) = 90,000.
The standard deviation of the present value is
=
_
90,000 = 300.
Arch MLC, Fall 2007 c _Yufeng Guo 257
www.archactuarial.com www.guo.coursehost.com 258
EXAMPLE:
Assuming (40) and (50) are independent lives and using the Illustrative Life Table,
nd values for
(A) A
40:50
,
(B) Var[ a
K+1
], where K is the number of complete years that pass before the
rst death between the two.
(C) A
40:50
.
Arch MLC, Fall 2007 c _Yufeng Guo 258
www.archactuarial.com www.guo.coursehost.com 259
SOLUTION:
(A) We want A
x:x+10
where x = 40, which equals 0.294.
(B)
Var
_
a
K+1
_
=
2
A
40:50
(A
40:50
)
2
d
2
=
0.124 (0.294)
2
(0.0566)
2
= 11.73.
where we used the fact that
d = 1
1
1 +i
= 1
1
1.06
= 0.0566.
(C) A
40:50
= A
40
+A
50
A
40:50
= 0.161 + 0.249 0.294 = 0.116.
Some nal formulas specic to multiple life functions - we will only show them in the con-
tinuous case, but they hold for the curtate case as well (we have seen a couple of these
already):
T(xy) +T(xy) = T(x) +T(y)
v
T(xy)
+v
T(xy)
= v
T(x)
+v
T(y)
v
T(xy)+T(xy)
= v
T(x)+T(y)
= v
T(xy)
v
T(xy)
= v
T(x)
v
T(y)
a
xy
+a
xy
= a
x
+a
y
A
xy
+A
xy
=A
x
+A
y
.
In addition, if T(x) and T(y) are independent, then
Cov [T(xy), T(xy)] =
_

e
x


e
xy
_ _

e
y


e
xy
_
,
(I included this one even though you have already seen it because the book messed up the
notation.)
Cov
_
v
T(xy)
, v
T(xy)
_
=
_
A
x
A
xy
_ _
A
y
A
xy
_
.
EXAMPLE:
(x) and (y) are independent lives. Find an expression for Cov
_
a
T(xy)
, a
T(xy)
_
in
terms of APV life formulas.
SOLUTION:
Cov
_
a
T(xy)
, a
T(xy)
_
= Cov
_
1 v
T(xy)

,
1 v
T(xy)

_
= Cov
_
v
T(xy)

,
v
T(xy)

_
=
1

2
Cov
_
v
T(xy)
, v
T(xy)
_
=
_
A
x
A
xy
_ _
A
y
A
xy
_

2
.
Arch MLC, Fall 2007 c _Yufeng Guo 259
www.archactuarial.com www.guo.coursehost.com 260
9.7.2 Special Two-Life Annuities
Option A reference: Actuarial Mathematics Chapter 9.7.2
Option B reference: Models for Quantifying Risk Chapter 13.4.2
In many pension plans, upon retirement of a married worker, an annuity is set up so that the
stream of payments will decrease after the death of one of the partners. This section gives
some insight into that sort of annuity.
The whole section is an example, so well walk through one similar to the books. You should
also work through the example in the book: Example 9.7.2 (Page 284 of the text).
EXAMPLE:
Mom and Pop want to buy a lifetime continuous two-life annuity to take care of their retire-
ment needs. They want the annuity to pay 1 as long as both are alive, but they gure either
spouse can get by on 0.6 once the other is deceased. So they have decided to buy an annuity
that pays:
(i) 1 per year while both (x) and (y) are alive; and
(ii) 0.6 per year while only one of (x) and (y) is alive.
(A) Derive an expression for the annuitys present value random variable:
The most obvious way to write the APV is:
1 a
T(xy)
+ (0.6)
_
a
T(xy)
a
T(xy)
_
.
Note that
_
a
T(xy)
a
T(xy)
_
indicates that we will pay 1 per year as long as either of
(x) or (y) is alive but we take that payment back while both are alive.
This present value random variable can be expressed as :
Z = (0.6)a
T(xy)
+ (0.4)a
T(xy)
.
(B) Derive an expression for the annuitys actuarial present value in terms of a
x
, a
y
, a
xy
.
The APV is the expected value of the equation in the solution to (A):
E [Z] = (0.6)a
xy
+ (0.4)a
xy
.
To get to a
x
, a
y
, and a
xy
, we can use
a
xy
+a
xy
= a
x
+a
y
to rewrite the APV as (0.6)a
x
+ (0.6)a
y
(0.2)a
xy
.
(C) Derive an expression for the variance of the random variable in (A) under the assump-
tion that Mom and Pop have independent lives.
Var[Z] = Var
_
(0.6) a
T(xy)
+ (0.4) a
T(xy)
_
.
Arch MLC, Fall 2007 c _Yufeng Guo 260
www.archactuarial.com www.guo.coursehost.com 261
Hopefully you remember from Statistics that
Var(aS +bT) = a
2
Var(S) +b
2
Var(T) + 2ab Cov(S, T).
Var[Z] = (0.36) Var
_
a
T(xy)
_
+ (0.16) Var
_
a
T(xy)
_
+ (0.48) Cov
_
a
T(xy)
, a
T(xy)
_
.
We know that
Var
_
a
T(xy)
_
=
2
A
xy

_
A
xy
_
2

2
, Var
_
a
T(xy)
_
=
2
A
xy

_
A
xy
_
2

2
.
Now for the covariance term:
Cov
_
a
T(xy)
, a
T(xy)
_
=
_
A
x
A
xy
_ _
A
y
A
xy
_

2
,
as we saw earlier. Therefore, Var[Z] is equal to
1

2
_
(0.36)
_
2
A
xy

_
A
xy
_
2
_
+ (0.16)
_
2
A
xy

_
A
xy
_
2
_
+ (0.48)
_
A
x
A
xy
_ _
A
y
A
xy
_
_
.
9.7.3 Reversionary Annuities
Option A reference: Actuarial Mathematics Chapter 9.7.3
Option B reference: Models for Quantifying Risk Chapter 13.4.4
Reversionary annuities are really nothing new except for the name and some notation. A
reversionary annuity is payable during the existence of one status (u), but only if another
status (v) has failed. For example, an annuity may be purchased to provide lifetime income
to Sue, but the income only kicks in once her husband Bob has passed away. In this case,
Sue would be (u) and Bob would be (v).
Consider an annuity of 1 per year payable continuously to (y) after the death of (x). The
present value at time 0 is
Z = a
T(y)
a
T(xy)
.
This is somewhat intuitive. It says Z is the present value of an annuity that pays as long as
(y) is alive minus the present value of an annuity that pays as long as both (x) and (y) are
alive. It is as if (y) is getting a life annuity but has agreed to give it back as long as both (x)
and (y) are alive. The symbol for the APV of this insurance is
a
x|y
and in the subscript, the person that has to die comes before the [.
a
x|y
= E[Z] = a
y
a
xy
.
Arch MLC, Fall 2007 c _Yufeng Guo 261
www.archactuarial.com www.guo.coursehost.com 262
EXAMPLE:
Find an expression for the actuarial present value of a single continuous life an-
nuity that pays as follows:
1 per year while both (x) and (y) are alive,
0.6 per year if (x) is alive and (y) is not,
0.5 per year if (y) is alive and (x) is not.
SOLUTION: The way this question is written, it is clear that this is the sum of
3 separate annuities. The rst is just a
xy
.
The second is (0.6)a
y|x
= (0.6) (a
x
a
xy
) .
The last one is (0.5)a
x|y
= (0.5) (a
y
a
xy
) .
Adding these up gives an APV equal to
(0.5)a
y
+ (0.6)a
x
(0.1)a
xy
.
EXAMPLE:
Pat and Kelly are independent lives ages 30 and 35, respectively. They want to
buy a two-life discrete annuity that will pay 1000 at the beginning of any year in
which either of them is alive and over 60. (So the maximum payment in any year
is 1000.) Find an expression for the APV of this annuity.
SOLUTION: This one is tricky since the annuity will start in 25 years if Kelly
is alive then, but wont start until later if Kelly dies before then. The key idea is
to think of this as an annuity that
pays 1000 per year to Pat if he is alive and over 60,
pays 1000 per year to Kelly if she is alive and over 60, and
takes back 1000 per year if both are alive and over 60.
Now it is easier to write the APV:
1000
30|
a
30
+ 1000
25|
a
35
1000
30|
a
30:35
.
Note that the joint-life piece is deferred 30 years because there is no way both
can be over 60 before that time.
One last example that I didnt manage to t in earlier in the Chapter: this one doesnt teach
a lot but is the kind of quirky question that shows up sometimes. I nd this type very hard
to get correct unless you have seen one or two like it.
QUIRKY EXAMPLE:
Assume that all lives are independent,
20
p
40:60
= 0.4, and
10
p
40
= 0.8.
Find
30
p
50
.
Arch MLC, Fall 2007 c _Yufeng Guo 262
www.archactuarial.com www.guo.coursehost.com 263
SOLUTION:
With no idea how to get started, lets start by writing down the two-life function
in terms of single-life functions.
0.4 =
20
p
40 20
p
60
.
If we are lucky, we will notice that this expression is also equal to
0.4 =
40
p
40
.
Now we can use the fact that
40
p
40
=
10
p
40

30
p
50
to see that
0.4 = (0.8)
30
p
50

30
p
50
= 0.5.
When you get questions like this, you might feel like the exam writer is testing
your cleverness as much as your knowledge of the exam material.
9.9 Simple Contingent Functions
Option A reference: Actuarial Mathematics Chapter 9.7.2
Option B reference: Models for Quantifying Risk Chapter 13.3
The last topic in this chapter is insurances that depend upon the order of death within a
status. For example, a policy might pay on the rst death of a couple if and only if the
husband is the rst to die. We have a little bit of new notation:
n
q
1
xy
where 1 over the x denotes (x) dies before (y) and the usual meaning for n that the death
occurs within n years. So
n
q
1
xy
gives the probability that (x) will die in the next n years and
(y) will be alive at the moment x dies. (I.e., (y) is alive when (x) kicks the bucket but (y)
does not necessarily live the full n years.)
If we arent told that (x) and (y) are independent, then we have an awkward formula:
n
q
1
xy
=
_
n
0
Pr[T(y) > t [ T(x) = t]
s
p
x
(x +t)dt.
If were lucky and the SOA gives a case where (x) and (y) are independent (this will almost
surely be the case), we can use the formula:
n
q
1
xy
=
_
n
0
t
p
y t
p
x
(x +t) dt.
Lets go over this formula in English: First, t is the time variable that were allowing to run
from 0 to n. For all times t, both (x) and (y) survive to t, but then (x) is hit by the force
Arch MLC, Fall 2007 c _Yufeng Guo 263
www.archactuarial.com www.guo.coursehost.com 264
of mortality in the interval (t, t + dt). Thus, in all cases analyzed here, (y) survives longer
than (x).
n
q
x
2
y
is the symbol representing the probability that (y) dies after (x), but within an n-yr
period from now. Note that above, we knew (y) died after (x), but there is no upper limit
on when (y) dies. Here there is an upper limit on when (y) dies n years.
In the independent case,
n
q
x
2
y
=
_
n
0
t
q
x t
p
y
(y +t)dt
The integral makes sense, it integrates over all times t, the probability that (x) is dead and
(y) is alive, times the probability that (y) gets hit by the force of mortality at that time.
n
q
x
2
y
=
n
q
y

n
q
x
1
y
,
This equation says that
n
q
x
2
y
equals the probability that (y) dies in the next n years minus
the probability that (y) dies in the next n years and dies rst. Note that the book has the 1
in the wrong place in this equation (Equation 9.9.4 in the book).
The non-independent case is messy and is unlikely (but possible) to be tested.
EXAMPLE:
(x) and (y) are subject to the constant forces of mortality
x
= 0.03 and
y
= 0.05,
respectively.
(A) Find the probability that (x) dies before (y).
(B) Find the probability that (x) dies before (y) and in the next 4 years.
(C) Find the probability that both (x) and (y) will die in the next 4 years with
(x) dying rst.
SOLUTION:
(A) This is

q
1
x:y
=
_

0
t
p
xy

x
(t) dt =
_

0
e
0.03t
e
0.05t
(0.03) dt =
3
8
.
(B)
4
q
1
x:y
=
_
4
0
e
0.08t
(0.03) dt =
0.03
0.08
_
e
0.08t
_
4
0
=
3
8
(1 0.726) = 0.103.
(C) This is
4
q
x:
2
y
. We can nd this one by calculating the integral
4
q
x:
2
y
=
_
4
0
t
q
x t
p
y

y
(t) dt =
_
4
0
(1 e
0.03t
)(e
0.05t
)
y
(t) dt.
Arch MLC, Fall 2007 c _Yufeng Guo 264
www.archactuarial.com www.guo.coursehost.com 265
Or we could save some trouble by noticing that
4
q
x:
2
y
=
4
q
y

4
q
1
x:y
=
_
1 e
0.05(4)
_
0.103 = 0.078.
If you think about it,
n
q
1
xy
and
n
q
x
2
y
are the same except that, with
n
q
x
2
y
, we know that (y)
did not survive the full n years. That is, in both cases, (x) dies within n years and before
(y). Therefore,
n
q
x
2
y
=
n
q
1
xy

n
p
y n
q
x
.
This can be thought of as the case in which (y) outlives (x), who died within the n-year
period. However, were taking away the possibility that (y) survives the full n-year period.
EXAMPLE:
A
1
x:y
represents the actuarial present value of an insurance that pays 1 at the
moment of death of (x) if (y) is alive at the moment (x) dies. Assuming (x) and
(y) are independent lives, nd an integral expression for A
1
x:y
.
SOLUTION:
A
1
x:y
=
_

0
v
T
t
p
xy
(x +t)dt =
_

0
v
T
t
p
x t
p
y
(x +t)dt.
EXAMPLE:
Assuming (x) and (y) are independent, write the integral expression for A
x:
2
y
.
SOLUTION:
A
x:
2
y
=
_

0
v
T
t
q
x t
p
y
(x +t)dt.
Chapter 9 Suggested Problems: 4(assume the lives are independent), 10, 12, 13,
22, 24, 25, 26, 28
(Solutions at archactuarial.com)
Arch MLC, Fall 2007 c _Yufeng Guo 265
www.archactuarial.com www.guo.coursehost.com 266
CHAPTER 9 Formula Summary
If (x) and (y) are independent lives,
t
p
xy
=
t
p
x t
p
y
and
t
q
xy
=
t
q
x
+
t
q
y

t
q
x t
q
y

xy
(t) = (x +t) +(y +t)
f
T(xy)
(t) =
t
p
x t
p
y
((x +t) +(y +t))
Force of Failure for non-independent lives:
xy
(t) =
f
T(xy)
(t)
1 F
T(xy)
(t)
The probability that a joint-life status terminates during the period (k, k + 1) is
k
p
xy

k+1
p
xy
=
k
p
xy
q
x+k:y+k
.
The Laws of addition:
T(xy) +T(xy) = T(x) +T(y) T(xy) T(xy) = T(x) T(y)
v
T(xy)
+v
T(xy)
= v
T(x)
+v
T(y)
v
T(xy)
v
T(xy)
= v
T(x)
v
T(y)
A
xy
+A
xy
=A
x
+A
y
a
xy
+a
xy
= a
x
+a
y
F
T(xy)
(t) +F
T(xy)
(t) = F
T(x)
(t) +F
T(y)
(t) f
T(xy)
(t) +f
T(xy)
(t) = f
T(x)
(t) +f
T(y)
(t)
t
p
xy
+
t
p
xy
=
t
p
x
+
t
p
y
Continuous Curtate

e
xy
=
_

0
t
p
xy
dt e
xy
=

1
k
p
xy

e
xy
=
_

0
t
p
xy
dt e
xy
=

1
k
p
xy

e
xy
=

e
x
+

e
y


e
xy
e
xy
= e
x
+e
y
e
xy
Var[T(xy)] = E[T(xy)
2
] E[T(xy)]
2
= 2
_

0
t
t
p
xy
dt (

e
xy
)
2
Var[T(xy)] = E[T(xy)
2
] E[T(xy)]
2
= 2
_

0
t
t
p
xy
dt (

e
xy
)
2
Cov[T(xy), T(xy)] = Cov[T(x), T(y)] + (

e
x


e
xy
)(

e
y


e
xy
)
Cov[T(xy), T(xy)] = (

e
x


e
xy
)(

e
y


e
xy
).
Arch MLC, Fall 2007 c _Yufeng Guo 266
www.archactuarial.com www.guo.coursehost.com 267
Common Shock:
s
T(x)
(t) = s
T

(x)
(t) s
Z
(t) = s
T

(x)
(t) e
t
,
s
T(y)
(t) = s
T

(y)
(t) s
Z
(t) = s
T

(y)
(t) e
t
,
s
T(x)T(y)
= s
T

(x)
(t) s
T

(y)
(t) s
Z
(t) = s
T

(x)
(t) s
T

(y)
(t) e
t
.
Insurances based on multiple lives:
A
xy
=

k=0
v
k+1
k
p
x k
p
y
(q
x+k
+q
y+k
q
x+k
q
y+k
)
A
xy
=

k=0
v
k+1
_
k
p
x
q
x+k
+
k
p
y
q
y+k

k
p
xy
q
x+k:y+k
_
If T(x) and T(y) are independent, then
Cov
_
v
T(xy)
, v
T(xy)
_
=
_
A
x
A
xy
_ _
A
y
A
xy
_
Arch MLC, Fall 2007 c _Yufeng Guo 267
www.archactuarial.com www.guo.coursehost.com 268
Past SOA/CAS Exam Questions:
1. For a last-survivor insurance of 10, 000 on independent lives (70) and (80), you are
given:
(i) The benet, payable at the end of the year of death, is paid only if the second
death occurs during year 5.
(ii) Mortality follows the Illustrative Life Table.
(iii) i = 0.03
Calculate the actuarial present value of this insurance.
(A) 235 (B) 245 (C) 255 (D) 265 (E) 275
SOLUTION: APV =
5
_
4
p
70:80

5
p
70:80
_
4
p
70
=
l
74
l
70
=
56, 640.51
66, 161.55
= 0.856094,
4
p
80
=
l
84
l
80
=
26, 607.34
39, 143.65
= 0.679736
4
p
70:80
=
4
p
70
+
4
p
80

4
p
70

4
p
80
= 0.953912
5
p
70
=
l
75
l
70
=
53, 960.81
66, 161.55
= 0.815592
5
p
80
=
l
85
l
80
=
23, 582.46
39, 143.65
= 0.602459
5
p
70:80
=
5
p
70
+
5
p
80

5
p
70

5
p
80
= 0.926690
APV =
5
(0.953912 0.926690) =
0.027222
1.03
5
= 0.02348 Key: A
2. For a last-survivor whole life insurance of 1 on (x) and (y):
(i) The death benet is payable at the moment of the second death.
(ii) The independent random variables T

(x), T

(y), and Z are the components of a


common shock model.
(iii) T

(x) has an exponential distribution with


T

(x)
x
(t) = 0.03, t 0.
(iv) T

(y) has an exponential distribution with


T

(y)
y
(t) = 0.05, t 0.
(v) Z, the common shock random variable, has an exponential distribution with

Z
(t) = 0.02, t 0.
(vi) = 0.06
Calculate the actuarial present value of this insurance.
(A) 0.216 (B) 0.271 (C) 0.326 (D) 0.368 (E) 0.423
Arch MLC, Fall 2007 c _Yufeng Guo 268
www.archactuarial.com www.guo.coursehost.com 269
Solution:

T(x)
x
=
T

(x)
x
+
Z
= 0.03 + 0.02 = 0.05

T(y)
y
=
T

(y)
y
+
Z
= 0.05 + 0.02 = 0.07

xy
=
T

(x)
x

T

(y)
y
+
Z
= 0.03 + 0.05 + 0.02 = 0.10
A
x
=

T(x)
x

T(x)
x
+
=
0.05
0.11
= 0.4545
A
y
=

T(y)
y

T(y)
y
+
=
0.07
0.13
= 0.5385
A
xy
=

xy

xy
+
=
0.1
0.16
= 0.6250
A
xy
= A
x
+A
y
A
xy
= 0.4545 + 0.5385 +0.6250 = 0.3680 Key: D
3. You have calculated the actuarial present value of a last-survivor whole life insurance
of 1 on (x) and (y). You assumed:
(i) The death benet is payable at the moment of death.
(ii) The future lifetimes of (x) and (y) are independent, and each life has a constant
force of mortality with = 0.06.
(iii) = 0.05
Your supervisor points out that these are not independent future lifetimes. Each mortal-
ity assumption is correct, but each includes a common shock component with constant
force 0.02.
Calculate the increase in the actuarial present value over what you originally calculated.
(A) 0.020 (B) 0.039 (C) 0.093 (D) 0.109 (E) 0.163
SOLUTION:
Original calculation (assuming independence):

x
= 0.06,
y
= 0.06

xy
= 0.06 + 0.06 = 0.12
A
x
=

x

x
+
=
0.06
0.06 + 0.05
= 0.54545
A
y
=

y

y
+
=
0.06
0.06 + 0.05
= 0.54545
Arch MLC, Fall 2007 c _Yufeng Guo 269
www.archactuarial.com www.guo.coursehost.com 270
A
xy
=

xy

xy
+
=
0.12
0.12 + 0.05
= 0.70588
A
xy
=A
x
+A
y
A
xy
= 0.54545 + 0.54545 0.70588 = 0.38502
Revised calculation (common shock model):

x
= 0.06,
T(x)
x
= 0.04

y
= 0.06,
T(y)
y
= 0.04

xy
=
T(x)
x
+
T(y)
y
+
Z
= 0.04 + 0.04 + 0.02 = 0.10
A
x
=

x

x
+
=
0.06
0.06 + 0.05
= 0.54545
A
y
=

y

y
+
=
0.06
0.06 + 0.05
= 0.54545
A
xy
=

xy

xy
+
=
0.10
0.10 + 0.05
= 0.66667
A
xy
=A
x
+A
y
A
xy
= 0.54545 + 0.54545 0.66667 = 0.42423
Dierence = 0.42423 0.38502 = 0.03921
Key: B
4. For independent lives (x) and (y):
(i) q
x
= 0.05
(ii) q
y
= 0.10
(iii) Deaths are uniformly distributed over each year of age.
Calculate
0.75
q
xy
.
(A) 0.1088 (B) 0.1097 (C) 0.1106 (D) 0.1116 (D) 0.1125
Solution:
0.75
p
x
= 1 (0.75)(0.05) = 0.9625
0.75
p
y
= 1 (0.75)(0.10) = 0.925
0.75
q
xy
= 1
0.75
p
xy
= 1 (0.9625)(0.925) = 0.1097 Key: B
Arch MLC, Fall 2007 c _Yufeng Guo 270
www.archactuarial.com www.guo.coursehost.com 271
5. For independent lives (50) and (60):
(x) =
1
100 x
, 0 x < 100
Calculate

e
50:60
.
(A) 30 (B) 31 (C) 32 (D) 33 (E) 34
Solution:
t
p
x
= exp
_

_
t
0
ds
100 x s
_
= exp
_
ln(100 x s)[
t
0
_
=
100 x t
100 x

e
50:60
=

e
50
+

e
60

e
50:60

e
50
=
_
50
0
50 t
50
dt =
1
50
_
50t
t
2
2
_
50
0
= 25

e
60
=
_
40
0
40 t
40
dt =
1
40
_
40t
t
2
2
_
40
0
= 20

e
50:60
=
_
40
0
50 t
50

40 t
40
dt =
_
40
0
1
2000
_
2000 90t +t
2
_
dt
=
1
2000
_
2000t 45t
2
+
t
3
3
[
40
0
_
= 14.67

e
50:60
= 25 + 20 14.67 = 30.33 Key: A
6. A continuous two-life annuity pays:
100 while both (30) and (40) are alive;
70 while (30) is alive but (40) is dead; and
50 while (40) is alive but (30) is dead.
The actuarial present value of this annuity is 1180. Continuous single life annuities
paying 100 per year are available for (30) and (40) with actuarial present values of 1200
and 1000, respectively.
Calculate the actuarial present value of a two-life continuous annuity that pays 100
while at least one of them is alive.
(A) 1400 (B) 1500 (C) 1600 (D) 1700 (E) 1800
Arch MLC, Fall 2007 c _Yufeng Guo 271
www.archactuarial.com www.guo.coursehost.com 272
SOLUTION:
1180 = 70a
30
+ 50a
40
20a
30:40
1180 = (70)(12) + (50)(10) 20a
30:40
a
30:40
= 8
a
30:40
= a
30
+a
40
a
30:40
= 12 + 10 8 = 14
100a
30:40
= 1400
Key: A
7. (x) and (y) are two lives with identical expected mortality.
P
x
= P
y
= 0.1
P
xy
= 0.06, where P
xy
is the annual benet premium for a fully discrete insurance
of 1 on (xy).
d = 0.06
Calculate the premium P
xy
, the annual benet premium for a full discrete insurance of
1 on (xy).
(A) 0.14 (B) 0.16 (C) 0.18 (D) 0.20 (E) 0.22
SOLUTION:
P
s
=
1
a
s
d, where s can stand for any of the statuses under consideration.
a
s
=
1
P
s
+d
a
x
= a
y
=
1
0.1 + 0.06
= 6.25
a
xy
=
1
0.06 + 0.06
= 8.333
a
xy
+ a
xy
= a
x
+ a
y
a
xy
= 6.25 + 6.25 8.333 = 4.167
P
xy
=
1
4.167
0.06 = 0.18
Key C
Arch MLC, Fall 2007 c _Yufeng Guo 272
www.archactuarial.com www.guo.coursehost.com 273
8. For a select-and-ultimate table with a 2-year select period:
x p
[x]
p
[x]+1
p
x+2
x + 2
48 0.9865 0.9841 0.9713 50
49 0.9858 0.9831 0.9698 51
50 0.9849 0.9819 0.9682 52
51 0.9838 0.9803 0.9664 53
Keith and Clive are independent lives, both age 50. Keith was selected at age 45 and
Clive was selected at age 50.
Calculate the probability that exactly one will be alive at the end of three years.
(A) Less than 0.115
(B) At least 0.115, but less than 0.125
(C) At least 0.125, but less than 0.135
(D) At least 0.135, but less than 0.145
(E) At least 0.145
SOLUTION:
Probonly 1 survives = 1 - Probboth survive - Probneither survives
= 1
3
p
50

3
p
[50]
(1
3
p
50
)
_
1
3
p
[50]
_
= 1 (0.9713)(0.9698)(0.9682)
. .
=0.91202
(0.9849)(0.9819)(0.9682)
. .
0.936320
(1 0.912012)(1 0.93632)
= 0.140461 Key: D
Arch MLC, Fall 2007 c _Yufeng Guo 273
www.archactuarial.com www.guo.coursehost.com 274
9. For two independent lives now age 30 and 34, you are given:
x q
x
30 0.1
31 0.2
32 0.3
33 0.4
34 0.5
35 0.6
36 0.7
37 0.8
Calculate the probability that the last death of these two lives will occur during the 3
rd
year from now (i.e.
2|
q
30:34
)
(A) 0.01 (B) 0.03 (C) 0.14 (D) 0.18 (D) 0.24
SOLUTION:
2|
q
30:34
=
2
p
30:34

3
p
30:34
Thanks to independence the following two expressions are true:
2
p
30:34
=
2
p
30

2
p
34
=
_
(0.9)(0.8)
_

_
(0.5)(0.4)
_
= 0.144
3
p
30:34
=
3
p
30

3
p
34
=
_
(0.72)(0.7)
_

_
(0.2)(0.3)
_
= 0.03024
Now
2
p
30:34
=
2
p
30
+
2
p
34

2
p
30:34
= 0.72 + 0.2 0.144 = 0.776
3
p
30:34
=
3
p
30
+
3
p
34

3
p
30:34
= 0.504 + 0.06 0.03024 = 0.53376
2|
q
30:34
= 0.776 0.53376 = 0.24224
10. You are given:
(i) Mortality follows DeMoivres law with = 105.
(ii) (45) and (65) have independent future lifetimes.
Calculate

e
45:65
(A) 33 (B) 34 (C) 35 (D) 36 (E) 37
Arch MLC, Fall 2007 c _Yufeng Guo 274
www.archactuarial.com www.guo.coursehost.com 275
SOLUTION:

e
45:65
=

e
45
+

e
65


e
45:65
Under DeMoivres Law,

e
x
=
x
2
, so we have

e
45
= 30,

e
65
= 20

e
45:65
=
_
40
0
t
p
45:65
dt =
_
40
0
t
p
45

t
p
65
dt =
_
40
0
_
1
t
60
__
1
t
40
_
dt
=
_
40
0
_
1
t
24
+
t
2
2400
_
dt = 15.56

e
45:65
= 30 + 20 15.56 = 34.4
11. For independent lives (35) and (45):
(i)
5
p
35
= 0.90
(ii)
5
p
45
= 0.80
(iii) q
40
= 0.03
(iv) q
50
= 0.05
Calculate the probability that the last death of (35) and (45) occurs in the 6
th
year.
(A) 0.0095
(B) 0.0105
(C) 0.0115
(D) 0.0125
(E) 0.0135
SOLUTION:
5|
q
35:45
=
5|
q
35
+
5|
q
45

5|
q
35:45
5
p
35
q
40
+
5
p
45
q
50

5
p
35:45
q
40:50
5
p
35
q
40
+
5
p
45
q
50

5
p
35

5
p
45
(1 p
40:50
)
5
p
35
q
40
+
5
p
45
q
50

5
p
35

5
p
45
(1 p
40
p
50
)
= (0.9)(0.3) + (0.8)(0.05) (0.9)(0.8)[1 (0.97)(0.95)]
= 0.01048.
Alternatively,
6
p
35
=
5
p
35
p
40
= (0.90)(1 0.03) = 0.873
Arch MLC, Fall 2007 c _Yufeng Guo 275
www.archactuarial.com www.guo.coursehost.com 276
6
p
45
=
5
p
50
p
50
= (0.80)(1 0.05) = 0.76
5|
q
35:45
=
5
p
35:45

6
p
35:45
= (
5
p
35
+
5
p
45

5
p
35:45
) (
6
p
35
+
6
p
45

6
p
35:45
)
= (
5
p
35
+
5
p
45

5
p
35

5
p
45
) (
6
p
35
+
6
p
45

6
p
35

6
p
45
)
= (0.90 + 0.80 0.90 0.80) (0.873 + 0.76 0.873 0.76)
= 0.98 0.96952 = 0.01048
Key: B
12. You are given:
(i) T(x) and T(y) are not independent.
(ii) q
x+k
= q
y+k
= 0.05, k = 0, 1, 2, . . .
(iii)
k
p
xy
= 1.02
k
p
x

k
p
y
, k = 1, 2, 3, . . .
Into which of the following ranges does e
x:y
, the curtate expectation of life of the last
survivor status, fall?
(A) e
x:y
25.7
(B) 25.7 < e
x:y
26.7
(C) 26.7 < e
x:y
27.7
(D) 27.7 < e
x:y
28.7
(E) 28.7 < e
x:y
SOLUTION:
e
x
= e
y
=

k=1
t
p
x
= 0.95 + 0.95
2
+. . .
=
0.95
10.95
= 19
e
xy
= p
xy
+
2
p
xy
+. . .
= 1.02(0.95)(0.95) + 1.02(0.95)
2
(0.95)
2
+. . .
= 1.02
_
0.95
2
+ 0.95
4
+. . .

=
1.02(0.95)
2
10.95
2
= 9.44152
e
xy
= e
x
+e
y
e
xy
= 28.56
Key: D
13. You are pricing a special 3-year annuity-due on two independent lives, both age 80.
The annuity pays 30,000 if both persons are alive and 20,000 if only one person is alive.
You are given:
Arch MLC, Fall 2007 c _Yufeng Guo 276
www.archactuarial.com www.guo.coursehost.com 277
(i)
k
k
p
80
1 0.91
2 0.82
3 0.72
(ii) i = 0.05
Calculate the actuarial present value of this annuity.
(A) 78,300
(B) 80,400
(C) 82,500
(D) 84,700
(E) 86,800
SOLUTION:
Calculate the probability that both are alive or both are dead.
P(both alive) =
k
p
xy
=
k
p
x

k
p
y
P(both dead) =
k
q
xy
=
k
q
x

k
q
y
P(exactly one alive) = 1
k
p
xy

k
q
xy
Only have to do two years worth so have table
Pr(both alive) Pr(both dead) Pr(only one alive)
1 0 0
(0.91)(0.91) = 0.8281 (0.09)(0.09) = 0.0081 0.1638
(0.82)(0.82) = 0.6724 (0.18)(0.18) = 0.0324 0.2952
APV Annuity = 30, 000
_
1
1.05
0
+
0.8281
1.05
1
+
0.6724
1.05
2
_
+ 20, 000
_
0
1.05
0
+
0.1638
1.05
1
+
0.2952
1.05
2
_
=
80, 431
Alternatively,
a
xy
= 1 +
0.8281
1.05
+
0.6724
1.05
2
= 2.3986
a
x
= a
y
= 1 +
0.91
1.05
+
0.82
1.05
2
= 2.6104
APV = 20, 000 a
x
+ 20, 000 a
y
10, 000 a
xy
(it pays 20,000 if x alive and 20,000 if y alive, but 10,000 less than that if both are
alive)
= (20, 000)(2.6104) + (20, 000)(2.6104) (10, 000)2.3986 = 80, 430
Other alternatives also work.Key: B
Arch MLC, Fall 2007 c _Yufeng Guo 277
www.archactuarial.com www.guo.coursehost.com 278
14. For a special fully continuous last survivor insurance of 1 on (x) and (y), you are given:
(i) T(x) and T(y) are independent.
(ii)
x
(t) = 0.08, t > 0
(iii)
y
(t) = 0.04, t > 0
(iv) = 0.06
(v) is the annual benet premium payable until the rst of (x) and (y) dies.
Calculate .
(A) 0.055
(B) 0.080
(C) 0.105
(D) 0.120
(E) 0.150
SOLUTION:

xy
(t) =
x
(t) +
y
(t) = 0.08 + 0.04 = 0.12

A
x
=
x
(t)/(
x
(t) +) = 0.5714

A
y
=
y
(t)/(
y
(t) +) = 0.4

A
xy
=
xy
(t)/(
xy
(t) +) = 0.6667
a
xy
= 1/(
xy
(t) +) = 5.556

A
xy
=

A
x
+

A
y


A
xy
= 0.5714 + 0.4 0.6667 = 0.3047
Premium = 0.304762/5.556 = 0.0549
Key: A
Arch MLC, Fall 2007 c _Yufeng Guo 278
www.archactuarial.com www.guo.coursehost.com 279
15. The mortality of (x) and (y) follows a common shock model with components T

(x),
T

(y) and Z.
(i) T

(x), T

(y) and Z are independent and have exponential distributions with re-
spective forces
1
,
2
and .
(ii) The probability that (x) survives 1 year is 0.96.
(iii) The probability that (y) survives 1 year is 0.97.
(iv) = 0.01
Calculate the probability that both (x) and (y) survive 5 years.
(A) 0.65
(B) 0.67
(C) 0.70
(D) 0.72
(E) 0.74
SOLUTION:
0.96 = e
(
1
+)

1
+ = ln(0.96) = 0.04082

1
= 0.04082 = 0.04082 0.01 = 0.03082.
Similarly

2
= ln(0.97) = 0.03046 0.01 = 0.02046

xy
=
1
+
2
+ = 0.03082 + 0.02046 + 0.01 = 0.06128.
5
p
xy
= e
(5)(0.06128)
= e
0.3064
= 0.736.
Key: E
Arch MLC, Fall 2007 c _Yufeng Guo 279
www.archactuarial.com www.guo.coursehost.com 280
16. For (80) and (84), whose future lifetimes are independent:
x p
x
80 0.50
81 0.40
82 0.60
83 0.25
84 0.20
85 0.15
86 0.10
Calculate the change in the value
2|
q
80:84
if p
82
is decreased from 0.60 to 0.30.
(A) 0.03 (B) 0.06 (C) 0.10 (D) 0.16 (E) 0.19
SOLUTION:
2|
q
80:84
=
2|
q
80
+
2|
q
84

2|
q
80:84
= 0.5 0.4 (1 0.6) + 0.2 0.15 (1 0.1) = 0.10136
Using new p
82
value of 0.3
= 0.5 0.4 (1 0.3) + 0.2 0.15 (1 0.1) = 0.16118
Change = 0.16118 0.10136 = 0.06
Alternatively,
2
p
80
= 0.5 0.4 = 0.20
3
p
80
=
2
p
80
0.6 = 0.12
2
p
84
= 0.20 0.15 = 0.03
3
p
84
=
2
p
84
0.10 = 0.003
2
p
80:84
=
2
p
80
+
2
p
84

2
p
80

2
p
84
since independent
= 0.20 + 0.03 (0.20)(0.03) = 0.224
3
p
80:84
=
3
p
80
+
3
p
84

3
p
80

3
p
84
= 0.12 + 0.003 (0.12)(0.003) = 0.12264
2
q
80:84
=
2
p
80:84

3
p
80:84
= 0.224 0.12264 = 0.10136
Revised
3
p
80
= 0.20 0.30 = 0.06
3
p
80:84
= 0.06 + 0.003 (0.06)(0.003) = 0.06282
2|
q
80:84
= 0.224 0.06282 = 0.16118
change = 0.16118 0.10136 = 0.06.
Key: B
Arch MLC, Fall 2007 c _Yufeng Guo 280
www.archactuarial.com www.guo.coursehost.com 281
Problems from Pre-2000 SOA-CAS exams
1. Individual A is subject to the following law of mortality:

x
= 0.04, for all x
Calculate

e
35:40
, assuming (35) and (40) are subject to the same law of mortality as
individual A.
(A) 11.5 (B) 12.0 (C) 12.5 (D) 13.0 (E) 13.5
2. For a fully continuous last-survivor whole life insurance of 1 on (x) and (y), you are
given:
T(x) and T(y) are independent.

x
(t) =
y
(t) = 0.06, t > 0
= 0.04
Premiums are payable until the rst death.
Calculate the annual benet premium.
(A) 0.033 (B) 0.042 (C) 0.055 (D) 0.072 (E) 0.120
3. (40) and (50) are independent lives. Mortality follows De Moivres law with = 100.
Calculate the probability that at least one of (40) and (50) will die within 10 years.
(A) 1/30 (B) 3/10 (C) 1/3 (D) 2/3 (E) 7/10
4. (Same setup as previous problem:) (40) and (50) are independent lives. Mortality
follows De Moivres law with = 100.
Calculate the probability that the second death occurs between times t = 10 and t = 20.
(A) 1/10 (B) 1/5 (C) 4/15 (D) 1/3 (E) 2/5
Arch MLC, Fall 2007 c _Yufeng Guo 281
www.archactuarial.com www.guo.coursehost.com 282
Use the following information for the next 4 questions:
You are given:
(30) and (50) are independent lives, each subject to constant force of mortality
with = 0.05.
= 0.03
5. Calculate
10
q
30:50
.
(A) 0.155 (B) 0.368 (C) 0.424 (D) 0.632 (E) 0.845
6. Calculate

e
30:50
.
(A) 10 (B) 20 (C) 30 (D) 40 (E) 50
7. Calculate Var[T(30 : 50)].
(A) 50 (B) 100 (C) 150 (D) 200 (E) 400
8. Calculate Cov
_
T(30 : 50), T
_
30 : 50
_
.
(A) 10 (B) 25 (C) 50 (D) 100 (E) 200
Arch MLC, Fall 2007 c _Yufeng Guo 282
www.archactuarial.com www.guo.coursehost.com 283
Solutions to Pre-2000 Exam Questions: Chapter 9
1. Key: C
Since the lives are independent, the joint-life force of mortality is

35:40
=
35
+
40
= 0.08
So the joint-life, force of mortality is also constant.

e
35:40
=
1
0.08
= 12.5
2. Key: D
Constant Force of Mortality:
A
x
=A
y
=

+
=
0.06
0.1
= 0.6

xy
=
x
+
y
A
xy
=
0.12
0.12 + 0.04
= 0.75
A
xy
=A
x
+A
y
A
xy
= 0.6 + 0.6 0.75 = 0.45
a
xy
=
1
+ +
=
1
0.16
= 6.25
P =
A
xy
a
xy
=
0.45
6.25
= 0.072
3. Key: C
10
q
40:50
= 1
10
p
40:50
= 1
10
p
40

10
p
50
Since this is De Moivres law:
10
p
40
=
5
6
and
10
p
50
=
4
5

10
q
40:50
= 1
_
5
6

4
5
_
=
1
3
4. Key: A
10|10
q
40:50
=
10|10
q
40
+
10|10
q
50

10|10
q
40:50
=
10
p
40

10
q
50
+
10
p
50

10
q
60

10
p
40:50

10
q
50:60
=
5
6
_
1
5
_
+
4
5
_
1
4
_

5
6
_
4
5
__
1
4
5
_
3
4
__
=
1
10
Arch MLC, Fall 2007 c _Yufeng Guo 283
www.archactuarial.com www.guo.coursehost.com 284
5. Key: A
10
q
30:50
= 1
10
p
30:50
10
p
30:50
=
10
p
30
+
10
p
50

10
p
30:50
e
10(0.05)
+e
10(0.05)
e
20(0.05)
= 0.845

10
q
30:50
= 1 0.845 = 0.155
6. Key: C
Constant Force of mortality:

xy
=
x
+
y
= 0.1

e
xy
=
1

xy
= 10

e
x
=

e
y
=
1
0.05
= 20

e
xy
=

e
x
+

e
y


e
xy
= 30
7. Key: B

xy
= 0.1, a constant. So,
Var[T] =
1
(
xy
)
2
=
1
0.01
= 100
8. Key: D
Cov =
_

e
x


e
xy
_ _

e
y


e
xy
_
= (20 10)(20 10) = 100
Arch MLC, Fall 2007 c _Yufeng Guo 284
Chapter 8
ACTUARIAL MATHEMATICS:
CHAPTER 10 MULTIPLE
DECREMENT MODELS
Option A reference: Actuarial Mathematics Chapter 10
Option B reference: Models for Quantifying Risk Chapter 15
For most of this text, weve encountered models with one life and one decrement (death).
We also looked at models with multiple lives and one decrement in Chapter 9. Now we are
back to models with one life, but these lives face multiple decrements. For example, consider
a group insurance plan provided by an employer. At time t = 0, there is some number
of employees covered (the radix l
0
). However, over time, this number will decrease due
to death, retirement, disability, or changing jobs (these are all dierent decrements). To
accurately model costs of such plans, along with retirement plans or disability insurance, we
need models that take into account both time of termination and cause of termination.
10.2 Two Random Variables
Option A reference: Actuarial Mathematics Chapter 10.2
Option B reference: Models for Quantifying Risk Chapter 15.1
In the models we will be developing in this chapter, there are two random variables. You are
already familiar with time of decrement, which is denoted by T(x). Now, we add a discrete
random variable for cause of decrement, J(x). For example, if death is decrement 1 and
withdrawal is decrement 2, then J(x) = 1 corresponds to decrement by death, and J(x) = 2
corresponds to decrement by withdrawal.
There are a few new notational concepts in this chapter. For example,
t
q
(j)
x
denotes the
probability of decrement in the next t years due to cause j. Also,
t
q
()
x
denotes the probability
of decrement due to all causes before time t.
285
www.archactuarial.com www.guo.coursehost.com 286
Suppose the only decrements to a retirement plan are (1) death and (2) retirement.
For example, if q
(1)
63
= 0.03 and q
(2)
63
= 0.3 , then
q
()
63
= q
(1)
63
+q
(2)
63
= 0.33.
And
p
()
63
= 1 q
()
63
= 0.67,
where p
()
63
is the probability of surviving all causes of decrement for 1 year. It is always true
that
t
p
()
x
= 1
t
q
()
x
.
The symbol
()
x
represents the force of decrement due to all causes combined, whereas
(j)
x
represents the force of decrement due only to decrement j. As you might guess,

()
x
=
m

j=1

(j)
x
,
where m is the number of dierent decrements trying to get at (x). Just as for single
decrement functions, it is also true that

()
x
=

d
dt
_
t
p
()
x
_
t
p
()
x
=
d
dt
_
t
q
()
x
_
t
p
()
x
,
and
t
p
()
x
= e

_
t
0

()
x
(s) ds
.
EXAMPLE:
If
(1)
x
(t) = 0.03 and
(2)
x
(t) = 0.1t for all t, nd a formula for
t
p
()
x
in terms of t.
SOLUTION:
t
p
()
x
= e

_
t
0

()
x
(s) ds
= e

_
t
0
(0.1s+0.03) ds
= e
(0.05)t
2
(0.03)t
.
Compare the following two integral expressions:
t
q
(j)
x
=
_
t
0
s
p
()
x

(j)
x
(s) ds,
t
q
()
x
=
_
t
0
s
p
()
x

()
x
(s) ds.
The rst integral adds up over all times s, the probability of surviving to s times the proba-
bility of decrementing in the next instant due to cause j. The second integral is the same
Arch MLC, Fall 2007 c _Yufeng Guo 286
www.archactuarial.com www.guo.coursehost.com 287
except that all decrements are taken into account. Note that in both integrals,
s
p
()
x
gives the
probability of surviving to time s, this is because no matter how you decrement at time s,
you must survive all decrements up to time s before decrementing at time s. These integrals
could be used to show the very intuitive fact that
t
q
()
x
=
m

j=1
t
q
(j)
x
.
Probability density functions:
Since there are several decrements now, the p.d.f. has two variables. f
T,J
(t, j) is the proba-
bility of dying at time t from cause j. The discussion above means that
f
T,J
(t, j) =
t
p
()
x

(j)
x
(t).
A nice summary of the p.d.fs you need to have in your bag of tricks is given below. Questions
on these are not plentiful on past tests but they do occur. It is important that you keep track
of the name of each one, since a question may ask for one by name without giving the
functional expression.
Joint PDF f
T,J
(t, j) =
t
p
()
x

(j)
x
(t)
Marginal PDF of J f
J
(j) =

q
(j)
x
Marginal PDF of T f
T
(t) =
t
p
()
x

()
x
(t).
Note that the Marginal PDF of J represents the probability of decrementing due to cause
j at some in the future. Similarly, the Marginal PDF of T represents the probability of
decrementing at time t due to any cause.
There is one other important PDF to know. It is a conditional probability that represents the
probability of decrementing due to cause j, given that you decrement at a specied time t.
The symbol is f
J|T
(j[t) (more nearly indecipherable notation!), and
The Conditional PDF f
J|T
(j[t) =

(j)
x
(t)

()
x
(t)
.
EXAMPLE:
You are given:

(1)
x
(t) =
2
25

(2)
x
(t) =
7
25
Arch MLC, Fall 2007 c _Yufeng Guo 287
www.archactuarial.com www.guo.coursehost.com 288
Find the p.d.f.s for
(A) the joint distribution,
(B) the marginal distributions, and
(C) the probability that (x) decrements due to cause 2 if decrement occurs at
t = 1.
Arch MLC, Fall 2007 c _Yufeng Guo 288
www.archactuarial.com www.guo.coursehost.com 289
SOLUTION:
(A) Joint Distribution:
f
T,J
(t, j) =
t
p
()
x

(j)
x
(t)
We know that

()
x
(t) =
(1)
x
(t) +
(2)
x
(t) =
9
25
.

t
p
()
x
= e

_
t
0
9/25 dt
= e
(0.36)t
Therefore,
f
T,J
(t, j) = e
(0.36)t

(j)
x
(t) =
_

_
e
(0.36)t

2
25
for j = 1
e
(0.36)t

7
25
for j = 2
(B) Marginal Distributions:
f
J
(j) =

q
(j)
x
=
_

0
t
p
()
x

(j)
(t) dt
=
_

_
_

0
e
(0.36)t 2
25
dt for j = 1
_

0
e
(0.36)t 7
25
dt for j = 2
=
_

_
2
9
for j = 1
7
9
for j = 2
Note that we could also have gotten these using
f
J
(j) =
_

0
f
T,J
(t, j)dt.
Now for the marginal distribution of T:
f
T
(t) =
t
p
()
x

()
x
(t) = e
(0.36)t
9
25
.
(C) Bypassing the ugly notation of the conditional distribution, this probability
is given by

(j)
x
(t)

()
x
(t)
,
where j = 2 and t = 1. So the probability we want is
7
25
9
25
=
7
9
The conditional distribution turned out to be a constant since both s were
constant, but that will not always be the case.
Arch MLC, Fall 2007 c _Yufeng Guo 289
www.archactuarial.com www.guo.coursehost.com 290
For a (much) more complex example of the same ideas, see Example 10.2.1 in the text.
EXAMPLE:
In a 2-decrement environment, you are given:

(1)
x
(t) = 0.3t, t 0

2
p
()
x
= 0.3
Find
e

_
2
0

(2)
x
(t) dt
.
SOLUTION:
t
p
()
x
= e

_
t
0

()
x
(s) ds
For t = 2, we have
2
p
()
x
= e

_
2
0

()
x
(t) dt
= e

_
2
0
_

(1)
x
(t)+
(2)
x
(t)
_
dt
= e

_
2
0

(1)
x
(t) dt
e

_
2
0

(2)
x
(t) dt

2
p
()
x
= e
(0.6)
e

_
2
0

(2)
x
(t) dt
e

_
2
0

(2)
x
(t) dt
=
0.3
0.549
= 0.547.
10.3 Random Survivorship Group
10.4 Deterministic Survivorship Group
Option A reference: Actuarial Mathematics Chapter 10.3, 10.4
Option B reference: Models for Quantifying Risk Chapter 15.1.1
Sections 10.3 and 10.4 are really just two dierent ways at looking at the same thing so we
will consider them together.
Suppose we have a group of people all at some age a at a given point in time. The number of
people in this group at time 0 is denoted by l
()
a
. (Well get to what the represents here.)
We assume that each member of the group has a joint pdf for time until decrement and cause
of decrement given by
f
T,J
(t, j) =
t
p
()
a

(j)
a
(t) for t 0, j = 1, 2, . . . , m.
Let
n
d
(j)
x
represent the expected number of lives to leave the group during the age-interval
(x, x+n) due to cause j. This is looking at the number of people that die in the next n years
after reaching age x as a random variable. If we were looking at things deterministically,
we would be saying that we know ahead of time that the number of people that will decrement
Arch MLC, Fall 2007 c _Yufeng Guo 290
www.archactuarial.com www.guo.coursehost.com 291
in the next n years due to cause j will denitely be
n
d
(j)
x
. In that case we dont have to talk
about it as an expected value. For the purposes of the exam, this distinction is not very
important.
Note that even though we are interested in who leaves the group between ages x and (x+n),
we are calculating this number while everyone is currently age a. So
3
d
(2)
x
refers to the number
of people among the original l
()
a
that live to age x and then decrement due to cause 2 before
reaching age (x + 3).
EXAMPLE:
1. Find an expression for
n
d
(j)
x
in terms of l
()
a
, p, and
n
q
(j)
x
.
2. Find an integral expression for
n
d
(j)
x
in terms of l
()
a
,
t
p
()
a
, and
(j)
a
(t).
SOLUTION:
1. This one is not too bad. The number that decrement from cause j among
the l
()
a
people that start out should just be
n
d
(j)
x
= l
()
a

xa
p
()
a n
q
(j)
x
2. We want to integrate over all times t between t = (xa) and t = (xa+n).
The probability of surviving all decrements for t years and then decrementing
in the next instant due to cause j:
n
d
(j)
x
= l
()
a
_
xa+n
xa
t
p
()
a

(j)
a
(t) dt.
Recall that indicates all causes, so we have
n
d
()
x
=
m

j=1
n
d
(j)
x
.
Similarly, the l
()
a
symbol represents the set of lives in the group that will eventually decrement
due to some cause (i.e. everyone!). If we write l
(j)
a
, we are referring to the people in the
group who eventually decrement due to cause j. Clearly,
l
()
a
=
m

j=1
l
(j)
a
.
For example, say that we begin with 100 people. And suppose that we expect that over time
40 will decrement due to retirement, 40 will leave to pursue other jobs, 10 will leave due to
disability, and 10 will leave due to death. Our total is the sum of these various decrement
pieces. Algebraically,
l
()
a
= l
(1)
a
+l
(2)
a
+l
(3)
a
+l
(4)
a
= 40 + 40 + 10 + 10.
Arch MLC, Fall 2007 c _Yufeng Guo 291
www.archactuarial.com www.guo.coursehost.com 292
We now have the formulas we need to construct a multiple decrement table. We saw
the idea of a single decrement (death) mortality table in Chapter 3. This table is simply
an extension to allow for multiple decrements. It allows the user to track the number of
terminations due to specic causes.
Read Example 10.3.1 in the text and then try to complete the multiple decrement mortality
table begun below (the solution is on the next page.) Assume l
()
40
= 100.
x q
(1)
x
q
(2)
x
p
()
x
l
()
x
d
(1)
x
d
(2)
x
40 0.01 0.05
41 0.02 0.06
42 0.03 0.07
43 0.04 0.08
Arch MLC, Fall 2007 c _Yufeng Guo 292
www.archactuarial.com www.guo.coursehost.com 293
Completed Multiple Decrement Table
x q
(1)
x
q
(2)
x
p
()
x
l
()
x
d
(1)
x
d
(2)
x
40 0.01 0.05 0.94 100.0 1.00 5.00
41 0.02 0.06 0.92 94.0 1.88 5.64
42 0.03 0.07 0.90 86.5 2.59 6.06
43 0.04 0.08 0.88 77.9 3.12 6.23
EXAMPLE:
Using the table above, determine each of the following.
2
q
(1)
42 2
q
()
42 1|2
q
(2)
40
SOLUTION: The rst one is the number of people who decrement due to cause
1 at ages 42 and 43 divided by the number alive at age 42.
2
q
(1)
42
=
d
(1)
42
+d
(1)
43
l
42
=
5.71
86.5
= 0.066
We could also have done this from earlier principles using ps and qs:
2
q
(1)
42
= q
(1)
42
+p
()
42
q
(1)
43
.
Next
2
q
()
42
=
d
()
42
+d
()
43
l
42
=
8.65 + 9.35
86.5
= 0.208.
1|2
q
(2)
40
=
d
(2)
41
+d
(2)
42
l
40
=
5.64 + 6.06
100
= 0.117.
EXAMPLE:
Using the table above, determine each of the following.
2
p
()
41 2
d
()
41
SOLUTION:
2
p
()
41
=
l
43
l
41
=
77.9
94
= 0.829,
2
d
()
41
= d
()
41
+d
()
42
= (1.88 + 5.64) + (2.59 + 6.06) = 16.17.
Arch MLC, Fall 2007 c _Yufeng Guo 293
www.archactuarial.com www.guo.coursehost.com 294
EXAMPLE:
Write q
(j)
x
in terms of d
(j)
x
and l
()
x
.
SOLUTION:
q
(j)
x
=
d
(j)
x
l
()
x
.
10.5 Associated Single Decrement Tables
Option A reference: Actuarial Mathematics Chapter 10.5
Option B reference: Models for Quantifying Risk Chapter 15.4.1
In Section 10.5 the text introduces an idea that has several names net probabilities of
decrement, independent rate of decrement, and the SOA-approved term absolute rate
of decrement. The idea also has a new symbol
t
q
(j)
x
as opposed to the
t
q
(j)
x
weve seen
previously. The symbol
t
q
(j)
x
represents the probability of decrement from cause j only,
ignoring the other possible causes. In other words, is the probability that (x) would decrement
from cause j in the next t years if there were no other causes of decrement.
A not-so-obvious, but important fact is the following:
t
q
(j)
x

t
q
(j)
x
. (Understand why this is true!!)
This is true because if the other decrements are not around for the next t years, the probability
that decrement j will get you is higher. For example suppose that every year, both death
and retirement have a 40 percent chance of acting on (x). If you take away the possibility
of retirement, the probability that (x) will decrement by dying is much higher we lose all
those cases in which (x) might have retired before dying. Understanding this concept should
help your intuition when trying to answer multiple decrement types of questions.
Since
t
q
(j)
x
takes into account the force of decrement for cause j only (forsaking all others),
it then follows that
t
p
(j)
x
also ignores all other possible causes of decrement:
1
t
q
(j)
x
=
t
p
(j)
x
= exp
_

_
t
0

(j)
x
(s)ds
_
.
EXAMPLE: Jake
Jake has an old car, age x, that faces two forces of decrement (as far as Jake is
concerned). They are (1) breakdown and (2) sale. Both forces of decrement are
constants:
(1)
x
= 0.03 and
(2)
x
= 0.1.
Find q

(1)
x
and q
(1)
x
.
Arch MLC, Fall 2007 c _Yufeng Guo 294
www.archactuarial.com www.guo.coursehost.com 295
SOLUTION:
q

(1)
x
=
_
1
0
t
p

(1)
x

(1)
x
(t) dt
Notice that the second force of decrement doesnt show up at all in the calculation
of q

(1)
x
. It doesnt exist if Jake agrees not to sell his car. We need
t
p

(1)
x
, so
t
p

(1)
x
= e

_
t
0

(1)
x
(s) ds
= e
0.03t
.
q

(1)
x
=
_
1
0
e
0.03t
(0.03)dt = 1 e
0.03
= 0.0296.
(We could also have used q

(1)
x
= 1 p

(1)
x
.)
q
(1)
x
=
_
1
0
t
p
()
x

(1)
x
(t) dt
Now there are two decrements so we have to survive all decrements up to the time
the rst decrements hit.
q
(1)
x
=
_
1
0
e
(0.13)t
(0.03) dt =
0.03
0.13
_
1 e
0.13
_
= 0.0281.
Note that q

(1)
x
> q
(1)
x
. If Jake agrees not to sell his car next year, there is a greater
probability that he will experience a breakdown.
10.5.1 Basic Relationships
Option A reference: Actuarial Mathematics Chapter 10.5.1
Option B reference: Models for Quantifying Risk Chapter 15.1-15.4
For m causes of decrement,
t
p
()
x
= exp
_

_
t
0
_

(1)
x
(s) +
(2)
x
(s) + +
(m)
x
(s)
_
ds
_
.
So, using a little algebra for exponential functions, we can see that
t
p
()
x
=
m

i=1
t
p
(i)
x
It follows from this relation that
t
p
(j)
x

t
p
()
x
and we have already seen that q
(j)
x
q
(j)
x
.
EXAMPLE: (I hope by now you are working all of the exercises before looking
at the solutions!)
You are given:
q
(1)
x
= 0.2, q
(2)
x
= 0.1, and q

(1)
x
= 0.21.
Find q

(2)
x
.
Arch MLC, Fall 2007 c _Yufeng Guo 295
www.archactuarial.com www.guo.coursehost.com 296
SOLUTION:
Often with associated decrement problems, it is easiest to work through the ps.
p
()
x
= 1 q
(1)
x
q
(2)
x
= 0.7
p
()
x
= p

(1)
x
p

(2)
x
=
_
1 q

(1)
x
_ _
1 q

(2)
x
_
0.7 = (0.79)
_
1 q

(2)
x
_
q

(2)
x
= 0.114
10.5.4 Uniform Distribution Assumption for Multiple Decrements
Option A reference: Actuarial Mathematics Chapter 10.5.4
Option B reference: Models for Quantifying Risk Chapter 15.4.1
Assuming a Uniform Distribution of Decrement over the interval (x, x + 1) implies that
t
q
(j)
x
= t q
(j)
x
and
t
q
()
x
= t q
()
x
.
Also, at any time t between 0 and 1,
t
p
()
x

(j)
x
(t) = q
(j)
x
. (UDD)
This tells us that the pdf over each year is a constant. Finally, it also gives us a handy
formula for
(j)
x
(t):

(j)
x
(t) =
q
(j)
x
t
p
()
x
=
q
(j)
x
1 t q
()
x
. (UDD)
EXAMPLE:
You are given:
q
(1)
x
= 0.05 and q
(2)
x
= 0.08,
All decrements are uniform in the multiple decrement context over the in-
terval (x, x + 1).
Find
(1)
x
(0.25) and
(1)
x
(0.75).
SOLUTION:

(1)
x
(0.25) =
q
(1)
x
1 (0.25) q
()
x
=
0.05
0.97
= 0.052

(1)
x
(0.75) =
q
(1)
x
1 (0.75) q
()
x
=
0.05
0.90
= 0.055
Arch MLC, Fall 2007 c _Yufeng Guo 296
www.archactuarial.com www.guo.coursehost.com 297
Note that if decrements are uniformly distributed, then the force of decrement
must increase over time. This could help you see that some answers you get are
reasonable compared to each other.
Letting 0 s 1, we can derive that
s
p
(j)
x
=
_
s
p
()
x
_
q
(j)
x
q
()
x
. (UDD)
When s = 1, this tells us that
p
(j)
x
=
_
p
()
x
_
q
(j)
x
q
()
x
. (UDD)
This formula is the same as the one in the constant force of decrement case if s = 1. I.e.
they are the same when you are only considering full-year decrements. Once again, note that
this formula cannot be used in the event that p
(j)
x
= 0 or p
()
x
= 0. More about this in Section
10.6.
IMPORTANT NOTE: These messy formulas result when we assume that the decrements
are uniform in the multiple decrement context, i.e., when we assume the
t
q
(j)
x
are uni-
formly distributed. A more common assumption that we will see in Section 10.6 is to assume
that the individual decrements are uniformly distributed in the associated single decre-
ment table, i.e., we assume the
t
q

(j)
x
are uniformly distributed. This leads to relatively
simple formulas. This is a distinction that you have to keep straight when reading questions
of this type or you may waste lots of time and still get the wrong answer.
END OF IMPORTANT NOTE
EXAMPLE: Jake 2
Jake has the same old car, age x, that faces two forces of decrement. They are (1)
breakdown and (2) sale. But now Jake believes that the decrements are uniform
over the interval (x, x+1) in the multiple decrement context, with q
(1)
x
= 0.2 and
q
(2)
x
= 0.1.
Find the probability that the car will break down before half a year has passed if
Jake agrees ahead of time NOT to sell the car next year.
SOLUTION:
We are looking for q

(1)
x
, and this one is trickier than it looks. Although
t
q
(1)
x
is
uniformly distributed on (x, x + 1),
t
q

(1)
x
is not. We can use
s
p
(j)
x
=
_
s
p
()
x
_
q
(j)
x
q
()
x
.
With s = 0.5 and j = 1, this becomes
0.5
p
(1)
x
=
_
0.5
p
()
x
_
q
(1)
x
q
()
x
=
_
1 (0.5)q
()
x
_2
3
=
Arch MLC, Fall 2007 c _Yufeng Guo 297
www.archactuarial.com www.guo.coursehost.com 298
= (0.85)
(0.66667)
= 0.897

0.5
q

(1)
x
= 1 0.897 = 0.103.
10.6 Construction of a Multiple Decrement Table
Option A reference: Actuarial Mathematics Chapter 10
Option B reference: Models for Quantifying Risk Chapter 15.1, 15.2
We have already done some Decrement Table development earlier in this chapter. The dier-
ence here is that we are including and using ideas from the Associated Single Decrement
Table (i.e. the primes). It turns out that life is often a lot easier if we are told the primes
are uniform, than if we are told the non-primes are uniform.
CASE I : Two decrements that are uniformly distributed in the associated
single decrement table
In this case
t
q

(j)
x
= tq

(j)
x
, the distribution of each decrement would be uniform if the other
decrement were removed. When this is true, there are two key formulas:
q
(1)
x
= q

(1)
x
_
1
1
2
q

(2)
x
_
q
(2)
x
= q

(2)
x
_
1
1
2
q

(1)
x
_
Read these equations carefully and think about them! One way to remember them is to keep
in mind that q
(1)
x
should be a little less than q

(1)
x
.
CASE II : Three decrements that are uniformly distributed in the asso-
ciated single decrement table
No need to memorize the following dicult formula. You can always derive it from the
scratch:
q
(1)
x
= q

(1)
x
_
1
1
2
q

(2)
x

1
2
q

(3)
x
+
1
3
q

(2)
x
q

(3)
x
_
Similar formulas hold for q
(2)
x
and q
(3)
x
.
CASE III : Multiple decrements some are uniformly distributed in the
associated single decrement table and some are not.
These have to be done on a case-by-case basis. Well will see them in the examples.
EXAMPLE: Jake 3
Jake has the same old car, age x, that faces two forces of decrement. They are (1) breakdown
and (2) sale. But now Jake believes that the decrements are uniform over the interval (x, x+1)
in the associated single decrement table. He also assumes that q
(1)
x
= 0.2 and q
(2)
x
= 0.1
Arch MLC, Fall 2007 c _Yufeng Guo 298
www.archactuarial.com www.guo.coursehost.com 299
Find the probability that the car will break down before a year has passed if Jake were to
agree ahead of time NOT to sell the car next year.
Arch MLC, Fall 2007 c _Yufeng Guo 299
www.archactuarial.com www.guo.coursehost.com 300
SOLUTION:
We are looking for q

(1)
x
. We know that
0.2 = q

(1)
x
_
1
1
2
q

(2)
x
_
0.1 = q

(2)
x
_
1
1
2
q

(1)
x
_
Now using the second equation to solve for q

(2)
x
, and substituting into the rst equation, we
get
0.2 = q

(1)
x
_
1
0.05
1 (0.5)q

(1)
x
_
Which leads to the following quadratic equation:
0.2 (0.1)q

(1)
x
= q

(1)
x
_
0.95 0.5q

(1)
x
_
or
(0.5)
_
q

(1)
x
_
2
(1.05)q

(1)
x
+ 0.2 = 0
Using the quadratic formula that we learned in (junior?) high school, we have
q

(1)
x
= 0.212
We can tell there is a good chance we did things right because the q

(1)
x
that we got is a little
bigger than q
(1)
x
.
EXAMPLE: Jake 4
Jake has the same old car, age x, that faces two forces of decrement but the model has
changed. The decrements are still (1) breakdown and (2) sale. But now Jake believes that
the breakdown decrement is uniform over the interval (x, x + 1) in the associated single
decrement table. The sale decrement can only take place at year-end. You are given:
q

(1)
x
= 0.25,
q

(2)
x
= 0.5.
Find q
(1)
x
, q
(2)
x
, and q
()
x
.
SOLUTION:
To do this one we should note that during the year, while the rst decrement is acting on
the car, the second decrement is in fact absent. After all, if Jake sells, he wont sell until
year-end. Therefore
q
(1)
x
= q

(1)
x
= 0.25.
For the second decrement, we note that Jake can only sell the car if it is still around after a
whole year of being hit by the rst decrement. So
q
(2)
x
= (1 q

(1)
x
)q

(2)
x
= (0.75)(0.5) = 0.375
Arch MLC, Fall 2007 c _Yufeng Guo 300
www.archactuarial.com www.guo.coursehost.com 301
Note that there was no
1
2
before q

(1)
x
in the expression above because the rst decrement
operated for a whole year before the sale decrement acted on the car.
Finally, q
()
x
= 0.375 + 0.25 = 0.625. We could have gotten this quantity directly from the
given information using
p
()
x
= p

(1)
x
p

(2)
x
=
_
1 q

(1)
x
_ _
q

(2)
x
_
= 0.375
Then q
()
x
= 1 0.375 = 0.625.
EXAMPLE: Jake 5
Jake has the same old car, age x, that faces two forces of decrement but the model has
changed. The decrements are still (1) breakdown and (2) sale. But now Jake believes that
the breakdown decrement is uniform over the interval (x, x + 1) in the associated single
decrement table. The sale decrement can only take place at mid-year. You are given:
q

(1)
x
= 0.25,
q

(2)
x
= 0.5.
Find q
(1)
x
and q
(2)
x
.
SOLUTION:
If you can do this one condently, you are probably ready for what the SOA can throw at
you! To do this one right, we really need to break the year into 3 parts: 1st half, mid-year,
2nd-half.
During the 1st half of the year, the only decrement acting on the car is (1). So the probability
of the car decrementing due to cause (1) during the 1st half of the year is
1
2
q

(1)
x
= 0.125.
We used the fact that the rst decrement is uniform over (x, x + 1) so that
t
q

(1)
x
= tq

(1)
x
.
At mid-year, in order for the car to decrement due to cause (2) it must have survived the 1st
half of the year, so the probability of decrement due to cause (2) is
q
(2)
x
=
_
1
1
2
q

(1)
x
_
q

(2)
x
= (0.875)(0.5) = 0.4375.
Now, during the second half of the year, the car can decrement due to cause (1) only if it
survived both the 1st half of the year and survived mid-year. So the probability of the car
decrementing due to cause (1) during the 2nd half of the year is
(1 0.125 0.4375)
_
_
1
2
q

(1)
x
1
1
2
q

(1)
x
_
_
= 0.0625
Arch MLC, Fall 2007 c _Yufeng Guo 301
www.archactuarial.com www.guo.coursehost.com 302
Therefore q
(1)
x
= 0.125 + 0.0625 = 0.1875 and q
(2)
x
= 0.4375. We can check our work by
checking that p
()
x
= 1 q
(1)
x
q
(2)
x
comes very close (equal except for rounding) to p
()
x
=
_
1 q

(1)
x
_ _
1 q

(2)
x
_
. On the exam you wont have to check since you will have one of the
5 answer choices!
EXAMPLE: Jake 6 Last Jake Problem
Jake has decided to rene his model by adding a 3rd decrement, collision. It is assumed
that decrements (1) and (3), breakdown and collision, are both uniform over (x, x + 1) in
the associated single decrement table. It is also assumed that the car may be sold only at
year-end.
You are given:
q

(1)
x
= 0.2,
q

(2)
x
= 0.5,
q

(3)
x
= 0.4 (Jake is a poor driver).
Find the probabilities of decrement for each of the three causes.
SOLUTION:
For the entire year until the last moment, decrements (1) and (3) are operating in the presence
of each other and both are uniform in the associated single decrement table so
q
(1)
x
= q

(1)
x
_
1
1
2
q

(3)
x
_
= (0.2)(0.8) = 0.16.
q
(3)
x
= q

(3)
x
_
1
1
2
q

(1)
x
_
= (0.4)(0.9) = 0.36.
Finally, decrement (2) operates only at year-end, so
q
(2)
x
=
_
1 q
(1)
x
q
(3)
x
_
q

(2)
x
= (0.48)(0.5) = 0.24.
As a last comment to this section, lets consider Examples 10.6.1 and 10.6.2 in the text.
The whole point to these examples is that in 10.6.1, q
(1)
x
, q
(2)
x
, q
(3)
x
were calculated assuming a
constant force of mortality whereas in Example 10.6.2, it was assumed that the decrements
were uniformly distributed in each year of age in the associated single-life decrement table.
You should be able to duplicate the rst couple of entries for either table. Finally, Example
10.6.3 gives a more complicated version of our examples above.
Chapter 10 Suggested Problems: 1, 2, 4, 5, 7, 8, 10, 14, 17, 28, 29
(Solutions at archactuarial.com)
Arch MLC, Fall 2007 c _Yufeng Guo 302
www.archactuarial.com www.guo.coursehost.com 303
CHAPTER 10 Formula Summary

()
x
=
m

j=1

(j)
x
,
t
p
()
x
= e

_
t
0

()
x
(s) ds
t
q
(j)
x
=
_
t
0
s
p
()
x

(j)
x
(s) ds,
t
q
()
x
=
_
t
0
s
p
()
x

()
x
(s) ds =
m

j=1
t
q
(j)
x
Joint PDF f
T,J
(t, j) =
t
p
()
x

(j)
x
(t)
Marginal PDF of J f
J
(j) =

q
(j)
x
Marginal PDF of T f
T
(t) =
t
p
()
x

()
x
(t).
n
d
(j)
x
= l
()
a

xa
p
()
a n
q
(j)
x
= l
()
a

_
xa+n
xa
t
p
()
a

(j)
a
(t) dt
n
d
()
x
=
m

j=1
n
d
(j)
x
l
()
a
=
m

j=1
l
(j)
a
q
(j)
x
=
d
(j)
x
l
()
x
t
q
(j)
x

t
q
(j)
x
1
t
q
(j)
x
=
t
p
(j)
x
= exp
_

_
t
0

(j)
x
(s)ds
_
Arch MLC, Fall 2007 c _Yufeng Guo 303
www.archactuarial.com www.guo.coursehost.com 304
For m causes of decrement,
t
p
()
x
= exp
_

_
t
0
_

(1)
x
(s) +
(2)
x
(s) + +
(m)
x
(s)
_
ds
_
t
p
()
x
=
m

i=1
t
p
(i)
x
t
p
(j)
x

t
p
()
x
UDD in the Multiple Decrement Table over the interval (x, x + 1):
t
q
(j)
x
= t q
(j)
x
and
t
q
()
x
= t q
()
x
t
p
()
x

(j)
x
(t) = q
(j)
x

(j)
x
(t) =
q
(j)
x
t
p
()
x
=
q
(j)
x
1 t q
()
x
s
p
(j)
x
=
_
s
p
()
x
_
q
(j)
x
q
()
x
Note that this formula cannot be used in the event that p
(j)
x
= 0 or p
()
x
= 0.
If two decrements are uniformly distributed in the associated single decrement table:
q
(1)
x
= q

(1)
x
_
1
1
2
q

(2)
x
_
q
(2)
x
= q

(2)
x
_
1
1
2
q

(1)
x
_
Read these equations carefully and think about them! One way to remember them is to keep
in mind that q
(1)
x
should be a little less than q

(1)
x
.
If three decrements are uniformly distributed in the associated single decrement table:
q
(1)
x
= q

(1)
x
_
1
1
2
q

(2)
x

1
2
q

(3)
x
+
1
3
q

(2)
x
q

(3)
x
_
Similar formulas hold for q
(2)
x
and q
(3)
x
.
Arch MLC, Fall 2007 c _Yufeng Guo 304
www.archactuarial.com www.guo.coursehost.com 305
Past SOA/CAS Exam Questions:
1. In a double decrement table:
(i) l
()
30
= 1000, l
()
32
= 472
(ii) q
(1)
30
= 0.100, q
(2)
30
= 0.300
(iii)
1|
q
(1)
30
= 0.075
Calculate q
(2)
31
.
(A) 0.11 (B) 0.13 (C) 0.14 (D) 0.15 (E) 0.17
Solution:
p
()
30
= p

(1)
30
p

(2)
30
= (0.9)(0.7) = 0.63
l
()
31
= p
()
30
l
()
30
= 630
d
(1)
31
=
1|
q
(1)
30
l
()
30
= 75
d
()
31
= l
()
31
l
()
32
= 630 472 = 158
d
(2)
31
= d
()
31
d
(1)
31
= 158 75 = 83
q
(2)
31
=
d
(2)
31
l
()
31
=
83
630
= 0.1317 0.13
Key: B
2. For a double decrement table with l
()
40
= 2000:
x q
(1)
x
q
(2)
x
q

(1)
x
q

(2)
x
40 0.24 0.10 0.25 y
41 - - - - 0.20 2y
Calculate l
()
42
.
(A) 800 (B) 820 (C) 840 (D) 860 (E) 880
Arch MLC, Fall 2007 c _Yufeng Guo 305
www.archactuarial.com www.guo.coursehost.com 306
Solution:
q
()
40
= q
(1)
40
+q
(2)
40
= 0.34
= 1 p

(1)
40
p

(2)
40
0.34 = 1 0.75p

(2)
40
p

(2)
40
= 0.88 q

(2)
40
= 0.12 = y
q

(2)
41
= 2y = 0.24
q
()
41
= 1 (0.8)(1 0.24) = 0.392
l
()
42
= 2000(1 0.34)(1 0.392) = 803
Key: A
3. For a multiple decrement table, you are given:
(i) Decrement (1) is death, decrement (2) is disability, and decrement (3) is with-
drawal.
(ii) q

(1)
60
= 0.010
(iii) q

(2)
60
= 0.050
(iv) q

(3)
60
= 0.100
(v) Withdrawals occur only at the end of the year.
(vi) Mortality and disability are uniformly distributed over each year of age in the
associated single decrement tables.
Calculate q
(3)
60
.
(A) 0.088 (B) 0.091 (C) 0.094 (D) 0.097 (E) 0.100
Solution:
Since only decrements (1) and (2) occur during the year, probability of reaching the
end of the year is
p

(1)
60
p

(2)
60
= (1 0.01)(1 0.05) = 0.9405
Probability of remaining through the year is
p

(1)
60
p

(2)
60
p

(3)
60
= (1 0.01)(1 0.05)(1 0.10) = 0.84645
Arch MLC, Fall 2007 c _Yufeng Guo 306
www.archactuarial.com www.guo.coursehost.com 307
Probability of exiting at the end of the year is
q
(3)
60
= 0.9405 0.84645 = 0.09405
Key: C
4. For students entering a three-year law school, you are given:
(i) The following double decrement table:
For a student at the beginning of that academic year,
probability of
Withdrawal for Survival
Academic Academic All Other Through
Year Failure Reasons Academic Year
1 0.40 0.20
2 0.30
3 0.60
(ii) Ten times as many students survive year 2 as fail during year 3.
(iii) The number of students who fail during year 2 is 40% of the number of students
who survive year 2.
Calculate the probability that a student entering the school will withdraw for reasons
other than academic failure before graduation.
(A) Less than 0.35
(B) At least 0.35, but less than 0.40
(C) At least 0.40, but less than 0.45
(D) At least 0.45, but less than 0.50
(E) At least 0.50
Solution:
Let l
()
0
= number of students entering in year 1.
Superscript (f) denotes academic failure,
superscript (w) denotes withdrawal,
subscript is age at start of year; equals year - 1.
p
()
0
= 1 0.40 0.20 = 0.40
l
()
s
= 10l
()
2
q
(f)
2
q
(f)
2
= 0.1
Arch MLC, Fall 2007 c _Yufeng Guo 307
www.archactuarial.com www.guo.coursehost.com 308
q
(w)
2
= q
()
2
q
(f)
2
= (1.0 0.6) 0.1 = 0.3
l
()
1
q
(f)
1
= 0.4
_
l
()
1
_
1 q
(f)
1
q
(w)
1
__
q
(f)
1
= 0.4
_
1 q
(f)
1
0.3
_
q
(f)
1
=
0.28
1.4
= 0.2
p
()
1
= 1 q
(f)
1
q
(w)
1
= 1 0.2 0.3 = 0.5
3
q
(w)
0
= q
(w)
0
+p
()
0
q
(w)
1
+p
()
0
p
()
1
q
(w)
2
= 0.2 + (0.4)(0.3) + (0.4)(0.5)(0.3) = 0.38
Key: B
5. For a multiple decrement model on (60):
(i)
(1)
60
(t), t 0, follows the Illustrative Life Table.
(ii)
()
60
(t) = 2
(1)
60
(t), t 0,
Calculate
10|
q
()
60
(A) 0.03 (B) 0.04 (C) 0.05 (D) 0.06 (E) 0.07
SOLUTION:
k
p
()
x
= e

_
k
0

()
x
(t)dt
o = e

_
k
0
2
(1)
x
(t)dt
=
_
e

_
k
0

(1)
x
(t)dt
_
2
= (
k
p
x
)
2
where
k
p
x
is from the ILT, since
(1)
follows ILT.
10
p
60
=
6,616,155
8,188,074
= 0.80802,
11
p
60
=
6,396,609
8,188,074
= 0.78121
10|
q
()
60
=
10
p
()
60

11
p
()
60
= (
10
p
60
)
2
(
11
p
60
)
2
from ILT.
= 0.80802
2
0.78121
2
= 0.0426
Key B
Arch MLC, Fall 2007 c _Yufeng Guo 308
www.archactuarial.com www.guo.coursehost.com 309
6. For students entering a college, you are given the following from a multiple decrement
model:
(i) 1000 students enter the college at t = 0.
(ii) Students leave the college for failure (1) or all other reasons (2).
(iii)
(1)
(t) = 0 t 4

(2)
(t) = 0.04 0 t < 4
(iv) 48 students are expected to leave the college during their rst year due to all
causes.
Calculate the expected number of students who will leave because of failure during their
fourth year.
(A) 8 (B) 10 (C) 24 (D) 34 (E) 41
SOLUTION:
d
()
0
= 1000
_
1
0
e
(+0.04)t
( + 0.04)dt = 1000
_
1 e
(+0.04)
_
= 48
e
(+0.04)
= 0.952
+ 0.04 = ln(0.952) = 0.049 = = 0.009
d
(1)
3
= 1000
_
4
3
e
0.049t
(0.009)dt
= 1000
0.009
0.049
_
e
(0.049)(3)
e
(0.049)(4)
_
= 7.6 Key A
7. Don, age 50, is an actuarial science professor. His career is subject to two decrements:
(i) Decrement 1 is mortality. The associated single decrement table follows De Moivres
law with = 100.
(ii) Decrement 2 is leaving academic employment, with

(2)
50
(t) = 0.05, t 0
Calculate the probability that Don remains an actuarial science professor for at least
ve but less than ten years.
(A) 0.22 (B) 0.25 (C) 0.28 (D) 0.31 (E) 0.34
Arch MLC, Fall 2007 c _Yufeng Guo 309
www.archactuarial.com www.guo.coursehost.com 310
SOLUTION:
5
p
()
50
=
5
p

(1)
50 5
p

(2)
50
=
_
100 55
100 50
_
e
(0.05)(5)
= (0.9)(0.7788) = 0.7009
Similarly
10
p
()
50
=
_
100 60
100 50
_
e
(0.05)(10)
= (0.8)(0.6065) = 0.4852
5|5
q
()
50
=
5
p
()
50

10
p
()
50
= 0.7009 0.4852 = 0.2157
Key: A
8. For a double decrement model:
(i) In the single decrement table associated with cause (1),
q

(1)
40
= 0.100 and decrements are uniformly distributed over the year.
(ii) In the single decrement table associated with cause (2),
q

(2)
40
= 0.125 and all decrements occur at time 0.7.
Calculate q
(2)
40
.
(A) 0.114 (B) 0.115 (C) 0.116 (D) 0.117 (E) 0.118
SOLUTION:
Only decrement 1 operates before t = 0.7
0.7
q

(1)
40
= (0.7)q

(1)
40
= (0.7)(0.10) = 0.07 since UDD
Probability of reaching t = 0.7 is 1 0.07 = 0.93
Decrement 2 operates only at t = 0.7, eliminating 0.125 of those who reached 0.7
q
(2)
40
= (0.93)(0.125) = 0.11625
Key: C
Arch MLC, Fall 2007 c _Yufeng Guo 310
www.archactuarial.com www.guo.coursehost.com 311
9. For a double-decrement model:
(i)
t
p
(1)
40
= 1
t
60
, 0 t 60.
(ii)
t
p
(2)
40
= 1
t
40
, 0 t 40.
Calculate
()
40
(20).
(A) 0.025 (B) 0.038 (C) 0.050 (D) 0.063 (E) 0.075
SOLUTION:
Note that above 40, decrement 1 is DeMoivre with omega = 100; decrement 2 is
DeMoivre with omega = 80.
That means
(1)
40
(20) = 1/40 = 0.025;
(2)
40
(20) = 1/20 = 0.05

()
40
(20) = 0.025 + 0.05 = 0.075.
Or from basic denition of ,
t
p
()
40
=
60t
60

40t
40
=
2400100t+t
2
2400
d(
t
p
()
40
)/dt = (100 + 2t)/2400
at t = 20 gives 60/2400 = 0.025
20
p
()
40
= (2/3) (1/2) = 1/3

()
40
(20) = [d(
t
p
()
40
)/dt]/
20
p
()
40
= 0.025/(1/3) = 0.075 Key: E
10. A fully discrete 3-year term insurance of 10,000 on (40) is based on a double-decrement
model, death and withdrawal:
(i) Decrement 1 is death.
(ii)
(1)
40
(t) = 0.02, t 0
(iii) Decrement 2 is withdrawal, which occurs at the end of the year.
(iv) q
(2)
40+k
(t) = 0.04, k = 0, 1, 2
(v) = 0.95.
Calculate the actuarial present value of the death benets for this insurance.
(A) 487 (B) 497 (C) 507 (D) 517 (E) 527
Arch MLC, Fall 2007 c _Yufeng Guo 311
www.archactuarial.com www.guo.coursehost.com 312
SOLUTION:
At any age, p
(1)
x
= e
0.02
= 0.9802
q
(1)
x
= 1 0.9802 = 0.0198, which is also q
(1)
x
, since decrement 2 occurs only at the end
of the year.
Actuarial present value (APV ) at the start of each year for that years death benets
= 10, 000 0.0198 = 188.1
p
()
x
= 0.9802 0.96 = 0.9410
E
x
= p
()
x
= 0.941 = 0.941 0.95 = 0.8940
APV of death benet for 3 years 188.1 +E
40
188.1 +E
40
E
41
188.1 = 506.60
Key: C
11. (50) is an employee of XY Z Corporation. Future employment with XY Z follows a
double decrement model:
(i) Decrement 1 is retirement.
(ii)
(1)
50
(t) =
_
0.00 0 t 5
0.02 5 t
(iii) Decrement 2 is leaving employment with XY Z for all other causes.
(iv)
(2)
50
(t) =
_
0.05 0 t 5
0.03 5 t
(v) If (50) leaves employment with XY Z, he will never rejoin XY Z.
Calculate the probability that (50) will retire from XY Z before age 60.
(A) 0.069 (B) 0.074 (C) 0.079 (D) 0.084 (E) 0.089
SOLUTION:
5
p
()
50
= e
(0.05)(5)
= e
0.25
= 0.7788
5
q
(1)
55
=
_
5
0
5

(1)
55
(t) e
(0.03+0.02)t
dt = (0.02/0.05)e
0.05t
[
5
0
= 0.4(1 e
0.25
)
= 0.0885
Probability of retiring before 60
5
p
()
50
5
q
(1)
55
= 0.7788 0.0885
= 0.0689
Key: A
Arch MLC, Fall 2007 c _Yufeng Guo 312
www.archactuarial.com www.guo.coursehost.com 313
12. For a double decrement table, you are given:
(i) q
(1)
x
= 0.2.
(ii) q
(2)
x
= 0.3.
(iii) Each decrement is uniformly distributed over each year of age in the double decre-
ment table.
Calculate
0.3
q
(1)
x+0.1
.
(A) 0.020
(B) 0.031
(C) 0.042
(D) 0.053
(E) 0.064
SOLUTION:
p
()
x
= p
(1)
x
p
(2)
x
= 0.8(0.7) = 0.56.
q
(1)
x
=
_
ln(p
(1)
x
)
ln(p
()
x
)
_
q
()
x
since UDD in double decrement table
=
_
ln(0.8)
ln(0.56)
_
0.44 = 0.1693
0.3
q
(1)
x+0.1
=
0.3q
(1)
x
10.1q
()
x
= 0.053
To elaborate on the last step:
0.3
q
(1)
x+0.1
=
_
Number dying from cause
1 between x + 0.1 and x + 0.4
_
Number alive at x+0.1
Since UDD in double decrement,
=
l
()
x
(0.3)q
(1)
x
l
()
x
(10.1q
()
x
)
Key: D
Arch MLC, Fall 2007 c _Yufeng Guo 313
www.archactuarial.com www.guo.coursehost.com 314
13. For a special fully discrete 3-year term insurance on (55), whose mortality follows a
double decrement model:
(i) Decrement 1 is accidental death; decrement 2 is all other causes of death.
(ii)
x q
(1)
x
q
(2)
x
55 0.002 0.020
56 0.005 0.040
57 0.008 0.060
(iii) i = 0.06
(iv) The death benet is 2000 for accidental deaths and 1000 for deaths from all other
causes.
(v) The level annual contract premium is 50.
(vi)
1
L is the prospective loss random variable at time 1, based on the contract pre-
mium.
(vii) K(55) is the curtate future lifetime of (55).
Calculate E[
1
L[K(55) 1].
(A) 5
(B) 9
(C) 13
(D) 17
(E) 20
SOLUTION:
Actuarial present value (APV ) of future benets =
= [(0.005)2000 +0.04(1000)]/1.06 +(1 0.005 0.04)(0.008 2000 +0.06 1000)/1.06
2
= 47.17 + 64.60
= 111.77
APV of future premiums = [1 + (1 0.005 0.04)/1.06]50
= (1.9009)(50)
= 95.05
E[
1
L[K(55) 1] = 111.77 95.05 = 16.72.
Key: D
Arch MLC, Fall 2007 c _Yufeng Guo 314
www.archactuarial.com www.guo.coursehost.com 315
Problems from Pre-2000 SOA-CAS Exams
1. Youre given the following double-decrement model for students writing actuarial exams:

x q
(1)
x
q
(2)
x
21 0.008 0.15
22 0.015 0.20
23 0.025 0.25
q
(1)
x
denotes the probability of decrement due to attaining fellowship.
q
(2)
x
denotes the probability of decrement due to all other causes.
Decrements occur at the end of the year.
You are also given:
After attaining fellowship, the only decrement is mortality.
After attaining fellowship, = 0.04.
Calculate the probability that a student writing exams now age 21, will be living and
a Fellow three years later.
(A) 0.031 (B) 0.036 (C) 0.038 (D) 0.043 (E) 0.046
2. For a triple-decrement model, you are given:
Decrement 1 is uniformly distributed over each year of age.
Decrement 2 occurs only at the end of the year.
Decrement 3 occurs only at the beginning of the year.

x l
()
x
q

(1)
x
q

(2)
x
q

(3)
x
60 100,000 0.14 0.1 0.1
61 0.1 0.2
62 45,516
Calculate q
(1)
61
.
(A) 0.070 (B) 0.074 (C) 0.078 (D) 0.082 (E) 0.086
Arch MLC, Fall 2007 c _Yufeng Guo 315
www.archactuarial.com www.guo.coursehost.com 316
3. For a double-decrement model, you are given:

(1)
10
(t) =
1
30 t
, 0 t < 30

()
10
(t) =
50 2t
600 50t +t
2
, 0 t < 20
Calculate the probability that (10) will terminate from cause 2 during the 6th year.
(A) 0.0225 (B) 0.0242 (C) 0.0392 (D) 0.0408 (E) 0.0650
Use the following information for the next 4 questions.
For a double-decrement model, you are given:

(1)
40
(t) =
2
60 +t
, t 0

(2)
40
(t) =
3
60 +t
, t 0
T is the time-until-decrement random variable for (40).
J is the cause-of-decrement random variable for (40).
4. Calculate f
T,J
(20, 2).
(A) 0.0059 (B) 0.0066 (C) 0.0076 (D) 0.0089 (E) 0.0099
5. Calculate f
T
(20).
(A) 0.0127 (B) 0.0136 (C) 0.0148 (D) 0.0159 (E) 0.0165
6. Calculate f
J
(2).
(A) 0.33 (B) 0.40 (C) 0.50 (D) 0.60 (E) 0.67
7. Calculate f
J|T
(1[10).
(A) 0.33 (B) 0.40 (C) 0.50 (D) 0.60 (E) 0.67
Arch MLC, Fall 2007 c _Yufeng Guo 316
www.archactuarial.com www.guo.coursehost.com 317
8. From a double-decrement table, you are given:
l
30
= 1000
q

(1)
30
= 0.1
q

(2)
30
= 0.2

1|
q
(1)
30
= 0.05
l
()
32
= 562
Calculate q
(2)
31
.
(A) 0.125 (B) 0.130 (C) 0.145 (D) 0.150 (E) 0.170
9. You are given the following extract from a triple-decrement table:
x q
(1)
x
q
(2)
x
q
(3)
x
q
()
x
l
()
x
q

(1)
x
q

(2)
x
p

(3)
x
60 0.010 0.050 0.020 10,000
61 0.076
62 0.023 0.033 0.990
63 0.098
Calculate l
()
63
.
(A) 7172 (B) 7175 (C) 7951 (D) 7954 (E) 8031
Arch MLC, Fall 2007 c _Yufeng Guo 317
www.archactuarial.com www.guo.coursehost.com 318
Solutions to Pre-2000 Problems: Chapter 10
1. Key: B
x q
(1)
x
q
(2)
x
q
()
x
p
()
x
n
p
()
x
n
21 0.008 0.15 0.158 0.842 0.842 1
22 0.015 0.20 0.215 0.785 0.661 2
23 0.025 0.25 0.275 0.725 0.479 3
The probability we are looking for is given by
q
(1)
21
p
22
p
23
+p
()
21
q
(1)
22
p
23
+p
()
21
p
()
22
q
(1)
23
= (0.008)e
0.04
e
0.04
+ (0.842)(0.015)e
0.04
+ (0.842)(0.785)(0.025)
= 0.007385 + 0.012135 + 0.016524 = 0.036
2. Key: B
First, we need l
()
61
p
()
60
= p

(1)
60
p

(3)
60
p

(2)
60
= (0.86)(0.9)(0.9) = 0.6966
l
()
61
= 69,660
To get to q
(1)
61
, we need d
(1)
61
. For the 69,660 people alive at age 61, the 3rd decrement
strikes rst and leaves 69,660(0.8) = 55,728 lives.
Next, the 55,728 remaining lives are hit by the 1st decrement, but to gure out how
many decrement due to cause 1, we need q
(1)
61
. We use the fact that
p

(1)
61
p

(2)
61
p

(3)
61
=
45,516
69,660
= 0.6534
p

(1)
61
=
0.6534
(0.9)(0.8)
= 0.9075
q

(1)
61
= 1 0.9075 = 0.0925
d
(1)
61
= 55,728(0.0925) = 5154.84
q
(1)
61
=
5154.84
69,660
= 0.074
Arch MLC, Fall 2007 c _Yufeng Guo 318
www.archactuarial.com www.guo.coursehost.com 319
3. Key: D
5|
q
(2)
10
=
_
6
5
t
p
()
10

(2)
10
(t) dt

()
10
(t) =
(1)
10
(t) +
(2)
10
(t)

(2)
10
(t) =
50 2t
600 50t +t
2

1
30 t
=
30 t
(30 t)(20 t)
=
1
20 t
t
p
()
10
=
t
p

(1)
10

t
p

(2)
10
=
30 t
30

20 t
20

_
6
5
30 t
30

20 t
20

1
20 t
dt
=
1
600
_
30t
t
2
2
_
6
5
=
1
600
_
180 18 150 +
25
2
_
=
24.5
600
= 0.0408
4. Key: D
f
T,J
(t, j) =
t
p
()
40

(j)
40
(t)

()
40
(t) =
(1)
40
(t) +
(1)
40
(t) =
5
60 +t
t
p
()
40
= e

_
t
0
5
60+s
ds
= e
5 ln(60+s)|
t
0
=
_
60
60 +t
_
5
f
T,J
(20, 2) =
_
60
80
_
5
_
3
80
_
= 0.0089
5. Key: C
f
T
(t) =
t
p
()
x

()
x
(t)
f
T
(20) =
_
60
80
_
5
_
5
80
_
= 0.0148
Arch MLC, Fall 2007 c _Yufeng Guo 319
www.archactuarial.com www.guo.coursehost.com 320
6. Key: D
F
J
(j) =

q
(j)
x
=
_

0
t
p
()
40

(j)
40
(t) dt
From work two problems ago, we know this equals
_

0
_
60
60 +t
_
5
_
3
60 +t
_
dt
= (60)
5
(3)
_

0
(60 +t)
6
ds = (60)
5
(3)
_
(60 +t)
5
5
_

0
= (60)
5
(3)
(60)
5
5
= 0.6
7. Key: B
f
J|T
(j[t) =

(j)
40
(t)

()
40
(t)
f
J|T
(1[10) =

(1)
40
(10)

()
40
(10)
=
_
2
70
_
_
5
70
_ = 0.4
8. Key: D
We need to know d
(2)
31
. Then dividing by l
()
31
will give the answer we are looking for.
l
()
31
= p
()
30
l
()
30
= (0.9)(0.8)(1000) = 720
d
()
31
= l
()
31
l
()
32
= 720 562 = 158
d
(1)
31
=
1|
q
(1)
30
l
()
30
= (0.05)(1000) = 50
d
(2)
31
= 158 50 = 108
q
(2)
31
=
108
720
= 0.15
9. Key: C
l
()
61
= l
()
60
p
()
60
= 10,000(1 0.01 0.05 0.02) = 9200
l
()
62
= l
()
61
p
()
61
= 9200(1 0.076) = 8501
l
()
63
= l
()
62
p
()
62
= 8501
_
p

(1)
62
_ _
p

(2)
62
_ _
p

(3)
62
_
= 8501 (0.977)(0.967)(0.99) = 7951
Arch MLC, Fall 2007 c _Yufeng Guo 320
Chapter 9
ACTUARIAL MATHEMATICS:
CHAPTER 11 APPLICATIONS
OF MULTIPLE DECREMENT
THEORY
Option A reference: Actuarial Mathematics Chapter 11
Option B reference: Models for Quantifying Risk Chapter 15.6
11.2 Actuarial Present Values and Their Numerical Estima-
tion
Multiple decrement theory comes into play when the benets to be paid vary based on
which decrement eventually gets the insured. For example, consider double indemnity
provisions in many life insurance policies. The benet paid might be X in the event of
a natural death, but will be 2X if the death is due to an accident. In general, what we
have to do to nd the actuarial present value of this type of insurance is to nd the APV
separately for all possible benets, and then sum them:
A =
m

j=1
__

0
B
(j)
x+t
v
t
t
p
()
x

(j)
x
(t)dt
_
,
where there are m decrements and B
(j)
refers to the benet paid for decrement j.
Note that if there is only one decrement (m = 1) and the benet is 1 (B
(1)
x+t
= 1), then the
formula above reduces to the standard actuarial present value for a whole life policy of 1 with
immediate payment of claims.
It is often easier to think of a policy with a double indemnity provision as two separate
policies - one that pays a unit upon death, regardless of cause, and a second that pays a unit
if the death is accidental. For example there are two ways to write down A for an n-year
321
www.archactuarial.com www.guo.coursehost.com 322
term policy with a double indemnity provision in which B
(1)
= 2 and B
(2)
= 1 (accidental
death is decrement 1 and natural death is decrement 2). The hard way is
A = 2
_
n
0
v
t
t
p
()
x

(1)
x
(t) dt +
_
n
0
v
t
t
p
()
x

(2)
x
(t) dt.
We can think of this as
A = 2A
(1)
1
x:n
+A
(2)
1
x:n
.
The more useful way to write this that is more often called for on the exam is as
A =A
1
x:n
+A
(1)
1
x:n
,
where A
1
x:n
is a standard n-year term policy that pays a unit death benet regardless of the
cause of death.
11.3 Benet Premiums and Reserves
This section (like the last one) doesnt really introduce anything new. We are just taking un-
usual benet patterns and breaking them down into pieces to calculate APV (in the previous
section) and benets and reserves (in this section).
Consider a fully discrete whole life policy issued to a person age x with a double indemnity
provision. It is common for the double indemnity provision to be applicable only prior to
age 65. The benet is 1 for non-accidental death and 2 for accidental death. Again, we will
break this policy into its two parts to calculate premiums. First, we calculate the premium
for the benet of 1 paid regardless of cause of death:
P
()
x
=

k=0
v
k+1
k
p
()
x
q
()
x+k

k=0
v
k
k
p
()
x
.
Second, we calculate the premium due to pay for the double indemnity rider. Note that this
benet only applies to age 65, so premiums are paid only to age 64 (since this is a fully
discrete policy):
65x
P
(2)
x
=
64x

k=0
v
k+1
k
p
()
x
q
(2)
x+k
64x

k=0
v
k
k
p
()
x
,
where (2) signies accidental death.
Arch MLC, Fall 2007 c _Yufeng Guo 322
www.archactuarial.com www.guo.coursehost.com 323
Each of the above premium calculations is of the form:
Benet Premium =
Exp PV of Benets
Exp PV of Corresponding Annuity
This is the pattern you are already familiar with from earlier sections.
The benet reserve is calculated in the usual way -
General Case: Reserve = (APV future benets) - (APV of future premiums)
In this case, it is the reserve for a usual death benet of 1 plus a reserve for a benet of 1 for
accidental death before age 65.
k
V =
k
V
x
+
k
V
(2)
x
=
_

h=0
v
h+1
h
p
()
x+k
q
()
x+k+h
P
()
x

h=0
v
h
h
p
()
x+k
_
+
_
64xk

h=0
v
h+1
h
p
()
x+k
q
(2)
x+k+h

65x
P
(2)
x
64xk

h=0
v
h
h
p
()
x+k
_
.
Before we get to an example, let me say that what you really need to get from this section is
that we can describe some pretty messy insurances by breaking a complex policy down into
its fairly easy parts, and then summing them back to get the whole.
EXAMPLE:
A certain 40-year old wishes to purchase a 3-year term life insurance policy that
pays 100,000 at the end of year of death in the event of a natural death (cause
1) and pays double indemnity in the event of an accidental death (cause 2). The
associated single-life decrement table is shown below. i = 0.06.
x q

(1)
x
q

(2)
x
40 0.02 0.005
41 0.03 0.005
42 0.04 0.005
Each of the decrements is uniformly distributed in the year of death in the asso-
ciated single-life decrement table.
1. Complete the multiple decrement table;
2. Find the actuarial present value of this insurance policy;
3. Find the level benet premium for this insurance;
4. Find the benet reserve at the end of the second year.
SOLUTION:
Arch MLC, Fall 2007 c _Yufeng Guo 323
www.archactuarial.com www.guo.coursehost.com 324
1. For each year we use the fact that
q
(1)
x
= q

(1)
x
_
1
1
2
q

(2)
x
_
,
and the corresponding formula for q
(2)
x
.
x q

(1)
x
q

(2)
x
q
(1)
x
q
(2)
x
q
()
x
40 0.02 0.005 0.01995 0.00495 0.0249
41 0.03 0.005 0.02993 0.00493 0.0349
42 0.04 0.005 0.03990 0.00490 0.0448
2. We can view this policy as a 3-year term policy on both causes plus a 3-year
term policy on cause 2 only. The value of the 3-year term policy that pays
regardless of cause of death is
A
()
1
40:3
= vq
()
40
+v
2
p
()
40
q
()
41
+v
3
p
()
40
p
()
41
q
()
42
= (1.06
1
)(0.0249) + (1.06
2
)(0.975)(0.0349) + (1.06
3
)(0.941)(0.0448)
= 0.08913
Similarly, the APV of the extra accidental death policy (a 3-year term policy
on cause 2) is given by
A
(2)
1
40:3
= vq
(2)
40
+v
2
p
()
40
q
(2)
41
+v
3
p
()
40
p
()
41
q
(2)
42
= (1.06
1
)(0.00495) + (1.06
2
)(0.975)(0.004925) + (1.06
3
)(0.941)(0.0049)
= 0.01282
So the actuarial present value of the entire policy is
100,000(0.08913 + 0.01282) = 10,195.05
3. The benet premium is
= 100, 000
A
()
1
40:3
+A
(2)
1
40:3
a
x:3
a
x:3
= 1 +vp
()
40
+v
2
p
()
40
p
()
41
= 1 + (1.06
1
)0.9751 + (1.06
2
)0.9411 = 2.757
=
10, 195.05
2.757
= 3, 697.21
4. The reserve is (PVFB PVFP), which equals
100,000v
_
q
()
42
+q
(2)
42
_

= 100,000(1.06
1
)(0.0448 + 0.0049) 3, 697.21 = 991.47
Chapter 11 Suggested Problems: 1,2
Arch MLC, Fall 2007 c _Yufeng Guo 324
www.archactuarial.com www.guo.coursehost.com 325
CHAPTER 11 Formula Summary
A =
m

j=1
__

0
B
(j)
x+t
v
t
t
p
()
x

(j)
x
(t)dt
_
P
()
x
=

k=0
v
k+1
k
p
()
x
q
()
x+k

k=0
v
k
k
p
()
x
ARCH Sample Exam Problem
1. Kyle enters a company pension plan at age 25. The only two decrements that might
aect Kyle at work are death and retirement. If Kyle survives to age 65, the retirement
plan at the company will begin making annual payments of $400 times the number of
years Kyle has worked at the company. If Kyle dies before retiring, he will receive a
at, end-of-year death benet of $100,000.
Mortality and interest follow the Illustrative Life Table with i = 6%.
Calculate the actuarial present value of these benets on the day Kyle begins work.
(A) 11,000 (B) 13,000 (C) 15,000 (D) 17,000 (E) 19,000
Solution:
1.
100,000A
1
25:40
+
40
E
25
(400)(40) a
65
= 100,000
_
A
25

1
(1.06)
40
40
p
25
A
65
_
+ 16,000
1
(1.06)
40
40
p
25
a
65
= 100,000
_
0.08165 0.09722
75,340
95,650
(0.43980)
_
Arch MLC, Fall 2007 c _Yufeng Guo 325
www.archactuarial.com www.guo.coursehost.com 326
+16,000(0.09722)
75,340
95,650
(9.8969)
= 4797 + 12,126 = 16,923 Key: D
Arch MLC, Fall 2007 c _Yufeng Guo 326
www.archactuarial.com www.guo.coursehost.com 327
Past SOA/CAS Exam Questions:
1. A special whole life insurance is issued on (x). The basic death benet is 1 during policy
year one and 2 thereafter. An additional benet of 2 is provided if death is accidental.
You are given
Benets are payable at the moment of death.
The force of mortality due to accidental death is

(ad)
(x +t) = 0.005, t 0.

()
(x +t) = 0.040, t 0.
= 0.06
Calculate the Actuarial Present Value for this insurance.
(A) 0.777 (B) 0.812 (C) 0.827 (D) 0.844 (E) 0.862
Solution:
APV =
_

0
v
t
t
p
()
x

()
dt +vp
()
x
_

0
v
t
t
p
()
x+1

()
dt +
_

0
2v
t
t
p
()
x

(ad)
dt
=
__

0
e
0.06t
e
0.04t
(0.04)dt +e
0.06
e
0.04
_

0
e
0.06t
e
0.04t
(0.04)dt
+
_

0
2e
0.06t
e
0.04t
(0.005)dt
_
= (0.0862)
_

0
e
0.1t
dt = 0.862 Key:E
2. For a triple-decrement model, you are given:
Decrement 1 is uniformly distributed over each year of age.
Decrement 2 occurs only at the end of the year.
Decrement 3 occurs only at the beginning of the year.

x l
()
x
q

(1)
x
q

(2)
x
q

(3)
x
60 100,000 0.14 0.1 0.1
61 0.1 0.2
62 45,516
Calculate q
(1)
61
.
(A) 0.070 (B) 0.074 (C) 0.078 (D) 0.082 (E) 0.086
Arch MLC, Fall 2007 c _Yufeng Guo 327
www.archactuarial.com www.guo.coursehost.com 328
Solution:
p
()
60
= p
(1)
60
p
(2)
60
p
(3)
60
= (0.86)(0.9)(0.9) = 0.6966
l
()
61
= 69, 660
At age 61, after decrement 3 occurs, there are 69,660(0.8) = 55,728 lives remaining. So
55,728q

(1)
61
lives leave due to Decrement 1.
_
1 q

(1)
61
_ _
p

(2)
61
_ _
p

(3)
61
_
=
45,516
69,660
= 0.6534

_
1 q

(1)
61
_
(0.9)(0.8) = 0.6534.
q

(1)
61
= 0.0925
d
(1)
61
= 55,728 q

(1)
61
= 5154.84
q
(1)
61
= 0.074
Key: B
3. For a special whole life insurance of 100,000 on (x), you are given:
(i) = 0.06.
(ii) The death benet is payable at the moment of death.
(iii) If death occurs by accident during the rst 30 years, the death benet is doubled.
(iv)
()
x
(t) = 0.008, t 0
(v)
(1)
x
(t) = 0.001, t 0, where
(1)
x
is the force of decrement due to death by
accident.
Calculate the single benet premium for this insurance.
(A) 11,765 (B) 12,195 (C) 12,622 (D) 13,044 (D) 13,235
Solution:
APV of regular death benet =
_

0
(100,000)(e
t
)(0.008)(e
t
)dt
=
_

0
(100,000)(e
0.06t
)(0.008)(e
0.008t
)dt
= 100,000
0.008
0.06 + 0.008
= 11,764.71
Arch MLC, Fall 2007 c _Yufeng Guo 328
www.archactuarial.com www.guo.coursehost.com 329
APV of accidental death benet =
_
30
0
(100,000)(e
t
)(0.001)(e
t
)dt
=
_
30
0
(100,000)(e
0.06t
)(0.001)(e
0.008t
)dt
= 100
1 e
2.04
0.068
= 1,279.37
Total APV = 11,765 + 1279 = 13,044 Key: D
4. A whole life policy provides that upon accidental death as a passenger on an airplane a
benet of 1,000,000 will be paid. If death occurs from other accidental causes, a death
benet of 500,000 will be paid. If death occurs from a cause other than an accident, a
death benet of 250,000 will be paid.
(i) Death benets are payable at the moment of death.
(ii)
(1)
=
1
2,000,000
where (1) indicates accidental death as a passenger on an air-
plane.
(iii)
(2)
=
1
250,000
(iv)
(3)
=
1
10,000
(v) = 0.06
Calculate the single benet premium for this insurance.
(A) 450 (B) 460 (C) 470 (D) 480 (E) 490
SOLUTION:

()
x
=
(1)
x
+
(2)
x
+
(3)
x
= 0.0001045
t
p
()
x
= e
0.0001045t
The present value of future benets equals
1,000,000
_

0
e
t
t
p
()
x

(1)
x
dt+500,000
_

0
e
t
t
p
()
x

(2)
x
dt+200,000
_

0
e
t
t
p
()
x

(3)
x
dt
=
1,000,000
2,000,000
_

0
e
0.0601045t
dt+
500,000
250,000
_

0
e
0.0601045t
dt+
250,000
10,000
_

0
e
0.0601045t
dt
= 27.5(16.6377) = 457.54 Key: B
Arch MLC, Fall 2007 c _Yufeng Guo 329
www.archactuarial.com www.guo.coursehost.com 330
5. For a special whole life insurance:
(i) The benet for accidental death is 50,000 in all years.
(ii) The benet for non-accidental death during the rst 2 years is return of the single
benet premium without interest.
(iii) The benet for non-accidental death after the rst 2 years is 50,000.
(iv) Benets are payable at the moment of death.
(v) Force of mortality for accidental death:
(1)
x
(t) = 0.01, t 0
(vi) Force of mortality for non-accidental death:
(2)
x
(t) = 2.29, t 0
(vii) = 0.10
Calculate the single benet premium for this insurance.
(A) 1,000
(B) 4,000
(C) 7,000
(D) 11,000
(E) 15,000
SOLUTION:

()
x
(t) =
(1)
x
(t) +
(2)
x
(t) = 0.01 + 2.29 = 2.30
P = P
_
2
0

t
p
()
x

(2)
x
(t)dt +50,000
_
2
0

t
p
()
x

(1)
x
(t)dt +50,000
_

2

t
p
()
x

()
x
(t)dt
P = P
_
2
0
e
0.1t
e
2.3t
2.29dt+50,000
_
2
0
e
0.1t
e
2.3t
0.01dt+50,000
_

2
e
0.1t
e
2.3t
2.3dt
P
_
1 2.29
1 e
2(2.4)
2.4
_
= 50000
_
0.01
1 e
2(2.4)
2.4
+ 2.3
e
2(2.4)
2.4
_
P = 11, 194
Key: D
Arch MLC, Fall 2007 c _Yufeng Guo 330
www.archactuarial.com www.guo.coursehost.com 331
6. A special whole life insurance on (x) pays 10 times salary if the cause of death is an
accident and 500, 000 for all other causes of death.
You are given:
(i)
()
x
(t) = 0.01, t 0
(ii)
(accident)
x
(t) = 0.001, t 0
(iii) Benets are payable at the moment of death.
(iv) = 0.05
(v) Salary of (x) at time t is 50, 000e
0.04t
, t 0.
Calculate the actuarial present value of the benets at issue.
(A) 78,000
(B) 83,000
(C) 92,000
(D) 100,000
(E) 108,000
SOLUTION:

(accid)
= 0.001

(total)
= 0.01

(other)
= 0.01 0.001 = 0.009
Actuarial present value
=
_

0
500,000e
0.05t
e
0.01t
(0.009)dt + 10
_

0
500,000e
0.04t
e
0.05t
e
0.01t
(0.001)dt
= 500,000
_
0.009
0.06
+
0.001
0.02
_
= 100,000
Key: D
Arch MLC, Fall 2007 c _Yufeng Guo 331
www.archactuarial.com www.guo.coursehost.com 332
Arch MLC, Fall 2007 c _Yufeng Guo 332
Chapter 10
ACTUARIAL MATHEMATICS:
CHAPTER 15 INSURANCE
MODELS INCLUDING
EXPENSES
Option A reference: Actuarial Mathematics Chapter 15
Option B reference: Models for Quantifying Risk Chapter 15.6-15.7
There are two important topics in this section. The rst is Expenses which are not too
dicult. The main point of the expenses section is to nally deal with the fact that all of
the premiums and reserves that we have talked about so far in this course have only been
sucient to cover benets. So far, theyve not covered any other cash demands an insurance
company faces (like the salary and study materials for actuarial students).
The second topic is Asset Shares, which will require some new notation and explanation.
One important point to asset shares is to have a method for assigning company assets to
dierent policyholders in the event the insurance company demutualizes or faces bankruptcy.
15.2 Expense Augmented Models
Expenses are not too hard to deal with as long as you treat them as just another benet
that the insurance company must provide. Then when calculating the Expense-loaded
Premium G, (G is for Gross Premium as opposed to Net Benet Premium P) you just need
to include the expenses in the equivalence relation. A couple of examples should make this
clear.
EXAMPLE:
You are given:
Mortality follows the Illustrative Life Table.
333
www.archactuarial.com www.guo.coursehost.com 334
i = 6%
A person aged (40) will purchase a fully discrete whole life insurance with
death benet equal to 1000.
Expenses include 10% of gross premium plus 5 per year, payable at the
beginning of each year.
1. Find the gross premium necessary to fund all benets and expenses.
2. Find the premium amount necessary to cover expenses. (This is called the
Expense Premium and is dened to be the gross premium minus the net
benet premium.)
3. Find the benet reserve and expense reserve (the portion of the total reserve
necessary to pay future expenses) at time t = 10.
Arch MLC, Fall 2007 c _Yufeng Guo 334
www.archactuarial.com www.guo.coursehost.com 335
SOLUTION:
1. The equivalence relation to fund both benets and expenses is
G a
40
= (0.1)G a
40
+ 5 a
40
+ 1000A
40
G =
5 a
40
+ 1000A
40
0.9 a
40
From the illustrative life table we obtain
G =
5(14.82) + 161.32
0.9(14.82)
= 17.65
2. The net benet premium is
1000A
40
a
40
=
161.32
14.82
= 10.885
So the expense premium is
17.65 10.885 = 6.77
3. There are lots of ways to calculate the benet reserve, well take a shortcut
with the annuity formula:
1000
_
1
a
50
a
40
_
= 1000
_
1
13.27
14.82
_
= 104.59
The expense reserve is the actuarial present value of Future Expenses minus
Future Expense Premiums
(0.1)(17.65) a
50
+ 5 a
50
6.77 a
50
= 0
For this example, the level expense premium exactly funds that years ex-
penses and the expense reserve is exactly zero in all years.
EXAMPLE:
You are given:
Mortality follows the Illustrative Life Table.
i = 6%
A person aged (40) will purchase a fully discrete whole life insurance with
death benet equal to F.
Expenses include 10% of gross premium, payable at the beginning of each
year. In addition there is an initial expense of 20 at policy issue and a nal
expense of 80 at the time the death benet is paid.
The level gross premium is equal to 30.
1. Find the face amount F of the policy.
2. Find the expense reserve time t = 10.
Arch MLC, Fall 2007 c _Yufeng Guo 335
www.archactuarial.com www.guo.coursehost.com 336
SOLUTION:
1.
G a
40
= (0.1)G a
40
+ 20 + 80A
40
+F A
40
Since G = 30, we have
(30) a
40
= (0.1)(30) a
40
+ 20 + 80A
40
+F A
40
Using values from the illustrative life table and solving for F, we arrive at a
face amount equal to
F = 2276
2. The net benet premium is
2276A
40
a
40
=
367.23
14.82
= 24.78
So the level expense premium is
30.00 24.78 = 5.22
The expense reserve is
(0.1)(30) a
50
+ 80A
50
5.21 a
50
= (3)(13.27) + 80(0.249) 5.22(13.27) = 9.54
The expense reserve is negative! This is due to the fact that the initial charge
was large in present value relative to the nal charge at time of issue. This
phenomenon will reverse in later years. For example, the expense reserve at
time t = 30 is
(0.1)(30) a
80
+ 80A
80
5.22 a
80
= (3)(5.91) + 80(0.666) 5.22(5.91) = 40.16
15.4 More Expenses
This section allows for more variation in types and level of expense than we saw in 15.2. We
will include one example here to complement Example 15.4.1 in the text but Example 15.4.1
gives excellent coverage of this material. Anyone who takes the time to understand it in all
its details should be ready for any questions on this material.
This section also includes a classication scheme for dierent types of expenses. They are
broken into Investment Expenses and Insurance Expenses
Investment Expenses - Costs of analyzing, buying, selling, and servicing the in-
vestments used to back insurance company reserves. Investment expenses are usually
accounted for by slightly reducing the interest rate assumed during the life of the policy.
Arch MLC, Fall 2007 c _Yufeng Guo 336
www.archactuarial.com www.guo.coursehost.com 337
Insurance Expenses
1. Acquisition Expense - Costs associated with acquiring the business. These can
include commissions, underwriting costs, advertising, and setting up new policy
records.
2. Maintenance Expense - regularly occurring expenses such as premium collection,
policy changes, policyholder correspondence.
3. General Expense - Expenses that arent easy to allocate to particular policies. Ex-
amples include research, actuarial and legal services, accounting, legal compliance.
4. Settlement Expenses - Expenses associated with validating and paying a claim:
claim investigation, legal defense, administrative costs of paying benets.
EXAMPLE:
For a fully discrete 10-year endowment insurance on (50), you are given the fol-
lowing:
Percent of premium expenses consist of commissions equal to 50% of the
gross premium in the rst year followed by 5% of gross premium in years 2
through 10.
Expenses include acquisition expense of 20, settlement expenses equal to 10
per 1000 of face amount plus 80, and annual maintenance expenses equal
to 5 plus 2 per 1000 of face amount.
Acquisition expense is due at time of policy issue, settlement expenses are
due when the benet is paid, and annual maintenance expenses are due at
the time annual premium is paid.
If the face amount of the policy is b, nd the gross premium G in terms of b and
the usual actuarial symbols for insurances and annuities.
SOLUTION:
G a
50:10
= 0.45G+ 0.05G a
50:10
. .
Percent of Premium
+ 20
..
Acquisition
+
10 b
1000
A
50:10
+ 80A
50:10
. .
Settlement
+5 a
50:10
+
2 b
1000
a
50:10
. .
Maintenance
+b A
50:10
Note that the right-hand side of the equation has been broken into the four types
of expenses plus the death benet. Rearranging gives us:
G a
50:10
=
_
b +
10b
1000
_
A
50:10
+0.45G+
_
0.05G+
2b
1000
_
a
50:10
+20+80A
50:10
+5 a
50:10
G =
(1.01b + 80)A
50:10
+ (5 + 0.002b) a
50:10
+ 20
0.95 a
50:10
0.45
Arch MLC, Fall 2007 c _Yufeng Guo 337
www.archactuarial.com www.guo.coursehost.com 338
We can separate this premium into a component that varies directly with the face
amount plus a component that is independent of the face amount:
G = b
_
1.01A
50:10
+ 0.002 a
50:10
0.95 a
50:10
0.45
_
+
_
80A
50:10
+ 5 a
50:10
+ 20
0.95 a
50:10
0.45
_

NOTES:
Insurance companies occasionally determine a price per unit of coverage for insurance
policies. So the number they are interested in is the per unit gross premium which
equals G/b. For the example just above, an expression for the per-unit premium would
be
G
b
=
_
1.01A
50:10
+ 0.002 a
50:10
0.95 a
50:10
0.45
_
+
1
b
_
80A
50:10
+ 5 a
50:10
+ 20
0.95 a
50:10
0.45
_
The per unit gross premium is awkward since it has a portion that depends on b. This
can be taken care of by calling the portion of G that is independent of b the policy fee
or the expense policy fee. The policy fee is the portion of each premium that goes
to pay per policy expenses (those expenses that are independent of the face amount of
the policy). In the example above, the policy fee is
80A
50:10
+ 5 a
50:10
+ 20
0.95 a
50:10
0.45
EXAMPLE:
For a fully discrete whole life insurance on (30), you are given:
Mortality follows the illustrative life table.
i = 0.06
Percent of premium expenses are 15% in each year
Per policy expenses include 100 at policy issue, 30 each year the policy is in
force, and 50 payable at the end of year of death.
Calculate the expense policy fee for this policy.
Arch MLC, Fall 2007 c _Yufeng Guo 338
www.archactuarial.com www.guo.coursehost.com 339
SOLUTION:
We need to keep in mind that percent-of-premium charges apply to the policy fee
as well as the rest of the gross premium. Therefore, the policy fee g must satisfy
g a
30
= 0.15g a
30
+ 100 + 30 a
30
+ 50A
30
g(15.86) =
100 + 30(15.86) + 50(0.1025)
0.85
g =
36.62
0.85
= 43.08
15.6.1 Asset Shares
Asset shares are a tool used to project the accumulation of assets backing a block of insurance
policies and to assign those assets to each policy.
First some terminology:
G - the level annual contract premium;

k
AS - the asset share assigned to the policy at time t = k;
c
k
- the fraction of the contract premium paid for expenses at time k (in other words
c G is the expense premium);
e
k
- expenses paid per policy at time t = k;
q
(1)
x+k
- the probability of death before age x +k + 1 for an insured who is now x +k;
q
(2)
x+k
- the probability of decrement by withdrawal before age x + k + 1 for an insured
who is now x +k;

k
CV - the cash amount due to the policyholder as a withdrawal benet in the event
they decrement by withdrawal. The CV stands for cash value.
b
k
- death benet due at time t = k for a death in the k-th policy year.
The denition of the k-th Asset Share
k
AS is the actuarially accumulated value at time k
of all past premiums, minus expenses, death benets, and withdrawal benets. If the initial
asset share is
0
AS (which may or may not be equal to zero), then
1
AS must satisfy
[
0
AS +G(1 c
0
) e
0
] (1 +i) = q
(1)
x
b
1
+q
(2)
x

1
CV +p
()
x

1
AS
This should remind you a lot of the reserve recursion from Chapter 8 of Bowers. The dier-
ences are
We are using contract premiums, expenses, and withdrawal benets,
Arch MLC, Fall 2007 c _Yufeng Guo 339
www.archactuarial.com www.guo.coursehost.com 340
The initial asset share may be non-zero (whereas the initial benet reserve is zero by
denition),
The nal asset share may also be non-zero.
The general recursion formula is
[
k
AS +G(1 c
k
) e
k
] (1 +i) = q
(1)
k
b
k+1
+q
(2)
k

k+1
CV +p
()
x+k

k+1
AS
EXAMPLE:
For fully discrete whole life insurance on (80), you are given the following:
The face amount is 1000, and the annual contract premium is 200.
Expenses include 10% of premium plus 20 in all years.
The cash value during any year is 50% of all premiums paid to that point.
i = 6%.
Mortality and withdrawal probabilities are given by the following table:
t q
(d)
x+t
q
(w)
x+t
0 0.10 0.30
1 0.15 0.25
2 0.20 0.10
3 0.25 0.10
Assuming that the initial asset share for the policy is 15, nd the asset share
amounts at times t = 1, 2, 3.
SOLUTION:
[
0
AS +G(1 c
0
) e
0
] (1 +i) = q
(1)
x
b
1
+q
(2)
x

1
CV +p
()
x

1
AS
[15 + 200(0.9) 20] (1 + 0.06) = (0.1) 1000 + (0.3)(100) + (0.6)
1
AS

1
AS = 92.5
Similarly, we can recursively produce the following:
t
t
AS
0 15
1 92.5
2 112.8
3 84.5

Chapter 15 Suggested Problems: 5,6,10


Arch MLC, Fall 2007 c _Yufeng Guo 340
www.archactuarial.com www.guo.coursehost.com 341
CHAPTER 15 Formula Summary
There are not a lot of formulas for the expense-augmented premium sections of this chapter.
Remember to treat expenses as a benet that has to be paid each year.
Asset Shares
[
k
AS +G(1 c
k
) e
k
] (1 +i) = q
(1)
k
b
k+1
+q
(2)
k

k+1
CV +p
()
x+k

k+1
AS
Past SOA/CAS Exam Questions:
1. For a special fully discrete whole life insurance on (x), you are given:
The net single premium is 450.
The level annual expense loaded premium determined by the equivalence principle
is 85.
Death is the only decrement.
Expenses, which occur at the beginning of the policy year, are as follows:
First Year Renewal Years
Percentage of Premium 80% 10%
Per policy 25 25
Calculate a
x
.
(A) 8.06 (B) 8.89 (C) 9.06 (D) 9.89 (E) 10.06
Arch MLC, Fall 2007 c _Yufeng Guo 341
www.archactuarial.com www.guo.coursehost.com 342
2. For a fully discrete 3-year endowment insurance of 1000 on (x), you are given:
i = 0.10
Expenses, which occur at the beginning of the policy year, are as follows:
First Year Renewal Years
Percentage of Premium 20% 6%
Per policy 8 2
The gross annual premium is equal to 314.
The following double-decrement table:
k p
()
x+k
q
(d)
x+k
q
(w)
x+k
0 0.54 0.08 0.38
1 0.62 0.09 0.29
2 0.50 0.50 0.00
The following table of cash values and asset shares:
k
k+1
CV
k
AS
0 247 0
1 571 173
Calculate
2
AS.
(A) 257 (B) 326 (C) 415 (D) 423 (E) 452
3. For a fully discrete whole life insurance of 1000 on (40), you are given:
Death and withdrawal are the only decrements.
Mortality follows the Illustrative Life Table.
i = 0.06
The probabilities of withdrawal are:
q
(w)
40+k
=
_
0.2, k = 0
0, k > 0
Withdrawals occur only at the end of the year. The following expenses are payable
at the beginning of the year:
Percent of Per 1000
Premium Insurance
All Years 10% 1.50

k
CV
40
=
1000k
3

k
V
40
, k 3

2
AS = 24
Calculate the gross premium, G.
(A) 15.33 (B) 15.79 (C) 16.25 (D) 16.77 (E) 17.02
Arch MLC, Fall 2007 c _Yufeng Guo 342
www.archactuarial.com www.guo.coursehost.com 343
Use the following information for numbers 4 through 7.
For a semi-continuous 20-year endowment insurance of 25,000 on (x), you are given:
The following expenses are payable at the beginning of the year:
Percent of Per 1000
Premium Insurance Per Policy
First Year 25% 2.00 15.00
Renewal 5% 0.50 3.00
Deaths are uniformly distributed over each year of age.
A
x:20
= 0.4058
A
x:
1
20
= 0.3195
a
x:20
= 12.522
i = 0.05
Premiums are determined using the equivalence principle.
4. Calculate the expense-loaded rst-year premium including policy fee assuming that
per-policy expenses are matched separately by rst-year and renewal policy fees.
(A) 884 (B) 899 (C) 904 (D) 909 (E) 924
5. Calculate the expense-loaded renewal premiums including policy fee assuming that per-
policy expenses are matched separately by rst-year and renewal policy fees.
(A) 884 (B) 887 (C) 899 (D) 909 (E) 912
6. Calculate the level annual policy fee to be paid each year.
(A) 3.00 (B) 3.60 (C) 3.82 (D) 4.24 (E) 4.51
7. Calculate the level annual expense-loaded premium.
(A) 884 (B) 888 (C) 893 (D) 909 (E) 913
Arch MLC, Fall 2007 c _Yufeng Guo 343
www.archactuarial.com www.guo.coursehost.com 344
8. For a fully discrete whole life insurance of 100, 000 on (35) you are given:
(i) Percent of premium expenses are 10% per year.
(ii) Per policy expenses are 25 per year.
(iii) Per thousand expenses are 2.50 per year.
(iv) All expenses are paid at the beginning of the year.
(v) 1000P
35
= 8.36
Calculate the level annual expense-loaded premium using the equivalence principle.
(A) 930 (B) 1041 (C) 1142 (D) 1234 (E) 1352
9. For a fully discrete whole life insurance of 1000 on (x):
(i) Death is the only decrement.
(ii) The annual benet premium is 80.
(iii) The annual contract premium is 100.
(iv) Expenses in year 1, payable at the start of the year, are 40% of contract premiums.
(v) i = 0.10
(vi) 1000
1
V
x
= 40
Calculate the asset share at the end of the rst year.
(A) 17 (B) 18 (C) 19 (D) 20 (E) 21
10. For a fully discrete whole life insurance of 1000 on (50), you are given:
(i) The annual per policy expense is 1.
(ii) There is an additional rst year expense of 15.
(iii) The claim settlement expense of 50 is payable when the claim is paid.
(iv) All expenses, except the claim settlement expense, are paid at the beginning of
the year.
(v) Mortality follows De Moivres law with = 100.
(vi) i = 0.05
Calculate the level expense-loaded premium using the equivalence principle.
(A) 27 (B) 28 (C) 29 (D) 30 (E) 31
Arch MLC, Fall 2007 c _Yufeng Guo 344
www.archactuarial.com www.guo.coursehost.com 345
SOLUTIONS to Past SOA-CAS Exam Problems:
1.
85 a
x
= 450 + 0.7(85) + 0.1(85) a
x
+ 25 a
x
a
x
=
450 + 0.7(85)
85 (0.1)(85) 25
= 9.89
a
x
= a
x
1 = 8.89
The Key is B
2.
2
AS =
[
1
AS +G(1 c
1
) e
1
] (1 +i) 1000q
(d)
x+1

2
CV q
(w)
x+1
p
()
x+1
[173 + 314(1 0.06) 2] (1.1) 1000(0.09) 571(0.29)
0.62
= 414.82
The Key is C
3. NOTE: Although it is not a strictly necessary assumption in actuarial science, this
problem requires you to assume that
0
AS = 0. Therefore, on the exam, it is a good
idea to assume the initial asset share is equal to zero unless otherwise stated.
k+1
AS p
()
x+k
= [
k
AS +G(1 c
k
) e
k
] (1 +i) 1000q
(1)
x+k
q
(2)
x+k

k+1
CV
The mortality comes from the single life table (even though there are two decrements),
while withdrawal is given in the problem:
q
(1)
40
= 0.00278, q
(2)
40
= 0.2
And we can use the illustrative life table to calculate:
1
CV
40
=
1000
3

1
V
40
=
1000
3
_
1
a
41
a
40
_
=
1000
3
_
1
14.686
14.817
_
= 2.947

1
AS(0.99722)(0.8) = [0 +G(0.9) 1.50] (1.06) 1000(0.00278) 0.2(2.947)

1
AS(0.79778) = G(0.954) 4.96

1
AS = 1.196G6.22
2
AS(0.99702) = [
1
AS +G(0.9) 1.50] (1.06) 1000(0.00298)
Arch MLC, Fall 2007 c _Yufeng Guo 345
www.archactuarial.com www.guo.coursehost.com 346
(24)(0.99702) = [2.096G7.72](1.06) 2.98
G = 15.79 Key: B
4. Since the policy fee is to be split between rst-year and renewal premiums with the
fees allocated accordingly, we rst want to calculate the level expense-loaded premium
necessary to pay the benet and expenses not covered by the policy fee. We will call
this premium K.
K a
x:20
= 25,000A
x:20
+ 25,000(0.0015 + 0.0005 a
x:20
) + 0.2K + 0.5K a
x:20
Note that 5 of the per premium expense for the rst year was carved out to go with the
renewal amounts to make the annuities we have to deal with more simple. The same
was done with the expenses that depend on face amount.
K =
25[405.80 + 1.5 + 0.5(12.522)]
0.95(12.522) 0.2
K = 883.99
They sneakily made this one of the options, but we need to add the expenses loaded
rst year policy fee which must pay the rst year per-policy expense (15) and must
take premium loading into account.
G
1
= 883.99 +
15
1 0.25
= 903.99 Key: C
5. For renewal years, the same base premium K is required to cover all but the policy fee
(3) plus we need 3/(0.95) to cover the policy fee.
G
2
= 883.99 +
3
1 0.05
= 887.15 Key: B
6. Now we assume instead that a level policy fee g is collected each year:
g a
x:20
= 12 + 3 a
x:20
+ 0.2g + 0.05g a
x:20
g =
12 + 3(12.522)
(0.95)(12.522) 0.2
= 4.24 Key: D
7.
G = K +g = 883.99 + 4.24 = 888.23 Key: B
Arch MLC, Fall 2007 c _Yufeng Guo 346
www.archactuarial.com www.guo.coursehost.com 347
8. Key: D
Let G be the expense-loaded premium.
Actuarial present value (APV) of benets = 100, 000A
35
APV of premiums = G a
35
APV of expenses = [0.1G+ 25 + (2.50)(100)] a
35
Equivalence principle:
G a
35
= 100, 000A
35
+ (0.1G+ 25 + 250) a
35
G = 100, 000
A
35
a
35
+ (0.1G+ 275)
0.9G = 100, 000P
35
+ 275
G =
(100)(8.36)+275
0.9
= 1234
9. Key: A
1000
1
V
x
= (1 +i) q
x
(1000 1000
1
V
x
)
40 = 80(1.1) q
x
(1000 40)
q
x
=
8840
960
= 0.05
1
AS =
(Gexpenses)(i+1)1000qx
px
=
(100(0.4)(100))(1.1)(1000)(0.05)
10.05
=
60(1.1)50
0.95
= 16.8
10. Key: E
Let G be the expense-loaded premium.
Actuarial present value (APV ) of benets = 1000A
50
.
APV of expenses, except claim expense = 15 + 1 a
50
APV of claim expense = 50A
50
(50 is paid when the claim is paid)
APV of premiums = G a
50
Equivalence principle: G a
50
= 1000A
50
+ 15 + 1 a
50
+ 50A
50
G1 =
1050A
50
+15
a
50
For De Moivres with = 100, x = 50 A
50
=
a
50
50
= 0.36512
a
50
=
1A
50
d
= 13.33248
Solving for G, G = 30.88
Arch MLC, Fall 2007 c _Yufeng Guo 347
www.archactuarial.com www.guo.coursehost.com 348
Arch MLC, Fall 2007 c _Yufeng Guo 348
Chapter 11
DANIEL CHAPTER 1 -
MULTI-STATE TRANSITION
MODELS FOR ACTUARIAL
APPLICATIONS
Once you think about it, and get used to the terminology, a Multi-State Transition Model
is very well named. For starters, its a model which has more than one state in it such
as living vs. dead, or healthy vs. partially impaired vs. totally disabled. Furthermore, the
model contains the likelihood of moving back and forth (or making transitions) among
these states. (For the living vs. dead model, you are already familiar with q
x
, the probability
of transition from the living state to the dead state.) The living versus dead model is
not a perfect example of a transition model because the transitions are usually regarded as
being only one way (for insurance purposes). Transition models are really designed to handle
more complex situations when it is possible to move back and forth between various states.
Daniels study note does a good job of introducing the concept (Chapter 1), layering in costs
associated with the movements among states (Chapter 2), and tying it all together through
examples throughout the note.
1.1 Introduction
Examples 1-6 in the study note outline several multi-state models, moving for simple to more
complex. As you read through these, it should be useful to sketch a diagram of each model
discussed. For all the models, the process of moving between the states over time is called
a Markov process. We will introduce the concepts of homogeneous and non-homogeneous
Markov Chains through an example and then make the denitions and terminology more
precise afterward.
EXAMPLE: NEWCO Auto Insurance 1
The NEWCO Auto Insurance company classies its policyholders into two groups:
Hi Risk and Low Risk. Each year a driver is assigned to one of the two categories
349
www.archactuarial.com www.guo.coursehost.com 350
based on his status for the previous year and his driving experience for the previous
year. Thus, for each policyholder there are two states:
State 0: Low Risk
State 1: High Risk
Suppose that each year, for a person in State 0, the probability of remaining in
State 0 is 0.85 and the probability of moving to State 1 is 0.15. These are referred
to as the transition probabilities for State 0 and using the symbols in the study
note, we have:
Q
(0,0)
= 0.85, Q
(0,1)
= 0.15
For drivers currently in State 1, we will assume that
Q
(1,0)
= 0.3, Q
(1,1)
= 0.7
Here is a transition diagram that represents the process:
Low Risk High Risk
0.30
0.15
It is handy to be able to represent all of these transition probabilities at once in
matrix form, the result is the transition probability matrix:
Q =
_
_
_
_
_
0.85 0.15
0.30 0.70
_
_
_
_
_
We will see later that the transition probability matrix is extremely useful for
modeling the transition process over time.
Two Important Notes
The probability that a policyholder will be in State 1 next year depends
only on what state they are in this year. It does not matter what state they
were in last year or any other year. This makes the process a memoryless
process and is what denes a Markov process.
In this example we have assumed that the probability of moving from State
0 to State 1 is the same in every year. If the transition probability matrix
remains the same in every period, the process is called a homogeneous
Markov Chain (or just Markov chain). In reality, the transition proba-
bility matrix is likely to change over time. For instance, a policyholder with
the company for 20 years might have a lower probability of moving from Low
Risk to High Risk than a brand new policyholder. If the transition proba-
bility matrix changes over time, we say the process is a non-homogeneous
Markov chain and we use the symbol
Q
(i,j)
n
to represent the probability that a policyholder in State i in period n will
move to State j for period n + 1. (See next example.)
Arch MLC, Fall 2007 c _Yufeng Guo 350
www.archactuarial.com www.guo.coursehost.com 351
EXAMPLE: NEWCO Auto Insurance 2
For a young policy holder (x), NEWCO models the risk class of the policyholder
using a non-homogeneous Markov Chain with 2 states, Low Risk and High Risk.
The transition probability matrices for the rst 5 years (time periods 0 to 4)are:
Q
0
=
_
_
_
_
_
0.7 0.3
0.2 0.8
_
_
_
_
_
, Q
1
=
_
_
_
_
_
0.8 0.2
0.2 0.8
_
_
_
_
_
, Q
2
=
_
_
_
_
_
0.8 0.2
0.3 0.7
_
_
_
_
_
Q
3
=
_
_
_
_
_
0.8 0.2
0.4 0.6
_
_
_
_
_
, Q
4
=
_
_
_
_
_
0.9 0.1
0.5 0.5
_
_
_
_
_
1. (Easy) If the policyholder is in State 0 during Period 2, what is the proba-
bility they will be in State 1 during Period 3?
2. If the policy holder is in State 0 during Period 3, what is the probability
they will be in State 1 during Period 5?
SOLUTION:
1. This is
Q
(0,1)
2
= 0.2
2. The symbol for what we are looking for here is
2
Q
(0,1)
3
The superscript (0, 1) refers to the transition we are interested in (from State
0 to State 1) as usual; the 3 subscript refers to the time period in which we
assume we start in State 0; and the 2 subscript before the Q refers to how
many time periods are involved in the transition.
There are two ways to approach this problem. First, we could note that
to get from state 0 to state 1 in 2 periods there are two possibilities. The
policyholder must pass through one of the following sequences of states at
periods 3, 4, and 5.
0 0 1
0 1 1
Therefore, using the matrices above, the probability of being in State 1 in
period 5 is
(0.8)(0.1) + (0.2)(0.5) = 0.18
Arch MLC, Fall 2007 c _Yufeng Guo 351
www.archactuarial.com www.guo.coursehost.com 352
Be sure you recognize where I got each of the numbers in the line above. In
the notation we have been using, the quantity we want is
2
Q
(0,1)
3
= Q
(0,0)
3
Q
(0,1)
4
+Q
(0,1)
3
Q
(1,1)
4
This notation reveals the second approach that can be used to solve this
problem. It turns out that you can form the 2-step transition matrix
2
Q
3
by matrix-multiplying Q
3
and Q
4
. That is
2
Q
3
= Q
3
Q
4
=
_
_
_
_
_
0.8 0.2
0.4 0.6
_
_
_
_
_

_
_
_
_
_
0.9 0.1
0.5 0.5
_
_
_
_
_
=
_
_
_
_
_
0.82 0.18
0.66 0.34
_
_
_
_
_
Now we can read o the probability we want, which is
2
Q
(0,1)
3
= 0.18.
Important Note:
The fact that we can obtain a k-step transition matrix by multiplying k 1-year transition
matrices is what makes Markov Chains useful. In the most general notation:
k
Q
n
= Q
n
Q
n+1
Q
n+k1
Reread these two examples if necessary. You should be ready now for the more careful
mathematical introduction in the next section.
1.2 Non-homogeneous Markov Chains
From the text, we have the following denition:
M is a non-homogeneous Markov Chain when M is an innite sequence of random variables
M
0
, M
1
, . . . with the following properties:
M
n
denotes the State number at time n.
Each M
n
is a discrete-type random variable over r values.
The transition probabilities are history independent:
Q
(i,j)
n
= Pr[M
n+1
= j[M
n
= i]
The transition probability Q
(i,j)
n
should be interpreted as the probability of moving to state
j at time n + 1 if you are in state i at time n. Also, history independent means that it
doesnt matter which states have been visited in the past. All that matters is where you are
at time n.
Arch MLC, Fall 2007 c _Yufeng Guo 352
www.archactuarial.com www.guo.coursehost.com 353
EXAMPLE: Mortality as a Non-homogeneous Markov Chain
Human mortality can be considered to be a non-homogeneous Markov chain with
two states, Alive and Dead. Suppose q
65
= 0.05. This number gives us the
probability of moving from the Alive state of living at age 65 to the state of dead
at age 66. In the language of Markov chains, we would say
q
65
= Q
(alive,dead)
0
= 0.05
If we were to dene State 0 to be the Alive state and State 1 to be the Dead state,
we would have the more familiar form:
q
65
= Q
(alive,dead)
0
= Q
(0,1)
0
= 0.05
The transition probability matrix for this example is
Q
0
=
_
_
_
_
_
0.95 0.05
0.00 1.00
_
_
_
_
_
and in general for a basic survival model,
Q
n
=
_
_
_
_
_
p
x+n
q
x+n
0.00 1.00
_
_
_
_
_
Note that State 1 is special. You can enter, but you can never leave.
More Notation:
P
(i)
n
= Q
(i,i)
n
is the probability of staying in state i over the next unit of time. So for
the mortality example above, P
(0)
n
= p
x+n
for all values of n and P
(1)
n
= 1 for all values
of n.
The probability that a subject in state i at time n remains in that state through time
n +k is
k
P
(i)
n
= Pr[M
n+1
= M
n+1
= = M
n+k
= i[M
n
= i] = P
(i)
n
P
(i)
n+1
P
(i)
n+k1
If were interested in transition probabilities across more than a single time period (and
we are), we need
k
Q
(i,j)
n
= Pr[M
n+k
= j[M
n
= i]
We use
k
Q
n
to denote the entire k-step r-by-r transition probability matrix in which
entry (i, j) represents the probability of being in State j at time n + k given that we
are in State i at time n. Note that
k
Q
(i,i)
n
represents the probability of being in State i
after k time periods given that we are in State i at time n. This is very dierent from
k
P
(i)
n
since
k
Q
(i,i)
n
allows for the possibility of leaving the state and re-entering it later,
whereas
k
P
(i)
n
allows only the case where the subject remains in State i throughout the
k periods.
Arch MLC, Fall 2007 c _Yufeng Guo 353
www.archactuarial.com www.guo.coursehost.com 354
You should be able to reason why it must be true in all cases that
k
P
(i)
n

k
Q
(i,i)
n
As we saw in the NEWCO Auto Insurance 2 example above, the following provides a handy
way to evaluate k-step probabilities for Markov Chains:
Theorem 1.18
k
Q
n
= Q
n
Q
n+1
Q
n+k1
Or, in the case of a homogeneous Markov chain,
k
Q
n
= Q
k
Arch MLC, Fall 2007 c _Yufeng Guo 354
www.archactuarial.com www.guo.coursehost.com 355
EXAMPLE:
Let Q be the transition probability matrix for a homogeneous Markov chain.
Q =
_
_
_
_
_
_
_
0.9 0.1 0
0.4 0.5 0.1
0.3 0.2 0.5
_
_
_
_
_
_
_
(A) How many dierent states does the Markov chain represented by this matrix
have?
(B) The rst time that the process enters State 2, what is the probability it will
next be in State 0?
(C) The fth time that the process enters State 2, what is the probability it will
next be in State 0?
(D) If the process is currently in State 1, what is the probability that the process
will be in State 1 two periods from now?
(E) If the process is currently in State 1, what is the probability that the process
will stay in State 1 throughout the next two periods?
SOLUTION:
(A) 3; usually we would call these States 0,1, and 2 with State 0 being the one
in which the process begins.
(B) Q
(2,0)
= 0.3
(C) 0.3 - the answer has to be the same as the one in (B) because this is a
Markov Chain.
(D) We could just multiply Q times itself and nd the(1,1) entry. A faster
method that works for the kinds of questions that end up on the exam is the
following:
Let (p
n
, q
n
, r
n
) be a vector that represents the probabilities of being in states
0, 1, and 2, respectively at time n. So we start out with
(p
0
, q
0
, r
0
) = (0, 1, 0)
since we are certain that we are in State 1 at time t = 0. Now
(p
1
, q
1
, r
1
) = (0, 1, 0)
_
_
_
_
_
_
_
0.9 0.1 0
0.4 0.5 0.1
0.3 0.2 0.5
_
_
_
_
_
_
_
= (0.4, 0.5, 0.1)
The vector on the right shows the probabilities of being in States 0, 1, and
2 at time t = 1. To get the probabilities of being in each state at time t = 2:
(p
2
, q
2
, r
2
) = (0.4, 0.5, 0.1)
_
_
_
_
_
_
_
0.9 0.1 0
0.4 0.5 0.1
0.3 0.2 0.5
_
_
_
_
_
_
_
= (0.59, 0.31, 0.1)
Arch MLC, Fall 2007 c _Yufeng Guo 355
www.archactuarial.com www.guo.coursehost.com 356
Now we can read o 0.31 as the probability of being in State 1 at time t = 2.
This method is especially handy if you need to nd two of the probabilities
for a couple of time periods after start. Also, note that you can check your
work by making sure the 3 probabilities in the state probability vector add
to 1 at each step.
(E) This is just
_
Q
(1,1)
_
2
= (0.5)
2
= 0.25
EXAMPLE:
For a certain Markov chain, the transition matrix given by
P =
_
_
_
_
_
0.9 0.1
0.4 0.6
_
_
_
_
_
If the process is in State 0 at time t = 4, what is the probability that the process
will be in State 1 at time t = 7, three periods later?
SOLUTION:
At t = 4, the probability vector is (1,0):
(1, 0)
_
_
_
_
_
0.9 0.1
0.4 0.6
_
_
_
_
_
= (0.9, 0.1); (0.9, 0.1)
_
_
_
_
_
0.9 0.1
0.4 0.6
_
_
_
_
_
= (0.85, 0.15)
(0.85, 0.15)
_
_
_
_
_
0.9 0.1
0.4 0.6
_
_
_
_
_
= (0.825, 0.175)
From this vector, we can see that the probability of being in State 1 at time t = 7
is 0.175.
Chapter 1 Suggested Problems: 1-8 (Solutions at archactuarial.com)
Also, we strongly recommend that you read all of Daniels examples. Focus especially on the
CCRC example since that has been a favorite with exam makers in the past.
Arch MLC, Fall 2007 c _Yufeng Guo 356
www.archactuarial.com www.guo.coursehost.com 357
CHAPTER 2 CASH FLOWS AND THEIR ACTUARIAL
PRESENT VALUES
Section 2.1 Introduction
Chapter 2 shows how the concepts of Chapter 1 can be t into an actuarial framework
involving present values of future contingent cash ows. There are two basic types of cash
ows we need to be able to deal with:
Cash Flows While in States - This is the most natural type of cash ow and the easiest
to deal with as far as calculating actuarial present values (APVs). Two Examples:
1. Premium payments for Hi Risk and Low Risk drivers. To gure out the APV of a
policyholders future premium payments, you need to gure out how many periods
that policyholder expects to spend in each state, calculate the premium due at
each time, and discount for the appropriate number of time periods.
2. Disability insurance payments payable to an insured during each time period that
they are in the disabled state.
Cash Flows upon Transitions - These are cash ows that are payable specically upon
transition from one state to another. For example, life insurance makes a one-time
payment upon transition from the Living state to the Dead state. Calculating the APV
of Cash Flows upon Transitions is a bit more complicated than for Cash Flows while
in States.
We will deviate from the study note authors approach and start with the easier case of Cash
Flows while in states. Also, the concepts of benet premiums and benet reserves will be
introduced throughout the chapter rather than separately at the end.
Cash Flows while in states
Notation: Let
l
C
(i)
denote the cash ow at time l if the subject is in State i at time l.
For example, if State 1 is the state of being a Low-risk driver, then
5
C
(1)
could be used to
represent the premium due from a policyholder at time t = 5 if they are in the Low-risk
state at that time. Suppose there are two possible states, Low-risk (State 1) and High-risk
(State 2), and we are now at time t = 0 with a policy holder in State 2. Then the premium
we expect to receive at time t = 5 is
_
5
Q
(2,1)
0
_

_
5
C
(1)
_
+
_
5
Q
(2,2)
0
_

_
5
C
(2)
_
That messy formula is just the payment for each state times the probability of being in that
state at time t = 5. To calculate the APV of this single payment at time t = 0, we need to
discount the payment for 5 years, or
APV
0
=
_
5
Q
(2,1)
0
_

_
5
C
(1)
_
v
5
+
_
5
Q
(2,2)
0
_

_
5
C
(2)
_
v
5
That, in a nutshell, is all you need to know for this section. The goal is to nd the APV of
all future payments for dierent states given that we start in a certain State i at time t = n.
Arch MLC, Fall 2007 c _Yufeng Guo 357
www.archactuarial.com www.guo.coursehost.com 358
Before writing down the one big formula for this section, it will be helpful to collect all of
the formal notation into one place.

l
C
(i)
- Cash ow due at time l if the subject is in State i.

k
Q
(s,i)
n
- The probability of being in State i at time t = n+k if in State s at time t = n.
APV
s@n
(C
(i)
) - The Actuarial Present Value at time t = n of all future payments to
be made while in State i, given that the subject is in State s at time t = n.

k
v
n
- The present value of 1 paid k years after time t = n.
Theorem 2.12 APV of cash ows while in states Let
l
C
(i)
denote the cash ow at time
l if the subject is in state i at time l. Suppose that the subject is in state s at time n. Then
the APV of these cash ows, as seen from time n, is
APV
s@n
(C
(i)
) =

k=0
[
k
Q
(s,i)
n
] [
n+k
C
(i)
] [
k
v
n
]
To interpret this formula, it is helpful to recall a very similar formula from Chapter 4 of the
Bowers text.
A
x
=

0
interest discount
..
v
k+1

k
p
x
..
survival from 0 to k

prob of death between times k and k + 1


..
q
x+k
Note that were summing, over all possible times of occurrence, the present value of a cash
ow (benet of 1). In this study note, we are doing the same type of calculation but the cash
ow is were calculating the APV of a C
(i)
instead of 1. In similar terms, our formula is:
APV
s@n
(C
(i)
) =

0
int. discount
..
k
v
n

k
Q
(s,i)
n
. .
prob of moving from s to i in k steps

cash ow amount
..
n+k
C
(i)
EXAMPLE: NEWCO 3 (Work this one before looking at the solution!)
For the NEWCO Auto Insurance 2 example given above in section 1.1, recall
that we had a non-homogeneous Markov Chain and we were provided the rst
5 transition probability matrices, Q
0
, Q
1
, ..., Q
4
. We were told the Policyholder
begins in State 0.
We are now given the additional information that premiums are equal to 500 per
year for those in the Low Risk state and 800 per year for those in the High Risk
state. Premiums are payable at the beginning of each year and the interest rate
for all years is i = 8%.
Find the actuarial present value of future premiums for this policyholder for the
rst 5 years of the policy.
Arch MLC, Fall 2007 c _Yufeng Guo 358
www.archactuarial.com www.guo.coursehost.com 359
SOLUTION:
We could do this exactly according to the formula in Theorem 2.12, but it will be
easier on exam day to apply probability state vectors. In Year 1, we are in State
0, so the premium is exactly 500. In Year 2, the probability state vector is
(1, 0) Q
0
= (1, 0)
_
_
_
_
_
0.7 0.3
0.2 0.8
_
_
_
_
_
= (0.7, 0.3)
So the expected premium to be collected in Year 2 is
(0.7)(500) + (0.3)(800) = 590
In Year 3, the state vector is
(0.7, 0.3) Q
1
= (0.62, 0.38)
So the expected premium for Year 3 is
(0.62)(500) + (0.38)(800) = 614
Similar calculations result in the following table:
Year Premium
1 500
2 590
3 614
4 617
5 606.8
Now the APV of all future premiums is
500 + 590v + 614v
2
+ 617v
3
+ 606.8v
4
= 500 + 590(0.926) + 614(0.857) + 617(0.794) + 606.8(0.735) = 2509
Two comments on this example:
For most people (certainly for me) it is easier to reason through the APV as in the
calculation above rather than to memorize a messy formula like the one in Theorem
2.12.
The APV of future premiums would have been dierent if the policyholder had begun
in State 1. For practice, it might be a good idea for you to repeat the entire calculation
in the example for that case.
Arch MLC, Fall 2007 c _Yufeng Guo 359
www.archactuarial.com www.guo.coursehost.com 360
The next example will be used to introduce the idea of benet premiums. Also, we will
consider for the rst time, interest rates that can vary by year.
EXAMPLE: Long Term Care 1
CARECO-LTC is a long term insurance company that sells 4-year Long-term
care insurance policies to 75-year olds. CARECO-LTC models the status of a
policyholder using a homogeneous Markov chain with 3 states:
State 1: Active
State 2: Disabled
State 3: Dead
The transition probability matrix for the process is
Q =
_
_
_
_
_
_
_
0.7 0.2 0.1
0.3 0.5 0.2
0.0 0.0 1.0
_
_
_
_
_
_
_
Each time period represents one year, benets are payable at the beginning of the
year and are equal to 20,000 per year spent in the disabled status. Interest rates
are assumed to be 4% for the rst year, 6% for the second, and 8% thereafter.
(Try to work out each of the following without looking at the solution. Do this
example twice if necessary. All parts of it involve essential skills for the exam.)
1. Premiums of P will be paid at the beginning of each year spent in the Active
State. If a policyholder always begins in the Active state, nd the APV of
future premiums.
2. Again, assuming policyholders always begin in the Active state, nd the
APV of future benets.
3. Find the benet premium for this insurance.
Arch MLC, Fall 2007 c _Yufeng Guo 360
www.archactuarial.com www.guo.coursehost.com 361
SOLUTION:
1. It will be helpful to have the probability state vectors for all years. Let
n
be the probability state vector for period n, then

0
= (1, 0, 0)

1
= (1, 0, 0)
_
_
_
_
_
_
_
0.7 0.2 0.1
0.3 0.5 0.2
0.0 0.0 1.0
_
_
_
_
_
_
_
= (0.7, 0.2, 0.1)

2
= (0.7, 0.2, 0.1)
_
_
_
_
_
_
_
0.7 0.2 0.1
0.3 0.5 0.2
0.0 0.0 1.0
_
_
_
_
_
_
_
= (0.55, 0.24, 0.21)
And similarly we can calculate

3
= (0.457, 0.23, 0.313)
The result is the following probability vectors:
Year
n
0 (1, 0, 0)
1 (0.70, 0.20, 0.10)
2 (0.55, 0.24, 0.21)
3 (0.46, 0.23, 0.31)
So we have the following table:
Time t 0 1 2 3
No. of active people 1 0.7 0.55 0.46
No. of disabled people 0 0.2 0.24 0.23
No. of the dead people 0 0.1 0.21 0.31
The active people pay the premium. The disabled people get the benets.
The APV of all future premiums is
(1)P+(0.7)
_
1
1.04
_
P+(0.55)
_
1
1.04

1
1.06
_
P+(0.46)
_
1
1.04

1
1.06

1
1.08
_
P
= 2.558P
2. The APV of all benet payments is
_
0+(0.2)
_
1
1.04
_
+(0.24)
_
1
1.04

1
1.06
_
+(0.23)
_
1
1.04

1
1.06

1
1.08
_
_
(20,000)
= 12,064
3. The benet premium is the premium such that the APV of future premiums
equals the APV of future benets, or
2.558P = 12,064
P = 4716
Arch MLC, Fall 2007 c _Yufeng Guo 361
www.archactuarial.com www.guo.coursehost.com 362
Cash Flows upon transitions
So far, all of the cash ows we have evaluated have involved payments made due to being
in a certain state for a particular time period. We also want to be able to value cash ows
that are made upon transition from one state to another. For example, life insurance benets
arent paid for every period you are dead, just for the single period in which you move from
the Alive state to the Dead state.
Notation 2.5 Cash ows at transition Let
l+1
C
(i,j)
denote the cash ow at time l +1
if the subject is in state i at time l and state j at time l + 1.
Actuarial Present Values
The formula for calculating the APV of a cash ow C
(i,j)
if the person moves from state i
to state j at any time in the future is very similar to the one for C
(i)
. The only dierence
is in the term that refers to probability payment, we need to recognize that for the payment
C
(i,j)
to occur at time l +1, two things must happen. We have to be in State i at time l and
we have to be in State j at time l + 1. Assuming we start out in some specic state s, the
probability of this occurring is
l
Q
(s,i)
n
Q
(i,j)
n+l
The resulting formula for the APV of a cash ow to be paid upon transition from State i to
State j is
APV
s@n
(C
(i,j)
) =

0
int. discount
..
k+1
v
n

k
Q
(s,i)
n
Q
(i,j)
n+k
. .
prob of moving from s to i, then i to j

cash ow amount
..
n+k+1
C
(i,j)
This formula is identical to the one for C
(i)
, except for the probability term mentioned above
and the fact that interest is discounted to the time of payment a year later. Rearranging of
the terms gives us
Theorem 2.7:
Let
l+1
C
(i,j)
denote the cash ow at time l +1 if the subject is in state i at time l and state
j at time l +1. Suppose that the subject is now in state s at time n. Then the APV of these
cash ows, as seen from time n, is
APV
s@n
(C
(i,j)
) =

0
[
k
Q
(s,i)
n
Q
(i,j)
n+k
] [
n+k+1
C
(i,j)
] [
k+1
v
n
]
Arch MLC, Fall 2007 c _Yufeng Guo 362
www.archactuarial.com www.guo.coursehost.com 363
EXAMPLE: Long Term Care 2
CARECO-LTC has decided to add a death benet to their product. If the pol-
icyholder dies while active, they will receive 50,000 for the period in which they
enter the Dead state. They will receive 25,000 if they die while disabled.
Find the additional premium necessary to fund the death benet.
SOLUTION:
Recall that the state probability vectors for each time period were given by
Year
n
0 (1, 0, 0)
1 (0.70, 0.20, 0.10)
2 (0.55, 0.24, 0.21)
3 (0.46, 0.23, 0.31)
Death benet payable in Period 2 can only occur as a transition from alive to
dead and occurs with probability 0.1. The APV of 2nd-period death benet is
_
1
1.04
_
(0.1)(50,000) = 4808
The present value of both types of death benets for entering the dead state in
Period 3 is
_
1
1.04
1
1.06
_
_
(0.7)(0.1)(50,000) + (0.2)(0.2)(25,000)
_
= 4082
The present value of 4th period death benets is
_
1
1.04
1
1.06
1
1.08
_
_
(0.55)(0.1)(50,000) + (0.24)(0.2)(25,000)
_
= 3318
The present value of death benet for dying upon transition from the 4th period
to the 5th is
_
1
1.04
1
1.06
1
1.08
1
1.08
_
_
(0.46)(0.1)(50,000) + (0.23)(0.2)(25,000)
_
= 2683
(Note that it makes sense that we need to calculate the APV of 4 dierent possible
times of death since 4 premiums are paid.) So the total actuarial present value
of all death benets is
4808 + 4082 + 3318 + 2683 = 14,891
2.558P = 14,891 P = 5821
Arch MLC, Fall 2007 c _Yufeng Guo 363
www.archactuarial.com www.guo.coursehost.com 364
Reserves could be calculated for cash ows upon transition just as they were for cash ows
while in states.
EXAMPLE: Long Term Care 3
CARECO-LTC is a long term insurance company that sells 3-year Long-term
care insurance policies to 75-year olds. CARECO-LTC models the status of a
policyholder using a homogeneous Markov chain with 3 states:
State 1: Active
State 2: Disabled
State 3: Dead
The transition probability matrix for the process is
Q =
_
_
_
_
_
_
_
0.7 0.2 0.1
0.3 0.5 0.2
0.0 0.0 1.0
_
_
_
_
_
_
_
Each time period represents one year. Premiums are paid by the active in the
beginning of each year. 5,000 disability benet is paid to the disabled at the end
of each year. 10,000 death benet is paid to the dead at the end of the year.
Interest rate is 5%.
1. Find the benet premium.
2. Using the retrospective method, nd the benet reserve
1
V for a policyholder
that is Active at t = 1.
3. Using the prospective method, nd the benet reserve
1
V for a policyholder
that is Active at t = 1.
4. Using the retrospective method, nd the benet reserve
2
V for a policyholder
that is Active at t = 2.
5. Using the prospective method, nd the benet reserve
2
V for a policyholder
that is Active at t = 2.
Arch MLC, Fall 2007 c _Yufeng Guo 364
www.archactuarial.com www.guo.coursehost.com 365
SOLUTION:
1. We previously produced the following table:
Time t 0 1 2 3
No. of the active 1 0.7 0.55 0.457
No. of the disabled 0 0.2 0.24 0.230
No. of the dead 0 0.1 0.21 0.313
Total: active+disabled+dead 1 1 1 1
PV of future premiums is P(1 + 0.7v + 0.55v
2
)
PV of future disability benets is 5, 000(0.2v + 0.24v
2
+ 0.23v
3
)
PV of future death benets is
10, 000[0.1v+(0.210.1)v
2
+(0.3130.21)v
3
] = 10, 000(0.1v+0.11v
2
+0.103v
3
)
Please note that PV of future death benets is not
10, 000(0.1v + 0.21v
2
+ 0.313v
3
)
The number of deaths in Row 4 is the accumulative deaths and the people
who died in the previous year still stay in the population (once dead, always
dead). This can be seen from the fact that the total population is always one
at Row 5 even though death occurs each year. Since one death gets 10,000
only once, we need to calculate the incremental deaths per year. Theres 0.1
death in Year 1; 0.21 death in Year 1 and Year 2; and 0.313 death in Year
1, Year 2, and Year 3. As a result, theres 0.1 death in Year 1; 0.11 death in
Year 2; and 0.103 death in Year 3.
Using Equivalence Principle, we get:
P(1+0.7v+0.55v
2
) = 5, 000(0.2v+0.24v
2
+0.23v
3
)+10, 000(0.1v+0.11v
2
+0.103v
3
)
P = 2712.54
2. Reserve at t = 1 using the retrospective method:
2712.54(1.05) 5, 000(0.2) 10, 000(0.1)
0.7
= 1211.67
3. Reserve at t = 1 using the prospective method:
5, 000(0.24v + 0.23v
2
) + 10, 000(0.11v + 0.103v
2
) 2712.54(0.7 + 0.55v)
0.7
= 1211.67
4. Reserve at t = 2 using the retrospective method:
2712.54(1.05
2
+ 0.7 1.05) 5, 000(0.2 1.05 + 0.24) 10, 000(0.1 1.05 + 0.11)
0.55
= 1062.35
5. Reserve at t = 1 using the prospective method:
5, 000(0.23v) + 10, 000(0.103v) 2712.54(0.55)
0.55
= 1062.35
Arch MLC, Fall 2007 c _Yufeng Guo 365
www.archactuarial.com www.guo.coursehost.com 366
This is a good time to point out that the Daniel note has a lot of examples worth reading.
Note that the material in Chapter 3 is there only to have some matrices for reference by some
examples in Chapter 1 and 2. We suggest that you read each of the examples carefully and
work out the numbers he obtains. Useful problems are indicated below.
Chapter 2 Suggested Problems: Page 15, #1-5, Page 18, #1-6, Page 20, #1-6
(Complete Solutions at archactuarial.com)
Arch MLC, Fall 2007 c _Yufeng Guo 366
www.archactuarial.com www.guo.coursehost.com 367
ARCH Warm-up Questions:
1. The Valley U. football team plays 1 game per week all year and has game results that
follow a Markov process. If the team wins this week then the probability of winning
next week is 0.8. If the team loses this week, then the probability of winning next week
is 0.3.
If Valley U. loses this week what is the probability that they will win two weeks from
now?
(A) 0.24 (B) 0.30 (C) 0.45 (D) 0.55 (E) 0.64
Use the following information for the next two questions:
Janet is a world class sprinter. As time passes, it becomes more and more likely that
shell drop from being a world class sprinter (state 0) to a decent sprinter (state 1). It
is possible, however, with lots of training and hard work, to move from being a decent
sprinter back to world class. For t = 1, . . . , 10, you are given:
Q
t
=
_
_
_
_
_
1 Q
(0,1)
t
(0.1)t
0.5 (0.05)t 1 Q
(1,0)
t
_
_
_
_
_
2. If Janet is a world class sprinter at time t = 4, what is the probability that shell still
be a world class sprinter 4 periods later?
(A) 0.086 (B) 0.114 (C) 0.162 (D) 0.192 (E) 0.213
3. What is the probability that shell remain a world class sprinter throughout the same
time period?
(A) 0.014 (B) 0.036 (C) 0.051 (D) 0.074 (E) 0.086
Arch MLC, Fall 2007 c _Yufeng Guo 367
www.archactuarial.com www.guo.coursehost.com 368
Use the following information for the next two questions:
An actuarial student has the following probabilities of changing titles (and salaries!)
each year. The titles and salaries (paid in a lump sum at the beginning of each year)
are as follows:
Actuarial Student (State 0), $40, 000
Actuarial Analyst (State 1), $60, 000
Associate Actuary (State 2), $80, 000
The probability matrix for career progression is as follows:
Q
t
=
_
_
_
_
_
_
_
0.35 0.60 0.05
0 0.50 0.50
0 0 1
_
_
_
_
_
_
_
4. If Luke is just starting his career in State 0, what is the probability that he will be an
Associate Actuary at the end of 3 years?
(A) 0.42 (B) 0.49 (C) 0.56 (D) 0.63 (E) 0.70
5. Suppose that i = 0.07 and that Luke receives his full annual salary at the beginning of
each year. What is the present value of Lukes salary over the next three years?
(A) 107,000 (B) 117,000 (C) 127,000 (D) 137,000 (E) 147,000
Arch MLC, Fall 2007 c _Yufeng Guo 368
www.archactuarial.com www.guo.coursehost.com 369
Use the following information for the next two questions:
You own a special two-year disability income policy. If you are healthy (state 0), you
pay a full premium at the beginning of the year. If you are partially disabled (state 1),
you pay a half premium. If you are fully disabled (state 2), you pay no premium.
All transitions take place at year end. At the moment you move to the partially disabled
state, you receive a benet of 10,000. If you move to the fully disabled state, you receive
a benet of 40,000. If you are healthy, you receive no benet.
The probability matrix is as follows:
Q
n
=
_
_
_
_
_
_
_
0.7 0.2 0.1
0.5 0.3 0.2
0.1 0.3 0.6
_
_
_
_
_
_
_
6. Ignoring the two year constraint on the policy, what is
3
Q
(1,0)
?
(A) 0.1 (B) 0.3 (C) 0.5 (D) 0.6 (E) 0.7
7. If the insured begins in the healthy state, what is the benet premium for this two-year
policy (i = 0.06)?
(A) 7,800 (B) 8,700 (C) 10,600 (D) 11,500 (E) 12,400
Arch MLC, Fall 2007 c _Yufeng Guo 369
www.archactuarial.com www.guo.coursehost.com 370
Solutions
1. The transition probability matrix for Valley U. is given by
P =

0.8 0.2
0.3 0.7

Where State 0 represents a win this week.


(0, 1)

0.8 0.2
0.3 0.7

= (0.3, 0.7)
(0.3, 0.7)

0.8 0.2
0.3 0.7

= (0.45, 0.55)
So the probability of winning two weeks after a loss is 0.45. Key: C
2. SOLUTION: Were looking for
4
Q
(0,0)
4
signifying starting and ending in State 0, be-
ginning at time t = 4, ending at time t = 8.
The state probability vector for time t = 4 is
_
_
_ 1 0
_
_
_, so the probability of being in
state zero four periods later is:
_
_
_ 1 0
_
_
_
_
_
_
_
_
0.60 0.40
0.30 0.70
_
_
_
_
_

_
_
_
_
_
0.50 0.50
0.25 0.75
_
_
_
_
_

_
_
_
_
_
0.40 0.60
0.20 0.80
_
_
_
_
_

_
_
_
_
_
0.30 0.70
0.15 0.85
_
_
_
_
_
_
_
_ 1 0
_
_
_
_
_
_
_
_
0.60 0.40
0.30 0.70
_
_
_
_
_
=
_
_
_ 0.6 0.4
_
_
_
_
_
_ 0.6 0.4
_
_
_
_
_
_
_
_
0.5 0.5
0.25 0.75
_
_
_
_
_
=
_
_
_ 0.4 0.6
_
_
_
_
_
_ 0.4 0.6
_
_
_
_
_
_
_
_
0.4 0.6
0.2 0.8
_
_
_
_
_
=
_
_
_ 0.28 0.72
_
_
_
_
_
_ 0.28 0.72
_
_
_
_
_
_
_
_
0.3 0.7
0.15 0.85
_
_
_
_
_
=
_
_
_ 0.192 0.808
_
_
_
So the answer is 0.192 D.
3. SOLUTION:
Were looking for
4
P
(0,0)
4
signifying staying in State 0, beginning at t = 4, ending at
time t = 8. We can multiply:
(0.6)(0.5)(0.4)(0.3) = 0.036
Arch MLC, Fall 2007 c _Yufeng Guo 370
www.archactuarial.com www.guo.coursehost.com 371
So the answer is B.
4. SOLUTION:
_
_
_ 1 0 0
_
_
_
_
_
_
_
_
_
_
0.35 0.60 0.05
0 0.50 0.50
0 0 1
_
_
_
_
_
_
_
=
_
_
_ 0.35 0.60 0.05
_
_
_
_
_
_ 0.35 0.60 0.05
_
_
_
_
_
_
_
_
_
_
0.35 0.60 0.05
0 0.50 0.50
0 0 1
_
_
_
_
_
_
_
=
_
_
_ 0.1225 0.5100 0.3675
_
_
_
_
_
_ 0.1225 0.5100 0.3675
_
_
_
_
_
_
_
_
_
_
0.35 0.60 0.05
0 0.50 0.50
0 0 1
_
_
_
_
_
_
_
=
_
_
_ 0.0429 0.3285 0.6286
_
_
_
so the answer is 0.6286 solution D.
5. SOLUTION:
He gets 40 (salary in 1000s) immediately. The APV of the payment at the beginning
of Year 2 is:
(0.35)(40) + (0.60)(60) + (0.05)(80)
1.07
= 50.467
The present value for potential salaries at the beginning of year 3 is:
(0.1225)(40) + (0.51)(60) + (0.3675)(80)
1.07
2
= 56.686
So the combined APV for all 3 years is
40 + 50.467 + 56.686 = 147.153
The answer is E.
6. SOLUTION:
Were looking for the probability of being partially disabled today and being in the
healthy state 3 periods later:
_
_
_ 0 1 0
_
_
_
_
_
_
_
_
_
_
0.7 0.2 0.1
0.5 0.3 0.2
0.1 0.3 0.6
_
_
_
_
_
_
_
=
_
_
_ 0.5 0.3 0.2
_
_
_
_
_
_ 0.5 0.3 0.2
_
_
_
_
_
_
_
_
_
_
0.7 0.2 0.1
0.5 0.3 0.2
0.1 0.3 0.6
_
_
_
_
_
_
_
=
_
_
_ 0.52 0.25 0.23
_
_
_
Arch MLC, Fall 2007 c _Yufeng Guo 371
www.archactuarial.com www.guo.coursehost.com 372
_
_
_ 0.52 0.25 0.23
_
_
_
_
_
_
_
_
_
_
0.7 0.2 0.1
0.5 0.3 0.2
0.1 0.3 0.6
_
_
_
_
_
_
_
=
_
_
_ 0.512 0.248 0.240
_
_
_
The answer is C.
7. SOLUTION:
The state probability vectors for times t=1 and t=2 are as follows:
_
_
_ 1 0 0
_
_
_
_
_
_
_
_
_
_
0.7 0.2 0.1
0.5 0.3 0.2
0.1 0.3 0.6
_
_
_
_
_
_
_
=
_
_
_ 0.7 0.2 0.1
_
_
_
_
_
_ 0.7 0.2 0.1
_
_
_
_
_
_
_
_
_
_
0.7 0.2 0.1
0.5 0.3 0.2
0.1 0.3 0.6
_
_
_
_
_
_
_
=
_
_
_ 0.60 0.23 0.17
_
_
_
The equivalence relation is:
P+
P(0.7)
1.06
+
(0.2)(0.5 P)
1.06
=
_
(10000)(0.2) + (40000)(0.1)
1.06
_
+
_
(0.23)(10000) + (0.17)(40000)
1.06
2
_
(1.754717)P = 13759.34
P = 7841
The answer is A.
Arch MLC, Fall 2007 c _Yufeng Guo 372
www.archactuarial.com www.guo.coursehost.com 373
Past SOA/CAS Exam Questions:
1. For Shoestring Swim Club, with three possible nancial states at the end of each year:
(i) State 0 means cash of 1500. If in state 0, aggregate member charges for the next
year are set equal to operating expenses.
(ii) State 1 means cash of 500. If in state 1, aggregate member charges for the next
year are set equal to operating expenses plus 1000, hoping to return the club to
state 0.
(iii) State 2 means cash less than 0. If in state 2, the club is bankrupt and remains in
state 2.
(iv) The club is subject to four risks each year. These risks are independent. Each of
the four risks occurs at most once per year, but may recur in a subsequent year.
(v) Three of the four risks each have a cost of 1000 and a probability of occurrence
0.25 per year.
(vi) The fourth risk has a cost of 2000 and a probability of occurrence 0.10 per year.
(vii) Aggregate member charges are received at the beginning of the year.
(viii) i = 0
Calculate the probability that the club is in state 2 at the end of three years, given that
it is in state 0 at time 0.
(A) 0.24 (B) 0.27 (C) 0.30 (D) 0.37 (E) 0.56
Solution:
P
02
= P
12
, and therefore it is not necessary to track states 0 and 1 separately.
P
12
= Pr [1 big or (no big and 2 or more smalls)]
= 0.1 + (0.9)
_
3(0.25)
2
(0.75) + (0.25)
3
_
= 0.1 + 0.14 = 0.24
If you enter state 2, you stay there.
Probability of not entering in 3 years = (1 0.24)
3
= 0.44.
Probability of being in 2 after 3 years = 1 0.44 = 0.56. Key: E
Arch MLC, Fall 2007 c _Yufeng Guo 373
www.archactuarial.com www.guo.coursehost.com 374
Use the following information for the next two questions:
The Simple Insurance Company starts at time t = 0 with a surplus of S = 3. At the
beginning of every year, it collects a premium of P = 2. Every year, it pays a random
claim amount:
Claim Amount Probability of Claim Amount
0 0.15
1 0.25
2 0.50
4 0.1
Claim amounts are mutually independent.
If, at the end of the year, Simples surplus is more than 3, it pays a dividend equal
to the amount of surplus in excess of 3. If Simple is unable to pay its claims, or if its
surplus drops to 0, it goes out of business. Simple has no administrative expenses and
its interest income is 0.
2. Determine the probability that Simple will ultimately go out of business.
(A) 0.00 (B) 0.01 (C) 0.44 (D) 0.56 (E) 1.00
3. Calculate the expected dividend at the end of the third year.
(A) 0.115 (B) 0.350 (C) 0.414 (D) 0.458 (E) 0.550
SOLUTION to 2:
Simples surplus at the end of each year follows a Markov process with four states:
State 0: out of business
State 1: ending surplus 1
State 2: ending surplus 2
State 3: ending surplus 3 (after dividend, if any)
State 0 is absorbing (recurrent). All other states are transient states. Thus, eventually
Simple must reach state 0. Key: E
SOLUTION to 3:
(See solution to problem 2 for denition of states)
t = 0:
_
0 0 0 1
_
_

_
1.0 0.0 0.00 0.00
0.1 0.5 0.25 0.15
0.1 0.0 0.50 0.40
0.0 0.1 0.00 0.90
_

_
=
_
0.0 0.1 0.0 0.9
_
at t = 1
Arch MLC, Fall 2007 c _Yufeng Guo 374
www.archactuarial.com www.guo.coursehost.com 375
t = 1:
_
0.0 0.1 0.0 0.9
_
_

_
1.0 0.0 0.00 0.00
0.1 0.5 0.25 0.15
0.1 0.0 0.50 0.40
0.0 0.1 0.00 0.90
_

_
=
_
0.01 0.14 0.025 0.825
_
at t = 2
Expected dividend at the end of the third year =
3

k=0
( probability in state k at t = 2) (expected dividend if in state k)
0.010+0.140+0.025(00.85+10.15)+0.825(00.6+10.25+20.15) = 0.4575
Key D
4. A machine is in one of four states (F, G, H, I) and migrates annually among them
according to a Markov process with transition matrix:
F G H I
F 0.20 0.80 0.00 0.00
G 0.50 0.00 0.50 0.00
H 0.75 0.00 0.00 0.25
I 1.00 0.00 0.00 0.00
At time 0, the machine is in State F. A salvage company will pay 500 at the end of 3
years if the machine is in State F.
Assuming v = 0.90, calculate the actuarial present value at time 0 of this payment.
(A) 150 (B) 155 (C) 160 (D) 165 (E) 170
Solution:
M = Initial state matrix = [ 1 0 0 0 ]
T = One year transition matrix =
_

_
0.20 0.80 0 0
0.50 0 0.50 0
0.75 0 0 0.25
1.00 0 0 0
_

_
M T = [ 0.20 0.80 0 0 ]
(M T) T = [ 0.44 0.16 0.40 0 ]
((M T) T) T = [ 0.468 0.352 0.08 0.10 ]
Arch MLC, Fall 2007 c _Yufeng Guo 375
www.archactuarial.com www.guo.coursehost.com 376
Probability of being in State F after three years = 0.468.
Actuarial present value =
_
0.468v
3
_
(500) = 171
Key: E
Notes:
Only the rst entry of the last matrix need be calculated (verifying that the four
sum to 1 is useful quality control.)
It would be valid to calculate T
3
here, but advancing M one year at a time seems
easier.
5. For a perpetuity-immediate with annual payments of 1:
(i) The sequence of annual discount factors follows a Markov chain with the following
three states:
State number 0 1 2
Annual discount factor, v 0.95 0.94 0.93
(ii) The transition matrix for the annual discount factors is:
_

_
0.0 1.0 0.0
0.9 0.0 0.1
0.0 1.0 0.0
_

_
Y is the present value of the perpetuity payments when the initial state is 1.
Calculate E[Y ].
(A) 15.67 (B) 15.71 (C) 15.75 (D) 16.82 (E) 16.86
SOLUTION: The process begins in State 1 and is guaranteed to return to State 1
two steps later with the same prospective value then as we have now.
Let E
0
be the event that the rst transition is to State 0 and let E
2
be the event that
the rst transition is to State 2.
E[Y ] = E[Y [E
0
] Pr[E
0
] + E[Y [E
2
] Pr[E
2
]
= 0.94
_
1 + 0.95 (1 + E[Y ])
_
0.9 + 0.94
_
1 + 0.93 (1 + E[Y ])
_
0.1
= 1.6497 + 0.8037 E[Y ] + 0.814 + 0.0874 E[Y ]
E[Y ] =
1.6497 + 0.1814
1 0.8037 0.0874
= 16.82
Key: D
Arch MLC, Fall 2007 c _Yufeng Guo 376
www.archactuarial.com www.guo.coursehost.com 377
6. For a Markov model with three states, Healthy (0), Disabled (1), and Dead (2):
(i) The annual transition matrix is given by
0 1 2
0
1
2
_

_
0.70 0.20 0.10
0.10 0.65 0.25
0 0 1
_

_
(ii) There are 100 lives at the start, all Healthy. Their future states are independent.
Calculate the variance of the number of the original 100 lives who die within the rst
two years.
(A) 11 (B) 14 (C) 17 (D) 20 (E) 23
SOLUTION:
Ways to go 0 2 in 2 years
0 0 2; p = (0.7)(0.1) = 0.07
0 1 2; p = (0.2)(0.25) = 0.05
0 2 2; p = (0.1)(1) = 0.1
Total = 0.22
Binomial m = 100 q = 0.22
V ar = (100)(0.22)(0.78) = 17
Key: C
Arch MLC, Fall 2007 c _Yufeng Guo 377
www.archactuarial.com www.guo.coursehost.com 378
7. An insurance company issues a special 3-year insurance to a high risk individual. You
are given the following homogenous Markov chain model:
(i) State 1: active
State 2: disabled
State 3: withdrawn
State 4: dead
Transition probability matrix:
1 2 3 4
1
2
3
4
_

_
0.4 0.2 0.3 0.1
0.2 0.5 0 0.3
0 0 1 0
0 0 0 1
_

_
(ii) Changes in state occur at the end of the year.
(iii) The death benet is 1000, payable at the end of the year of death.
(iv) i = 0.05
(v) The insured is disabled at the end of year 1.
Calculate the actuarial present value of the prospective death benets at the beginning
of year 2.
(A) 440 (B) 528 (C) 634 (D) 712 (E) 803
SOLUTION:
A For death occurring in year 2
APV =
0.31000
1.05
= 285.71
For death occurring in year 3, two cases:
(1) State 2 State 1 State 4 : (0.2 0.1) = 0.02
(2) State 2 State 2 State 4 : (0.5 0.3) = 0.15
Total 0.17
APV =
0.171000
1.05
2
= 154.20
Total APV = 285.71 + 154.20 = 439.91.
Key: A
Arch MLC, Fall 2007 c _Yufeng Guo 378
www.archactuarial.com www.guo.coursehost.com 379
8. For a Markov model for an insured population:
(i) Annual transition probabilities between health states of individuals are as follows:
Healthy Sick Terminated
Healthy 0.7 0.1 0.2
Sick 0.3 0.6 0.1
Terminated 0.0 0.0 1.0
(ii) The mean annual health care cost each year for each health state is:
Mean
Healthy 500
Sick 3000
Terminated 0
(iii) Transitions occur at the end of the year.
(iv) i = 0.
Calculate the expected future health care costs (including the current year) for an
insured individual whose current state is healthy.
Recall:
_
a
11
a
12
a
21
a
22
_
1
=
_
a
22
/d a
12
/d
a
21
/d a
11
/d
_
where d = a
11
a
22
a
12
a
21
.
(A) 5100
(B) 5600
(C) 6100
(D) 6600
(E) 7100
Arch MLC, Fall 2007 c _Yufeng Guo 379
www.archactuarial.com www.guo.coursehost.com 380
SOLUTION:
Non absorbing matrix T =
_
0.7 0.1
0.3 0.6
_
, the sub-matrix excluding Terminated,
which is an absorbing state.
I T =
_
0.3 0.1
0.3 0.4
_
(I T)
1
=
_
0.4
0.09
0.1
0.09
0.3
0.09
0.3
0.09
_
=
_
4.4

4 1.1

1
3.3

3 3.3

3
_
Future costs for a healthy = 4.4

4 500 + 1.1

1 3000 = 5555. Key: B


9. For a Markov model for an insured population:
(i) Annual transition probabilities between health states of individuals are as follows:
Healthy Sick Terminated
Healthy 0.7 0.1 0.2
Sick 0.3 0.6 0.1
Terminated 0.0 0.0 1.0
(ii) The mean annual healthcare cost each year for each health state is:
Mean
Healthy 500
Sick 3000
Terminated 0
(iii) Transitions occur at the end of the year.
(iv) i = 0.
A contract premium of 800 is paid each year by an insured not in the terminated state.
Calculate the expected value of contract premiums less healthcare costs over the rst
3 years for a new healthy insured.
(A) 390
(B) 200
(C) 20
(D) 160
(E) 340
Arch MLC, Fall 2007 c _Yufeng Guo 380
www.archactuarial.com www.guo.coursehost.com 381
SOLUTION:
T =
_
_
_
0.7 0.1 0.2
0.3 0.6 0.1
0 0 1
_
_
_ T
2
=
_
_
_
0.52 0.13 0.35
0.39 0.39 0.22
0 0 1
_
_
_
Actuarial present value (A.P.V.) prem = 800(1 + (0.7 + 0.1) + (0.52 + 0.13)) = 1, 960.
A.P.V. claim = 500(1 + 0.7 + 0.52) + 3000(0 + 0.1 + 0.13) = 1800.
Dierence = 160
Key: D
Arch MLC, Fall 2007 c _Yufeng Guo 381
www.archactuarial.com www.guo.coursehost.com 382
Arch MLC, Fall 2007 c _Yufeng Guo 382
Chapter 12
PROBABILITY MODELS:
CHAPTER 5 THE
EXPONENTIAL DISTRIBUTION
AND THE POISSON PROCESS
5.3 The Poisson Process
In the life contingencies book, the two most important mortality rules to keep track of
are the constant force of mortality and DeMoivres Law. In the same way, the Poisson
process is denitely the most important stochastic process to know and understand well in
the probability book. (Markov is probably second!)
5.3.1 Counting Processes
This section formalizes a few ideas that are fairly simple.
A stochastic process is also a counting process if N(t) represents the total number of
occurrences of some event up to time t. For example, if N(t) is the number of people who
have entered a store at or before time t, then N(t), t 0, is a counting process.
Some quick facts about a counting process N(t):
1. N(t) 0.
2. N(t) 0, 1, 2, . . ..
3. If s < t, then N(s) N(t).
4. For s < t, N(t) N(s) equals the number of events that have occurred in the time
interval (s, t].
A counting process has independent increments if the number of events occurring in
383
www.archactuarial.com www.guo.coursehost.com 384
dierent time periods are unrelated. For example, if you believe the number of car accidents
in July does not depend on the number in May, you would say the increments are independent.
A counting process has stationary increments if the number of events in an interval depend
only on the length of the interval. This is a stronger statement than that of independence.
For the car accident example, a stationary increments assumption would imply that you
expect July and May to have the same number of accidents since they have the same number
of days.
5.3.2 Denition of the Poisson Process
The most important counting process for this exam is the Poisson process. A Poisson Process
is one for which
PN(s +t) N(s) = k = e
t
(t)
k
k!
k = 0, 1, . . . .
This is true for any time s and any length of time t. The constant is called the rate or
parameter of the process.
Notice that in the right hand side of the formula, s doesnt show up at all. This tells us
that no matter when we start a period of time of length t, the distribution for the number of
occurrences is the same. So a Poisson Process has stationary increments.
EXAMPLE: Car Counting
You have been assigned to watch cars pass at a rural intersection and to model
the number of cars passing as a Poisson process, N(t), with parameter 3 where t
is measured in minutes.
1. What is the probability that 0 cars will pass by in the next 2 minutes?
2. Same question for 1, 2, 3, 4 cars respectively.
SOLUTION:
1. PN(2) = 0 = e
t
(t)
0
0!
= e
32
(3 2)
0
0!
= e
6
.
2.
PN(2) = 1 = e
6
(6)
1
1!
= 6e
6
PN(2) = 2 = e
6
(6)
2
2!
= 18e
6
PN(2) = 3 = e
6
(6)
3
3!
= 36e
6
PN(2) = 4 = e
6
(6)
4
4!
= 54e
6
.
Arch MLC, Fall 2007 c _Yufeng Guo 384
www.archactuarial.com www.guo.coursehost.com 385
We could use a bunch of calculus to prove the following important fact that is true for any
Poisson Process:
E[N(t)] = t and Var[N(t)] = t.
So, in the example above, the expected number of cars to pass by each minute is 3 and the
standard deviation in the number that pass each minute is

3.
5.3.3 Interarrival and Waiting Time Distributions
Continuing with the cars at the intersection in the last example, there is another important
random variable closely related to the Poisson process for the number of cars that have passed.
It is called the interarrival time between occurrences the waiting time between events.
So for the intersection, the interarrival time random variable T
n
is concerned with the time
we have to wait after one car passes for the next one to arrive. It turns out that if the random
variable for number of cars passing is Poisson with rate , then the random variable for the
interarrival time is a continuous random variable and is exponentially distributed with
mean
1

.
In the car example, the waiting time is exponentially distributed with mean
1
3
(minutes).
The expected waiting time between cars is 20 seconds, which makes sense if we expect to see
3 per minute.
More formally:
Let T
1
be the rst time an event in a Poisson process occurs. For n > 1, let T
n
be the time
since occurrence T
n1
. Given a Poisson process, T
n
, n = 1, 2, . . . are independent random
variables with an exponential distribution with mean
1

.
Let S
n
be the time of the n-th occurrence of the event that is being modeled as a Poisson
process. It is clear that
S
n
=
n

i=1
t
i
for n 1.
It turns out that S
n
has a gamma distribution with parameters n and . This is less important
than the fact that the interarrival time is exponentially distributed but you still should know
it for the exam. The probability density function for S
n
is
f
Sn
(t) = e
t
(t)
n1
(n 1)!
.
F
Sn
(t) =

j=n
e
t
(t)
j
j!
.
E[S
n
] =
n

, Var[S
n
] =
n

2
.
Note: For the exam, you will be given the probability density functions for Poisson, Expo-
nential, and Gamma distributions. So, it is up to you whether to memorize the formulas or
not but it certainly cannot hurt to know these by memory (in addition to the mean and
Arch MLC, Fall 2007 c _Yufeng Guo 385
www.archactuarial.com www.guo.coursehost.com 386
variance for each) so they will be automatic when you need them. I would probably memorize
the ones for Poisson and Exponential, and use the formula sheet for Gamma. In any case,
it is essential that you become very familiar with the relations between the Poisson process,
the interarrival time, and the waiting time distributions and that you are able to work your
way between them comfortably. Note that if you memorize the Gamma distribution stu,
then you already have the exponential distribution stu by letting n = 1 in the formulas for
the Gamma Distribution.
EXAMPLE:
The following questions refer to the car counting question above.
1. If a car has just passed, what is the probability of not seeing another car for
60 seconds?
2. If no car has passed for the last 20 seconds, what is the probability of not
seeing another car for 60 seconds?
3. What is the expected waiting time for the arrival of the next car?
4. What is the variance of the waiting time until the arrival of the next car?
SOLUTION:
1. For an exponential distribution with mean equal to
1

, F(t) = 1 e
t
. This
implies that the probability of seeing no cars for 1 minute is
1 F(1) = e
(1)
= e
(3)(1)
= 0.0498.
2. This is a memoryless process, so the answer must be the same as in (1)
0.0498.
3. This is the mean =
1

=
1
3
, or 20 seconds.
4. The variance for an exponential distribution with mean
1

is
_
1

_
2
=
1
9
, or 6.67 seconds.
EXAMPLE:
The following questions also refer to the car counting question.
1. What is the expected waiting time until the 10th car passes?
2. What is the variance in the time until the 10th car passes?
3. What is the probability that S
4
2?
Arch MLC, Fall 2007 c _Yufeng Guo 386
www.archactuarial.com www.guo.coursehost.com 387
SOLUTION:
1. E[S
10
] =
10

=
10
3
minutes.
2.
Var[S
10
] =
10

2
=
10
9
.
3. You can use the Gamma distribution, or you can note that this is the same
as the probability that, for the Poisson Process, N(2) 4. (That is, the 4th
car pass in two minutes or less if and only if the number of cars passing in
the next two minutes is 4 or more.
Pr[N(2) 4] = 1 Pr[N(2) < 4]
= 1 (Pr[N(2) = 0] + Pr[N(2) = 1] + Pr[N(2) = 2] + Pr[N(2) = 3])
These are exactly the numbers we calculated in the original car counting
example. So our answer is
1 e
6
6e
6
18e
6
36e
6
= 1 61e
6
= 0.849
5.3.4 Further Properties of Poisson Processes
Suppose that customers arrive at a certain convenience store at a Poisson rate of 20 per hour.
So N(t), the number of people arriving at the store before time t, is a Poisson process with
parameter = 20. Furthermore, suppose that each person that enters the station is male
with probability p and is female with probability 1p. Finally, let N
1
(t) and N
2
(t) represent
the number of men and women, respectively, that enter the store before time t. Then N
1
(t)
and N
2
(t) are both Poisson processes with parameters p and (1 p), respectively.
In addition, although it is somewhat surprising, the two random variables N
1
(t) and N
2
(t) are
independent as well. Even if 50 men enter the store in the next hour, the expected number
of women that will enter the store is still (1 p).
EXAMPLE:
An emergency veterinary clinic cares for only cats and dogs. Animals arrive at a poisson rate
of 5 per hour and each arrival is a dog with probability 0.6.
1. What is the expected time until the eighth dog arrives?
2. What is the probability that no cats will be treated in the next hour?
3. If the vet wants to have treatment capacity to treat 1 standard deviation above the
expected number of cats to arrive each hour and 2 standard deviations more than the
expected number of dogs to arrive each hour, the clinic must be equipped to handle the
arrival of how many cats and dogs in a single hour?
Arch MLC, Fall 2007 c _Yufeng Guo 387
www.archactuarial.com www.guo.coursehost.com 388
SOLUTION:
1. Let N
d
(t) be the random variable for the number of dogs that arrive each hour. Then
N
d
(t) is Poisson with rate = (0.6)(5) = 3. This means that the expected waiting
time until the 8th canine arrival has a gamma distribution with mean
8
3
= 2.67. So the
expected waiting time until the eighth arrival is 2.67 hours.
2. Cat arrivals follow a Poisson distribution with parameter = (0.4)(5) = 2, so the
probability of no arrivals in the next hour is
P [N(1) = 0] = e
t
(t)
n
n!
= e
21
(2 1)
0
0!
= e
2
.
3. The variances in the number of cats and dogs that arrive each hour are 2 and 3,
respectively, so the clinic must be ready to handle the arrival of (2 +

2) cats and
(3 + 2

3) dogs in a single one-hour period.


EXAMPLE: Car Care
A very specialized car care center accepts only Fords and Chevys. It is assumed that the
number of Fords that arrive at the center each day is given by a Poisson distribution with
parameter
F
= 6. The number of Chevys that arrive at the center each day is given by a
Poisson distribution with parameter
C
= 8.
(A) What is the expected waiting time until the 4th car arrives?
(B) What is the variance in the number of cars that arrives in a 5-day week?
SOLUTION:
(A) Since N
F
and N
C
are both Poisson distributions, N = N
F
+N
C
is also Poisson and has
parameter = 14. Therefore the waiting time until the 4th car arrives has a Gamma
distribution with mean
E[S
4
] =
4
14
= 0.286.
(B) The number of cars arriving in a 5 day period has a Poisson distribution with =
5 14 = 70, so the variance in the number arriving is 70.
The next application is to determine the probability that n events occur in one Poisson process
before m events occur in a dierent, independent Poisson process. For example, if we were
counting the number of cars passing at two dierent intersections, we might be interested in
the probability that 20 cars pass one intersection before 10 cars pass at the other.
Let N
1
(t) and N
2
(t) be independent Poisson processes with rates
1
and
2
, respectively. Let
S
1
n
be the time of the n-th event of N
1
(t) and S
2
m
be the time of the m-th event of N
2
(t).
So some facts that we already know (and want to remember for the exam!) are
Arch MLC, Fall 2007 c _Yufeng Guo 388
www.archactuarial.com www.guo.coursehost.com 389
1. S
1
1
and S
2
1
are exponentially distributed with means
1

1
and
1

2
, respectively.
2. S
1
n
and S
2
m
have gamma distributions with means
n

1
and
m

2
, respectively.
Back to the point of this application we are interested in nding
PS
1
n
< S
2
m
,
which is the probability that event n of the N
1
process occurs before event m the N
2
process.
To make a long story a little shorter,
PS
1
n
< S
2
m
=
n+m1

k=n
_
n +m1
k
_
_

1

1
+
2
_
k
_

2

1
+
2
_
n+m1k
.
This formula is ugly and as well see in the example below, this type of problem can be
approached more simply. At a minimum, though, be comfortable with the two special cases
below:
PS
1
n
< S
2
1
=
_

1

1
+
2
_
n
and PS
1
1
< S
2
1
=

1

1
+
2
.
EXAMPLE: Car Care
For the Car Care center described in the previous example:
(A) What is the probability that 3 Chevys arrive before the rst Ford arrives?
(B) What is the probability that 3 Chevys arrive before 2 Fords arrive?
SOLUTION:
(A)
PS
C
3
< S
F
1
=
_

C

F
+
C
_
3
=
_
8
14
_
3
= 0.187.
Note that although a total of four cars are involved, we only need to see the
rst three cars to know if three Chevys will arrive before the rst Ford
(B) As in the last problem, we will know if 3 Chevys will arrive before 2 Fords
by looking at the next 4 cars that arrive. We need to nd the probability
that 3 or more Chevys are among the next 4 cars. The probability that all
4 cars are Chevys is
_
8
14
_
4
_
6
14
_
0
while the probability that there will be exactly 3 Chevys among the next
four cars is
_
4
1
_
_
8
14
_
3
_
6
14
_
1
Arch MLC, Fall 2007 c _Yufeng Guo 389
www.archactuarial.com www.guo.coursehost.com 390
Note that we had to include the term
_
4
1
_
to account for the fact that the
Ford could have been any of the next four cars. All this information give us
the following:
PS
C
3
< S
F
2
= 4
_
8
14
_
3
_
6
14
_
+ 1
_
8
14
_
4
_
6
14
_
0
= 0.320 + 0.107 = 0.427.
5.3.5 Conditional Distribution of the Arrival Times
Although this sections contains a lot of interesting math and examples, it is not easy to make
much of it relevant to the exam. The following is a short explanation of some of the points
and one testable example.
Concept: In a Poisson process, if we know that exactly one event occurred between time 0
and time t, then the distribution of the time at which the event occurred is uniform over the
interval [0, t].
For the cars passing the intersection example: if we know that 1 car passed by in the last 30
seconds, it is equally likely to have passed by at any moment during that 30 seconds. The
probability that it passed by in the rst x seconds of that interval is
x
30
.
Concept: In the cats and dogs example, it was assumed that the probability an arriving
animal was a dog with probability 0.6 no matter what time the animal arrived. It is possible
to let this probability change with time - for example, it could be 0.6 for the rst two hours
(0 < t < 2), 0.5 for the next two hours, etc. If the probability P
1
that the arriving animal
is a dog is kept constant, and if N
1
is the random variable for the number of dogs arriving,
then
E[N
i
(t)] = P
i
t.
If the probability varies with time P
i
= P
i
(t), then
E[N
i
(t)] =
_
t
0
P
i
(s)ds.
EXAMPLE:
An emergency veterinary clinic cares for only cats and dogs. Animals arrive at a Poisson rate
of 5 per hour and the clinic is open for 6 hours each day. The likelihood that the arriving
animal is a dog, given that it arrives at time t during the day, is given in the following table.
t Pr[dog]
0 t < 2 0.6
2 t < 4 0.5
4 t 6 0.2
What is the expected number of dogs per day to arrive at the clinic?
Arch MLC, Fall 2007 c _Yufeng Guo 390
www.archactuarial.com www.guo.coursehost.com 391
SOLUTION:
(If P
i
were always 0.6, this would just be 5 6 (0.6).)
E[N
i
(t)] =
_
t
0
P
i
(s)ds
= 5
__
2
0
(0.6)ds +
_
4
2
(0.5)ds +
_
6
4
(0.2)ds
_
= 5 [1.2 + 1 + 0.4] = 13.
EXAMPLE:
A hotel shuttle currently runs to and from the airport once per hour. Patrons arrive at the
airport shuttle stop according to a Poisson process with parameter = 10.
(A) What is the expected amount of time that each arriving (to the stop) patron expects
to wait for the shuttle at the shuttle stop?
(B) The Hotel would like to pick a second time t between 0 and 1 (the current shuttle arrival
times) to send a second shuttle. What value of t will minimize the total expected waiting
time of all arriving patrons during the hour?
SOLUTION:
These are somewhat intuitive, and if at all possible, I would rely on your intuition, rather
than memorizing lots of formulas around this point in the book.
(A) Since this is a Poisson process the arrival time of each customer during the hour is
uniformly distributed. Therefore, the expected arrival time is at t = 1/2 and the
expected waiting time is 0.5 hours.
(B) Once the hotel has picked time t, the expected waiting time (till rst shuttle) of all of
those expected to arrive before t is
(Number expected to arrive) (Expected waiting time of each arrival)
= (10t)
_
t
2
_
Similarly, the expected waiting time (till second shuttle) of those arriving after t is
10(1 t)
_
1t
2
_
. The hotel wants to minimize the sum of these waiting times which is
5t
2
+ 5(1 t)
2
.
Dierentiating this function and setting the derivative equal to 0 gives us
10t 10(1 t) = 0
t =
1
2
.
(Pretty intuitive, this question would have been a lot more interesting if the answer
had come out to be t = 0.9 or something.)
Arch MLC, Fall 2007 c _Yufeng Guo 391
www.archactuarial.com www.guo.coursehost.com 392
5.4 Generalizations of the Poisson Process
5.4.1 Nonhomogeneous Poisson Process
A Nonhomogeneous Poisson Process has the quality that can vary over time. We express
that concept by calling N(t) a non-homogeneous Poisson process with intensity function
(t).
Whereas the mean under a homogeneous Poisson process was just m = , we now have a
mean value function m(t) of the process, and
m(t) =
_
t
0
(y)dy.
Finally, the book uses more than a page of equations to prove that
PrN(t) = n = e
m(t)
[m(t)]
n
n!
, for n 0.
Notice that this is the same old formula for homogenous Poisson process, with the dierence
that m(t) has replaced t in the formula. This actually gives us a pretty good sense of what
m(t) represents. m(t) is the expected number of arrivals between times 0 and t. This used
to be t, back when was a constant.
Even though this material is complex, it has made it onto exams in the new system. Example
5.20 in the book is an excellent example - it shows the skills of developing a formula for (t)
and then using that formula to answer several questions. Work through that example and
try to do it without looking at the solution (just as you do all of our examples!). I dont
think we can improve on that example, but here is a similar one for practice see also the
recommended questions at the end of this chapter.
EXAMPLE:
Each our of the day, customers enter a 24-hour grocery store according to a non-homogeneous
poisson process with intensity function
(t) = t(24 t)
where t is measured in hours and t = 0 corresponds to midnight.
(A) What is the expected number of customers per day?
(B) What is the probability that there will be fewer than 3 customers between t = 0 and
t = 1?
SOLUTION:
Arch MLC, Fall 2007 c _Yufeng Guo 392
www.archactuarial.com www.guo.coursehost.com 393
(A) We need the mean value function that plays the role usually played by in a homoge-
neous Poisson setting. There are 24 hours in a day, so
m(24) =
_
24
0
(t)dt =
_
24
0
t(24 t) dt =
_
12t
2

t
3
3
_
24
0
= 2304
Since m(24) = 2304, the expected number of customers entering the store in one day
is 2304.
(B) We need
m(1) =
_
1
0
(t)dt =
_
1
0
t(24 t) dt =
_
12t
2

t
3
3
_
1
0
= 11.67
We are looking for
Pr[N = 0] + Pr[N = 1] + Pr[N = 2]
= e
11.67
_
(11.67)
0
0!
+
(11.67)
1
1!
+
(11.67)
2
2!
_
= 91.39(e
11.67
)
5.4.2 Compound Poisson Process
The idea behind a Compound Poisson process has to do with casualty insurance. Suppose
you are an automobile insurer and you model the number of claims with a Poisson random
variable N(t) - a very reasonable thing to do over short periods of time. There is another
random variable involved that will aect total claims over a period of time. It is the claim
size of each individual claim, which can be represented by Y
i
(t), where the i refers to the i-th
accident claim. It is assumed that the Y
i
s are independent of each other and have identical
distributions.
In this situation, we can see that our total claims X(t) over a period of time are a combination
of a Poisson random variable and some other random variable used to model claim size. To
be more precise:
X(t) =
N(t)

i=1
Y
i
, t 0.
The random variable X(t) is called a Compound Poisson random variable, and the
stochastic process X(t), t 0 is a Compound Poisson process.
For a compound Poisson process where N(t) has rate , there are two very important rela-
tions:
E[X(t)] = tE[Y
1
]
Var[X(t)] = tE
_
Y
2
1
_
Arch MLC, Fall 2007 c _Yufeng Guo 393
www.archactuarial.com www.guo.coursehost.com 394
These two equations are a special case of two important relations that hold in this situation
regardless of whether N(t) is Poisson or not. They are
E[X(t)] = E[N(t)] E[Y
1
]
Var[X(t)] = E[N(t)] Var(Y
1
) + Var(N(t)) (E[Y
1
])
2
The rst of these is more intuitive than the second but both MUST be committed to memory.
The formulas for a Poisson process can be derived from these using the fact that
E[N(t)] = Var[N(t)] = t.
EXAMPLE: Collision Insurance
A certain auto insurer models the number of claims it receives each month as a
Poisson process with = 50. Claim sizes are assumed to be independent and are
assumed to follow an exponential distribution with mean equal to 500.
Find the mean and standard deviation of the total aggregate claims for
(A) the next month, and (B) the next year.
SOLUTION:
(A) This is a compound Poisson distribution.
E[X] = E[N] E[Y ] = 50 500 = 25,000
Var[X] = E[N] Var(Y ) + Var(N) (E[Y ])
2
= 50(500)
2
+ 50(500)
2
= 25,000,000

X
= 5000.
(B) This is exactly the same problem except that now = 600.
E[X] = 300,000
Var[X] = 300,000,000
X
= 17,321.
Arch MLC, Fall 2007 c _Yufeng Guo 394
www.archactuarial.com www.guo.coursehost.com 395
EXAMPLE:
X
1
has a compound Poisson distribution with Poisson parameter = 5 and
claim amounts that are 1 or 2 with probabilities 0.6 and 0.4, respectively.
X
2
has a compound Poisson distribution with Poisson parameter = 3
and claim amounts that are 1, 2, or 3 with probabilities 0.3, 0.3 and 0.4,
respectively.
If X
1
and X
2
are independent, describe the distribution of X = X
1
+X
2
.
SOLUTION: Note that
5
8
of the claims occurring under the combined distribu-
tion will be from the X
1
distribution. So when a claim occurs, the probability
is
5
8
that it has the distribution of X
1
and the probability is
3
8
that it has the
distribution of X
2
. The frequency distribution of X (call it N) is Poisson with
parameter = 8. When a claim occurs, the claim amount will have the following
distribution:
y Pr(Y = y)
1
5
8
(0.6) +
3
8
(0.3) = 0.4875
2 0.3625
3 0.15
So the overall distribution is compound Poisson with parameter = 8, and claim
amount distribution as shown in the table.
EXAMPLE:
X is a compound Poisson distribution with = 4 and claim amount distribution
given by the following table.
y Pr(Y = y)
1 0.3
2 0.3
3 0.4
Find the mean and standard deviation of X.
SOLUTION:
E[N] = Var[N] = 4
E[Y ] = 2.1 Var[Y ] = E[Y
2
] (E[Y ])
2
= 5.1 (2.1)
2
= 0.69.
Using the Expected Value and Variance equations for a compound distribution,
we have
E[X] = E[N] E[Y ] = 8.4
Var[X] = E[N] Var(Y ) + Var(N) (E[Y ])
2
Arch MLC, Fall 2007 c _Yufeng Guo 395
www.archactuarial.com www.guo.coursehost.com 396
= 4(0.69) + 4(2.1)
2
= 20.4,

X
= 4.5.
A good source of questions for the SOA-CAS is the idea that as t gets large (whatever
that means), the distribution of X(t) approaches a normal distribution. This will allow us
to use E[X(t)] and Var[X(t)] to create a standard normal approximation from which we can
determine probabilities of specied events.
EXAMPLE: Collision Insurance
In the Collision insurance example earlier, we found that the mean and standard
deviation for annual aggregate claims were E[X] = 300,000 and
X
= 17,321,
respectively. Use the Normal Approximation to nd a 95% condence interval for
total aggregate claims over the next year.
SOLUTION: For a 95% condence interval, we want to go 1.96 standard devi-
ations above and below the mean. That is, the interval we want (assuming our
distribution is approximately normal) is
(
X
1.96
X
,
X
+ 1.96
X
)
= (300,000 33,949 , 300,000 + 33,949) = (266,051 , 333,949).
Note that I made this problem a lot shorter by starting with information from
another example. You need to be ready to do this type of problem from scratch!
5.4.3 Conditional or Mixed Poisson Processes: Gamma-Poisson Model
This chapter looks scary. Before you dismiss it as overly complex and utterly useless for
passing Exam MLC, please note the concept in this chapter was repeatedly tested in the
past. So lets spend some time and gure out what we need to know.
The only core concept you need to know about this chapter is the Poisson-Gamma model. The
Poisson-Gamma model arises when the N[ is a Poisson random variable and the parameter
is also a random variable.
Probability Models Example 5.27 derives the Poisson-Gamma model, even though it doesnt
say that this is the Poisson-Gamma model. If you are interested, you can go through Example
5.27 and learn how to derive the formula. If you are short on time, you can skip how to derive
the formula and just memorize the conclusion.
Im going to explain the Poisson-Gamma model through an example. If you understand this
example, you have understood the core concept of this chapter.
Arch MLC, Fall 2007 c _Yufeng Guo 396
www.archactuarial.com www.guo.coursehost.com 397
EXAMPLE Sample M Problem 87:
On his walk to work, Lucky Tom nds coins on the ground at a Poisson rate. The Poisson
rate, expressed in coins per minute, is constant during any one day, but varies from day to
day according to a gamma distribution with mean 2 and variance 4.
Calculate the probability that Lucky Tom nds exactly one coin during the sixth minute of
todays walk.
SOLUTION:
Method 1 - calculating from scratch without using the Gamma-Poisson model
N[ is Poisson with mean , but varies by day (ie. is a random variable).
Since during any day the parameter is constant, the probability of nding a coin in the
sixth minute is the same as the probability of nding a coin in any minute. Using the total
probability formula, we have:
P(N = 1) =
_

0
P(N = 1[)f()d
Next,lets nd the two parameters for . is gamma with mean 2 and variance 4. From
Exam M Table, youll get the gamma mean and variance formula:
E() = = 2
E(
2
) = ( + 1)
2
V ar() = E(
2
) [E()]
2
=
2
= 4
This gives us:
=
[E()]
2
V ar()
=
2
2
4
= 1
=
V ar()
E()
= 2
f() =
(

()
e

=
1
2
e

2
P(N = 1) =
_

0
P(N = 1[)f()d =
_

0
e

1
2
e

2
d =
1
2
_

0
e

3
2
d =
2
9
Method 2 - using the Gamma-Poisson model
If N[ is Poisson and is gamma, then the unconditional random variable N is a negative
binomial random variable. Now our job is to nd the two parameters of the negative random
variable N.
Arch MLC, Fall 2007 c _Yufeng Guo 397
www.archactuarial.com www.guo.coursehost.com 398
First, N[ is Poisson. Then E(N[) = V ar(N[) =
Double expectation:
E(N) = E

[E(N[)] = E

() = E()
Variance formula:
V ar(N) = E

[V ar(N[)] +V ar

[E(N[)] = E() +V ar()


Next, look up the mean and variance formula from Exam M Table. For the negative binomial
random variable N with parameters and r:
E(N) = r
V ar(N) = r(1 +)
P(N = k) =
r(r 1)(r 2)...(r +k 1)
k
k!(1 +)
r+k
The two parameters of the negative binomial random variable N is:
=
V ar(N)
E(N)
1 =
E() +V ar()
E()
1 =
V ar()
E()
=
4
2
= 2
r =
E(N)

=
E()

=
E()
V ar()
E()
=
[E()]
2
V ar()
=
2
2
4
= 1
P(N = 1) =
r
(1 +)
r+1
=
2
3
2
=
2
9
Now your see that the two parameters of the negative binomial random variable N are exactly
the same as the two parameters of the gamma random variable .
Key Point to Remember:
If N[ is Poisson random variable and is a gamma random variable with
parameters and , then the unconditional random variable N is a negative
binomial random variable with parameters
r = =
[E()]
2
V ar()
= =
V ar()
E()
Chapter 5 Suggested Problems:
37, 38, 39, 42, 57, 58, 60, 77, 78, 85, 88
(Solutions at archactuarial.com)
Arch MLC, Fall 2007 c _Yufeng Guo 398
www.archactuarial.com www.guo.coursehost.com 399
CHAPTER 5 Formula Summary
Poisson:
PN(s +t) N(s) = k = e
t
(t)
k
k!
k = 0, 1, . . . .
E[N(t)] = t and Var[N(t)] = t.
PS
1
n
< S
2
m
=
n+m1

k=n
_
n +m1
k
_
_

1

1
+
2
_
k
_

2

1
+
2
_
n+m1k
PS
1
n
< S
2
1
=
_

1

1
+
2
_
n
PS
1
1
< S
2
1
=

1

1
+
2
If N[ is Poisson random variable and is a gamma random variable with parameters
and , then the unconditional random variable N is a negative binomial random variable
with parameters:
r = =
[E()]
2
V ar()
= =
V ar()
E()
Non-homogeneous Poisson:
m(t) =
_
t
0
(y)dy.
PrN(s +t) N(s) = n = e
[m(s+t)m(s)]
[m(s +t) m(s)]
n
n!
, for n 0.
E[X(t)] = E[N(t)] E[Y
1
]
Var[X(t)] = E[N(t)] Var(Y
1
) + Var(N(t)) (E[Y
1
])
2
Arch MLC, Fall 2007 c _Yufeng Guo 399
www.archactuarial.com www.guo.coursehost.com 400
ARCH Warm-up Problems:
1. Telemarketers call your home according to a Poisson process at a rate of = 2 per day.
(A) What is the distribution of the time between calls?
(B) What is the expected value of the time between calls?
(C) What is the probability that the time between calls 5 and 6 is more than 3 days?
(D) What is the variance in the number of calls received in any 5-day period?
Use the following information for the next ve questions:
An application processor for a small life insurance and annuity company processes all
applications for both products. Applications arrive according to a Poisson process at a
rate of 6 per hour and any one application has probability 0.4 of being for an annuity
policy.
2. Calculate the probability that exactly 3 annuity applications will arrive in the next
hour.
(A) 0.05 (B) 0.10 (C) 0.15 (D) 0.20 (E) 0.25
3. Calculate the probability that the next 3 applications received are for life insurance.
(A) 0.18 (B) 0.22 (C) 0.26 (D) 0.30 (E) 0.34
4. Calculate the probability that the next 3 life applications will arrive before the next 3
annuity applications.
(A) 0.5 (B) 0.6 (C) 0.7 (D) 0.8 (E) 0.9
5. We are told that 12 life applications arrived between 8 a.m. and 9 a.m this morning.
What is the expected number of annuity applications that arrived during that hour?
(A) 2.4 (B) 3.6 (C) 4.8 (D) 8.0 (E) 12.0
6. We are told that 12 life applications arrived between 8 a.m. and 9 a.m this morning.
What is the best estimate of the total number of applications that arrived between 8:00
a.m. and 8:30 a.m.?
(A) 3.0 (B) 5.0 (C) 7.2 (D) 8.4 (E) 13.2
Arch MLC, Fall 2007 c _Yufeng Guo 400
www.archactuarial.com www.guo.coursehost.com 401
Solutions:
1. (A) Interarrival times for a Poisson distribution with rate are Exponential with mean
1

. So, our answer is exponential, with . . .


(B) . . . mean equal to
1
2
.
(C) This is the same as the probability of no calls in the next 3 days.
= e
t
= e
23
= 0.0025
(D) For a ve-day period, the process is Poisson with a mean of 10, therefore the
variance is also 10.
2. The process for annuities is also Poisson with parameter = 0.4 (6) = 2.4. So the
probability of exactly 3 annuity applications in the next hour is
e
2.4
(2.4)
3
3!
= 0.209 Key: D
3.
_

life

ann
+
life
_
3
=
_
3.6
6
_
3
= 0.216 Key: B
4. Note that we only need to observe the following 5 applications to know if 3 life applica-
tions will arrive before the 3rd annuity application. We need all combinations for the
rst 5 applications that include 3 life apps. The equation below adds the probabilities
of 5 life apps, 4 life apps and 3 life apps:
(0.6)
5
+
_
5
1
_
(0.6)
4
(0.4)
1
+
_
5
2
_
(0.6)
3
(0.4)
2
= 0.07776 + 5 (0.05184) + 10 (0.03456) = 0.683 Key: C
5. The annuity applications form a Poisson process that is independent of the number of
life applications, so the answer is 2.4. Key: A
6. The best estimate for the number of annuity applications is (0.5) 2.4 = 1.2. The best
estimate for the number of life applications, given that 12 arrived during the hour, is
(0.5) (12) = 6. So the best estimate for the number that arrived between 8:00 and 8:30
is 7.2. Key: C
Arch MLC, Fall 2007 c _Yufeng Guo 401
www.archactuarial.com www.guo.coursehost.com 402
Past SOA/CAS Exam Questions:
1. Lucky Tom nds coins on his way to work at a Poisson rate of 0.5 coins/minute. The
denominations are randomly distributed:
(i) 60% of the coins are worth 1;
(ii) 20% of the coins are worth 5; and
(iii) 20% of the coins are worth 10.
Calculate the conditional expected value of the coins Tom found during his one-hour
walk today, given that among the coins he found, exactly ten were worth 5 each.
(A) 108 (B) 115 (C) 128 (D) 165 (E) 180
Solution:
A priori, expect 30 coins: 18 worth one, 6 worth 5, 6 worth 10.
Given 10 worth 5, expect: 18 @1; 10 @5; 6 @10.
Total = 18 + 50 + 60 = 128
Key: C
2. Taxicabs leave a hotel with a group of passengers at a Poisson rate = 10 per hour.
The number of people in each group taking a cab is independent and has the following
probabilities:
Number of People Probability
1 0.60
2 0.30
3 0.10
Using the normal approximation, calculate the probability that at least 1050 people
leave the hotel in a cab during a 72-hour period.
(A) 0.60 (B) 0.65 (C) 0.70 (D) 0.75 (E) 0.80
Solution:
Let X(t) be the number leaving by cab in a t hour interval and let Y
i
denote the number
of people in the ith group. Then:
E [Y
i
] = (1 0.6) + (2 0.3) + (3 0.1) = 1.5
E
_
Y
2
i
_
= (1
2
0.6) + (2
2
0.3) + (3
2
0.1) = 2.7
E [X(72)] = 10 72 1.5 = 1, 080
Arch MLC, Fall 2007 c _Yufeng Guo 402
www.archactuarial.com www.guo.coursehost.com 403
Var [X(72)] = 10 72 2.7 = 1, 944
P [X(72) 1, 050] = P [X(72) 1, 049.5]
= P
_
X(72) 1, 080

1944

1, 049.5 1, 080

1944
_
= 1 (0.691754) = (0.691754) = 0.7553
Key: D With or without using the 0.5 continuity correction.
3. An insurance company has two insurance portfolios. Claims in Portfolio P occur in
accordance with a Poisson process with mean 3 per year. Claims in portfolio Q occur
in accordance with a Poisson process with mean 5 per year. The two processes are
independent.
Calculate the probability that 3 claims occur in Portfolio P before 3 claims occur in
Portfolio Q.
(A) 0.28 (B) 0.33 (C) 0.38 (D) 0.43 (E) 0.48
Solution:
For each claim,the probability it is from portfolio is P =
3
8
.
Prob (3 from P before 3 from Q) = Prob (3 or more of rst 5 are from P)
=
_
5
3
_
_
3
8
_
3
_
5
8
_
2
+
_
5
4
_
_
3
8
_
4
_
5
8
_
1
+
_
5
5
_
_
3
8
_
5
_
5
8
_
0
=
5 4 3
3
5
2
2 8
5
+
5 3
4
5
8
5
+
3
5
8
5
=
6750 + 2025 + 243
32,768
= 0.2752
Key: A
4. Workers compensation claims are reported according to a Poisson process with mean
100 per month. The number of claims reported and the claim amounts are indepen-
dently distributed. 2% of the claims exceed 30,000.
Calculate the number of complete months of data that must be gathered to have at
least a 90% chance of observing at least 3 claims each exceeding 30,000.
(A) 1 (B) 2 (C) 3 (D) 4 (E) 5
Arch MLC, Fall 2007 c _Yufeng Guo 403
www.archactuarial.com www.guo.coursehost.com 404
Solution:
Serious claims are reported according to a Poisson process at an average rate of 2 per
month.
P(3+) 0.9 is the same as P(0, 1, 2) 0.1, which is the same as
[P(0) +P(1) +P(2)] 0.1
0.1 e

+e

+
_

2
2
_
e

The expected value is 2 per month, so we would expect it to be at least 2 months


( = 4). Plug in and try
e
4
+ 4e
4
+
_
4
2
2
_
e
4
= 0.238, too high, so try 3 months ( = 6).
e
6
+ 6e
6
+
_
6
2
2
_
e
6
= 0.062, okay. The answer is 3 months.
[While 2 is a reasonable rst guess, it was not critical to the solution. Wherever you
start, you should conclude that 2 is too few and 3 is enough.] Key: C
5. Job oers for a college graduate arrive according to a Poisson process with mean 2 per
month. A job oer is acceptable if the wages are at least 28,000. Wages oered are
mutually independent and follow a lognormal distribution with = 10.12 and = 0.12.
Calculate the probability that it will take a college graduate more than 3 months to
receive an acceptable job oer.
(A) 0.27 (B) 0.39 (C) 0.45 (D) 0.58 (E) 0.61
Solution:
Let T denote the random variable of time until the college graduate nds a job. Let
N(t), t > 0 denote the job oer process.
Each oer can be classied as either
_
Type I- - accept with probability p N
1
(t)
Type II- - reject with probability (1 p) N
2
(t)
By proposition 5.2 N
1
(t) is a Poisson process with
1
= p
p = Pr(w > 28,000) = Pr(ln w > ln 28,000) = Pr(ln w > 10.24)
= Pr
_
ln w 10.12
0.12
>
10.24 10.12
0.12
_
= 1 (1) = 0.1587

1
= 0.1587 2 = 0.3174
Arch MLC, Fall 2007 c _Yufeng Guo 404
www.archactuarial.com www.guo.coursehost.com 405
T has an exponential distribution with =
1
0.3174
= 3.15
Pr(T > 3) = 1 F(3) = e
3
3.15
= 0.386 Key: B
6. For a claims process, you are given:
(i) The number of claims N(t), t 0 is a nonhomogeneous Poisson process with
intensity function;
(t) =
_

_
1, 0 t < 1
2, 1 t < 2
3, 2 t
(ii) Claims amounts Y
i
are independently and identically distributed random variables
that are also independent of N(t).
(iii) Each Y
i
is uniformly distributed on [200, 800].
(iv) The random variable P is the number of claims with claim amount less than 500
by time t = 3.
(v) The random variable Q is the number of claims with claim amount greater than
500 by time t = 3.
(vi) R is the conditional expected value of P, given Q = 4.
Calculate R.
(A) 2.0 (B) 2.5 (C) 3.0 (D) 3.5 (E) 4.0
SOLUTION
_
3
0
(t)dt = 6 so N(3) is Poisson with = 6.
P is Poisson with mean 3 since Prob(y
i
< 500) = 0.5
P and Q are independent, so the mean of P is 3 no matter what the value of Q is.
Key C
7. The number of accidents follows a Poisson distribution with mean 12. Each accident
generates 1, 2, or 3 claimants with probabilities
1
2
,
1
3
,
1
6
,
respectively.
Calculate the variance in the total number of claimants.
(A) 20 (B) 25 (C) 30 (D) 35 (E) 40
Arch MLC, Fall 2007 c _Yufeng Guo 405
www.archactuarial.com www.guo.coursehost.com 406
SOLUTION:
Treat as three independent Poisson variables, corresponding to 1, 2, or 3 claimants.
rate
1
= 6
_
=
1
2
12
_
, rate
2
= 4, rate
3
= 2
V ar
1
= 6, V ar
2
= 16
_
= 4 2
2
_
, V ar
3
= 18
Total Variance = 6 + 16 + 18 = 40 since independent.
Alternatively,
E(X
2
) =
1
2
2
+
2
2
3
+
3
2
6
=
10
3
For compound Poisson, V ar[S] = E[N]E[X
2
] = (12)
_
10
3
_
= 40 Key E
8. The claims department of an insurance company receives envelopes with claims for in-
surance coverage at a Poisson rate of = 50 envelopes per week. For any period of time,
the number of envelopes and the numbers of claims in the envelopes are independent.
The numbers of claims in the envelopes have the following distribution:
Number of Claims Probability
1 0.20
2 0.25
3 0.40
4 0.15
Using the normal approximation, calculate the 90th percentile of the number of claims
received in 13 weeks.
(A) 1690 (B) 1710 (C) 1730 (D) 1750 (E) 1770
SOLUTION
Let Y
i
be the number of claims in the i-th envelope.
Let X(13) be the aggregate number of claims received in 13 weeks.
E[Y
i
] = (1 0.2) + (2 0.25) + (3 0.4) + (4 0.15) = 2.5
E
_
Y
2
i
_
= (1 0.2) + (4 0.25) + (9 0.4) + (16 0.15) = 7.2
E[X(13)] = 50 13 2.5 = 1625
Var[X(13)] = 50 13 7.2 = 4680
Prob X(13) Z = 0.90 = (1.282)
X(13) 1625

4680
1.282
Arch MLC, Fall 2007 c _Yufeng Guo 406
www.archactuarial.com www.guo.coursehost.com 407
X(13) 1712.7
Note: The formula for Var[X(13)] took advantage of the frequency being Poisson. The
more general formula for the variance of a compound distribution,
Var(S) = E(N) Var(X) + Var(N) E(X)
2
,
would give the same result. Key: B
9. A Poisson claims process has two types of claims, Type I and Type II.
(i) The expected number of claims is 3000.
(ii) The probability that a claim is Type I is 1/3.
(iii) Type I claim amounts are exactly 10 each.
(iv) The variance of aggregate claims is 2,100,000.
Calculate the variance of aggregate claims with Type I claims excluded.
(A) 1,700,000 (B) 1,800,000 (C) 1,900,000
(D) 2,000,000 (E) 2,100,000
SOLUTION
Poisson processes are separable. The aggregate claims process is therefore equiva-
lent to two independent processes, one for Type I claims with expected frequency
_
1
3
_
(3000) = 1000 and one for Type II claims. Let
S
I
= aggregate Type I claims.
N
I
= number of Type I claims.
X
I
= severity of a Type I claim (here = 10).
Since X
I
= 10, a constant, E(X
I
) = 10; Var (X
I
) = 0.
Var (S
I
) = E(N
I
) Var (X
I
) + Var (N
i
) [E(X
I
)]
2
= (1000)(0) + (1000)(10)
2
= 100,000
Var(S) = Var(S
I
) + Var(S
II
) since independent
2,100,000 = 100,000 + Var(S
II
) Var(S
II
) = 2,000,000 Key: D
Arch MLC, Fall 2007 c _Yufeng Guo 407
www.archactuarial.com www.guo.coursehost.com 408
10. Subway trains arrive at a station at a Poisson rate of 20 per hour. 25% of the trains
are express and 75% are local. The type of each train is independent of the types of
preceding trains. An express gets you to the stop for work in 16 minutes and a local
gets you there in 28 minutes. You always take the rst train to arrive. Your co-worker
always takes the rst express. You both are waiting at the same station.
Calculate the probability that the train you take will arrive at the stop for work before
the train your co-worker takes.
(A) 0.28 (B) 0.37 (C) 0.50 (D) 0.56 (E) 0.75
SOLUTION:
You arrive rst if both
(A) The rst train to arrive is a local, and
(B) No express train arrives in the 12 minutes after the local arrives.
Pr(A) = 0.75
Expresses arrive at a Poisson rate of (0.25)(20) = 5 per hour, hence 1 per 12 minutes.
f(0) =
e
1
1
0
0!
= 0.368
Events (A) and (B) are independent,
Pr[A and B] = (0.75)(0.368) = 0.276
Key: A
11. A member of a high school math team is practicing for a contest. Her advisor has given
her three practice problems: #1, #2, and #3.
She randomly chooses one of the problems, and works on it until she solves it. Then
she randomly chooses one of the remaining unsolved problems, and works on it until
solved. Then she works on the last unsolved problem.
She solves problems at a Poisson rate of 1 problem per 5 minutes.
Calculate the probability that she has solved problem #3 within 10 minutes of starting
the problems.
(A) 0.18 (B) 0.34 (C) 0.45 (D) 0.51 (E) 0.59
Arch MLC, Fall 2007 c _Yufeng Guo 408
www.archactuarial.com www.guo.coursehost.com 409
SOLUTION:
The number of problems solved in 10 minutes is Poisson with mean 2.
If she solves exactly one, there is 1/3 probability that it is #3.
If she solves exactly two, there is a 2/3 probability that she solved #3.
If she solves 3 or more, she got #3.
f(0) = 0.1353
f(1) = 0.2707
f(2) = 0.2707
P =
_
1
3
_
(0.2707) +
_
2
3
_
(0.2707) + (1 0.1353 0.2707 0.2707) = 0.594 Key: E
12. Kings of Fredonia drink glasses of wine at a Poisson rate of 2 glasses per day.
Assassins attempt to poison the kings wine glasses. There is a 0.01 probability that
any given glass is poisoned. Drinking poisoned wine is always fatal instantly and is the
only cause of death.
The occurrences of poison in the glasses and the number of glasses drunk are indepen-
dent events.
Calculate the probability that the current king survives at least 30 days.
(A) 0.40 (B) 0.45 (C) 0.50 (D) 0.55 (E) 0.60
SOLUTION:
Poisoned wine glasses are drunk at a Poisson rate of 2 0.01 = 0.02 per day.
Number of glasses in 30 days is Poisson with = 0.02 30 = 0.60
f(0) = e
0.60
= 0.55.
Key: E
13. Subway trains arrive at your station at a Poisson rate of 20 per hour. 25% of the
trains are express and 75% are local. The types and number of trains arriving are
independent. An express gets you to work in 16 minutes and a local gets you there in
28 minutes. You always take the rst train to arrive. Your co-worker always takes the
rst express. You are both waiting at the same station.
Calculate the conditional probability that you arrive at work before your co-worker,
given that a local arrives rst.
(A) 37% (B) 40% (C) 43% (D) 46% (E) 49%
Arch MLC, Fall 2007 c _Yufeng Guo 409
www.archactuarial.com www.guo.coursehost.com 410
SOLUTION:
Local comes rst. I board
So I get there rst if he waits more than 28 16 = 12 minutes after the local arrived.
His wait time is exponential with mean 12
The wait before the local arrived is irrelevant; the exponential distribution is memoryless
Prob(exp with mean 12 > 12) = e
12/12
= e
1
= 36.8%
Key: A
14. Beginning with the rst full moon in October deer are hit by cars at a Poisson rate of
20 per day. The time between when a deer is hit and when it is discovered by highway
maintenance has an exponential distribution with a mean of 7 days. The number hit
and the times until they are discovered are independent.
Calculate the expected number of deer that will be discovered in the rst 10 days
following the rst full moon in October.
(A) 78 (B) 82 (C) 86 (D) 90 (E) 94
SOLUTION:
This problem is a direct application of Example 5.18 in Probability Models (p. 308); it
follows from proposition 5.3 (p. 303).
Deer hit at time s are found by time t (here, t = 10) with probability F(t s), where
F is the exponential distribution with mean 7 days.
We can split the Poisson process deer being hit into deer hit, not found by day 10
and deer hit, found by day 10. By proposition 5.3, these processes are independent
Poisson processes.
Deer hit, found by day 10, at time s has Poisson rate 20 F(t s). The expected
number hit and found by day 10 is its integral from 0 to 10.
E(N(t)) = 20
_
t
0
F(t s)ds
E(N(10)) = 20
_
10
0
1 e
(10s)
7
ds
= 20
_
10 7e
s10
7
[
10
0
_
= 20
_
10 7 + 7e
10/7
_
= 94
Key: E
Arch MLC, Fall 2007 c _Yufeng Guo 410
www.archactuarial.com www.guo.coursehost.com 411
15. In a certain town the number of common colds an individual will get in a year follows
a Poisson distribution that depends on the individuals age and smoking status. The
distribution of the population and the mean number of colds are as follows:
Proportion of population Mean number of colds
Children 0.30 3
Adult Non-Smokers 0.60 1
Adult Smokers 0.10 4
Calculate the conditional probability that a person with exactly 3 common colds in a
year is an adult smoker.
(A) 0.12 (B) 0.16 (C) 0.20 (D) 0.24 (E) 0.28
SOLUTION:
Let c denote child; ANS denote Adult Non-Smoker; AS denote Adult Smoker.
P(3[c)P(c) =
3
3
e
3
3!
0.3 = 0.067
P(3[ANS)P(ANS) =
1e
1
1!
0.6 = 0.037
P(3[AS)P(AS) =
4
3
e
4
3!
0.1 = 0.020
P(AS)P(N = 3) =
0.020
(0.067+0.037+0.020)
= 0.16
Key: B
16. For a water reservoir:
(i) The present level is 4999 units.
(ii) 1000 units are used uniformly daily.
(iii) The only source of replenishment is rainfall.
(iv) The number of rainfalls follows a Poisson process with = 0.2 per day.
(v) The distribution of the amount of a rainfall is as follows:
Amount Probability
8000 0.2
5000 0.8
(vi) The numbers and amounts of rainfalls are independent.
Calculate the probability that the reservoir will be empty sometime within the next 10
days.
(A) 0.27 (B) 0.37 (C) 0.39 (D) 0.48 (E) 0.50
Arch MLC, Fall 2007 c _Yufeng Guo 411
www.archactuarial.com www.guo.coursehost.com 412
SOLUTION:
Since the rate of depletion is constant there are only 2 ways the reservoir can be empty
sometime within the next 10 days.
Way #1:
There is no rainfall within the next 5 days
Way #2:
There is one rainfall in the next 5 days
And it is a normal rainfall
And there are no further rainfalls for the next ve days
Pr(Way #1) = Pr(0 in 5 days) = e
(0.25)
= 0.3679
Pr(Way #2) = Pr(1 in 5 days) 0.8 Pr(0 in 5 days)
= 5 0.2e
(0.25)
0.8 e
(0.25)
= 1e
1
0.8 e
1
= 0.1083
Hence Prob empty at some time = 0.3679 + 0.1083 = 0.476.
Key: D
17. Customers arrive at a store at a Poisson rate that increases linearly from 6 per hour at
1 : 00 p.m. to 9 per hour at 2 : 00 p.m.
Calculate the probability that exactly 2 customers arrive between 1 : 00 p.m. and 2 : 00
p.m.
(A) 0.016
(B) 0.018
(C) 0.020
(D) 0.022
(E) 0.024
SOLUTION:
This is a nonhomogeneous Poisson process with intensity function
(t) = 3 + 3t, 0 t 2, where t is time after noon
Average =
_
2
1
(t)dt
1
=
_
2
1
(3+3t)dt
1
=
_
3t +
3t
2
2
_

2
1
= 7.5
f(2) =
e
7.5
7.5
2
2!
= 0.0156
Key: A
Arch MLC, Fall 2007 c _Yufeng Guo 412
www.archactuarial.com www.guo.coursehost.com 413
18. The annual number of accidents for an individual driver has a Poisson distribution with
mean . The Poisson mean, , of a heterogenous population of drivers have a gamma
distribution with mean 0.1 and variance 0.01.
Calculate the probability that a driver selected at random from the population will
have 2 or more accidents in a year.
(A)
1
121
(B)
1
111
(C)
1
100
(D)
1
90
(E)
1
81
SOLUTION:
N is a negative binomial random variable
r =
[E()]
2
V ar()
=
0.1
2
0.01
= 1
=
V ar()
E()
=
0.01
0.1
= 0.1
P(N = 0) =
1
(1 +)
r
=
1
1.1
P(N = 1) =
r
(1 +)
r+1
=
0.1
1.1
2
P(N 2) = 1P(N = 0)P(N = 1) = 1
1
1.1

0.1
1.1
2
=
1.1
2
1.1 0.1
1.1
2
=
0.01
1.1
2
=
1
121
Key: A
19. Actuaries have modeled auto windshield claim frequencies. They have concluded that
the number of windshield claims led per year follows a Poisson distribution with pa-
rameter , where follows a gamma distribution with mean 3 and variance 3.
Calculate the probability that a driver selected at random will le no more than 1
windshield claim next year.
(A) 0.22 (B) 0.24 (C) 0.26 (D) 0.28 (E) 0.30
SOLUTION:
N is a negative binomial random variable
r =
[E()]
2
V ar()
=
3
2
3
= 3
Arch MLC, Fall 2007 c _Yufeng Guo 413
www.archactuarial.com www.guo.coursehost.com 414
=
V ar()
E()
=
3
3
= 1
P(N = 0) =
1
(1 +)
r
=
1
2
3
= 0.125
P(N = 1) =
r
(1 +)
r+1
=
3
2
4
= 0.1875
P(N 1) = P(N = 0) +P(N = 1) = 0.125 + 0.1875 = 0.3125
Key: E
Arch MLC, Fall 2007 c _Yufeng Guo 414
Chapter 13
ARCH Practice Exam
1. Jim, a 30-year old male, does not smoke. Sarah, a 31-year old female, does indeed
partake of cigarettes. Consider a fully discrete 3-year term policy for $1,000. Given
i = 0.04 and the table of q
x
s below:
MALE FEMALE
Age Smoker Non Smoker Smoker Non Smoker
30 0.015 0.010 0.007 0.006
31 0.029 0.025 0.012 0.010
32 0.036 0.032 0.021 0.018
33 0.044 0.040 0.028 0.024
What is the absolute dierence between Jim and Sarahs premium as determined by
the equivalence principle?
(A) 5.63 (B) 5.15 (C) 2.32 (D) 1.83 (E) 0.75
2. You are given:


e
0
= 35
l
x
= x, for 0 x < .
Calculate
15
m
10
, the central-death-rate over the interval from 10 to 25.
(A) 0.011 (B) 0.013 (C) 0.015 (D) 0.017 (E) 0.019
3. Customers arrive at a bank according to a Poisson process at the rate of 80 per hour.
20% of them make only a deposit, 25% make only a withdrawal, and the remaining
55% are there only to complain. Deposit amounts are distributed with mean 7,000 and
standard deviation 1,200. Withdrawal amounts have mean 5,000 and standard deviation
1,000. The number of customers and their activities are mutually independent.
Using the normal approximation, calculate the probability that for an 8-hour day the
total withdrawals of the bank will exceed the total deposits.
415
www.archactuarial.com www.guo.coursehost.com 416
(A) 0.18 (B) 0.22 (C) 0.26 (D) 0.30 (E) 0.34
4. A risky investment with constant rate of default will pay:
Principal and accumulated interest at 14% compounded annually at the end of 15
years if it does not default, and
Nothing if it does default.
A risk-free investment will pay principal and accumulated interest at 8% compounded
annually at the end of 15 years. Principal amounts of the two investments are equal.
Actuarial present values of the two investments are equal at time of purchase.
Calculate the probability that the risky investment defaults no later than time t = 12.
(A) 0.214 (B) 0.385 (C) 0.477 (D) 0.523 (E) 0.684
5. At the end of a season, a baseball player will nd himself either in the majors, in the
minors, or released from the team.
If he is in the majors, he will remain there next year with probability 0.75. If he
leaves the majors, he will go to the minors.
If he is in the minors, he will be moved up to the majors with probability 0.40 or
released with probability 0.30.
If he is released, he could get back to the minors with probability 0.20, but will
not get back to the majors.
Joe just nished a season in the minors where the salaries never change. He can purchase
a special two-year term Income Guarantee insurance policy that will pay Joes minor
league salary for each of the next two seasons that Joe spends in the Released state. If
the benet is paid at year-end and the benet premium for this insurance policy using
i = 0.06 is 11, 534.35, what is Joes minor league salary?
(A) 10,000 (B) 20,000 (C) 30,000 (D) 40,000 (E) 50,000
6. T is the random variable for the future lifetime of a newborn, (0). You are given:
f
1
(t) is the p.d.f. corresponding to the illustrative life table.

f
T
(t) =
_

_
k f
1
(t), 0 t 40
0.02, 40 < t 80
0 80 < t
Arch MLC, Fall 2007 c _Yufeng Guo 416
www.archactuarial.com www.guo.coursehost.com 417
Find
30
p
10
.
(A) 0.800 (B) 0.825 (C) 0.850 (D) 0.875 (E) 0.900
7. For a fully discrete whole life insurance of b on (x), you are given:
q
x+4
= 0.025
i = 0.06.
the initial benet reserve for Policy Year 5 is 437.5
the net amount of risk for Policy Year 5 is 550
a
x
= 11.64
Calculate the terminal reserve for Policy Year 4
(A) 350 (B) 375 (C) 400 (D) 425 (E) 450
8. You are pricing a special 3-year annuity-due on two independent lives, both age 80.
The annuity pays 30,000 if both persons are alive and 70,000 if only one person is alive.
You are given:

k
k
p
80
1 0.95
2 0.85
3 0.75
i = 0.08
Calculate the actuarial present value of this annuity.
(A) 9,200 (B) 9,300 (C) 9,400 (D) 9,500 (E) 9,600
9. For a whole life insurance of 1 on (50) with death benet payable at the end of year of
death, you are given:
(i) i = 0.05
(ii) p
49
= 0.95
(iii) A
50
A
49
= 0.01
(iv)
2
A
50

2
A
49
= 0.007
(v) Z is the present-value random variable for this insurance.
Arch MLC, Fall 2007 c _Yufeng Guo 417
www.archactuarial.com www.guo.coursehost.com 418
Calculate Var(Z).
(A) 0.0075 (B) 0.0100 (C) 0.0125 (D) 0.0150 (E) 0.0175
10. For a fully continuous, unit benet whole life insurance policy, you are given:
= 0.08
i = 0.07
T(x) > 25
Calculate the variance of loss at time 25.
(A) 0.372 (B) 0.214 (C) 0.834 (D) 1.026 (E) 0.453
11. A group of 50 lives all age 35 set up a fund to pay each member 1000 at the end of the
year of their death. The members assume i = 0.06 for all years and use a mortality
table that contains the following information:
x A
x
l
x
35 0.1287 94, 207
36 0.1347 94, 017
37 0.1409 93, 816
38 0.1475 93, 602
39 0.1542 93, 374
Actual experience for the group is shown below:
Year Deaths i
1 0 0.06
2 0 0.08
3 2 0.08
4 1 0.10
If the plan was initially fully funded, what is the dierence between the expected size
of the fund and the actual size of the fund at the end of the fourth year of experience?
(A) 0 (B) 630 (C) 1350 (D) 2090 (E) 2640
12. For a fully discrete whole life insurance on (40), you are given:
Mortality follows the illustrative life table.
i = 0.06
Arch MLC, Fall 2007 c _Yufeng Guo 418
www.archactuarial.com www.guo.coursehost.com 419
Percent of premium expenses are 20% in each year
Per policy expenses include 50 at policy issue, 15 each year the policy is in force,
and 30 payable at the end of year of death.
Calculate the expense policy fee for this policy.
(A) 20.8 (B) 21.5 (C) 22.2 (D) 22.8 (E) 23.4
13. You are given:
T

(x), T

(y), and Z are independent random variables which are components of


a common shock model.
T

(x) follows an exponential distribution with


x
= 0.10.
T

(y) follows an exponential distribution with


y
= 0.08.
Z follows an exponential distribution with
z
= 0.02 for t 0.
= 0.06
T(t) represents the future lifetime random variable for (t) in the presence of a
common shock.
What is the probability that T(x) = T(y)?
(A) 0.00 (B) 0.02 (C) 0.10 (D) 0.20 (E) 0.30
14. For a special fully discrete whole life insurance of 1000 on (30):
(i) The level benet premium for the rst 10 years is .
(ii) The benet premium payable thereafter at age x is 1000v q
x
, x = 40, 41, . . .
(iii) Mortality follows the Illustrative Life Table.
(iv) i = 0.06
Calculate .
(A) 1.83 (B) 2.21 (C) 3.42 (D) 4.12 (E) 5.07
15. Visitors to a historic cemetery are either relatives of the buried or tourists. Visitors who
are relatives come according to a Poisson process with variance of 5 per day. Tourists
visit according to a Poisson process with variance of 7 per day.
What is the probability that in a day, there are 11 relatives and 8 tourists?
(A) 0.0005 (B) 0.0011 (C) 0.0082 (D) 0.0106 (E) 0.0154
Arch MLC, Fall 2007 c _Yufeng Guo 419
www.archactuarial.com www.guo.coursehost.com 420
16. For a fully discrete insurance of unit benet on (50), you are given:

6
V = 0.486

7
V = 0.750
l
56
= 23,285
l
57
= 23,000
i = 0.06
What premium is required at age 56 for these factors to hold?
(A) 0.124 (B) 0.224 (C) 0.324 (D) 0.424 (E) 0.524
17. The distribution of Sandras future lifetime, T, is a two point
The probability is 0.3 that T has constant force of mortality with = 0.03.
The probability is 0.7 that f(t) =
1
50
for 0 t 50, 0 elsewhere.
i = 0.05
A fully continuous whole life policy of 1,000 is issued on Sandra at age 45. What is the
benet premium for this insurance?
(A) 24 (B) 29 (C) 34 (D) 39 (E) 44
18. Consider the following multiple decrement table:
x q
(1)
x
q
(2)
x
65 0.010 0.005
66 0.025 0.012
67 0.030 0.012
68 0.000 1.000
l
65
= 1000
Using the multiple decrement table above, calculate d
(1)
67
+d
(2)
68
.
(A) 920 (B) 940 (C) 960 (D) 980 (E) 1000
19. Anna, age 37, purchases a fully continuous 10-year deferred whole life insurance policy
of unit benet.
= 0.04
= 0.10
Arch MLC, Fall 2007 c _Yufeng Guo 420
www.archactuarial.com www.guo.coursehost.com 421
Calculate the variance of the present value of the benet payment.
(A) 0.010 (B) 0.247 (C) 0.298 (D) 0.417 (E) 0.552
20. For a life table with a one-year select period, you are given:

x l
[x]
d
[x]
l
(x+1)

e
[x]
50 1000 50 8.5
51 800 120
Deaths are uniformly distributed over each year of age.
Calculate

e
[51]
(A) 8.4 (B) 8.7 (C) 9.0 (D) 9.3 (E) 9.6
21. An insurance company issues whole life insurance policies for 21 year olds according to
a Poisson process with mean of 15 per year. All of the policies have a face amount of
2,500. Mortality and interest follow the Illustrative Life Table.
The random variable S represents the total present value of all payments on policies
issued in one year.
Find
V ar[S]
E[S]
.
(A) 128 (B) 292 (C) 344 (D) 414 (E) 552
22. S represents the aggregate claims amount from a compound distribution with number
of claims N and claim amount distribution X.
Pr[S 1.5E(S)] = 1 (3)
Mean
Number of Claims 8 y
Individual Losses 1200 280
What is the value of y?
(A) 1.12 (B) 1.16 (C) 1.20 (D) 1.24 (E) 1.28
Arch MLC, Fall 2007 c _Yufeng Guo 421
www.archactuarial.com www.guo.coursehost.com 422
23. For a whole life insurance of 1000 on (x) with benets payable at the moment of death:
(i)

t
=
_
0.02, 0 < t 5
0.04, 5 < t
(ii)

x
(t) =
_
0.1, 0 < t 5
0.2, 5 < t
Calculate the single benet premium for this insurance.
(A) 433 (B) 533 (C) 633 (D) 733 (E) 833
24. For a fully discrete 3 year term insurance of 1,000 on (45), you are given
x l
x
45 12,500
46 12,275
47 12,105
48 11,977
In addition, i = 0.05 and 1000
2
V
1
45:3
= 3.067.
Calculate 1000P
1
45:3
.
(A) 7.00 (B) 11.56 (C) 13.58 (D) 16.87 (E) 18.67
25. For a fully discrete whole life insurance of 1000 on (30), you are given:
the expenses, payable at the beginning of the year, are:
Expense Type First Year Renewal Year
% of premium 15% 8%
Per policy 10 5
the level expense-loaded premium is 51.8
i = 8%
Calculate the value of the expense augmented loss variable,
0
L
e
, if the insured dies in
the third policy year.
(A) 550 (B) 600 (C) 650 (D) 700 (E) 750
Arch MLC, Fall 2007 c _Yufeng Guo 422
www.archactuarial.com www.guo.coursehost.com 423
26. For a fully discrete 2-year term insurance of 500 on (x), you are given:
450 is the lowest premium such that theres zero chance of loss in Year 1
p
x
= 0.4
p
x+1
= 0.7.
Z is the random variable for the present value at issue of future benets
(A) 30,000 (B) 35,000 (C) 40,000 (D) 45,000 (E) 50,000
27. In a triple decrement table, lives are subject to decrements of death (d), disability (i),
and withdrawals (w).
All decrements have a uniform distribution of decrement in the multiple decrement
table over (x, x + 1)

()
x
= 25,000

()
x+1
= 23,000
d
(i)
x
= 550
Calculate
0.6
p
(i)
x
.
(A) 0.987 (B) 0.115 (C) 0.638 (D) 0.552 (E) 0.237
28. John enjoys small time gambling. He begins his day with twenty-ve dollars with which
to gamble. He gambles twice with the following possible results for each play:
Result Probability
+2 0.1
0 0.2
4 0.3
5 0.4
What is the probability that John will be down ve or more dollars after two plays?
(A) 0.26 (B) 0.48 (C) 0.65 (D) 0.73 (E) 0.91
29. A consulting company has 3 possible nancial results: lose money, break even, or make
money.
(i) If money is lost in a given month, it will lose money, break even, or gain money
with respective probabilities 0.6, 0.3, and 0.1.
Arch MLC, Fall 2007 c _Yufeng Guo 423
www.archactuarial.com www.guo.coursehost.com 424
(ii) If the rm breaks even in a given month, it will lose money, break even, or gain
money with respective probabilities 0.3, 0.3, and 0.4.
(iii) If money is made in a given month, it will lose money, break even, or gain money
with respective probabilities 0.1, 0.3, and 0.6.
Given that the rm broke even in January, what is the probability that it will make
money in March?
(A) 0.09 (B) 0.19 (C) 0.29 (D) 0.39 (E) 0.49
30. For a fully discrete whole life policy on (55), you are given:
The death benet pattern is as follows:
Pol Yr Benet Premium
1 1000 100
2 2000 200
3 4000 300
4 8000 400
5+ 16000 500
p
55+h
= 0.98
(h+1)
for h 0
i = 0.08
Calculate the benet reserve at duration 3.
(A) 186 (B) 252 (C) 298 (D) 374 (E) 441
31. An early retiree is oered 3 pension options on January 1 her 55th birthday.
(A) 40,000 per year each Jan. 1 for life,
(B) A lump sum of 400,000,
(C) 32,000 per year each Jan. 1, certain for 15 years, then continuing for life.
Given:
i = 0.06
A
55
= 0.45, A
70
= 0.72, A
1
55:15
= 0.20
How do the APVs rank?
(A) A > B > C (B) B > A > C (C) C > A > B
(D) A > C > B (E) B > C > A
Arch MLC, Fall 2007 c _Yufeng Guo 424
www.archactuarial.com www.guo.coursehost.com 425
32. For a fully discrete 3-year endowment insurance of 1000 on (x), you are given:

k
L is the prospective loss random variable at time k.
i = 0.05
a
x:3
= 2.105
Premiums are determined by the equivalence principle.
Calculate the value of the expense augmented loss variable,
1
L, given that (x) dies in
the second year from issue.
(A) 510 (B) 515 (C) 520 (D) 525 (E) 530
33. For a continuous whole life annuity of 1 on (x),
(x) is subject to constant force of mortality, = 0.08.
The force of interest is 0.05.
T is the random variable for the future lifetime of (x).
Find Var
_
a
T
_
.
(A) 12.21 (B) 20.54 (C) 26.27 (D) 29.42 (E) 41.58
34. A small insurance company has an insurance portfolio such that the number of claims
follows a binomial distribution with parameters n and p = 0.25. Individual claims
follow the following distribution:
Claim Amt, X Pr(X = x)
0 0.3
20 0.3
30 0.3
40 0.1
If the variance of aggregate claims is 919.5, what is n?
(A) 4 (B) 8 (C) 12 (D) 16 (E) 20
35. For a fully discrete whole life insurance on (x), you are given:
the death benet is 1000 in the rst year and 5000 thereafter
level benet premiums are payable for life
q
x
= 0.02
Arch MLC, Fall 2007 c _Yufeng Guo 425
www.archactuarial.com www.guo.coursehost.com 426
v = 0.90.

10
V
x
= 0.6

10
V is the benet reserve at the end of Year 10 for this insurance
Calculate
10
V
(A) 2,000 (B) 2,500 (C) 3,000 (D) 3,500 (E) 4,000
36. For a whole life annuity-due on (x),
q
x
= 0.01, q
x+1
= 0.05.
i = 0.05
a
x+2
= 6.951
Payment No. Benet A Benet B
1 1 3
2 2 2
3+ 3 1
What is the absolute dierence in single premium required to purchase such an annuity
with benet pattern A versus benet pattern B?
(A) 14.32 (B) 13.85 (C) 11.31 (D) 9.86 (E) 8.47
37. Catherine is a waitress at a very nice restaurant downtown. You are given:
On a given night, she expects to receive 30 tips with a standard deviation of 2.5.
The distribution of each tips has a mean of 20 with a standard deviation of 4.
Using the normal approximation, calculate the probability that Catherines tip income
on a given night is less that $548.
(A) 0.17 (B) 0.20 (C) 0.23 (D) 0.26 (E) 0.29
38. Evan works for a multinational corporate conglomerate. He is considering spending the
next year on temporary assignment aboard an oil rigger.
Usually, the pdf of Evans future lifetime is f(t) =
1
50
.
On the oil rigger, however, f(t) =
1
10
.
= 0.05
Arch MLC, Fall 2007 c _Yufeng Guo 426
www.archactuarial.com www.guo.coursehost.com 427
What is the dierence in the actuarial present value at issue of a 5-year fully continuous
term policy if he takes the assignment versus not taking it?
(A) 0.02 (B) 0.05 (C) 0.07 (D) 0.12 (E) 0.17
39. On his walk to work, Lucky Tom nds coins on the ground at a Poisson rate. The
Poisson rate, expressed in coins per minute, is constant during any one day, but varies
from day to day according to a gamma distribution with mean 5 and variance 25.
Calculate the probability that Lucky Tom nds exactly one coin during the third minute
of todays walk.
(A) 0.10 (B) 0.14 (C) 0.18 (D) 0.22 (E) 0.26
40. For two independent lives (40) and (44)
Table 1 Table 2
x q
x
q
x
40 0.1 0.2
41 0.2 0.3
42 0.3 0.4
43 0.4 0.5
44 0.5 0.6
45 0.6 0.7
46 0.7 0.8
Calculate the absolute dierence between
2|
q
40:44
under Table 1 and under Table 2
(A) 0.02 (B) 0.04 (C) 0.06 (D) 0.08 (E) 0.10
Arch MLC, Fall 2007 c _Yufeng Guo 427
www.archactuarial.com www.guo.coursehost.com 428
Answer Key for Practice Exam
Question Answer Question Answer
1 D 21 E
2 E 22 B
3 A 23 E
4 C 24 A
5 B 25 D
6 D 26 C
7 C 27 A
8 D 28 C
9 C 29 D
10 A 30 D
11 D 31 B
12 E 32 D
13 C 33 C
14 A 34 B
15 B 35 C
16 B 36 D
17 B 37 A
18 B 38 C
19 A 39 B
20 D 40 A
Arch MLC, Fall 2007 c _Yufeng Guo 428
www.archactuarial.com www.guo.coursehost.com 429
ARCH Solutions Practice Exam: SOLUTIONS
1. SOLUTION:
For Jim,
A
1
30:3
=
(1)(0.01)
1.04
+
(1)(0.99)(0.025)
1.04
2
+
(1)(0.99)(0.975)(0.032)
1.04
3
= 0.0096 + 0.0229 + 0.0275 = 0.06
a
30:3
= 1 +
(1)(0.99)
1.04
+
(1)(0.99)(0.975)
1.04
2
= 1 + 0.9519 + 0.8924 = 2.8443
Then, for Jim, 1000P
1
30:3
=
1000(0.06)
2.8843
= 21.095.
For Sarah,
A
1
31:3
=
(1)(0.012)
1.04
+
(1)(0.988)(0.021)
1.04
2
+
(1)(0.988)(0.979)(0.028)
1.04
3
= 0.0115 + 0.0192 + 0.0241 = 0.0548
a
31:3
= 1 +
(1)(0.988)
1.04
+
(1)(0.988)(0.979)
1.04
2
= 1 + 0.95 + 0.8943 = 2.8443
Then, for Sarah, 1000P
1
31:3
=
1000(0.0548)
2.8843
= 19.267.
Finally, 21.095 19.267 = 1.828 answer D.
2. SOLUTION:
n
m
x
=
l
x
l
x+n
n
L
x
, so
15
m
10
=
l
10
l
25
15
L
10
.
To nd the ls and L, we need to know what is.

e
0
gives it to us.
35 =

e
0
=
_

0
t
p
0
dt =
_

0
l
t
l
0
dt =
_

0
t

dt =
_

0
_
1
t

_
dt
=
_
t
t
2
2
_

0
=

2
=

2
.
Therefore = 70.
Alternatively, since this is DeMoivre,

e
0
=
0
2
= 35 = 70.
l
10
= 60, and l
25
= 45,
n
L
x
=
_
t
0
l
x+t
dt =
_
t
0
( x t)dt.
Arch MLC, Fall 2007 c _Yufeng Guo 429
www.archactuarial.com www.guo.coursehost.com 430

15
L
10
=
_
15
0
(70 10 t) dt =
_
60t
t
2
2
_
15
0
= 787.5,
and
15
m
10
=
60 45
787.5
= 0.019 The Answer is E.
3. SOLUTION:
Split into 2 Poisson processes
Deposit:
D
= 80 8 0.2 = 128
Withdrawal:
W
= 80 8 0.25 = 1604
Individual deposit X
D
:
E(X
D
) = 7, 000
(X
D
) = 1, 200
Individual withdrawal X
W
:
E(X
W
) = 5, 000
(X
W
) = 1, 000
Total deposit S
D
=

N
D
i=1
X
D
i
E(S
D
) = E(N
D
)E(X
D
) = 128 7, 000 = 896, 000
V ar(S
D
) =
D
[V ar(X
D
) +E
2
(X
D
)] = 128 (7, 000
2
+ 1, 200
2
) = 6, 456, 320, 000
Total deposit S
W
=

N
W
i=1
X
W
i
E(S
W
) = E(N
W
)E(X
W
) = 160 5, 000 = 800, 000
V ar(S
W
) =
W
[V ar(X
W
) +E
2
(X
W
)] = 160 (5, 000
2
+ 1, 000
2
) = 4, 160, 000, 000
Y = S
W
S
D
E(Y ) = E(S
W
) E(S
D
) = 800, 000 896, 000 = 96, 000
V ar(Y ) = V ar(S
W
) +V ar(S
D
) = 6, 456, 320, 000 + 4, 160, 000, 000 = 10, 616, 320, 000
P(Y > 0) = 1 P(Y 0) = 1
_
0 (96, 000)

10, 616, 320, 000


_
= 1 (0.932) = 0.1757
The answer is A.
Arch MLC, Fall 2007 c _Yufeng Guo 430
www.archactuarial.com www.guo.coursehost.com 431
4. SOLUTION:
APV (risk free) = (1.08)
15
v
15
APV (risky) = (1.14)
15
e
15
v
15
Note that e
15
is the probability of surviving decrement by default for 15 years.
Setting these equal gives us 1.08 = 1.14e

= 0.05407.
Pr (risky defaults by year 12) = Pr(X 12) = 1 e
_

_
12
0
0.05407dt
_
= 1 e
[0.05407t]
12
0
= 1 e
[0.05407(12)+0]
= 0.4773
Faster Solution: Pr(X 12) = F(12) = 1 e
12
= 0.4773 The answer is C.
5. SOLUTION:
The only way the insurance pays o is if Joe is released in either of the next two seasons.
This can happen only as follows:
Minors Minors Released.
Minors Released.
Minors Released Released.
We dont know the amount of the benet, so call it X. We then combine the above
three cases into the following equation:
Benet Premium = 11, 534.35 =
(0.3)(0.3)(X)
1.06
2
+
(0.3)(X)
1.06
+
(0.3)(0.8)(X)
1.06
2
This solves to X = 20, 000. The answer is B.
6. SOLUTION:
First, we need to nd k. Since f
T
(t) is a pdf, it must be true that
_

0
f
T
(t) dt = 1
1 =
_
40
0
kf
1
(t) dt +
_
80
40
0.02 dt
1 = kF
1
(40) + 0.8
Since F
1
(40) corresponds to the Illustrative Life Table, we know that
F
1
(40) = 1
0
p
40
= 1
l
40
l
0
= 1
93,132
100,000
= 0.06868
1 = k(0.06868) + 0.8 k =
0.2
0.06868
= 2.912
Arch MLC, Fall 2007 c _Yufeng Guo 431
www.archactuarial.com www.guo.coursehost.com 432
Now, for t < 40,
F
T
(t) =
_
t
0
2.912f
1
(t) dt = 2.912F
1
(t)
where F
1
(t) corresponds to the Illustrative Life Table.
10
p
30
=
s(40)
s(10)
=
1 F
T
(40)
1 F
T
(10)
F
T
(40) = 2.912F
1
(40) = 2.912
_
1
l
40
l
0
_
= 0.2
F
T
(10) = 2.912F
1
(10) = 2.912
_
1
l
10
l
0
_
= 2.912 (1 0.97056) = 0.0857

10
p
30
=
1 0.2
1 0.0857
= 0.875
The answer is D.
7. SOLUTION: Policy Year 5 runs from t = 4 to t = 5
Initial benet reserve for Policy Year 5 = terminal reserve at t = 4 + benet premium
at t = 5
NAR(net amount of risk) for Policy Year 5 = Death benet paid at t = 5 - terminal
reserve at t = 5
(
4
V +P)(1 +i) = bq
x+4
+
5
Vp
x+4
= bq
x+4
+
5
V(1 q
x+4
) = (b
5
V)q
x+4
+
5
V
437.5 1.06 = 550 0.025 +
5
V
5
V = 450
b =
5
V + (b
5
V) = 550 + 450 = 1, 000
P = b(
1
ax
d) = 1000(
1
11.64

1
1.06
) = 29.31
4
V = (
4
V +P) P = 437.5 29.31 = 408.19
The answer is C.
8. Solution
Time t 0 1 2
Prob(both alive) 1 0.95
2
0.85
2
Prob(both dead) 0 0.05
2
0.15
2
Prob(one alive and one dead) 0 1 0.95
2
0.05
2
1 0.85
2
0.15
2
30, 000(1+0.95
2
v+0.85
2
v
2
)+70, 000[(10.95
2
0.15
2
)v+(10.85
2
0.15
2
)v
2
] = 95, 113.17
Key: D
Arch MLC, Fall 2007 c _Yufeng Guo 432
www.archactuarial.com www.guo.coursehost.com 433
9. Key: C
Var(Z) =
2
A
50
(A
50
)
2
A
50
A
49
= 0.01 = A
50
(vq
49
+vp
49
A
50
)
= A
50

_
0.05
1.05
+
0.95
1.05
A
50
_
A
50
= 0.605
2
A
50

2
A
49
= 0.007 =
2
A
50

_
v
2
q
49
+v
2
p
49
2
A
50
_
=
2
A
50

_
0.05
1.05
2
+
0.95
1.05
2

2
A
50
_

2
A
50
= 0.3785
Var(Z) = 0.3785 (0.605)
2
= 0.01245
10. SOLUTION:
First, = ln(1 +i) = ln(1.07) = 0.0677
Also, A
x
=A
x+t
since constant is independent of age.
=

+
=
0.08
0.1477
= 0.5416
In addition,
2
A
x
=
2
A
x+t
since independent of age.
=

+ 2
=
0.08
0.2154
= 0.3714.
a
x
=
1
+
= 6.7705
Then we can calculate P(A
x
) =
Ax
ax
= 0.08.
Alternatively, we know that for constant force of mortality,
P(A
x
) = = 0.08
V ar[
t
L] =
_
_
1 +
P
_
A
x
_

_
_
2
_
2
A
x+t

_
A
x+t
_
2
_
=
_
_
1 +
P
_
A
x
_

_
_
2
_
2
A
x

_
A
x
_
2
_
=
_
1 +
0.08
0.0677
_
2 _
0.3714 (0.5416)
2
_
= (4.7597)(0.0781) = 0.3716
The answer is A 0.372.
Arch MLC, Fall 2007 c _Yufeng Guo 433
www.archactuarial.com www.guo.coursehost.com 434
11. SOLUTION:
Initial fund is 50 1000A
35
= 6435.
After 4 years, we would expect to have a fund (reserve) equal to the PVFB for the
members expected to be alive at that time:
50 1000A
39
l
39
l
35
= 7642
Actual Fund experience is as follows
Time Fund
0 6435
1 6435(1.060) = 6821
2 6821(1.08) = 7367
3 7956 2000 = 5956
4 6551 1000 = 5551
So the dierence between actual and expected fund after 4 years is 7642 5551 = 2090
The answer is D.
12. SOLUTION:
Note that percent-of-premium charges apply to the policy fee as well as the rest of the
gross premium. Therefore, the policy fee g must satisfy
g a
40
= 0.2g a
40
+ 50 + 15 a
40
+ 30A
40
g(14.82) =
50 + 15(14.82) + 30(0.1613)
0.8
g = 23.38 Key: E
13. SOLUTION:
From Bowers 9.6 (especially ex. 9.6.2c),
Pr[T(x) = T(y)] =
_

0
f
T(x)T(y)
(t, t) dt
=
_

0
e
(x+y+)t
dt =

+
x
+
y
=
0.02
0.2 + 0.1 + 0.08
= 0.1
The answer is C.
14.
APV Benets = 1000A
30
= 1000A
1
30:10
+

k=10
k
E
30
1000v q
30+k
APV Premiums = a
30:10
+

k=10
k
E
30
1000v q
30+k
Arch MLC, Fall 2007 c _Yufeng Guo 434
www.archactuarial.com www.guo.coursehost.com 435
The equivalence principle gives us
1000A
1
30:10
+

k=20
k
E
30
1000v q
30+k
= a
30:10
+

k=20
k
E
30
1000v q
30+k
= 1000
A
1
30:10
a
30:10
A
1
30:10
= A
30

10
E
30
A
40
= 0.1025
1
1.06
10

10
p
30
(0.1613) = 0.0142
a
30:10
= a
30

10
E
30
a
40
= 7.75
=
14.2
7.75
= 1.83
The answer is A.
15. SOLUTION:
From Probability Models 5.3.2, the answer is given by:
e
tp
(tp)
n
n!
e
t(1p)
(t (1 p))
m
m!
In this problem, = 5 + 7 = 12
p =
5
12
, 1 p =
7
12
n = 11, m = 8
Pr[11 relatives, 8 tourists] = e
5
(5)
1
1
11!
e
7
(7)
8
8!
= (0.008242)(0.1304) = 0.0011
The answer is B.
16. SOLUTION:
From Bowers using formula 8.3.10,
h
V +
h
= b
h+1
vq
x+h
+
h+1
V vp
x+h
0.486 + = (1)
_
1
1.06
__
285
23, 285
_
+ (0.75)
_
1
1.06
__
23, 000
23, 285
_
= 0.224 the answer is B.
17. SOLUTION:
A
Sandra
= (0.30)A
constant
45
+ (0.70)A
f(x)
45
A
constant
45
=

+
=
0.03
0.03 + ln(1.05)
= 0.3808
Arch MLC, Fall 2007 c _Yufeng Guo 435
www.archactuarial.com www.guo.coursehost.com 436
A
f(x)
45
=
_
50
0
v
t
f
t
(t)dt =
_
50
0
e
t
1
50
dt
=
1 e
50
50
=
1 0.0872
2.4395
= 0.374
So, A
Sandra
= (0.30)0.3808 + (0.70)0.374 = 0.376.
a
Sandra
=
1 A
Sandra

=
1 0.376
ln(1.05)
= 12.79
Finally, 1000P
Sandra
=
10000.376
12.79
= 29.4 answer B.
18. SOLUTION:
x q

x
p

x
l

x
d
(1)
x
= l

x
q
(1)
x
d
(2)
x
= l

x
q
(2)
x
65 0.015 0.985 1000
66 0.037 0.963 985
67 0.042 0.958 949 28.5
68 1.000 0 909 909
28.5 + 909 = 937.5, the answer is B.
19. SOLUTION:
10|
A
x
=
_

10
e
t
e
t
dt =

+
e
10(+)
Alternatively,
10|
A
x
= e
10
10
p
x
A
x+10
= e
10(+)

+
So,
10|
A
x
=
0.04
0.14
e
1.4
= 0.0705
2
10|
A
x
=
0.04
0.04 + 0.20
e
10(0.04+0.20)
=
0.04
0.24
e
2.4
= 0.0151
Then V ar(Z) = (0.0151) (0.0705)
2
= 0.0101 the answer is A.
20. SOLUTION:
Imagine two customers one aged 50 and other 51 each bought a life insurance policy
at the same day. Their health is better than the health of someone randomly chosen
from the general population. As a result, we use [50] and [51] to represent these two
customers. However, the selection eect wears o in one year (one year select period).
So one year later, [50] becomes 51. Another year later, 51 becomes 52. Similarly, one
year later, [51] will become 52.
[50] in 2 years = [51] in one year=someone aged 52 randomly chosen from
the general population.
So we can calculate

e
52
in two ways:
Arch MLC, Fall 2007 c _Yufeng Guo 436
www.archactuarial.com www.guo.coursehost.com 437


e
[50]


e
51


e
52


e
[51]


e
52
Since these two [50] and [51] will merge into one person (52) in the future, the two ways
of calculation should produce the same result.
So the solution process looks like this:

e
[50]


e
51


e
52


e
[51]
.
x l
[x]
d
[x]
l
(x+1)

e
[x]
50 1000 50 1000 50 = 950 8.5
51 800 120 800 120 = 680 ?
p
[50]
=
950
1000
= 0.95 p
[51]
=
680
800
= 0.85
p
51
=
680
950
= 0.715789
Recursive formula if [50] moves one forward and becomes (51):

e
[50]
=

e
[50]:1
+p
[50]

e
51

e
[50]:1
=
_
1
0
t
p
[50]
dt = 0.5 (1 +
t
p
[50]
) = 0.5 (1 + 0.95) = 0.975
Please note that

e
[50]:1
=
_
1
0
t
p
[50]
dt is the area of the graph
t
p
[50]
from t = 0 to t = 1
under UDD.
0.85 = 0.975 + 0.925

e
51
. This gives us:

e
51
= 7.921053
Recursive formula if 51 moves one forward and becomes (52):

e
51
=

e
51:1
+p
51

e
52

e
51:1
=
_
1
0
t
p
51
dt = 0.5 (1 +
t
p
51
) = 0.5 (1 + 0.715789) = 0.857895
Please note that

e
51:1
=
_
1
0
t
p
51
dt is the area of the graph
t
p
51
from t = 0 to t = 1
under UDD.
7.921053 = 0.857895 + 0.715789

e
52
. This gives us:

e
52
= 9.867647

e
[51]
=

e
[51]:1
+p
[51]

e
52

e
[51]:1
=
_
1
0
t
p
[51]
dt = 0.5 (1 +
t
p
[51]
) = 0.5 (1 + 0.85) = 0.925

e
[51]
= 0.925 + 0.85 9.867647 = 9.3125
The Key is D.
21. SOLUTION:
S =
N

j=1
Z
j
, Var(S) = E(N) Var(Z
i
) +E(Z
i
)
2
Var(N)
Arch MLC, Fall 2007 c _Yufeng Guo 437
www.archactuarial.com www.guo.coursehost.com 438
So, Var(Z
i
) = 2500
2
(
2
A
21
A
2
21
)
= 2500
2
_
15.0569
1000

_
68.2423
1000
_
2
_
= 64,999.30
E[Z
i
] = 2500A
21
= 170.6
So, Var(S) = (15)(64,999.30) + (170.6)
2
(15) = 1,411,555
and E(S) = E(N)E(Z
i
) = (15)(170.6) = 2559
V ar[S]
E[S]
=
1,411,555
2559
= 551.6. Key: E
22. SOLUTION:
There are several ways to do this one, here is one.
1 (3) = Pr(Z 3) = Pr
_
S E(S)

1.5E(S) E(S)

s
_
.
Since the right-hand-side of the inequality must equal 3, we have
0.5E(S)

s
= 3 0.5E(S) = 3
s
.
Using the relations,
E(S) = E(N) E(X)
Var(S) = E(N) Var(X) + Var(N) (E(X))
2
,
E(S) = 8 (1200) = 9600 and Var(S) = 8 (280)
2
+y
2
(1200)
2
,
Then, 0.5E(S) = 3
s
becomes:
0.5(8 1200) = 3
_
8(280)
2
+y
2
(1200)
2
1600 =
_
627,200 + 1,440,000y
2
2,560,000 = 627,200 + 1,440,000y
2
y = 1.1585 Key: B
23.
A
x
=

+
, A
1
x:5
=

+
(1
5
E
x
)
and
5
E
x
= e
5(+)
1000A
x
= 1000
_
A
1
x:5
+
5
E
x
A
x+5
_
Arch MLC, Fall 2007 c _Yufeng Guo 438
www.archactuarial.com www.guo.coursehost.com 439
= 1000
__
0.1
0.1 + 0.02
_
_
1 e
5(0.1+0.02)
_
+e
5(0.1+0.02)
_
0.2
0.2 + 0.04
__
= 1000
_
(0.833)
_
1 e
0.6
_
+ 0.833e
0.6
_
= 833
The answer is E.
24. SOLUTION:
1000
2
V
1
45:3
= 1000A
1
45+2:1
1000P
1
45:3
a
45+2:1
Using the given informtion,
3.067 = 1000
__
12, 105 11, 977
12, 105
__
1
1.05
__
[1000P
1
45:3
][1]
7.004 = 1000P
1
45:3
The answer is A.
25. SOLUTION:
Time t 0 1 2 3
Age 30 31 32 33
A Premium 51.80 51.80 51.80
B Per Policy Fee 10.00 5.00 5.00
C % of Premium Expense 7.77 4.144 4.144
D Death Benet Paid 1000
E=B+C+D-A Loss per Year -34.03 -42.656 -42.656 1000
Please note that 15%51.8 = 7.77 and 8%51.8 = 4.144
0
L
e
= 34.03 42.656v 42.656v
2
+ 1000v
3
= 683.735 The answer is D.
26. SOLUTION:
If the insured dies in Year 1, the present value of death benet is 500v. If the insurer
charges 500v, then theres zero chance of loss in Year 1. 500v = 450. This gives us
v = 0.9.
Next, lets calculate V ar(Z) assuming the death benet is 1.
Time to die T
(x)
0 1 2
Prob of being alive 1 0.4 0.4 0.7 = 0.28 0
Prob of deaths (also prob of getting Z) 0 1 0.4 = 0.6 0.4 0.28 = 0.12 0.28
Present value of death benet Z 0 v v
2
0
E(Z) = 0.6v + 0.12v
2
+ 0.28 0 = 0.6372
E(Z
2
) = 0.6v
2
+ 0.12(v
2
)
2
+ 0.28 0
2
= 0.564732
Arch MLC, Fall 2007 c _Yufeng Guo 439
www.archactuarial.com www.guo.coursehost.com 440
V ar(Z) = E(Z
2
) E(Z)
2
= 0.564732 0.6372
2
= 0.15870816
If the death benet is 500, then the variance of the present value of death benet is:
V ar(500Z) = 500
2
V ar(Z) = 500
2
0.15870816 = 39, 677.04. The answer is C.
27. SOLUTION:
From Bowers in 10.5.4,
s
p
(i)
x
=
_
s
p
()
x
_
q
(i)
x
q
()
x
0.6
p
()
x
= 1 0.6q
()
x
= 1 0.6
_
2,000
25,000
_
= 0.952
q
(i)
x
=
550
25,000
= 0.022, q
()
x
=
25,000 23,000
25,000
= 0.08
So,
0.6
p
(i)
x
= (0.952)
(
0.022
0.08
)
= 0.987 The answer is A.
28. SOLUTION:
It is important to not make a problem too complicated! First, gure out how Johns
position could be 20 or less after day 2. All such possibilities are shown below:
Init Amt Play 1 result Play 2 Result Final Amt Probability
25 0 5 20 (0.2)(0.4) = 0.08
25 4 4 17 (0.3)(0.3) = 0.09
25 4 5 16 (0.3)(0.4) = 0.12
25 5 0 20 (0.4)(0.2) = 0.08
25 5 4 16 (0.4)(0.3) = 0.12
25 5 5 15 (0.4)(0.4) = 0.16
So the total probability of ending up with 20 or fewer dollars is the sum of the above
probabilities, which is 0.65. The answer is C.
29. SOLUTION:
The transition probability matrix, P, looks like:
Lose Even Gain
Lose 0.6 0.3 0.1
Even 0.3 0.3 0.4
Gain 0.1 0.3 0.6
So, the transition probability matrix over two months is P
2
= P P which looks like:
Lose Even Gain
Lose 0.46 0.30 0.24
Even 0.31 0.30 0.39
Gain 0.21 0.30 0.49
Arch MLC, Fall 2007 c _Yufeng Guo 440
www.archactuarial.com www.guo.coursehost.com 441
So, given break even in January, we move to Gain in two months with probability 0.39
answer D.
Note: If you know which element in a matrix youre looking for, you could just do the
multiplication which leads to that element in order to save time! But be careful dont
make a careless error while trying to save time!
30. SOLUTION:
First,
Age p
x
q
X
55 0.98 0.02
56 0.9604 0.0396
57 0.9412 0.0588
58 0.9224 0.0776
3
p
55
= (0.98)(0.9604)(0.9412) = 0.8855
2
p
56
= (0.9604)(0.9412) = 0.9039
From Bowers, Section 8.3:
k
V =
k1

h=0
(
h
vb
h+1
q
x+h
)
(1 +i)
kh
kh
p
x+h
With k = 3 in this problem, at h = 0, we have (using the given info):
_
100
1000
1.08
q
55
_
_
(1.08)
3
3
p
55
_
=
_
100
1000
1.08
0.02
_
_
(1.08)
3
0.8855
_
= 115.92
Likewise, at h = 1:
_
200
2000
1.08
q
56
_
_
(1.08)
2
2
p
56
_
=
_
200
2000
1.08
0.0396
_
_
(1.08)
2
0.9039
_
= 163.45
And, at h = 2:
_
300
4000
1.08
q
57
_
_
(1.08)
1
p
57
_
=
_
300
4000
1.08
0.0588
__
(1.08)
0.9412
_
= 94.35
These are all the pieces we need, so adding them up gives us
3
V = 115.92 + 163.45 + 94.35 = 373.72
Arch MLC, Fall 2007 c _Yufeng Guo 441
www.archactuarial.com www.guo.coursehost.com 442
The answer is D. Note that we found this reserve retrospectively!
31. SOLUTION:
For A,
a
55
=
1 A
55
d
=
1 0.45
0.06
1.06
= 9.72 40,000 a
55
= 388,667
For C, we need
32,000
_
a
15 0.06
+
15
E
55
a
70
_
A
55
= A
1
x:15
+
15
E
55
A
70
0.45 = 0.20 +
15
E
55
(0.72)
15
E
55
= 0.3472
Using the handy SOA approved calculator, a
15 0.06
= 10.29.
And, a
70
=
1 A
70
d
=
1 0.72
.057
= 4.95
So for C we have,
32,000 (10.29 + 0.3472 (4.95)) = 384,276
So, B > A > C and the answer is B!
32. SOLUTION:
1
L is the loss random variable of a 2-year endowment insurance on
(x + 1) given that (x + 1) dies in Year 1.
Time t 0 1
Attained age (x + 1) (x + 2)
Premium collected 1000P
30:3
Death benet paid 1000
1
L = 1000v 1000P
30:3
(P
30:3
+d) a
x:3
= 1 P
30:3
= 0.42744
1
L = 1000(v P
30:3
) = 1000(1.05
1
0.42744) = 524.941
The answer is D.
33. SOLUTION:
In Bowers, formula 5.2.9 gives
Var(a
T
) =
_
2
A
x
_
(A
x
)
2

2
As shown in example 5.2.1,
A
x
=

+
=
0.08
0.05 + 0.08
= 0.6154
Arch MLC, Fall 2007 c _Yufeng Guo 442
www.archactuarial.com www.guo.coursehost.com 443
2
A
x
=

2 +
=
0.08
0.10 + 0.08
= 0.4444
So, Var(a
T
) =
0.44440.6154
2
0.05
2
= 26.27. The answer is C.
34. SOLUTION:
V ar(S) = E(N) V ar(X) +E
2
(X) V ar(N)
With binomial distribution,
E(N) = np and V ar(N) = npq
In this problem, E(N) = n(0.25) and V ar(N) = n(0.25)(0.75) With this claim distri-
bution,
E(X) = 0(0.3) + 20(0.3) + = 19,
E(X
2
) = 0
2
(0.3) + 20
2
(0.3) + = 550, so
V ar(X) = 550 19
2
= 189
So,
V ar(S) = 919.5 = [n(0.25)] [189] + [19
2
] [n(0.25)(0.75)]
which leads to n = 8, the answer is B.
35. SOLUTION:
10
V = 5000A
x+10
a
x+10
First, calculate
=
5000A
x+10
4000vqx
ax
=
5000A
x+10
ax

4000vqx
ax
= 5000(
1
ax
d)
4000vqx
ax
= 5000(
1
4
0.1) 4000
0.90.02
4
= 732
Next, calculate A
x+10
and a
x+10
10
V
x
= 1
a
x+10
ax
a
x+10
= (1
10
V
x
) a
x
= (1 0.6)0.4 = 1.6
A
x+10
= 1 d a
x+10
= 1 0.1 1.6 = 0.84
Finally,
10
V = 5000A
x+10
a
x+10
= 5000 0.84 732 1.6 = 3, 028.8
36. SOLUTION:
A single premium is equal to the APV at issue. It is critical to recognize that
APV = a
x
= b
x
+ (b
x+1
)(v)(p
x
) + (b
x+3
)(v
2
)(p
x
)(p
x+1
)( a
x+2
)
Arch MLC, Fall 2007 c _Yufeng Guo 443
www.archactuarial.com www.guo.coursehost.com 444
APV
A
= 1 + (2)(0.9524)(0.99) + (3)(0.9524)
2
(0.99)(0.95)(6.951) = 20.675
APV
B
= 3 + (2)(0.9524)(0.99) + (1)(0.9524)
2
(0.99)(0.95)(6.951) = 10.815
Then, 20.675 10.815 = 9.86 the answer is D!
Note: in both calculations of APV, you use the number (2)(0.9524)(0.99) = 1.8857. As
you work an exam problem, try to recognize when you need numbers youve already
calculated this can be a time saver!!
37. SOLUTION:
E(S) = (30)(20) = 600
V ar(S) = (30)(4)
2
+ (2.5)
2
(20)
2
= 2980
Normal Approximation:
Pr(S < 548) = Pr
_
Z <
548 600
54.589
_
= Pr(Z < 0.95)
1 (0.95) = 1 0.83 = 0.17
The answer is A.
38. SOLUTION: If he doesnt go:
A
1
x:5
=
_
5
0
v
t
f
T(nogo)
(t)dt =
_
5
0
e
0.05t
1
50
dt =
1 e
0.05(5)
50(0.05)
= 0.0885
If he does go:
A
1
x:5
=
_
1
0
v
t
f
T(go)
(t)dt +
_
5
1
v
t
1
p
x

t1
p
x+1

no go
(x +t)dt
From age x + 1 on, the probability distribution for the future lifetime is uniform with
maximum 49 years to live so
t1
p
x+1

no go
(x +t) =
1
49
This gives us
=
_
1
0
e
0.05t
1
10
dt +
_
5
1
e
0.05t
(0.9)
1
49
dt
=
1
10
_
e
0.05t
0.05
_
1
0
+
0.9
49
_
e
0.05t
0.05
_
5
1
=
1
10
_
e
0.05
1
0.05
_
+
0.9
49
_
e
0.05(5)
e
0.05(1)
0.05
_
Arch MLC, Fall 2007 c _Yufeng Guo 444
www.archactuarial.com www.guo.coursehost.com 445
= 0.0975 + 0.0633 = 0.161
So the dierence is APV(Go) - APV(No Go) = 0.161 0.0885 = 0.0725. The key is C.
39. SOLUTION:
N[ Poisson but is gamma. Then the unconditional distribution of N is negative
binomial.
First, nd the parameters of N. N[ is Poisson. Then E(N[) = V ar(N[) =
Double expectation:
E(N) = E

[E(N[)] = E

() = E() = 5
Variance formula:
V ar(N) = E

[V ar(N[)] +V ar

[E(N[)] = E() +V ar() = 5 + 25 = 30


Next, look up the mean and variance formula from Exam M Table. For negative
binomial random variable N with parameters and r:
E(N) = r
V ar(N) = r(1 +)
P(N = k) =
r(r 1)(r 2)...(r +k 1)
k
k!(1 +)
r+k
The two parameters of negative binomial random variable N is:
=
V ar(N)
E(N)
1 = 5
r =
E(N)

= 1
P(N = 1) =
r
(1 +)
r+1
=
5
6
2
= 0.1389
Please note that in a given day, the parameter is constant. Consequently, the prob-
ability of nding one coin in the third minute is the same of probability of nding one
coin in one minute, no matter its the rst minute, second minute, or any other one
minute. Only the length of the time interval matters; the starting point doesnt matter.
The answer is B.
Arch MLC, Fall 2007 c _Yufeng Guo 445
www.archactuarial.com www.guo.coursehost.com 446
40. SOLUTION:
2|
q
40:44
=
3
q
40:44

2
q
40:44
3
q
40:44
= (
3
q
40
)(
3
q
44
) = (1
3
p
40
)(1
3
p
43
) = (1 p
40
p
41
p
42
)(1 p
43
p
44
p
45
)
2
q
40:44
= (
2
q
40
)(
2
q
44
) = (1
2
p
40
)(1
2
p
43
) = (1 p
40
p
41
)(1 p
43
p
44
)
Table 1:
3
q
40:44
= (1 0.9 0.8 0.7)(1 0.5 0.4 0.3) = 0.46624
2
q
40:44
= (1 0.9 0.8)(1 0.5 0.4) = 0.22400
2|
q
40:44
= 0.46624 0.22400 = 0.24224
Table 2:
3
q
40:44
= (1 0.8 0.7 0.6)(1 0.4 0.3 0.2) = 0.648064
2
q
40:44
= (1 0.8 0.7)(1 0.4 0.3) = 0.387200
2|
q
40:44
= 0.648064 0.387200 = 0.260864
Absolute dierence: 0.260864 0.24224 = 0.018624
The answer is A.
Arch MLC, Fall 2007 c _Yufeng Guo 446
Chapter 14
DELUXE SOLUTION TO MAY
2007 MLC
447
www.archactuarial.com www.guo.coursehost.com 448
Deluxe solution to May 2007 MLC
Problem 1
The only tricky part is the notation of the force of mortality. The force of mortality is expressed
as a function of the attained age j
r
= j(r) instead of a function of future life time j(r +t). Most
candidates are familiar with j(r +t) and are less familiar with j
r
.
Make sure you understand how j
r
and j(r +t) are related. Consider (r), whose future life time
is T (r). If we treat the attained age A as the random variable, then the force of mortality at age r is
j
r
= j(r) =
)

(r)
1 1

(r)
=
:
0
(r)
: (r)
Using the basic calculus rule d [lnq (n)] =
q
0
(n)
q (n)
dn, we have:
j(r) =
:
0
(r)
: (r)
=
d [ln: (r)]
dr
Change r to j: j(j) dj = d [ln: (j)]

_
r+|
r
j(j) dj =
_
r+|
r
d [ln: (j)] = ln: (r +t) ln: (r)
= ln
: (r +t)
: (r)
=
|
j
r

|
j
r
= exp
_

_
r+|
r
j(j) dj
_
Set : = j r. Then : becomes the future life time (r). We have:
_
r+|
r
j(j) dj =
_
|
0
j(r +:) d: (1)
|
j
r
= exp
_

_
r+|
r
j(j) dj
_
= exp
_

_
|
0
j(r +:) d:
_
(2)
Remember Equation 1 and 2. Now we are ready to tackle the problem.
We are given
_
75
71
j(r) dr = 0.107

4
j
71
= exp
_

_
71+4
71
j(r) dr
_
= c
0.107
5
j
70
= j
70
(
4
j
71
) =
3
j
70
2
j
71
(
4
j
71
) =
0.95
0.96
c
0.107
= 0.889 17
c _Yufeng Guo 448
www.archactuarial.com www.guo.coursehost.com 449
Problem 2
This is a simple problem.
\ ar
_
a
T(r)j
_
=
2

2
c
2
=
j
j +c
=
1
1 +c,j
0.3443 =
1
1 +c,j
c,j = 1. 904 4
2
=
j
j + 2c
=
1
1 + 2 (c,j)
=
1
1 + 2 (1. 904 4)
= 0.207 95
\ ar
_
a
T(r)j
_
=
0.207 95 0.3443
2
0.08
2
= 13. 970
Problem 3
5
\
[60]
= 1
..
a
[60]+5
..
a
[60]
The above formula holds for a fully discrete whole life regardless of mortality.
The selection period is 3 years. Hence
..
a
[60]+5
=
..
a
65
5
\
[60]
= 1
..
a
65
..
a
[60]
..
a
65
= 9.8969
..
a
[60]
=
1
[65]
d
=
1 0.359
1 1.06
1
= 11. 324 3
5
\
[60]
= 1
9.8969
11. 324 3
= 0.126
1000
5
\
[60]
= 126
Problem 4
\ ar (1) =
2

2
(1
r
)
2
=
0.0143 0.0653
2
(1 0.0653)
2
= 0.01 148 7
_
\ ar (1) =
_
0.01 148 7 = 0.1072
150000
_
\ ar (1) = (150000) 0.1072 = 16080
c _Yufeng Guo 449
www.archactuarial.com www.guo.coursehost.com 450
Problem 5
Exponentially distributed inter-arrival time

Number of arrivals is a Poisson distribution


To specify the parameter of the Poisson distribution, use common sense. If the inter-arrival time
is 0 months, then every 0 months theres one more event. So events occur at the average rate of
1
0
per
month or
2
0
per 2-month.
No events by 1/31/ 2007 doesnt have any impact on the distribution of future events (the Poisson
arrival process always starts afresh and doesnt remember what happened in the past). Let represent
the number of events that have occurred during 1/31/2007 and 3/31/2007. is a Poisson random
variable with an average arrival rate of ` =
2
0
= 2 per 2-month.
1 ( = :) = c
2 2
n
n!
1 ( _ 3) = 1 [1 ( = 0) +1 ( = 1) +1 ( = 2)]
= 1 c
2
_
1 + 2 +
2
2
2!
_
= 0.323
Problem 6
The key to solving this problem is precisely specifying the model.
Total donation at the end of 7 days:
o
1
= A
1
+A
2
+... +A
1
A
I
is the donation by the i-th person who donates.
A
I
s are independent identically distributed with a common mean and common variance:
1 (A) = 15, \ ar (A) = 75
' is the total number of people who donate.
' is Poisson with parameter of an average rate ` = 10 (7) (0.8) = 56 arrivals per 7-day.
1 (') = \ ar (') = 56
A and ' are independent
Total withdrawal at the end of 7 day:
o
V
= 1
1
+1
2
+... +1

is the withdrawal by the ,-th person who draws food.


1

s are independent identically distributed with a common mean and common variance:
1 (1 ) = 40, \ ar (1 ) = 533
is the total number of people who withdraw food
is Poisson with parameter of an average rate ` = 10 (7) (0.2) = 14 arrivals per 7-day
1 () = \ ar () = 14
1 and are independent
Net amount the end of 7 days: l = o
1
o
V
o
1
and o
V
are independent and approximately normal.
Hence l is approximately normal (linear combination of independent normal random variables is
also normal)
Net amount at t = 0 is zero. We are asked to nd 1 (l 600) at the end of 7 days.
1 (l 600) = 1 1 (l _ 600) = 1
_
600J(I)
_
\ o:(I)
_
1 (l) = 1 (o
1
) 1 (o
V
) \ ar (l) = \ ar (o
1
) +\ ar (o
V
)
1 (o
1
) = 1 (A) 1 (') = 15 (56) = 840
c _Yufeng Guo 450
www.archactuarial.com www.guo.coursehost.com 451
\ ar (o
1
) = \ ar (A) 1 (') +1
2
(A) \ ar (') = 75 (56) + 15
2
(56) = 16 800
1 (o
V
) = 1 (1 ) 1 () = 40 (14) = 560
\ ar (o
V
) = \ ar (1 ) 1 () +1
2
(1 ) \ ar () = 533 (14) + 40
2
(14) = 29 862
1 (l) = 840 560 = 280
\ ar (l) = 16 800 + 29 862 = 46 662
1 (l 600) = 1
_
600280
p
46 662
_
= 1 (1. 48) = 0.069 4
Problem 7
Since we know the benet premium for the rst 3 years, we can use the retrospective method.
Reserve at t = 3 is a /, where
a=accumulating benet premiums for the rst 3 years to t = 3
/=accumulating death benets for the rst 3 years to t = 3
/ is the same regardless of the benet premium pattern
Choice B,C, and D have same benet premium for the rst 3 years. They have the same reserve
at t = 3.
B,C, and D can be eliminated.
Of A and E, the accumulated value of the benet premiums for the rst 3 years is higher for E
than for A.
E produces higher reserve than for A (and also higher than B,C, and D).
Problem 8
This is a dicult problem. If you cant solve it, dont feel too bad.
Dene being called by the parent is death. Then we have two lives Kevin and Kira. Set the unit
of time is an hour (instead of one year in a typical life contingency problem). The constant force of
mortality is
j
1tuIn
= 0.7 j
1I:o
= 0.6
Kiras total score is
l =
_
0 if T
1tun
(t) _ T
1I:o
(t)
100, 000T
1I:o
(t) if T
1tun
(t) < T
1I:o
(t)
1 (l) = 1 [T
1tuIn
(t) _ T
1I:o
(t)] 0 +1 [T
1tuIn
(t) < T
1I:o
(t)] 1 [100, 000T
1I:o
(t)]
= 100, 0001 [T
1tuIn
(t) < T
1I:o
(t)] 1 [T
1I:o
(t)]
= 100, 0001 [T
1tuIn
(t) < T
1I:o
(t)]

c
1I:o
1 [T
1tuIn
(t) < T
1I:o
(t)] =

Kevin

Kevin
+
Kira
=
0.7
0.7+0.6
To see why, notice
)
1tuIn
(:) =
J
Js
(
|
j
1tuIn
) =
J
J|
_
c
0.7s
_
= 0.7c
0.7s
)
1I:o
(t) =
J
J|
(
|
j
1I:o
) =
J
J|
_
c
0.6|
_
= 0.6c
0.6|
So the future lives of Kevin and Kira are both exponentially distributed. The joint life distribution
is
)
1tuIn,1I:o
(:, t) = )
1tuIn
(:) )
1I:o
(t) = 0.7c
0.7s
0.6c
0.6|
1 (: < t) =
_
1
0
_
|
0
)
1tuIn,1I:o
(:, t) d:dt =
_
1
0
_
|
0
0.7c
0.7s
0.6c
0.6|
d:dt
c _Yufeng Guo 451
www.archactuarial.com www.guo.coursehost.com 452
=
_
1
0
0.6c
0.6|
__
|
0
0.7c
0.7s
d:
_
dt =
0.7
0.7+0.6
You can save time if you know the following rule about two independent exponential random
variables competing:
Two independent exponential random variables T
1
and T
2
:
)
T1
(t) = `
1
c
X1|
)
T2
(t) = `
2
c
X2|
Then the probability that T
1
arrives before T
2
is:
1 (T
1
< T
2
) =
X1
X1+X2
You can use common sense to memorize why 1 (T
1
< T
2
) =
X1
X1+X2
. In this example, the Kevins
death arrives at the rate of 0.7 death per hour and Kira 0.6 death per hour. This is similar to:
Car A arrives at 0.7 car per hour. Car B arrives at 0.6 car per hour. So on average we see a total
of 1.3 car per hour. If we see one car, then theres 0.7/1.3=7/13 chance that this car is Car A (i.e.
theres 7/13 chance that Car A arrives before Car B) and 6/13 chance that this car is Car B (i.e.
theres 6/13 chance that Car B arrives before Car A).
Lets continue.

c
1I:o
=
1
0.6
1 (l) = 100, 0001 [T
1tun
(t) < T
1I:o
(t)]

c
1I:o
= 100, 000
_
0.7
0.7+0.6
_
1
0.6
= 89, 743. 59
Problem 9
The typical formula is

(2)
25
=
_
1
0
j
(2)
(t)
|
j
(r)
25
dt =
_
1
0
j
(2)
(t)
|
j
0
(1)
25

|
j
0
(2)
25
dt
Cause (2) works only at t = 1,5 and t = 3,5. During [0, 1,5), (1,5, 3,5), and (3,5, 1], only cause
1 is at work. Hence the above formula needs to be modied as

(2)
25
=
1/5
j
0
(1)
25
[(2)s death rate at t = 1,5] +
3/5
j
0
(1)
25
[(2)s death rate at t = 3,5]
time t 0 1,5 3,5
j
0
(1)
25
1 1
0.1
5
= 0.98 1
3(0.1)
5
= 0.94
Cause (2) decrement rate
3
4
(0.12) = 0.09
1
4
(0.12) = 0.03
# of people hit by cause (2) (0.98) 0.09 = 0.088 2 0.94 (0.03) = 0.028 2
During [0, 1,5), only cause 1 is at work (so we are in the world of a single decrement table). There
will be 0.98 survivor immediately before t = 1,5. Then at the next instant, cause (2) hits (0.98)
0.09 = 0.088 2 people.
Similarly, the interval (1,5, 3,5) is in the world of single decrement table. Immediately before
t = 3,5, 0.94 people survive cause (1). In the next instant, cause (2) hits 0.94 (0.03) = 0.028 2 people.
So the total number of people hit by cause (2) in the rst year is

(2)
25
= 0.088 2 + 0.028 2 = 0.116 4
c _Yufeng Guo 452
www.archactuarial.com www.guo.coursehost.com 453
Problem 10
This is a clever problem testing the recursive formula of whole life insurance.
However, wording is a little vague. This is what SOA means:
On 12/31/2007, two insureds (65) and (66) each buy a whole life insurance with $1000 death
benet. The interest rate is 10% for the rst policy year and 6% thereafter.
65
= 0.01 and
66
= 0.012
. In addition, 1000
66
= 300. Calculate 1000
65
.
Use the recursive formula
r
=
r
+j
r

r+1
. Remember that the interest rate may change.

65
= 1.1
1
(0.01) + 1.1
1
(0.99)
6%
66

6%
66
= 1.06
1
(0.012) + 1.06
1
(0.988)
6%
67
To nd
6%
67
:

66
= 1.1
1
(0.012) + 1.1
1
(0.988)
6%
67
0.3 = 1.1
1
(0.012) + 1.1
1
(0.988)
6%
67

6%
67
= 0.321 86
Final answer:

6%
66
= 1.06
1
(0.012) + 1.06
1
(0.988) 0.321 86 = 0.311 32

65
= 1.1
1
(0.01) + 1.1
1
(0.99) 0.311 32 = 0.289 279
1000
65
= 289 .3
Problem 11
time t 0 1+
% of premium expense (for 10 years) 9% + 20% 9%
xed expense (for 20 years) 5 + 5 5
G
..
a
40:10j
= 1000 1
40:20j
+ 0.2G+ 5 + 0.09G
..
a
40:10j
+ 5
..
a
40:20j
illustrative life table
r
..
a
rj
1000
r
1000
10
1
r
1000
20
1
r
40 14.8166 161.32 536.67 274.14
50 13.2668
60 11.1454 369.13
..
a
40:10j
=
..
a
40

10
1
40
..
a
50
= 14.8166 0.53667 13.2668 = 7. 696 7
1
40:20j
=
40

20
1
40

60
= 0.16132 0.27414 0.36913 = 0.060 13
..
a
40:20j
=
..
a
40

20
1
40
..
a
60
= 14.8166 0.27414 11.1454 = 11. 761 2
7. 696 7G = 1000 (0.060 13) + 0.2G+ 5 + 0.09G(7. 696 7) + 5 (11. 761 2)
G = 18. 215
c _Yufeng Guo 453
www.archactuarial.com www.guo.coursehost.com 454
Problem 12
r
(1)
r

(2)
r

(r)
r
=
(1)
r
+
(2)
r
j
(r)
r
= 1
(r)
r
56 0.005 0.04 0.054 0.955
57 0.008 0.06 0.068 0.932
1 (55) _ 1 means that the insured is still alive at age 56. Now we are standing at t = 1, where
the insured is aged 65 and still alive.
1
1[1 (55) _ 1 is distributed as follows:
death time cause
1
1[1 (55) _ 1 Probability
1 < t _ 2 accident 2000
_
1.06
1
_
50 = 1836. 79
(1)
56
= 0.005
1 < t _ 2 other 1000
_
1.06
1
_
50 = 893. 40
(2)
56
= 0.04
2 < t _ 3 accident 2000
_
1.06
2
_
50
_
1 + 1.06
1
_
= 1682. 82
1j

(1)
56
= 0.007 64
2 < t _ 3 other 1000
_
1.06
2
_
50
_
1 + 1.06
1
_
= 792. 83
1j

(2)
56
= 0.05 73
t 3 any 50
_
1 + 1.06
1
_
= 97. 17
2
j
(r)
56
= 0.890 06
Total 1
1j

(1)
56
= j
(r)
56

(1)
57
= 0.955 0.008 = 0.007 64
1j

(2)
56
= j
(r)
56

(2)
57
= 0.955 0.06 = 0.057 3
2
j
(r)
56
= j
(r)
56
j
(r)
57
= 0.955 0.932 = 0.890 06
Check total probability:
0.005 + 0.04 + 0.007 64 + 0.05 73 + 0.890 06 = 1 OK
If your total probability is not one, you made some errors.
Lets reorganize the above table:
. 1 [
1
1 = .[1 (55) _ 1] 1 [
1
1 _ .[1 (55) _ 1]
97. 17 0.890 06 0.890 06
792. 83 0.05 73 0.890 06 + 0.05 73 = 0.947 36
893. 40 0.04 0.947 36 + 0.04 = 0.987 36
1682. 82 0.007 64 0.987 36 + 0.007 64 = 0.995
1836. 79 0.005 0.995 + 0.005 = 1.0
Total 1
So the smallest . such 1 [
1
1 _ .[1 (55) _ 1] _ 0.95 is . = 893. 40
SOAs answer 893 is problematic because
1 [
1
1 _ 893[1 (55) _ 1]
= 1 [
1
1 = 97. 17[1 (55) _ 1] +1 [
1
1 = 792. 83[1 (55) _ 1]
= 0.890 06 + 0.05 73 = 0.947 36
Of the SOAs 5 choices, (E) 943 is the smallest . given by SOA that satises
1 [
1
1 _ .[1 (55) _ 1] _ 0.95
1 [
1
1 _ 943[1 (55) _ 1]
= 1 [
1
1 = 97. 17[1 (55) _ 1] +1 [
1
1 = 792. 83[1 (55) _ 1] +1 [
1
1 = 893. 40[1 (55) _ 1]
= 0.890 06 + 0.05 73 + 0.04 = 0.987 36
If you chose (E) and your exam score is 5, you might consider writing a letter to SOA justifying
your answer.
c _Yufeng Guo 454
www.archactuarial.com www.guo.coursehost.com 455
Problem 13
SOA wants you to the Hattendorf theorem (Actuarial Mathematics Section 8.5 page 244):
\ ar [
|
1[1 (r) _ /] = [ (/
|+1

|+1
\ )]
2
j
r+|

r+|
+
2
j
r+|
\ ar [
|+1
1[1 (r) _ / + 1]
If you are one of the lucky few who have memorized this dicult formula, congratulations.
Since the policy ends at t = 3, \ ar [
3
1[1 (r) _ 3] = 0
\ ar [
2
1[1 (r) _ 2]
= [ (/
3

3
\ )]
2
j
r+2

r+2
+
2
j
r+2
\ ar [
3
1[1 (r) _ 3]
= [ (/
3

3
\ )]
2
j
r+2

r+2
=
_
1.1
1
(1000 0)

2
(0.5) (0.5) = 206611. 57
\ ar [
1
1[1 (r) _ 1]
= [ (/
2

2
\ )]
2
j
r+1

r+1
+
2
j
r+1
\ ar [
2
1[1 (r) _ 2]
=
_
1.1
1
(1000 120.833)

2
(0.6) (0.4) + 1.1
2
(0.6) (206611. 57)
= 255761. 36
If you are one of many who dont bother memorizing the Hattendorf theorem, congratulations.
You can solve this problem using basic principles. To simplify calculation, assume the death benet
is $1 (instead of $1000).
Calculate the benet premium:
time t 0 1 2 3
people alive 1 0.7 0.7 (0.6) = 0.42 0.7 (0.6) (0.5) = 0.21
deaths 1 0.7 = 0.3 0.7 0.42 = 0.28 0.42 0.21 = 0.21
Equivalence principle:
1
_
1 + 0.7 + 0.42
2
_
= 0.3 + 0.28
2
+ 0.21
3
1 =
0.3 1.1
1
+ 0.28 1.1
2
+ 0.21 1.1
3
1 + 0.7 1.1
1
+ 0.42 1.1
2
= 0.333 71
Find the distribution of the random variable
1
1[1 (r) _ 1. We are standing at t = 1.
1
1[1 (55) _
1 is distributed as follows:
death time
1
1[1 (r) _ 1 Probability
1 < t _ 2 1 = 1.1
1
0.333 71 = 0.575 38 0.4
2 < t _ 3
2
1 (1 +) = 1.1
2
0.333 71
_
1 + 1.1
1
_
= 0.189 36 0.6 (0.5) = 0.3
t 3 1 (1 +) = 0.333 71
_
1 + 1.1
1
_
= 0.637 08 1 0.4 0.3 = 0.3
1 [
1
1[1 (r) _ 1] = 0.575 38 (0.4) + 0.189 36 (0.3) 0.637 08 (0.3) = 0.095 836
1
_
1
1
2
[1 (r) _ 1

= 0.575 38
2
(0.4) + 0.189 36
2
(0.3) + (0.637 08)
2
(0.3)
= 0.264 943 298 56
\ ar [
1
1[1 (r) _ 1] = 0.264 943 298 56 0.095 836
2
= 0.255 759
Since the death benet is 1000, the variance is
1000
2
0.255 759 = 255, 759
You can use BA II Plus or BA II Professional calculator 1-V Statistics Worksheet to quickly
calculate the variance. To 1-V Statistics Worksheet, we need to scale the probability to an integer.
So 0.4, 0.3, and 0.3 are scaled to 4, 4, and 3. Enter
c _Yufeng Guo 455
www.archactuarial.com www.guo.coursehost.com 456
A01 = 0.575 38 1 01 = 4
A02 = 0.189 36 1 02 = 3
A03 = 0.637 08 1 03 = 3
You should get:
A = 0.095836 o

= 0.50572597
Then o
2

= 0.50572597
2
= 0.255 759 1000
2
0.255 759 = 255, 759
You can use 1-V statistics worksheet to calculate the mean and variance of a discrete random
variable as long as you scale the probabilities to integers.
By the way, the reserve given to you 0.95833, 0.120833, and 0 at t = 1, 2, 3 is really not necessary
because you can calculate the reserve yourself. The reserve at t = 1 under the retrospective method
is:
1 (1 +i)
r
j
r
=
0.333 71 (1.1) 0.3
0.7
= 0.095 83
The reserve at t = 2 under the prospective method is:

r+2
1 = 0.5
_
1.1
1
_
0.333 71 = 0.120 835
SOA purposely gives you the reserves because it wants you to use the Hattendorf theorem.
Problem 14
To have 0 < T (30) T (35) < 5, we let two things happen:
Let (35) die at time t or T (35) = t. Since the probability that a continuous random variable
takes on a single value is zero (i.e. 1 [T (35) = t] = 0), to produce probability we have to express
T (35) in terms of a range. So we write t < T (35) < t +dt. This means that (35) dies in the tiny
interval [t, t +dt], which is the statistical statement that (35) dies at time t. The probability is
)
T(35)
(t) dt
Let (30) die during the interval [t, t + 5]. The probability is 1 [t < T (30) < 5 +t] = 1 [T (30) t]
1 [T (30) t + 5] = (
|
j
30
) (
|+5
j
30
)
1 [0 < T (30) T (35) < 5]
=
_
1
0
1 [t < T (35) < t +dt] (1 [T (30) t] 1 [T (30) t + 5])
=
_
1
0
1 [t < T (35) < t +dt] 1 [T (30) t]
_
1
0
1 [t < T (35) < t +dt] 1 [T (30) t + 5]
_
1
0
1 [t < T (35) < t +dt] 1 [T (30) t] = 1 [T (30) T (35)] = 1 /
So the correct answer should have the term 1 /. This immediately tells us that 1 is the right
answer.
Lets nish up the problem.
1 [T (30) t + 5] = 1 [T (30) 5] 1 [T (30) + 5 t[T (30) 5] = (
5
j
30
) (
|
j
35
)
_
1
0
1 [t < T (35) < t +dt] 1 [T (30) t + 5]
=
_
1
0
_
)
T(35)
(t) dt

(
5
j
30
) (
|
j
35
) = (
5
j
30
)
_
1
0
_
)
T(35)
(t) dt

(
|
j
35
)
= (
5
j
30
)
_
1
0
[j
35
(t) (
|
j
35
)] (
|
j
35
) dt
c _Yufeng Guo 456
www.archactuarial.com www.guo.coursehost.com 457
The rst
|
j
35
in the integration is the survival function of someone who is initially 35. The second
|
j
35
in the integration is the survival function of someone who was initially 30 years old 5 years ago
and who becomes age 35 now. By the phrase "The future lifetimes of (30) and (35) are independent
and identically distributed," SOA wants to say that two
|
j
35
terms are identical. In other words, SOA
wants to say that two people with the same attained age have the same survival function.
Then
_
1
0
[j
35
(t) (
|
j
35
)] (
|
j
35
) dt refers to the following probability: "There are two independent
lives each aged 35. Their future lifetimes are identically distributed. Whats the chance that one (35)
will die before the other (35)?" Clearly, the probability is 1/2 because each (35) has 50% chance of
dying before the other.
(
5
j
30
)
_
1
0
[j
35
(t) (
|
j
35
)] (
|
j
35
) dt = 0.5 (
5
j
30
) = 0.5a
Then the nal answer is 1 / 0.5a.
Please note that the phrase "The future lifetimes of (30) and (35) are independent and identically
distributed" is problematic. It means that T (30) and T (35) are independent identically distributed
(instead of meaning
|
j
35
is the same regardless of you are initially 35 or you are initially younger but
grow up to age 35, as SOA wants us to interpret).
c _Yufeng Guo 457
www.archactuarial.com www.guo.coursehost.com 458
Problem 15
A probability tree diagram will enable you to quickly solve the problem.
L=Independent Living. C=Health Center. D=Dead.
Next, draw the following diagram.
t = 0 t = 1 t = 2 t = 3
0.21
0.4L 0.1C
0.71
01
0.6L 0.4C 0.1C
0.91
0.21
01
0
1 L 0.3C 0.4C 0.1C
0.91
0.61
0.11
For example, 1 Independent Living at t = 0 will become [0.61, 0.3C, 0.11] at t = 1. Similarly,
11 at t = 1 becomes [0.41, 0.4C, 0.21] at t = 2. Finally, after t = 3, each state remains unchanged
(i.e. each state becomes an absorbing state). So nobody collects benets at t 3.
time t 0 1 2 3
# of people in 1 1 0.6 0.6 (0.4) = 0.24
# of people 1 C 0.3 0.6 (0.4) = 0.24 0.6 (0.4) (0.1) = 0.024
To calculate how many people collect benets at t = 1, 2, 3, pay attention to the bold numbers in
the diagram.
From t = 0 to t = 1, 11 0.3C. Hence 0.3C collects 1000 benets at t = 1.
From t = 1 to t = 2, 0.61 0.4C. Hence 0.6 (0.4) = 0.24C collects 1000 benets at t = 2.
c _Yufeng Guo 458
www.archactuarial.com www.guo.coursehost.com 459
From t = 2 to t = 3, 0.41 0.1C. Hence 0.6 (0.4) (0.1) = 0.024C collects 1000 benets at t = 3.
The equivalence principle:
1
_
1 + 0.6 + 0.24
2
_
= 1000
_
0.3 + 0.24
2
+ 0.024
3
_
1 = 1000
0.3 1.25
1
+ 0.24 1.25
2
+ 0.024 1.25
3
1 + 0.6 1.25
1
+ 0.24 1.25
2
= 248. 46
c _Yufeng Guo 459
www.archactuarial.com www.guo.coursehost.com 460
Problem 16
First, label the transition matrix so we can quickly nd the probability.
0 1 2
0 0.2 0.5 0.3
1 0.1 0.6 0.3
2 0.1 0.5 0.4
For example, from the above table, we easily see that the probability of "1 coin today and 1 coin
tomorrow" is 1 (1 1) = 0.6.
Next, draw the following diagram:
t = 0 t = 1 t = 2
0, Pr0.2
0, Pr 0.1 1, Pr0.5
2, Pr0.3
0, Pr 0.1
1, Pr 1 1, Pr 0.6 1, Pr 0.6
2, Pr 0.3
0, Pr 0.1
2, Pr 0.3 1, Pr 0.5
2, Pr 0.4
A =# of coins found at t = 1
1 =# of coins found at t = 2
We need to nd 1 (A +1 _ 3)
Possible combinations of (A, 1 ):
(A = 1, 1 = 2)
1 (A = 1, 1 = 2) = 0.6 (0.3) = 0.18
(A = 2, 1 = 1 or 2)
1 (A = 2, 1 = 1 or 2) = 0.3 (0.5 + 0.4) = 0.27
1 (A +1 _ 3) = 0.18 + 0.27 = 0.45
c _Yufeng Guo 460
www.archactuarial.com www.guo.coursehost.com 461
Problem 17
N=No accident. Y=Yes, theres at least one accident.
Draw the following diagram:
t = 0 t = 1 t = 2 t = 3
N0.8
N0.8
1 0.2
N0.8
N0.7
1 0.2
1 0.3
1
N0.8
N0.7
1 0.2
1 0.2
N0.7
1 0.3
1 0.3
At t = 0, theres no accident with potability 1 (so 1 at t = 0). Then at t = 1, theres 0.8 chance
of no accident (0.8) and 0.2 chance of accident (1 0.2). So on and so forth.
Payout #1: Pay 100 for each node at t = 1, 2, 3
time t Pr ()
1 0.8
2 0.8
2
+ 0.2 (0.7) = 0.78
3 0.8
3
+ 0.8 (0.2) (0.7) + 0.2 (0.7) (0.8) + 0.2 (0.3) (0.7) = 0.778
Payout #2: Pay 1 at t = 3 if the pattern is at t = 1, 2, 3
1 () = 0.8
3
Equivalence:
100
_
0.8 + 0.78
2
+ 0.778
3
_
= 0.8
3

3
1
1 =
100
_
0.8 + 0.78
2
+ 0.778
3
_
0.8
3

3
= 100 0.8
3
_
0.8 (1 +i)
2
+ 0.78 (1 +i) + 0.778
_
= 100 0.8
3
_
0.8 1.04
2
+ 0.78 1.04 + 0.778
_
= 479. 39
c _Yufeng Guo 461
www.archactuarial.com www.guo.coursehost.com 462
Problem 18
A logic approach is

[67]+1

[67]
|
67+2
= |
69
|
[67]+1
|
[67]
We are given: 3
[r]+1
= 4
[r+1]
4
r+2
= 5
[r+1]+1
To simplify, set r + 1 = j
3
[1]+1
= 4
[]
4
+1
= 5
[]+1

[67]+1
=
4
5

67+1
=
4
5

68
=
4
5
_
1
7700
8000
_
= 0.03

[67]
=
3
4

[671]+1
=
3
4

[66]+1
=
3
4

4
5

67
=
3
4

4
5
_
1
8000
8200
_
= 0.01 463
|
[67]
_
1
[67]
_ _
1
[67]+1
_
= |
69
|
[67]
(1 0.03) (1 0.01 463) = 7700
|
[67]
=
7700
(1 0.03) (1 0.01 463)
= 8056
Problem 19
10
\
40
= 1
..
a
50
..
a
40
13
\
40
= 1
..
a
53
..
a
40
10
\
40
=
13
\
40

..
a
50
=
..
a
53
= 10
..
a
50
= 1 +j
50
+ (
2
j
50
)
2
+ (
3
j
50
)
3
..
a
53
Set j
50
= j
51
= j
52
= j

..
a
50
= 1 +j +j
2

2
+j
3

3
..
a
53
10 = 1 +j +j
2

2
+ 10j
3

3
10 = 1 + 1.06
1
j +j
2
1.06
2
+ 10j
3
1.06
3
Try out the 5 answers. The correct answer is j = 0.954
c _Yufeng Guo 462
www.archactuarial.com www.guo.coursehost.com 463
Problem 20
Year 1
Person D only D & A A only
j 0.001 0.0002 = 0.000 8 0.0002 0.001 0.0002 = 0.000 8
Probability that both D and A survive the rst year
c
0.0008
c
0.0002
c
0.0008
= 0.998 2
Year 2
Person D only D & A A only
j 0.001 0 0.001
Probability that both D and A survive the second year
c
0.001
c
0
c
0.001
= 0.998
Probability that both D and A survive 2 years:
0.998 2 0.998 = 0.996 2
Problem 21
Old model (DeMoivre model): |
r
= . r
New model (general DeMoivre model): |
r
= (. r)
o
The new and the old have the same ..
To solve this problem quickly, youll need memorize the following formula:
under the general DeMoivre model

c
r
=
. r
c + 1
This is why.
|
j
r
=
|
r+|
|
r
=
(. r t)
o
(. r)
o
=
_
1
t
. r
_
o

c
r
=
_
.r
0
(
|
j
r
) dt =
_
.r
0
_
1
t
. r
_
o
dt
=
. r
c + 1
_
_
1
t
. r
_
o+1
_
.r
0
=
. r
c + 1
Set c = 1. Then the general DeMoivre model becomes the familiar DeMoivre model and

c
r
=
. r
2
.
We are given:

c
tu
30
=
4
3

c
OlJ
30
. 30
c + 1
=
4
3

. 30
1 + 1
1
c + 1
=
2
3
c + 1 = 1.5

c
tu
60
=
. 60
c + 1
=
. 60
1.5
= 20 . = 90

c
OlJ
70
=
90 70
2
= 10
c _Yufeng Guo 463
www.archactuarial.com www.guo.coursehost.com 464
Problem 22
Let t = T (40)
7 =
_
1000c
0.1|
if t _ 10
2500c
0.1|
if t 10
1000c
0.1|
700 c
0.1|
0.7 t <
1
0.1
ln0.7 = 3. 57
2500c
0.1|
700 c
0.1|

700
2500
t <
1
0.1
ln
700
2500
= 12. 73
7 700 =
_
t < 3. 57 if t _ 10
t < 12. 73 if t 10
t 0 ... 3. 57 ... 10 ... 12. 73 ... . 40 = 60
[ 7 700 [ [ 7 700 [
DeMoivre ) (t) =
1
. r
=
1
100 40
=
1
60
1 (7) =
_
3.57
0
) (t) dt +
_
12. 73
10
) (t) dt =
3.57 + 2.73
60
= 0.105
Problem 23
In a single decrement table,
1j1

r
= j
r

r+1
. In a double decrement table,
1j1

(2)
r
,= j
(2)
r

(2)
r+1
; theres
no such a symbol as j
(2)
r
. The correct formula is
1j1

(2)
r
= j
(r)
r

(2)
r+1
. To be hit by cause (2) in Year 2,
the person needs to survive all causes in Year 1 and then be hit by cause (2) in Year 2.
The rst sentence of the problem says that force of decrement for each cause is constant in each
year of age. So j
(2)
r
= 0.2 means that cause (2) has a constant mortality 0.2 in Year 1. j
(1)
r+1
= 0.15
means that cause (1) has a constant mortality 0.15 in Year 2.
j
(r)
r
= j
0
(1)
r
j
0
(2)
r
=
_
1
0
(1)
r
_
exp
_
j
(2)
r
_
= (1 0.1) exp(0.2) = 0.736 9

(2)
r+1
=
j
(2)
r+1
j
(r)
r+1

(r)
r+1
j
(r)
r+1
= j
0
(1)
r+1
j
0
(2)
r+1
= exp
_
j
(1)
r+1
__
1
0
(2)
r+1
_
= exp(0.15) (1 0.25) = 0.645 5
exp
_
j
(r)
r+1
_
= j
(r)
r+1
= 0.645 5 j
(r)
r+1
= ln0.645 5 = 0.437 7
j
(2)
r+1
= j
(r)
r+1
j
(1)
r+1
= 0.437 7 0.15 = 0.287 7

(2)
r+1
=
0.287 7
0.437 7
(1 0.645 5) = 0.233

1j1

(2)
r
= j
(r)
r

(2)
r+1
= 0.736 9 (0.233) = 0.171 7
c _Yufeng Guo 464
www.archactuarial.com www.guo.coursehost.com 465
Problem 24
..
a
r:3j
= 1 +j
r
+
2
j
r

2
By Equation 2,
|
j
75
= exp
_

_
75+|
75
j(j) dj
_
= exp
_

__
75+|
0
j(j) dj
_
75
0
j(j) dj
__
_
75+|
0
j(j) dj
_
75
0
j(j) dj = 0.01 (75 +t)
1.2
0.01 (75)
1.2

|
j
75
= exp
_
0.01 (75)
1.2
0.01 (75 +t)
1.2
_
j
75
= exp
_
0.01 (75)
1.2
0.01 (75 + 1)
1.2
_
= 0.971 9
2
j
75
= exp
_
0.01 (75)
1.2
0.01 (75 + 2)
1.2
_
= 0.944 5

..
a
r:3j
= 1 + 0.971 9
_
1.11
1
_
+ 0.944 5
_
1.11
2
_
= 2. 642
100
..
a
r:3j
= 100 (2. 642 ) = 264. 2
Problem 25
The average number of trains that arrive during 7am to 7:25am is:
_
25
0
`(t) dt =
_
10
0
0.05 dt +
_
20
10
t
200
dt +
_
25
20
0.1 dt = 0.5 + 0.75 + 0.5 = 1. 75
1 ( = 4) = c
1.75
1.75
4
4!
= 0.0679 - 0.07
Problem 26
Mistakes occur at the rate of 4 per day. So on average theres one mistake every 1,4 day. The
inter-arrival time (i.e. the time between two failures) is an exponential random variable with mean
1,4 day.
T = time (in days) when the 289th mistake occurs
T
1
= time (in days) when the 1st mistake occurs
T
2
= time (in days) between the occurrence of the 1st mistake and the 2nd mistake
...
T
289
= time (in days) between the occurrence of the 288th mistake and the 289th mistake
T = T
1
+ T
2
+ ... + T
289
where T
I
s are independent identically distributed exponential random
variables with a common mean 1,4 and a common variance (1,4)
2
. Under the central limit theorem,
T is approximately normal.
1 (T) = 289 (1,4) = 72. 25 \ ar (T) = 289 (1,4)
2
_
\ ar (T) =
_
289 (1,4) = 4. 25
1 (T 68) = 1 1 (T _ 68) = 1
_
68 72. 25
4. 25
_
= 0.841 3
c _Yufeng Guo 465
www.archactuarial.com www.guo.coursehost.com 466
Problem 27
1 (7) =
_
1
0
jc
|
c
o|
/
|
dt =
_
1
0
jc
|
c
0.06|
c
|
dt =
j
1.06 +j
= 0.03636
j = 0.04
1
_
7
2
_
=
_
1
0
jc
|
_
c
o|
c
|
_
2
dt =
_
1
0
jc
|
c
2.12|
dt
=
j
2.12 +j
=
0.04
2.12 + 0.04
= 0.01 851 9
\ ar (7) = 0.01 851 9 0.03636
2
= 0.017 2
Problem 28
Dont be scared by the cash value. Just treat it as another expense.
Asset share is similar to reserve EXCEPT
1. Reserve considers death only, ignoring lapse; asset share considers both death and lapse,
2. Reserve uses the benet premium; asset share uses the contract (gross) premium.
3. Reserve considers the death benets only, ignore expenses; asset share consider death benet
and expenses.
Consider the time interval t [15, 16]
Time t 0 ... 15 16 ...
Age 40 ... 45 46 ...
Gross premium 90
Renewal expense 0.05 (90) = 4. 54
Cash value C\
Asset share 1150 1320
The source of funding at t = 15 is: 90 4.45 + 1150 = 1235. 55
This accumulates with interest to t = 16: 1235. 55 (1.08) = 1334. 394
The fund pays the following 3 items at t = 16:
Death benet for each death. 10000
(Jto||)
55
= 10000 (0.004) = 40
Lapse expense (i.e. CV) for each lapse. C\
(lost)
55
= 0.05C\
Asset share for each policy remaining. 1320
_
1
(Jto||)
55

(lost)
55
_
= 1320 (1 0.004 0.05) =
1248. 72
Funding=Spending
1334. 394 = 40 + 0.05C\ + 1248. 72 C\ = 913. 48
You dont need to memorize any complex recursive formula about the asset share. Just remember
that the funding is equal to the spending.
c _Yufeng Guo 466
www.archactuarial.com www.guo.coursehost.com 467
Problem 29
1 = 1 1
30:30j
+ (
30
1
30
) 200
..
a
60
1
30:30j
=
30
(
30
1
30
)
60
30
1
30
= (
20
1
30
) (
10
1
50
) = (
20
1
30
) (
10
1
50
) = 0.29374 0.51081 = 0.150 05
..
a
60
= 11.1454
30
= 0.10248
60
= 0.36913
1
30:30j
= 0.10248 (0.150 05) 0.36913 = 0.047 09
1 = 0.047 091 + 200 0.150 05 11.1454 1 = 351
Problem 30
1
_
1
0
c
o|
c

()
|
dt = 3
_
1
0
c
o|
c

()
|
j
(1)
r
dt +
_
1
0
c
o|
c

()
|
j
(2)
r
dt
j
(r)
= j
(1)
r
+j
(2)
r
= 0.02 + 0.04 = 0.06
1
_
1
0
c
o|
c
0.06|
dt = 3
_
1
0
0.02c
o|
c
0.06|
dt +
_
1
0
0.04c
o|
c
0.06|
dt
1
0.06 +c
=
0.06
0.06 +c
+
0.04
0.06 +c
1 = 0.1
c _Yufeng Guo 467

You might also like